高等数学

ε-δ语言与极限存在

极限的概念与性质

待更

2022年数三一道

利用极限定义证明极限存在

问题f:Uo(0;δ)Rf:U^o(0;\delta)\to\Bbb{R}满足

limx0(f(x)+1f(x))=2.\lim_{x \to 0}\left( f(x)+\frac{1}{f(x)}\right)=2.

求证:limx0f(x)=1\begin{aligned}\lim_{x\to 0}f(x)=1\end{aligned}


证明 根据函数极限定义,原问题转化为:ε>0,δ0>0\forall \varepsilon>0,\exists \delta_0>0有 当0<x<δ00<|x|<\delta_0 时,

f(x)+1f(x)2<ε(1)\left|f(x)+\frac 1 {f(x)}-2\right|<\varepsilon \tag{1}

成立,求证:η>0δ1>0\forall \eta>0,\exists \delta_1>0使得,当0<x<δ10<|x|<\delta_1时,

f(x)1<η(2)|f(x)-1|<\eta\tag{2}

其中,对 (1)(1),可作如下化简:

f(x)+1f(x)2=[f(x)1]2f(x)=f(x)121f(x)<ε\left|f(x)+\frac 1 {f(x)}-2\right|=\left|\frac{\left[f(x)-1\right]^2}{f(x)}\right|=|f(x)-1|^2\left|\frac 1 {f(x)}\right|<\varepsilon

所以有:

f(x)12<f(x)ε=εf(x)1+1εf(x)1+ε|f(x)-1|^2<|f(x)|·\varepsilon=\varepsilon|f(x)-1+1|\leq\varepsilon|f(x)-1|+\varepsilon

f(x)1=u|f(x)-1|=uu0u\ge0,则:

u2εuε<0u^2-\varepsilon u-\varepsilon<0

要使上式成立,有:

εε2+4ε2<u<ε+ε2+4ε2\frac{\varepsilon-\sqrt{\varepsilon^2+4\varepsilon}}{2}<u<\frac{\varepsilon+\sqrt{\varepsilon^2+4\varepsilon}}{2}

其中,εε2+4ε2<0\frac{\varepsilon-\sqrt{\varepsilon^2+4\varepsilon}}{2}<0,所以:

0<u=f(x)1<ε+ε2+4ε20<u=|f(x)-1|<\frac{\varepsilon+\sqrt{\varepsilon^2+4\varepsilon}}{2}

因此,η=ε+ε2+4ε2\exists \eta=\frac{\varepsilon+\sqrt{\varepsilon^2+4\varepsilon}}{2},使得f(x)1<η|f(x)-1|<\eta 成立,证毕

单调有界与海涅定理

单调有界证收敛

在求数列极限之前,我们通常应该先证明数列收敛。否则所求出来极限可能不具备正确性!

【单调有界准则】

【例题/必备题】an=1+12++1nlnna_n=1+\frac 1 2+\cdots+\frac 1 n-\ln n,试证{an}\{a_n\} 收敛(2011年真题)

【证明】单调性:

an+1an=1n+1ln(n+1)lnn=1n+1ln(1+1n)<0(x1+x<ln(1+x))\begin{aligned}a_{n+1}-a_n&=\frac1{n+1}-\ln(n+1)-\ln n\\&=\frac1{n+1}-\ln(1+\frac1n)\lt0\quad(\frac{x}{1+x}\lt\ln (1+x))\end{aligned}

有界性:

an=1+12++1nlnn>ln1+ln(1+12)++ln(1+1n)lnn=ln(n+1)lnn>0\begin{aligned}a_n&=1+\frac12+\cdots+\frac1n-\ln n\\&\gt\ln1+\ln(1+\frac12)+\cdots+\ln(1+\frac1n)-\ln n\\&=\ln(n+1)-\ln n\gt0\end{aligned}

{an}\{a_n\} 单调递减有下界,故{an}\{a_n\} 收敛,证毕!

注:本题同样可以利用数列收敛与级数收敛的关系来证明。

事实上,本题源于 1999 年的一道数学真题,关于其具体内容可参阅本文章在积分专栏的《积分与求和的统一》部分。

本题我们证明了该数列收敛。那么它的极限值是多少呢?
事实上我们称其极限值为欧拉-马歇罗尼常数(Euler-Mascheroni constant),用γ\gamma 表示。

γ=limn(k=1n1klnn)=1(1[x]1x)dx0.5772156649\gamma=\lim_{n\to\infty}\left(\sum_{k=1}^n\frac1k-\ln n\right)=\int_1^\infty\left(\frac1{[x]}-\frac1x\right)\mathrm dx\approx0.57721 56649


【番外】一种逆用牛顿莱布尼茨公式的有界性证明
f(x)f(x)[1,+)[1,+\infty) 上有连续一阶导数,f(x)=11+f2(x)(1xln(1+1x))\begin{aligned}f'(x)=\frac 1 {1+f^2(x)}\left(\sqrt{\frac 1 x}-\sqrt{\ln\left(1+\frac 1 x\right)}\right)\end{aligned}
证明limx+f(x)\lim\limits_{x\to+\infty}f(x) 存在.


二级结论:an+1=f(an)a_{n+1}=f(a_n)f(x)f(x) 的单调性

海涅定理

limxaf(x)=L\lim\limits_{x\to a}f(x)=L 存在的充分必要条件是,对于函数f(x)f(x)定义域内任意数列{an}\{a_n\} ,都有:

  1. limnan=a,ana\lim\limits_{n\to\infty}a_n=a,\quad a_n\neq a
  2. limnf(an)=L\lim\limits_{n\to\infty}f(a_n)=L

海涅定理阐述了以任意数列来遍历aa 的领域中的所有实数,在一定程度上仍旧是连续逼近,而不是去跳跃性地靠近极限。这就是以离散性去描述连续性。

海涅定理解释了数列极限与函数极限之间的关系,应用海涅定理,我们可以计算函数极限来间接的计算数列极限;反之,可以通过数列极限来判别函数极限是否存在。

例:计算limn(ntan1n)1n2\lim\limits_{n\to\infty}(n\tan\frac 1 n)^{\frac 1 {n^2}}

limn(ntan1n)1n2=let x=1/nlimx0+(1xtanx)x2=e13\lim_{n\to\infty}(n\tan\frac 1 n)^{\frac 1 {n^2}}\xlongequal{\text{let }x=1/n}\lim_{x\to0^+}(\frac1 x\tan x)^{x^2}=e^{\frac 1 3}

例:证明limx0+sin1x\lim\limits_{x\to0^+}\sin\frac 1 x 不存在
an=12πn,bn=12πn+π/2a_n=\frac 1 {2\pi n},b_n=\frac 1 {2\pi n+\pi/2}. 显然有:

limnsin1an=0limnsin1bn=1\lim_{n\to\infty}\sin\frac 1 {a_n}=0\neq\lim_{n\to\infty}\sin\frac 1 {b_n}=1

由海涅定理可知,原极限不存在.

🦄压缩映像原理

压缩映像原理

设 axb,  af(x)ba\leq x\leq b,\;a\leq f(x)\leq b  且存在常数 k(0,1)k∈(0,1)  使得 f(x2)f(x1)k(x2x1)|f(x_2)−f(x_1)|≤k(x_2−x_1)

  1. 证明:存在唯一的 ξ[a,b]ξ∈[a,b],使得 f(ξ)=ξf(ξ)=ξ
  2. 对于 x0[a,b]∀x_0∈[a,b] ,定义 xn+1=f(xn)x_{n+1}=f(x_n) ,证明 xn{x_n} 收敛且 limnxn=ξ\lim\limits_{n→∞}x_n=ξ

分析】根据待证结论的形式不难发现,应该构造辅助函数+零点定理来证明。
而想要使用零点定理,需要f(x)f(x) 连续,因此我们需要先证明函数连续。

证明
第一步:证明连续性

  x0[a,b],0f(x)f(x0)kxx0limxx0kxx0=0  limxx0f(x)f(x0)=0(夹逼准则)limxx0f(x)=f(x0)\begin{aligned}\because\;&\forall x_0\in[a,b],\quad0≤|f(x)−f(x_0)|≤k|x−x_0|\\&\lim_{x\to x_0}k|x-x_0|=0\\\\\therefore\;&\lim_{x\to x_0}|f(x)-f(x_0)|=0\quad\text{(夹逼准则)}\\&\lim_{x\to x_0}f(x)=f(x_0)\end{aligned}

第二步:证明 ξ 的存在性
令 F(x)=f(x)x,  F(a)=f(a)a0,  F(b)=f(b)b0F(x)=f(x)−x,\;F(a)=f(a)−a≥0,\;F(b)=f(b)−b≤0

① 当 F(a)=0F(a)=0 时,取 ξ=aξ=a 即可
② 当 F(b)=0F(b)=0 时,取 ξ=bξ=b 即可
③ 当 F(a)>0,F(b)<0F(a)>0,F(b)<0 时,由零点定理可得 ξ(a,b)∃ξ∈(a,b) 使得 F(ξ)=0F(ξ)=0

第三步:证明 ξ 的唯一性
利用反证法: 假设 ξ,ηξ,η 都是 f(x)=xf(x)=x 的根,且 ξηξ≠η ,所以有:

0<ξη=f(ξ)f(η)kξη<ξη0<|ξ−η|=|f(ξ)−f(η)|≤k|ξ−η|<|ξ−η|

显然矛盾,故 ξ 唯一

第四步:证明极限存在
注意到:

0xnξ=f(xn1)f(ξ)kxn1ξknx0ξ0≤|x_n−ξ|=|f(x_{n−1})−f(ξ)|≤k|x_{n−1}−ξ|≤⋯≤k^n|x_0−ξ|

显然 limnkn=0\lim\limits_{n→∞}k^n=0 ,因此由夹逼准则可得:

limnxn=ξ\lim_{n→∞}x_n=ξ


压缩映像的推广形式 设 f(x)f(x) 为可导函数,kk 为常数且 k(0,1)k∈(0,1) ,若 f(x)k|f'(x)|≤k ,则由 xn+1=f(xn)x_{n+1}=f(x_n) 产生的数列一定收敛于 f(x)=xf(x)=x 的根 x=ξx=ξ

【证明】由拉格朗日中值定理,可得:

0xnξ=f(xn1)f(ξ)=f(η)xn1ξkxn1ξknx0ξ\begin{aligned}0\leq|x_n-\xi|&=|f(x_{n-1})-f(\xi)|\\&=|f'(\eta)|·|x_{n-1}-\xi|\\&\leq k|x_{n-1}-\xi|\leq\cdots\leq k^n|x_0-\xi|\end{aligned}

从而由夹逼准则可得结论,证毕.


利用压缩映像原理,可以快速“秒杀”一类数列收敛及其极限值的问题。

例:已知x1=2,xn+1=3+4xn\begin{aligned}x_1=2,x_{n+1}=3+\frac 4 {x_n}\end{aligned} ,求 limnxn\lim\limits_{n→∞}x_n
解:根据已知条件,显然xn+1>3n=1,2,3,..x_{n+1}\gt3\quad n=1,2,3,.. 有:

0xn+14=3+4xn4=xn4xn13xn419xn14(13)3xn24(13)n1x24\begin{aligned}0\leq|x_{n+1}-4|&=\bigg|3+\frac 4 {x_n}-4\bigg|=\bigg|\frac {x_n-4} {x_n}\bigg|\\&\leq\frac13|x_n-4|\leq\frac 1 9|x_{n-1}-4|\\&\leq\left(\frac 1 3\right)^3|x_{n-2}-4|\\&\leq\cdots\leq\left(\frac 13\right)^{n-1}|x_2-4|\\\end{aligned}

由夹逼准则可得,limnxn=4\lim\limits_{n→∞}x_n=4 .

事实上,本题相当于把压缩映像原理的推广形式再证明了一遍。这也是考研答题时,为了步骤分而直接利用压缩映像原理的正确解法。上述步骤中的结果 4 则是提前通过:

A=3+4AA=4A=3+\frac 4A\Rightarrow A=4

直接得到的。即是先求出极限结果,再反过来验证,所以利用压缩映像原理求解也被其他考研辅导书描述为“先斩后奏法”。

本题直接利用压缩映像原理说明的写法:
f(x)=3+4x(x>3)\begin{aligned}f(x)=3+\frac 4 x\quad(x\gt3)\end{aligned},由:

f(x)=4x249<1|f'(x)|=\bigg|-\frac 4 {x^2}\bigg|\leq\frac 4 9\lt1

可得,数列必收敛,且收敛于使得A=3+4/AA=3+4/A 成立的AA.

上述内容仅仅只是压缩映像原理在一元函数中的推导。事实上该原理属于不动点定理的一种:巴拿赫不动点定理。具体内容可参阅本站文章:

※极限相关的概念问题与易错点

极限相关的概念易错点

💦limx0sin(xsin1x)xsin1x\begin{aligned}\lim_{x\to0}\frac{\sin(x·\sin\frac 1 x)}{x·\sin\frac 1 x}\end{aligned}不存在
xxx=1nπ,nx=\frac 1 {n\pi},n\to\infty 的方式趋于0 时,因为分母为0没有定义,所以原极限不存在。


💦 设limx0φ(x)=0\begin{aligned}\lim_{x\to 0}\varphi(x)=0\end{aligned} 则下列选项中,哪些是正确的?

  1. limx0φ2(x)φ(x)=0\begin{aligned}\lim_{x\to0}\frac{\varphi^2(x)}{\varphi(x)}=0\end{aligned}
  2. limx0sin(φ(x))φ(x)=0\begin{aligned}\lim_{x\to0}\frac{\sin(\varphi(x))}{\varphi(x)}=0\end{aligned}
  3. limt0f(t)=0\begin{aligned}\lim_{t\to0}f(t)=0\end{aligned} ,则limx0f(φ(x))=0\begin{aligned}\lim_{x\to0}f(\varphi(x))=0\end{aligned}
  4. f(0)=Af'(0)=A ,则limx0f(φ(x))\begin{aligned}\lim_{x\to0}f(\varphi(x))\end{aligned} 存在
  5. f(0)limx0f(x)\begin{aligned}f(0)\neq\lim_{x\to0}f(x)\end{aligned} ,则limx0f(φ(x))\begin{aligned}\lim_{x\to0}f(\varphi(x))\end{aligned} 存在
答案

举反例.

当函数φ(x)0\varphi(x)\equiv 0 是常函数时:

  • 对于 1. , 2. 其对应的函数均无定义,则极限不存在。
  • 对于 3. 由于f(x)f(x) 在 0 处的连续性未知,顾不一定成立。
  • 对于 5. 直接说明f(x)f(x) 在 0 处不连续,因此不成立。

💦设有数列{xn},{yn}\{x_n\},\{y_n\},以下结论正确的是?

  1. limnxnyn=0\begin{aligned}\lim_{n\to\infty}x_ny_n=0\end{aligned},则必有limnxn=0\begin{aligned}\lim_{n\to\infty}x_n=0\end{aligned}limnyn=0\begin{aligned}\lim_{n\to\infty}y_n=0\end{aligned}
  2. limnxnyn=\begin{aligned}\lim_{n\to\infty}x_ny_n=\infty\end{aligned},则必有limnxn=\begin{aligned}\lim_{n\to\infty}x_n=\infty\end{aligned}limnyn=\begin{aligned}\lim_{n\to\infty}y_n=\infty\end{aligned}
  3. xnynx_ny_n 有界,则必有xn,ynx_n,y_n 均有界
  4. xnynx_ny_n 无界,则必有xnx_n 无界 或yny_n 无界
答案

举反例.

  • 对于1. 设xn=1+(1)n,yn=1(1)nx_n=1+(-1)^n,y_n=1-(-1)^n.
    从而xnyn=1(1)2n0(n)x_ny_n=1-(-1)^{2n}\to0\quad(n\to\infty) 但是nn\to\inftyxn,ynx_n,y_n 不存在

  • 对于2. 设xn=1/[1+(1)n],yn=1/[1(1)n]x_n=1/[1+(-1)^n],y_n=1/[1-(-1)^n]

  • 对于3.1xsinx1(x0)\frac 1 x\sin x\to1\quad(x\to0) 有界,而1/x1/x(0,+)(0,+\infty) 无界

💦 设函数y=f(x)y=f(x)(0,+)(0,+\infty)内有界且可导

求证:当limx+f(x)\begin{aligned}\lim_{x\to+\infty}f'(x)\end{aligned} 存在时,必有limx+f(x)=0\begin{aligned}\lim_{x\to+\infty}f'(x)=0\end{aligned}.

取材自 2002研究生考试🔔数一真题

答案

证明limx+f(x)=A\begin{aligned}\lim_{x\to+\infty}f'(x)\end{aligned}=A,利用反证法.

A>0A>0,则ε>0,δ>0\forall\varepsilon>0,\exists \delta>0 使得x>δx>\delta 时,f(x)A<ε|f'(x)-A|<\varepsilon 成立。

即:

Aε<f(x)<A+εA-\varepsilon\quad<f'(x)<\quad A+\varepsilon

在区间[δ,x][\delta,x]上,由 拉格朗日中值定理 知:

f(x)=f(δ)+f(ξ)(xδ)>f(δ)+(Aε)(xδ)f(x)=f(\delta)+f'(\xi)(x-\delta)>f(\delta)+(A-\varepsilon)(x-\delta)

所以

limx+f(x)>f(δ)(Aε)δ+limx+(Aε)x=\lim_{x\to+\infty}f(x)>f(\delta)-(A-\varepsilon)\delta+\lim_{x\to+\infty}(A-\varepsilon)x\\=\infty

与题设“f(x)f(x)(0,+)(0,+\infty)有界”矛盾。

同理可证A<0A<0时同样矛盾,因此:limx+f(x)=0\begin{aligned}\lim_{x\to+\infty}f'(x)=0\end{aligned}.

无穷小与泰勒展开式

常见等价无穷小与泰勒公式

常见等价无穷小与泰勒公式

1cosαxα2x2ln(x+x2+1)x16x3+340x5\begin{aligned}1-\cos^\alpha x&\sim\frac \alpha2x^2\\\ln(x+\sqrt{x^2+1})&\sim x-\frac 1 6x^3+\frac 3 {40}x^5\end{aligned}

(x+y)n=Cnmx..(x+y)^n=C_n^mx..

选择题技巧:利用奇偶性快速辨别展开式系数,见武忠祥强化课


  • 如果f(x)g(x)f(x)\sim g(x),则有0f(x)P(x)dx0g(x)P(x)dx\begin{aligned}\int_0^{f(x)}P(x)dx\sim\int_0^{g(x)}P(x)dx\end{aligned}
  • 如果P(x)Q(x)P(x)\sim Q(x),则有0f(x)P(x)dx0f(x)Q(x)dx\begin{aligned}\int_0^{f(x)}P(x)dx\sim\int_0^{f(x)}Q(x)dx\end{aligned}
    综合以上二者,还可以得到:

If there are {f(x)g(x)P(x)Q(x)then 0f(x)P(x)dx0g(x)Q(x)dx\begin{aligned}\text{If there are }\begin{cases}f(x)\sim g(x)\\P(x)\sim Q(x)\end{cases}\quad\text{then }\int_0^{f(x)}P(x)dx\sim\int_0^{g(x)}Q(x)dx\end{aligned}

  • 如果f(x)0,g(x)0f(x)\to0,g(x)\to0,则有ef(x)eg(x)f(x)g(x)e^{f(x)}-e^{g(x)}\sim f(x)-g(x)
  • 如果f(x)g(x)1\begin{aligned}\frac{f(x)}{g(x)}\to1\end{aligned} ,则有g(x)[lnf(x)lng(x)]f(x)g(x)g(x)[\ln f(x)-\ln g(x)]\sim f(x)-g(x)
求解高阶无穷小问题

求解高阶无穷小

直接给出结论

If F(x)=0f(x)g(x)dx , where f(x)xn,g(x)xm Then F(x)xn(m+1)\begin{aligned}\text{If }F(x)&=\int_0^{f(x)}g(x)\mathrm{d}x\quad\text{ , where }f(x)\sim x^n,g(x)\sim x^m\\\\\text{ Then } F(x)&\sim x^{n(m+1)}\end{aligned}

注:上述等价的前提是x0x \to 0 .

具体推到和扩展详见本站文章:

极限求解方法概述

等价无穷思想与泰勒求极限

等价无穷小与泰勒

上一节

等价无穷大

“奥特曼大法”

i    ai>0,  limna1n+a2n++annn=max{a1,a2,,an}\begin{aligned}\forall i\;\;a_i\gt 0,\;\lim_{n\to\infty}\sqrt[n]{a_1^n+a_2^n+\cdots+a_n^n}=\max\{a_1,a_2,\cdots,a_n\}\\\end{aligned}

黎曼级数及其余项的等价

nn\to\infty 情况下:

  1. a>1a\gt -1 时,1a+2a++nana+1a+1\begin{aligned}1^a+2^a+\cdots+n^a\sim\frac{n^{a+1}}{a+1}\end{aligned}
  2. a=1a=-1 时,1a+2a++nalnn1^a+2^a+\cdots+n^a\sim \ln n
  3. a<1a\lt -1 时,1a+2a++na1^a+2^a+\cdots+n^a 收敛,特别地,在a=2a=-2 时,有k=11k2=π26\begin{aligned}\sum_{k=1}^\infty\frac{1}{k^2}=\frac{\pi^2}{6}\end{aligned}
    我们

https://www.bilibili.com/video/BV1hN4y1F79E
https://zhuanlan.zhihu.com/p/115256300

洛必达法则及其推广形式

洛必达法则| L’Hospital

https://www.bilibili.com/video/BV1gp4y197wb
广义洛必达

带变限积分分式的极限问题

一道很容易想到洛必达,但实际上得不到结果的问题,如下:

已知f(x)=0xcos1t  dt\begin{aligned}f(x)=\int_0^x\cos\frac1t\;\mathrm dt\end{aligned},求f(0)f'(0).

【解析】利用导数的定义:

f(0)=limx00xcos1t  dt0x0f'(0)=\lim_{x\to0}\frac{\int_0^x\cos\frac1t\;dt-0}{x-0}

显然该极限的形式满足0/00/0 型,可以尝试洛必达。于是就有:

f(0)=limx0cos1x∄f'(0)=\lim_{x\to0}\cos\frac1x\Rightarrow\not\exists

然而这个方法是错误的。我们应该清楚,洛必达法则是一个后验法则。也就是说,洛必达之后的极限存在能够反推原极限存在,但是反过来,如果洛必达之后的极限震荡或是不存在(无穷大的情况除外),那么,我们得不到任何结论。

也就是说,此时原极限存在与否是不知道的,不能因为洛之后不存在就反推原极限不存在。同理,即使原极限存在,你也不能下结论洛之后的极限存在。

所以,本题应该另寻他法。
事实上,我们可以通过”转换研究对象“的思想,改造这道题。虽然我们对原极限束手无策,但是如果能换一个变限积分的话,再洛或许就能有奇效了。而对一个变限积分什么办法可以把它转换为另一个变限积分呢?答案是分部积分。

f(x)=0xcos1t  dt=tcos1t0x0x1tsin1t  dtf(x)=\int_0^x\cos\frac1t\;\mathrm dt=t\cos\frac1t\bigg|_0^x-\int_0^x\frac1t\sin\frac1t\;\mathrm dt

然后你会发现,即使如此,此变限积分在x0x\to0 时甚至是无界震荡的!
所以,我们分部积分还需要再变通。

f(x)=0xt2×(1t2cos1t)  dt=0xt2d(sin1t)=t2sin1t0x+20xtsin1t  dt  f(0)=LHospitallimx0xsin1x=0\begin{aligned}f(x)&=\int_0^xt^2\times\left(\frac1{t^2} \cos\frac1t\right)\;\mathrm dt\\&=\int_0^xt^2\mathrm d\left(-\sin\frac1t\right)=-t^2\sin\frac1t\bigg|_0^x+2\int_0^xt\sin\frac1t\;\mathrm dt\\\\\therefore\;f'(0)&\xlongequal{L'Hospital}\lim_{x\to0}x\sin\frac1x=0\end{aligned}

夹逼准则与定积分定义求极限

夹逼准则

在考研数学中,夹逼准则的使用常常结合着拉格朗日中值定理、定积分定义求极限的方法一起使用。而夹逼准则本身的定义和使用方法较为简单,此处略过。

为了方便记录一些解题技巧,此处放上常见 放缩/不等式 结论用于速查。

常见 放缩/不等式 结论:

1.1+2+3++n=n(n+1)22.12+22+32++n2=n(n+1)(2n+1)63.13+23+33++n3=(n(n+1)2)24.ln(n+1)1+12++1n1+lnn5.k=1nf(k)=k=1nkk+1f(k)  dxk=1nkk+1f(x)  dx=1n+1f(x)  dxk=1nf(k)=k=1nkk+1f(k)  dxk=1nk1kf(x)  dx=0nf(x)  dxwhere f(x) on [0,+)\begin{aligned}1.&1+2+3+\cdots+n=\frac{n(n+1)}{2}\\2.&1^2+2^2+3^2+\cdots+n^2=\frac{n(n+1)(2n+1)}{6}\\3.&1^3+2^3+3^3+\cdots+n^3=\left(\frac{n(n+1)}{2}\right)^2\\4.&\ln(n+1)\leq1+\frac1 2+\cdots+\frac 1 n\leq1+\ln n\\5.&\sum_{k=1}^nf(k)=\sum_{k=1}^n\int_k^{k+1}f(k)\;\mathrm dx\leq\sum_{k=1}^n\int_k^{k+1}f(x)\;\mathrm dx=\int_1^{n+1}f(x)\;\mathrm dx\\&\sum_{k=1}^nf(k)=\sum_{k=1}^n\int_k^{k+1}f(k)\;\mathrm dx\geq\sum_{k=1}^n\int_{k-1}^{k}f(x)\;\mathrm dx=\int_0^nf(x)\;\mathrm dx\\\\&\text{where }f(x)\text{ on }[0,+\infty)\nearrow\end{aligned}

定积分定义求极限

基本理论见 本文章积分专题中《积分与求和的统一》一栏

一般情况下,考研数学中并不会给出“完美”的积分定义形式让考生求积分,而是结合一定的放缩法,借助夹逼准则来出题。因此,下面我们将介绍一些可用于应对这类问题的技巧。

作差法夹逼出结果

https://www.bilibili.com/video/BV1iD4y1S7qR

看似等价无穷小的形式

写出佩亚诺余项,证明余项之和为零。https://www.bilibili.com/video/BV1vz4y1k7xn

f(0)=0,f(0)=1f(0)=0,f'(0)=1,考察limn[f(1n2)+f(2n2)++f(nn2)]\begin{aligned}\lim_{n\to\infty}\left[f(\frac{1}{n^2})+f(\frac{2}{n^2})+\cdots+f(\frac{n}{n^2})\right]\end{aligned}

其他特殊性技巧与结论总结

利用中值定理求极限

二重积分中值定理的应用

逆用等价无穷小

exx+1,  ln(x+1)xe^x\sim x+1,\;\ln(x+1)\sim x

例题1limx01cosxcos2xcos3xx2\begin{aligned}\lim_{x\to0}\frac{1-\cos x·\cos 2x·\cos 3x}{x^2}\end{aligned}

答案

方法一: 逆用等价无穷小.

I=limx0cosxcos2xcos3x1x2=limx0ln(cosxcos2xcos3x)x2=(limx0ln(cosx)x2+limx0ln(cos3x)x2+limx0ln(cos3x)x2)=limx01cosxx2+limx01cos2xx2+limx01cos3xx2=12+2+92=7\begin{aligned}I&=-\lim_{x\to0}\frac{\cos x·\cos 2x·\cos 3x-1}{x^2}\\&=-\lim_{x\to0}\frac{\ln(\cos x·\cos 2x·\cos 3x)}{x^2}\\&=-\left(\lim_{x\to0}\frac{\ln(\cos x)}{x^2}+\lim_{x\to0}\frac{\ln(\cos 3x)}{x^2}+\lim_{x\to0}\frac{\ln(\cos 3x)}{x^2}\right)\\&=\lim_{x\to0}\frac{1-\cos x}{x^2}+\lim_{x\to0}\frac{1-\cos 2x}{x^2}+\lim_{x\to0}\frac{1-\cos 3x}{x^2}\\&=\frac 1 2+2+\frac 9 2=7\end{aligned}

方法二:配凑法.

  1cosxcos2xcos3x=1cosx+cosx(1cos2x)+cosxcos2x(1cos3x)I=limx01cosxx2+limx0cosx1cos2xx2+limx0cosxcos2x1cos3xx2=7\begin{aligned}\because\;&1-\cos x·\cos 2x·\cos 3x\\=&1-\cos x+\cos x(1-\cos 2x)+\cos x\cos 2x(1-\cos 3x)\\\\\therefore I&=\lim_{x\to0}\frac{1-\cos x}{x^2}+\lim_{x\to0}\cos x\frac{1-\cos 2x}{x^2}+\lim_{x\to0}\cos x\cos 2x\frac{1-\cos 3x}{x^2}\\&=7\end{aligned}

方法三:泰勒硬展.

  1cosxcos2xcos3xx2={1[112x2+o(x2)][112(2x)2+o(x2)][112(3x)2+o(x2)]}/x2=[(12292)x2+o(x2)]/x2=7+o(x2)I=7\begin{aligned}\because\;&\frac{1-\cos x·\cos 2x·\cos 3x}{x^2}\\&=\left\{1-\left[1-\frac 1 2x^2+o(x^2)\right]·\left[1-\frac 1 2(2x)^2+o(x^2)\right]·\left[1-\frac 1 2(3x)^2+o(x^2)\right]\right\}/x^2\\&=-\left[(-\frac 1 2-2-\frac 9 2)x^2+o(x^2)\right]/x^2=7+o(x^2)\\\therefore I&=7\end{aligned}

例题2

级数收敛反推项函数极限

x, if limnk=1nuk(x) ,then limnun(x)=0\forall x,\text{ if }\quad\exists \lim_{n\to\infty}\sum_{k=1}^nu_k(x)\text{ ,then }\lim_{n\to\infty}u_n(x)=0

【例题】求limn2nn!nn\begin{aligned}\lim_{n\to\infty}\frac{2^n·n!}{n^n}\end{aligned}.

解:对于无穷级数n=02nn!nn\begin{aligned}\sum_{n=0}^\infty\frac{2^n·n!}{n^n}\end{aligned},利用比值判别法:

limn2n+1(n+1)!(n+1)n+1/2nn!nn=2limn(nn+1)n=2/e<1\lim_{n\to\infty}\frac{2^{n+1}·(n+1)!}{(n+1)^{n+1}}\bigg/\frac{2^n·n!}{n^n}=2\lim_{n\to\infty}\left(\frac{n}{n+1}\right)^n=2/e\lt1

因此该无穷级数收敛,从而其一般项极限为0,即n=02nn!nn=0\begin{aligned}\sum_{n=0}^\infty\frac{2^n·n!}{n^n}=0\end{aligned}.

三角函数巧用 nπ

例题1limnsin(πn2+a)\begin{aligned}\lim_{n\to\infty}sin(\pi\sqrt{n^2+a})\end{aligned}

limnsin(πn2+a)=limnsin(nπ1+an2)=limnsin(nπ+aπ2n)(1+x1+x2)=limnsinnπcos(aπ2n)+limncosnπsin(aπ2n)=0\begin{aligned}&\lim_{n\to\infty}\sin(\pi\sqrt{n^2+a})\\&=\lim_{n\to\infty}\sin\left(n\pi\sqrt{1+\frac a {n^2}}\right)\\&=\lim_{n\to\infty}\sin\left(n\pi+\frac{a\pi}{2n}\right)\quad(\sqrt{1+x}\sim1+\frac x 2)\\&=\lim_{n\to\infty}\sin n\pi\cos\left(\frac{a\pi}{2n}\right)+\lim_{n\to\infty}\cos n\pi\sin\left(\frac{a\pi}{2n}\right)\\&=0\end{aligned}

例题2limnsin2(πn2+n)\begin{aligned}\lim_{n\to\infty}\sin^2(\pi\sqrt{n^2+n})\end{aligned}

limnsin2(πn2+n)=limnsin2(πn2+nnπ)=limnsin2[(n2+nn)π]=limnsin2(nπn2+n+n)(分子有理化)=limnsin2(π1+1n+1)=sin2(π2)=1\begin{aligned}&\lim_{n\to\infty}\sin^2(\pi\sqrt{n^2+n})\\&=\lim_{n\to\infty}\sin^2(\pi\sqrt{n^2+n}-n\pi)\\&=\lim_{n\to\infty}\sin^2[(\sqrt{n^2+n}-n)\pi]\\&=\lim_{n\to\infty}\sin^2\left(\frac{n\pi}{\sqrt{n^2+n}+n}\right)\quad(\text{分子有理化})\\&=\lim_{n\to\infty}\sin^2\left(\frac{\pi}{\sqrt{1+\frac1n}+1}\right)=\sin^2(\frac \pi 2)=1\end{aligned}

不难发现,这类含三角函数的数列极限问题,可以利用±nπ\pm n\pi 的方式进行处理。如例题1除了利用等价无穷小外,还可以:

limnsin(πn2+a)=limnsin(nπ+π(n2+an))=limn(1)nsin(πan2+a+n)(分子有理化)=limn(1)nsin0=0\begin{aligned}&\lim_{n\to\infty}\sin(\pi\sqrt{n^2+a})=\lim_{n\to\infty}\sin\left(n\pi+\pi(\sqrt{n^2+a}-n)\right)\\&=\lim_{n\to\infty}(-1)^n\sin\left(\pi\frac{a}{\sqrt{n^2+a}+n}\right)\quad(\text{分子有理化})\\&=\lim_{n\to\infty}(-1)^n\sin0=0\end{aligned}

化长链为短链

经典例题limncosx2cosx22cosx2n(x0)\begin{aligned}\lim_{n\to\infty}\cos\frac x 2\cos\frac x {2^2}·\cdots ·\cos\frac x {2^n}\quad (x\neq 0)\end{aligned}

Tip:右乘一个sin(x/2n)\sin(x/2^n) 利用二倍角公式
答案:sinx/x\sin x/x

高级定理的化简之路

施笃兹定理|O’stolz定理

内容

相关例题

例题1limn12+222++n2×nnn3\begin{aligned}\lim_{n\to\infty}\frac{1^2+2^2\sqrt 2 +\cdots+n^2\times \sqrt[n]{n}}{n^3}\end{aligned}

例题2limx+0xtsintdtx2\begin{aligned}\lim_{x\to+\infty}\frac{\int_0^x t|\sin t|\mathrm dt}{x^2}\end{aligned}

例题3limncos1+cos12++cos1n1n1costtdt\begin{aligned}\lim_{n\to\infty}\frac{\cos 1+\cos \frac 1 2+\cdots+\cos \frac 1 n}{\int_{\frac 1 n}^1\frac{\cos t}{t}dt}\end{aligned}

例题4limx+0xtcost  dtx\begin{aligned}\lim_{x\to+\infty} \frac{\int_0^x \sqrt{t} \cos t \;\mathrm dt}{x}\end{aligned}

答案

利用 O’stolz 定理。

斯特林公式| Stirling’s approximation

斯特林公式给出了计算阶乘n!n! 的近似值的数学公式。即:

n!2πn(ne)nn!\approx\sqrt{2\pi n}\left(\frac n e\right)^n

更进一步地,有:

limnn!2πn(n/e)n=1,  n!=2πn(ne)n(1+Θ(1n))\lim_{n\to\infty}\frac{n!}{\sqrt{2\pi n}(n/e)^n}=1,\;n!=\sqrt{2\pi n}\left(\frac n e\right)^n\left(1+\Theta(\frac 1 n)\right)


Stirling 公式在实际工程应用上具有重要价值,甚至在算法设计中时间复杂度的阶数判断也起到作用。

例题limn1n!n(1n+ln1+2n+ln2+nn+lnn)\begin{aligned}\lim_{n\to\infty}\frac 1 {\sqrt[n]{n!}}\left(\frac 1 {n+\ln 1}+\frac 2 {n+\ln 2}+\cdots \frac n {n+\ln n}\right)\end{aligned}

本题选自 武忠祥每日一题 2022.6.5

答案

解:

\begin{aligned}I&=\lim_{n\to\infty}(2\pi n)^{-\frac 1 {2n}}\frac e n\sum_{k=1}^n\frac k {n+\ln k}\\&=e\lim_{n\to\infty}\frac 1 n\sum_{k=1}^n\frac k {n+\ln k}\;,\quad(2\pi n)^{-1/2n}=\exp\{-\frac 1 {2n}\ln(2\pi n)\}\to1,\;n\to\infty\\\\\because&\;0\leq \ln k\leq \ln n\\\\\therefore&\frac k{n+\ln n}\leq \frac k{n+\ln k}\leq \frac k n\\\\\therefore&\;e\frac {\sum_\limits{k=1}^n k} {n(n+\ln n)}\leq I\leq e\frac {\sum_\limits{k=1}^n k} {n^2}\Rightarrow I=\frac e 2.\quad(\sum_{k=1}^nk=\frac{n(n+1)}{2})\end{aligned}

【注】本题同样可以直接先做如下处理:

nn!n=exp(1nk=1nkn)exp(01lnxdx)=e  ,  n\begin{aligned}\frac n {\sqrt[n]{n!}}=\exp(-\frac 1 n\sum_{k=1}^n\frac k n)\to\exp(-\int_0^1\ln xdx)=e\;,\;n\to\infty\end{aligned}

从而化简原式,避免了 Stirling 公式 的使用。

一元函数及其微分学

连续、间断与渐近线

曲线的渐近线及其求解问题

曲线的渐近线

如果存在直线LL 使得当xx0x\to x_0xx\to\infty (包括正负无穷)时,曲线y=f(x)y=f(x) 上的任意动点M(x,y)M(x,y) 到直线LL 的距离d(M,L)0d(M,L)\to0 则称直线LL 为曲线y=f(x)y=f(x)渐近线

  • x0,  s.t.limxx0f(x)=\exists x_0,\;s.t.\lim\limits_{x\to x_0}f(x)=\infty 则 直线L:x=x0L:x=x_0 为曲线的铅直渐近线
  • limxf(x)=A\lim\limits_{x\to \infty}f(x)=A 则 直线L:y=AL:y=A 为曲线的水平渐近线
  • L:y=kx+b,k0\exists L:y=kx+b,k\neq0 是曲线的渐近线,称此直线为其斜渐近线
  • 无穷远处的一侧最多只会存在水平渐近线或斜渐近线的一种

对于斜渐近线y=kx+by=kx+b 我们有结论:

k=limxyx,  b=limx(ykx)k=\lim_{x\to\infty}\frac yx,\;b=\lim_{x\to\infty}(y-kx)

此外,只考虑斜渐近线的可能,我们还有另一种稍微快捷一点的计算方法,这依托于我们对无穷小和泰勒公式的理解。

斜渐近线与等价无穷小

如果y=f(x)y=f(x) 可以通过泰勒展开写为y=f(x)=kx+b+α(x)y=f(x)=kx+b+\alpha(x) 的形式(xx\to\infty),则该曲线的斜渐近线就是y=kx+by=kx+b. 其中,α(x)0\alpha(x)\to0.

例题 求曲线y=x1+x(1+x)x(x>0)\begin{aligned}y=\frac{x^{1+x}}{(1+x)^x}\quad(x\gt0)\end{aligned} 的斜渐近线方程。(2020年)

【解】因为求斜渐近线要求xx\to\infty,而我们熟悉的等价无穷小都属于x0x\to0 的情况。由此我们时常需要将无穷小量α(x)\alpha(x) 化为o(1x)o(\frac1x) 的形式。把1/x1/x 看作一个整体处理。

y=x1+x(1+x)x=x(x1+x)x=x(11+1x)x=xexln(1+1x)=xexp{x[1x+12x2+o(1x2)]}=xexp{1+12x+o(1x)}=xeexp{12x+o(1x)}=xe(1+12x+o(1x))=xe+12e+o(1)\begin{aligned}y&=\frac{x^{1+x}}{(1+x)^x}=x\left(\frac x{1+x}\right)^x\\&=x\left(\frac1{1+\frac1x}\right)^x=xe^{-x\ln(1+\frac1x)}\\&=x·exp\{-x\left[\frac1x+\frac1{2x^2}+o(\frac1{x^2})\right]\}\\&=x·exp\{-1+\frac1{2x}+o(\frac1x)\}\\&=\frac{x}{e}·exp\{\frac1{2x}+o(\frac1x)\}\\&=\frac xe\left(1+\frac1{2x}+o(\frac1x)\right)\\&=\frac xe+\frac1{2e}+o(1)\end{aligned}

由此可得,该曲线的斜渐近线方程为y=(2x+1)/(2e)y=(2x+1)/(2e).

隐函数确定的曲线的渐近线

例题 设曲线由x3+y3=y2x^3+y^3=y^2 确定,求其斜渐近线方程。

【解】根据斜渐近线的公式,将原方程凑出待求极限的形式。

  1. 求斜率k=limxyxk=\lim\limits_{x\to\infty}\frac yx. 将y/xy/x 看作一个整体。对原方程两边同时除以x3x^3,得:

1+(yx)3=(yx)21x1+(\frac yx)^3=(\frac y x)^2·\frac1x

xx\to\infty ,得:1+k3=0k=11+k^3=0\Rightarrow k=-1.

  1. 求截距b=limx(y+x)b=\lim\limits_{x\to\infty}(y+x). 将(y+x)(y+x) 看作一个整体。对原方程利用立法和公式:

x3+y3=(y+x)(y2xy+x2)=y2y+x=y2y2xy+x2=11+xy+(xy)2\begin{aligned}x^3+y^3&=(y+x)(y^2-xy+x^2)=y^2\\\Rightarrow y+x&=\frac{y^2}{y^2-xy+x^2}=\frac1{1+\frac xy+(\frac xy)^2}\end{aligned}

xx\to\infty ,得:b=1/(1+1k+1k2)=1/3b=1/(1+\frac 1k+\frac1{k^2})=1/3.

从而,该曲线的斜渐近线方程为:y=x+13y=x+\frac13.

反函数相关的问题

反函数的积分(一道图像题)
反函数的导数(包括二阶导)

函数的平均值

导数定义及常用导数表

反函数的导数
参数确定的一元函数的导数 dy/dx=dy/dt/(dx/dt)

导数极限定理

https://zhuanlan.zhihu.com/p/427887032

区间内均有定义的导函数无可去、跳跃和无穷间断点。
不能忘记前提条件!参考:导函数的间断点问题

基本概念结论与反例全收录

基本概念结论与反例全收录

【注】可能用到的函数:

  1. 狄利特雷函数(Dirichlet function)

D(x)=limk(limj(cos(k!πx))2j)D(x)=\lim_{k\to\infty}\left(\lim_{j\to\infty}(\cos(k!\pi x))^{2j}\right)

其中,k,jk,j 为整数。事实上,D(x)D(x) 还可以写成:

D(x)={1,xQ0,xQD(x)=\begin{cases}1,&x\in Q\\0,&x\notin Q\end{cases}

其中,QQ 为有理数集.

  1. 魏尔特斯拉函数(Weierstrass function)

W(x)=n=0ancos(bnπx)W(x)=\sum_{n=0}^\infty a^n\cos(b^n\pi x)

其中,0<a<10\lt a\lt1bb 为正奇数,使得ab>1+3π/2ab\gt1+3\pi/2,满足该条件的最小b=7b=7.

  1. Volterra 函数

处处可导,导函数有界,但导函数不可积(黎曼可积)

f(x)f(x) 在某点连续

f(x)f(x)x=x0x=x_0 处连续

  1.   f(x)\Rightarrow\;f(x)x0x_0 的邻域内有定义
  2.   limxx0f(x)=f(x0)\Rightarrow\;\lim\limits_{x\to x_0}f(x)=f(x_0)
  3. ⇏  f(x)\not\Rightarrow\;f(x)x0x_0 的邻域内连续【反例:f(x)=xD(x)f(x)=xD(x) 只在x=0x=0 处连续】
  4. ⇏  f(x)\not\Rightarrow\;f(x)x0x_0 处可导 【反例:处处连续但处处不可导的函数 魏尔特斯拉函数

f(x)f(x) 在某邻域内连续

f(x)f(x)x0x_0 的某邻域U(x0,δ)U(x_0,\delta) 内连续

  1.   f(x)\Rightarrow\;f(x)U(x0,δ)U(x_0,\delta) 有定义
  2.   f(x)\Rightarrow\;f(x)U(x0,δ)U(x_0,\delta) 处处连续
  3. ⇏  f(x)\not\Rightarrow\;f(x)x0x_0 处可导【反例: 魏尔特斯拉函数

f(x)f(x) 在去心邻域内连续

f(x)f(x)x0x_0 的去心邻域U˚(x0,δ)\mathring{U}(x_0, \delta) 内连续

  1.   f(x)\Rightarrow\;f(x)U˚(x0,δ)\mathring{U}(x_0, \delta) 有定义
  2.   f(x)\Rightarrow\;f(x)U˚(x0,δ)\mathring{U}(x_0, \delta) 处处连续
  3. ⇏  f(x)\not\Rightarrow\;f(x)x0x_0 处可导【反例: 魏尔特斯拉函数
  4. ⇏  f(x)\not\Rightarrow\;f(x)x0x_0 处极限存在【(x0δ),(x0+δ)(x_0-\delta),(x_0+\delta) 分别连续,但左右极限未必相等】

f(x)f(x) 在某点可导

f(x)f(x)x=x0x=x_0 处导数存在

  1.   f(x)\Rightarrow\;f'(x)x0x_0 处有定义
  2.   limxx0f(x)=limxx0+f(x)\Rightarrow\;\lim\limits_{x\to x_0^-}f'(x)=\lim\limits_{x\to x_0^+}f'(x)
  3.   f(x)\Rightarrow\;f(x)x0x_0 处连续
  4. ⇏  f(x)\not\Rightarrow\;f(x)x0x_0 处光滑
  5. ⇏  f(x)\not\Rightarrow\;f(x)x0x_0 邻域内连续
  6. ⇏  f(x)\not\Rightarrow\;f(x)x0x_0 邻域内可导
  7. ⇏  f(x)\not\Rightarrow\;f'(x)x0x_0 处可导

关于 (4,5)的反例如下:

f(x)={x2sin1x,x00,x=0\begin{aligned}f(x)=\begin{cases}\begin{aligned}x^2\sin\frac{1}{x}\end{aligned}&,x\neq0\\0&,x=0\end{cases}\end{aligned}

高阶导数的求解问题

高阶导数的求解问题

在考研范围内,求解高阶导数问题(通常是求f(n)(0)f^{(n)}(0))的这类问题一般的求解方法有:
找规律法、数学归纳法、泰勒级数莱布尼茨求导公式等方法。

下面将通过例题一一介绍相关方法。

泰勒级数法

【例题1】 已知f(x)=x1x2\begin{aligned}f(x)=\frac x {1-x^2}\end{aligned},求fn(0)=?  (n=1,2,3,)f^n(0)=?\ \ (n=1,2,3,\dots)

泰勒展开法

泰勒公式法(优先选择) 利用 长除法 对本 多项式函数 进行处理:

x+x3+x5+1x2) x xx3 x3x3x5\begin{array}{} & x+x^3+x^5+\cdots \\ 1-x^2 & \overline{)\ x\quad\quad\quad\quad\quad\quad\quad } \\ & \underline{\ x-x^3\quad\quad}\quad\quad\ \\ & x^3\quad\quad \\& \underline{x^3-x^5} \\ & \cdots \end{array}

于是有:

f(x)=x+x3+x5++x(2n1)+f(x)=x+x^3+x^5+\cdots+x^{(2n-1)}+\cdots

与泰勒展开式进行对比,显然有:

f(2n)(0)(2n)!=0f(2n1)(0)(2n1)!=1\frac {f^{(2n)}(0)}{(2n)!}=0\\\frac {f^{(2n-1)}(0)}{(2n-1)!}=1

因此:

f(2n)(0)=0;f(2n1)(0)=(2n1)!f^{(2n)}(0)=0;\quad f^{(2n-1)}(0)=(2n-1)!

数学归纳法

另解 数学归纳法

f(x)=x1x2=(1+x)+(1x)2(1+x)(1x)=12(11x11+x)f(x)=\frac x {1-x^2}=\frac{(1+x)+(1-x)}{2(1+x)(1-x)}=\frac 1 2 \left(\frac 1 {1-x}-\frac 1 {1+x}\right)

易归纳得:

(11+ax)(n)=(1)nn!an(1+ax)n+1\left(\frac 1 {1+ax}\right)^{(n)}=\frac{(-1)^nn!a^n}{(1+ax)^{n+1}}

因此有:

f(n)(x)=n!2[1(1x)n+1(1)n(1+x)n+1]f(n)(0)=n!2[1+(1)n+1]={n!,n is odd,0,n is even.\begin{aligned}f^{(n)}(x)=\frac{n!} 2\left[\frac 1 {(1-x)^{n+1}}-\frac {(-1)^n}{(1+x)^{n+1}}\right]\\\\f^{(n)}(0)=\frac {n!} 2\left[1+(-1)^{n+1}\right]=\begin{cases}n!\quad ,\text{n is odd,} \\0\quad ,\text{n is even.}\end{cases}\end{aligned}

莱布尼茨求导公式

设函数u,vu,vnn 阶导函数,则:

(uv)(n)=Cn0uv(n)+Cn1uv(n1)+Cn2uv(n2)++Cnnu(n)v(uv)^{(n)}=C_n^0uv^{(n)}+C_n^1u'v^{(n-1)}+C_n^2u''v^{(n-2)}+\cdots+C_n^nu^{(n)}v

  • 在求解高阶导数问题式,通常我们需要构造两个函数相乘且其中一个函数是幂函数的情况,以希望借助幂函数在有限次求导结果变为0的这个性质化简答案,得到线性递推关系

【例题2】f(x)=(xa)nφ(x)f(x)=(x-a)^n\varphi(x), 其中φ(x)\varphi(x)x=ax=a 的某邻域内有n1n-1 阶连续导数,试求f(n)(a)f^{(n)}(a).


【例题3】 已知f(x)=arcsinx1x2\begin{aligned}f(x)=\frac {\arcsin x} {\sqrt{1-x^2}}\end{aligned},求fn(0)=?  (n=1,2,3,)f^n(0)=?\ \ (n=1,2,3,\dots)(李永乐《660题》)

分析与求解

显然有:

f(x) dx=arcsinx d(arcsinx)=12arcsin2x+C\int f(x)\ \text{d}x=\int \arcsin x \ \text{d}(\arcsin x)=\frac 1 2\arcsin^2x+C

而:

arcsinx=x+12x33+1×32×4x55++(2n1)!!(2n)!!x2n+12n1+arcsin2x=?\begin{aligned}\arcsin x&=x+\frac 1 2·\frac {x^3} 3+\frac{1\times 3}{2\times4}·\frac{x^5} 5+\cdots+\frac{(2n-1)!!}{(2n)!!}·\frac{x^{2n+1}}{2n-1}+\cdots\\\arcsin ^2x&=?\end{aligned}

显然本题如果继续用泰勒进行处理,会十分麻烦,难以总结出确切的规律,并且总结的过程中已然用到了 数学归纳法 因此,不如考虑直接归纳。

f(x)=11x2[1+xf(x)]f(x)=11x2[f(x)+3xf(x)]f(x)=11x2[4f(x)+5xf(x)]f(4)(x)=11x2[9f(x)+7xf(x)]f(5)(x)=11x2[16f(x)+9xf(4)(x)]f(n)(x)=11x2[Anf(n2)(x)+Bnxf(n1)(x)]\begin{aligned}f'(x)&=\frac 1 {1-x^2}\left[1+xf(x)\right]\\f''(x)&=\frac 1 {1-x^2}\left[f(x)+3xf'(x)\right]\\f'''(x)&=\frac 1 {1-x^2}\left[4f'(x)+5xf''(x)\right]\\f^{(4)}(x)&=\frac 1 {1-x^2}\left[9f''(x)+7xf'''(x)\right]\\f^{(5)}(x)&=\frac 1 {1-x^2}\left[16f'''(x)+9xf^{(4)}(x)\right]\\\cdots&\\f^{(n)}(x)&=\frac 1 {1-x^2}\left[A_nf^{(n-2)}(x)+B_nxf^{(n-1)}(x)\right]\end{aligned}

可归纳出An=An1+Bn1,Bn=2n1A_n=A_{n-1}+B_{n-1}\quad ,B_n=2n-1

而令x=0x=0并带入上述各式,有res=0if n is even,RESn=An×RESn1res=0\quad \text{if n is even,}\quad RES_n=A_n\times RES_{n-1}\quad

于是问题转化为求解数列AnA_n通项公式

  AnAn1=Bn1=2n3  i=2n(AnAn1)=  RESn=RESn1i=2n(RESnRESn1)=RESn=\begin{aligned}\because&\; A_n-A_{n-1}=B_{n-1}=2n-3\\\therefore&\;\sum_{i=2}^{n}\left(A_n-A_{n-1}\right)=\\\therefore&\; RES_n=RES_{n-1}\\&\prod_{i=2}^n \left(\frac{RES_n}{RES_{n-1}}\right)=\\&RES_n=\end{aligned}

中值定理证明问题

中值定理全收录

介值定理/零点定理

设函数f(x)f(x) 在闭区间[a,b][a,b] 上连续,并且f(a)f(b)f(a)\neq f(b),则x0(a,b)\exists x_0\in(a,b) ,使得:

min{f(a),f(b)}<f(x0)<max{f(a),f(b)}\min\{f(a),f(b)\}\lt f(x_0)\lt\max\{f(a),f(b)\}

该定理被称为介值定理(Intermediate Value Theorem),事实上应该是连续函数的介值定理

如果取F(x)=f(x)f(x0)F(x)=f(x)-f(x_0),显然有F(a),F(b)F(a),F(b) 异号,也即是F(a)F(b)<0F(a)F(b)\lt0.
于是可得出零点定理
设函数f(x)f(x) 在闭区间[a,b][a,b] 上连续,并且f(a)f(b)<0f(a)f(b)\lt0,则x0(a,b)\exists x_0\in(a,b) ,使得:

f(x0)=0f(x_0)=0

费马引理

y=f(x)y=f(x) 在点x=x0x=x_0 处可导且取得极值,则必有f(x0)=0f'(x_0)=0.

事实上,该定理就是可导函数取得极值的必要条件.

罗尔定理

设函数f(x)f(x)[a,b][a,b]连续,在(a,b)(a,b) 内可导且f(a)=f(b)f(a)=f(b) ,则ξ(a,b)\exists\xi\in(a,b) 使得:

f(ξ)=0f'(\xi)=0

罗尔定理其实就是特殊的拉格朗日中值定理.

拉格朗日中值定理

设函数f(x)f(x)[a,b][a,b]连续,在(a,b)(a,b) 内可导,则ξ(a,b)\exists\xi\in(a,b) 使得:

f(b)f(a)=f(ξ)(ba)f(b)-f(a)=f'(\xi)(b-a)

也可以写作:Δy=f(x+θΔx)Δx\Delta y=f'(x+\theta\Delta x)·\Delta x.
因此该定理也被称为 有限增量定理/公式

拉格朗日中值定理其实就是g(x)=xg(x)=x 时的柯西中值定理,也是特殊的泰勒中值定理.

柯西中值定理

设函数f(x),g(x)f(x),g(x)[a,b][a,b]连续,在(a,b)(a,b) 内可导且g(x)0g'(x)\neq0,则ξ(a,b)\exists\xi\in(a,b) 使得:

f(b)f(a)g(b)g(a)=f(ξ)g(ξ)\frac{f(b)-f(a)}{g(b)-g(a)}=\frac{f'(\xi)}{g'(\xi)}

泰勒中值定理

如果函数f(x)f(x) 在含有x0x_0 的某个开区间(a,b)(a,b) 内具有直到(n+1)(n+1) 阶的导数,则对任一x(a,b)x\in(a,b) 有:

f(x)=f(x0)+f(x0)1!(xx0)+f(x0)2!(xx0)2++f(n)(x0)n!(xx0)n+Rn(x)f(x)=f(x_0)+\frac {f'(x_0)}{1!}(x-x_0)+\frac {f''(x_0)}{2!}(x-x_0)^2+\cdots+\frac{f^{(n)}(x_0)}{n!}(x-x_0)^n+R_n(x)

其中,ξ(x0,x)\exists \xi\in(x_0,x)(x,x0)(x,x_0) 使得:

Rn(x)=f(n+1)(ξ)(n+1)!(xx0)n+1R_n(x)=\frac{f^{(n+1)}(\xi)}{(n+1)!}(x-x_0)^{n+1}

积分中值定理

积分第一中值定理

如果函数f(x)f(x)闭区间[a,b][a,b] 上连续,则ξ[a,b]\exists \xi\in[a,b] 使得:

abf(x)dx=f(ξ)(ba)\int_a^bf(x)dx=f(\xi)(b-a)

二重积分的中值定理

如果函数f(x,y)f(x,y)有界闭区域DD 上连续,则在(ξ,η)D\exists (\xi,\eta)\in D ,使得:

Df(x,y)dσ=f(ξ,η)SD\iint_Df(x,y)d\sigma=f(\xi,\eta)·S_D

推广形式

如果函数f(x),g(x)f(x),g(x)闭区间[a,b][a,b] 上连续,且g(x)g(x)[a,b][a,b]不变号,则ξ[a,b]\exists \xi\in [a,b] 使得:

abf(x)g(x)dx=f(ξ)abg(x)dx\int_a^bf(x)g(x)dx=f(\xi)\int_a^bg(x)dx

注意,无论是积分中值定理还是其推广形式,都指出存在的ξ\xi 是在闭区间[a,b][a,b] 内的。这是因为证明之成立的原理是介值定理。但事实上,积分中值定理可以改写在开区间,即ξ(a,b)\exists \xi\in(a,b) ,此时需要借助 拉格朗日中值定理 加以说明。

如 设F(x)=axf(x)dxF(x)=\int_a^xf(x)dx,由拉格朗日中值定理,有:

ξ(a,b),s.t.F(b)F(a)ba=F(ξ)=f(ξ)\exists \xi\in(a,b),\quad s.t.\frac{F(b)-F(a)}{b-a}=F'(\xi)=f(\xi)

abf(x)dx=(ba)f(ξ)\int_a^bf(x)dx=(b-a)f'(\xi).

※达布定理及其导出结论

达布定理及其导出结论

介值定理中,我们强调如果函数f(x)f(x) 在闭区间连续,那么介于其两个端点值之间的任意一个实数值kk,我们都能够在区间内找到这样的一个点x0x_0 使得f(x0)=kf(x_0)=k.
这说明,定义在闭区间上的连续函数满足介值性质

而达布定理想要说明的是,定义在闭区间上的导函数满足介值性质

达布定理 设函数f(x)f(x) 在闭区间[a,b][a,b] 上可导,并且f+(a)f(b)f'_+(a)\neq f'_-(b),则x0(a,b)\exists x_0\in(a,b) ,使得:

f(x0)=kf'(x_0)=k

其中kk 介于f+(a),f(b)f'_+(a),f'_-(b) 之间。

不难发现,达布定理一定程度上弱化了介值定理。达布定理指出,如果一个函数A是另一个函数B的导函数,那么即使函数A不连续,也满足介值性质。

达布定理的证明

F(x)=f(x)kxF(x)=f(x)-kx,其中kk 介于f+(a),f(b)f'_+(a),f'_-(b) 之间。
F(x)=f(x)kF+(a)F(b)<0F'(x)=f'(x)-k\quad \Rightarrow F'_+(a)F'_-(b)\lt0
不妨设F+(a)>0,F(b)<0F'_+(a)\gt0,F'_-(b)\lt0,于是有:

F+(a)=limxa+F(x)F(a)xa>0F(x)>F(a)(极限保号性)F(b)=limxbF(x)F(b)xb<0F(x)>F(b)(同上)\begin{aligned}F'_+(a)&=\lim_{x\to a^+}\frac{F(x)-F(a)}{x-a}\gt0\Rightarrow F(x)\gt F(a)\quad(\text{极限保号性})\\F'_-(b)&=\lim_{x\to b^-}\frac{F(x)-F(b)}{x-b}\lt0\Rightarrow F(x)\gt F(b)\quad(\text{同上})\end{aligned}

可见Fmax(x)>max{F(a),F(b)}F_{\max}(x)\gt\max\{F(a),F(b)\},因此由最大最小值定理知,x0(a,b)\exists x_0\in(a,b) 使得F(x0)=Fmax(x)F(x_0)=F_{\max}(x)

再由F(x)F(x)[a,b][a,b] 可导,可得F(x)F(x) 连续,根据费马引理,得F(x0)=0F'(x_0)=0.

参考介值定理与零点定理的关系,我们根据达布定理也能得到相应的导数零点定理

🔔导数零点定理
设函数f(x)f(x) 在闭区间[a,b][a,b] 上可导,并且f+(a)f(b)<0f'_+(a) f'_-(b)\lt0,则x0(a,b)\exists x_0\in(a,b) ,使得:f(x0)=0f'(x_0)=0.

♾️推论
区间内均有定义的导函数不存在第一类间断点。若存在,就表示区间内任意一点的导数值不可能取导左导数极限与右导数极限之间的某个值,这与达布定理矛盾。

更加具体的证明参考:关于导函数间断点的探讨

方程的根与函数零点问题
常用的辅助函数构造法

积分因子法

积分因子法即利用指数函数的辅助函数构造方法。

拓展:构造二元辅助函数的证明题

乘积求导法则逆用

  • f=ff''=f 型 可推广到f(n)=ff^{(n)}=f
  • fg=gff''g=gf''

f(x)f(x) 二阶可导,且f(0)=0f(0)=0,求证ξ(π2,π2)  ,s.t.f(ξ)=f(ξ)(1+2tan2ξ)\begin{aligned}\exists \xi\in(-\frac\pi2,\frac\pi2)\;,\quad s.t. f''(\xi)=f(\xi)(1+2\tan^2\xi)\end{aligned}

微分方程构造法

辅助多项式法

常数 K 值法

当题目要求证明的 formula 形如f(n)(ξ)=g(a,b)f^{(n)}(\xi)=g(\boldsymbol a,b) 时,通常可以利用 常数K值法解决. (其中g(a,b)g(a,b) 是关于a,b\boldsymbol a,b 的常数,a=[a1,a2,...,an]T\boldsymbol a=[a_1,a_2,...,a_n]^T

具体步骤如下:

  1. K=g(a,b)K=g(\boldsymbol a,b)
  2. 构造辅助函数F(x)=g(a,x)KF(x)=g(\boldsymbol a,x)-K(去分母),显然F(b)=0F(b)=0,若辅助函数能使得F(ai)=F(b)=0F(a_i)=F(b)=0,则可利用多次罗尔定理得到新的 formula
  3. 对新的 formula 利用拉格朗日、泰勒展开、达布定理甚至再用一次罗尔定理 等方法,想方设法 引入ξ\xi 及其nn 阶导
  4. 化简式子证明K=f(n)(ξ)K=f^{(n)}(\xi)
  5. 证毕

参考:中值定理杀手锏-常数 K值法|▷考研竞赛凯哥

经典例题

问题f(x)f(x)[0,2][0,2] 具有连续导数,其中f(0)=0,f(1)=2,f(2)=1f(0)=0,f(1)=2,f(2)=1
(1) 证明:至少存在一点ξ(0,1)\xi \in (0,1),使得f(ξ)=2f'(\xi)=2.
(2) 证明:至少存在一点η(0,2)\eta\in(0,2),使得f(η)1=2ηf(η)2f(η)2η2+2ηf'(\eta)-1=2\eta f(\eta)-2f(\eta)-2\eta^2+2\eta.

答案
  1. 构造F(x)=f(x)2xF(x)=f(x)-2x. 则F(0)=f(1)=0F(0)=f(1)=0,从而ξ(0,1),F(ξ)=0\exists \xi \in (0,1),F'(\xi)=0
  2. 构造F(x)=e2xx2[f(x)x]F(x)=e^{2x-x^2}[f(x)-x]. 则F(0)=0,F(1)=e>0,F(2)=1<0F(0)=0,F(1)=e\gt 0,F(2)=-1\lt 0
    从而μ(1,2),F(μ)=0=F(0)\exists \mu\in(1,2),F(\mu)=0=F(0); 于是η(0,μ)(0,2),F(η)=0\exists \eta\in(0,\mu)\subset (0,2),F'(\eta)=0

问题 设函数f(x)f(x) 在闭区间[0,1][0,1] 上具有一阶导数,且f(0)=0f(0)=0
证明:ξ(0,1)\exists \xi \in(0,1),使得f(ξ)=201f(x)dxf'(\xi)=2\int_0^1f(x)\mathrm dx

答案

g(x)=0xf(x)dxg(x)=\int_0^xf(x)\mathrm dx,由泰勒中值定理,有:

g(x)=g(x0)+11!g(x0)(xx0)+12!g(ξ)(xx0)2ξ(x0,x)g(x)=g(x_0)+\frac 1{1!}g'(x_0)(x-x_0)+\frac 1{2!}g''(\xi)(x-x_0)^2\quad\xi\in(x_0,x)

x0=0,x=1x_0=0,x=1,则有:

01f(x)dx=g(1)=0+0+12g(ξ)=12f(ξ)ξ(0,1)\int_0^1f(x)\mathrm dx=g(1)=0+0+\frac 1 2g''(\xi)=\frac 1 2f'(\xi)\quad\xi\in(0,1)

证毕.

真假双中值证明问题的求解

双中值证明问题

中值 θ 的极限问题

中值 θ 的极限

在各类中值定理的表述中,我们通常有写ξ(a,b),s.t.  f(ξ)=0\exists \xi\in (a,b),\quad s.t.\;f(\xi)=0. 而这里的ξ\xi 可以改写为:

ξ=a+(ba)θ,  θ(0,1)\xi=a+(b-a)\theta,\;\theta\in(0,1)

例如:设f(x)f(x)(x0δ,x0+δ)(x_0-\delta,x_0+\delta)nn 阶连续导数,则对于任意h,h<δh,|h|\lt\delta 有:

f(x0+h)=f(x0)+hf(x0)++hm1(m1)!f(m1)(x0)+hmm!f(m)(x0+θh)f(x_0+h)=f(x_0)+hf'(x_0)+\cdots+\frac{h^{m-1}}{(m-1)!}f^{(m-1)}(x_0)+\frac{h^m}{m!}f^{(m)}(x_0+\theta h)

在理解上述概念后,我们通常需要考察当bab\to a 时,θ\theta极限问题,对于上述泰勒公式的表达,则是考察h0h\to0 时的极限。


例题1】已知f(x)=arctanx,  f(h)f(0)=f(θh)hf(x)=\arctan x,\;f(h)-f(0)=f'(\theta h)h,求limh0θ2\lim_{h\to0}\theta^2.

由于表达式已知,我们可以直接由已知条件得:

arctanh=h11+θ2h2\arctan h=h\frac{1}{1+\theta^2h^2}

从而,有:

limh0θ2=limh0harctanhh2arctanh=limh013h3h3=13\lim_{h\to0}\theta^2=\lim_{h\to0}\frac{h-\arctan h}{h^2\arctan h}=\lim_{h\to0}\frac{\frac 1 3h^3}{h^3}=\frac 1 3

从该例题中不难发现,θ\theta 通常是hh 的函数,而求θ\theta 的极限问题通常我们需要将其剥离出来,这里正好我们知道f(x)f(x) 的表达式,所以可以直接将其分离。

例题2】已知函数f(x)f(x) 具有n+1n+1 阶连续导数,且有:
f(a+h)=f(a)+f(a)h++f(n1)(a)(n1)!hn1+f(n)(a+θh)n!hn\begin{aligned}f(a+h)=f(a)+f'(a)h+\cdots+\frac{f^{(n-1)}(a)}{(n-1)!}h^{n-1}+\frac{f^{(n)}(a+\theta h)}{n!}h^n\end{aligned},其中0<θ<10\lt\theta\lt1,并且f(n+1)(a)0f^{(n+1)}(a)\neq0. 证明limh0θ=1n+1\begin{aligned}\lim_{h\to0}\theta=\frac 1{n+1}\end{aligned}.

证明:重新对f(a+h)f(a+h)aa 处进行泰勒展开到n+1n+1 项(因为已知有这么多阶可导)

f(a+h)=f(a)+f(a)h++f(n)(a)n!hn+f(n+1)(a)(n+1)!hn+1+o(hn+1)(1)f(a+h)=f(a)+f'(a)h+\cdots+\frac{f^{(n)}(a)}{n!}h^n+\frac{f^{(n+1)}(a)}{(n+1)!}h^{n+1}+o(h^{n+1})\quad(1)

又对f(n)(a+θh)f^{(n)}(a+\theta h) 泰勒展开(为了剥离出待求参数)

f(n)(a+θh)=f(n)(a)+f(n+1)(a)θh+o(h)(2)f^{(n)}(a+\theta h)=f^{(n)}(a)+f^{(n+1)}(a)\theta h+o(h)\qquad(2)

将 (2) 式代入已知原式,即有:

f(a+h)=f(a)+f(a)h++f(n1)(a)(n1)!hn1+f(n)(a)+f(n+1)(a)θh+o(h)n!hn=f(a)+f(a)h++f(n1)(a)(n1)!hn1+f(n)(a)n!hn+f(n+1)(a)n!hn+1θ+o(hn+1)\begin{aligned}f(a+h)&=f(a)+f'(a)h+\cdots+\frac{f^{(n-1)}(a)}{(n-1)!}h^{n-1}+\frac{f^{(n)}(a)+f^{(n+1)}(a)\theta h+o(h)}{n!}h^n\\&=f(a)+f'(a)h+\cdots+\frac{f^{(n-1)}(a)}{(n-1)!}h^{n-1}+\frac{f^{(n)}(a)}{n!}h^n+\frac{f^{(n+1)}(a)}{n!}h^{n+1}·\theta+o(h^{n+1})\end{aligned}

将该结果与 (1) 式对比可得:

f(n+1)(a)(n+1)!hn+1+o(hn+1)=f(n+1)(a)n!hn+1θ+o(hn+1)f(n+1)(a)(n+1)!+o(hn+1)hn+1=f(n+1)(a)n!θ+o(hn+1)hn+1f(n+1)(a)(n+1)!=f(n+1)(a)n!limh0θ( let h0)limh0θ=1n+1( 已知 f(n+1)(a)0)\begin{aligned}&\frac{f^{(n+1)}(a)}{(n+1)!}h^{n+1}+o(h^{n+1})=\frac{f^{(n+1)}(a)}{n!}h^{n+1}·\theta+o(h^{n+1})\\&\Rightarrow\frac{f^{(n+1)}(a)}{(n+1)!}+\frac{o(h^{n+1})}{h^{n+1}}=\frac{f^{(n+1)}(a)}{n!}·\theta+\frac{o(h^{n+1})}{h^{n+1}}\\&\Rightarrow\frac{f^{(n+1)}(a)}{(n+1)!}=\frac{f^{(n+1)}(a)}{n!}·\lim_{h\to0}\theta\quad(\text{ let }h\to0)\\&\Rightarrow\lim_{h\to0}\theta=\frac 1 {n+1}\quad(\text{ 已知 }f^{(n+1)}(a)\neq0)\end{aligned}

证毕!


解题模板 从上述例题中,我们不难总结出对于此类问题的解法。

通常题目给定的等式是泰勒展开或拉格朗日 “不完全” 的情况。
如 【例题2】 中已知 n+1 阶可导,但是给定等式却只展到 n 阶。

此时,我们需要自行对其进行展开,利用带佩亚诺余项的方式得到新的等式。
然后再利用泰勒展开式将含θ\theta 项的式子展开,以达到剥离出参数的目的,余项同样是佩亚诺余项。

最终联立以上式子,即可得到关于θ\theta 的等式,最后再结合已知条件移项求极限即可。

事实上,本问题还可以利用带拉格朗日余项的方法进行求解,基本思路其实是相通的。
实操参看:求中值θ的极限|▷考研竞赛凯哥

不等式证明问题

常见的不等式结论

在考研数学、竞赛数学中,对于不等式证明的考察常常是一大难点,这类问题需要考生充分发挥数学思维能力,借助放缩、各类抽象函数的定理等方式进行构造
这类题常常没有唯一的解题思路,需要考生的不断 刷题 日积月累,形成数学思维。

下面,我将总结出数学中,常见的不等式证明结论与具体的证明方法,以供参考。

1.x1+xln(1+x)x(泰勒不等式)2.π2xsinxx,  x[0,π/2](若尔当不等式)3.xtanx4πx,  x[0,π/4](琴生不等式)4.ba<b+ca+c  where a>b;  ba>b+ca+c  where a<b(糖水不等式)\begin{aligned}1.&\frac{x}{1+x}\leq \ln(1+x)\leq x\quad(\text{泰勒不等式})\\2.&\frac\pi 2x\leq\sin x\leq x,\;x\in[0,\pi/2]\quad(\text{若尔当不等式})\\3.&x\leq\tan x\leq\frac 4 \pi x,\;x\in[0,\pi/4]\quad(\text{琴生不等式})\\4.&\frac b a\lt\frac{b+c}{a+c}\;\text{where }a\gt b;\;\frac b a\gt\frac{b+c}{a+c}\;\text{where }a\lt b\quad(\text{糖水不等式})\\\end{aligned}

其他不等式,如下列这些不等式对解题非常有帮助,因此我们将穿插在具体的题型中给出并适当证明。

  • JensenJensen 不等式
  • YoungYoung 不等式
  • 流行积分不等式
  • HadamardHadamard 不等式
  • PoincarePoincare 不等式
  • CauchyCauchy 不等式
  • ChebyshevChebyshev 不等式
  • MinkowskiMinkowski 不等式
利用泰勒/函数凹凸性证明微分不等式

琴生不等式| Jesen

函数凹凸性的性质

函数f(x)f(x) 在区间[a,b][a,b] 有二阶导数,且f(x)>0f''(x)\gt0 ,则 证明:

  1. x0[a,b],  f(x)f(x0)+f(x0)(xx0)\forall x_0\in[a,b],\;f(x)\geq f(x_0)+f'(x_0)(x-x_0)
  2. x1,x2[a,b],  f(x1+x22)f(x1)+f(x2)2\begin{aligned}\forall x_1,x_2\in[a,b],\;f\left(\frac{x_1+x_2}{2}\right)\leq\frac{f(x_1)+f(x_2)}{2}\end{aligned}

证明:将函数在x=x0x=x_0 处展开,有:f(x)=f(x0)+f(x0)(xx0)+0.5f(ξ)(xx0)2f(x)=f(x_0)+f'(x_0)(x-x_0)+0.5f''(\xi)(x-x_0)^2
显然,当f(x)>0f''(x)\gt0 时,问题1得证。
下面证明问题2. 取x0=(x1+x2)/2x_0=(x_1+x_2)/2 ,依次取x=x1,x=x2x=x_1,x=x_2 可得到方程:

f(x1)=f(x1+x22)+f(x1+x22)x1x22f(x2)=f(x1+x22)f(x1+x22)x1x22\begin{aligned}f(x_1)&=f\left(\frac{x_1+x_2}{2}\right)+f'\left(\frac{x_1+x_2}{2}\right)·\frac{x_1-x_2}{2}\\f(x_2)&=f\left(\frac{x_1+x_2}{2}\right)-f'\left(\frac{x_1+x_2}{2}\right)·\frac{x_1-x_2}{2}\\\end{aligned}

两式相加即可得到结论。

该性质具有十分有用的几何意义,即:凹函数的切线在曲线下方,割线在曲线上方。

凹凸性的几何性质

加权琴生不等式

上面我们介绍的性质是琴生不等式的特殊形式,事实上该不等式还有对应的加权版本:

f(x)>0,λ+μ=1f(λx1+μx2)λf(x1)+μf(x2)f''(x)\gt0,\lambda+\mu=1\Rightarrow f(\lambda x_1+\mu x_2)\leq\lambda f(x_1)+\mu f(x_2)

证明方法同样可以泰勒展开并联立方程组,当然还有构造辅助函数证明等。
同样地,该结论可以继续推广:

f(x)>0,i=1nki=1,ki>0f(i=1nkixi)i=1nkif(xi)f''(x)\gt0,\sum_{i=1}^nk_i=1,k_i\gt0\Rightarrow f\left(\sum_{i=1}^nk_ix_i\right)\leq\sum_{i=1}^nk_if(x_i)

经典例题

一旦出现有λ+μ=1\lambda+\mu=1 形式的“系数之和为一”这种情况出现,最先想到凹凸性/泰勒展开。

例题2 证明sinxsinyxycosy12xy,x,y(,+)\begin{aligned}\bigg|\frac{\sin x-\sin y}{x-y}-\cos y\bigg|\leq \frac 1 2 |x-y|,\quad x,y\in(-\infty,+\infty)\end{aligned}

本题在求解时,很容易想到利用拉格朗日中值定理对绝对值内的左边进行处理。
而引入cosξ\cos\xi 之后,又不得不再次使用“拉中”,从而连续的两次拉中使得精度不够。因此,应该使用泰勒。

【证明】将sinx\sin xx=yx=y 处展开:

sinx=siny+cosy(xy)0.5sinξ(xy)2\sin x=\sin y+\cos y(x-y)-0.5\sin\xi(x-y)^2

代入原式,得:

LHS=12sinξ(xy)12xy=RHSLHS=\bigg|\frac 1 2\sin\xi(x-y)\bigg|\leq\frac 1 2|x-y|=RHS

证毕。

杨氏不等式| Young

a,b>0,    p,q>1,1p+1q=1a,b\gt0,\;\;p,q\gt1,\frac 1 p+\frac 1 q=1 则有:abapp+bqq\begin{aligned}ab\leq\frac{a^p}{p}+\frac{b^q}{q}\end{aligned}


证明:两边取对数,再适当变形可得:

lna+lnb=ln(1pap+1qbq)1plnap+1qlnbq=ln(1pap+1qbq)\begin{aligned}\ln a+\ln b&=\ln\left(\frac1{p}a^p+\frac 1 qb^q\right)\\\Leftrightarrow\frac 1 p\ln a^p+\frac 1 q\ln b^q&=\ln\left(\frac1{p}a^p+\frac 1 qb^q\right)\\\end{aligned}

f(x)=lnxf(x)=\ln x,为凹函数,取x1=ap,x2=bqx_1=a^p,x_2=b^q 利用琴生不等式即可证明。

利用中值定理证明不等式

两边拉格朗日型

如果已知条件中出现对抽象函数f(x)f(x) 的单调性的描述,通常可将待证不等式通过移项+拉中的方法处理。从而得出两个隶属于不同区间的中值ξ1,ξ2\xi_1,\xi_2 .
进而将待证结论化为证明f(ξ1)  ()  f(ξ2)f(\xi_1)\;\leq(\geq)\; f(\xi_2) 的形式,最后利用单调性解决。


【例题1】利用本节思想证明带权琴生不等式

证明:根据题意,要证

f(x)>0,λ+μ=1f(λx1+μx2)λf(x1)+μf(x2)f''(x)\gt0,\lambda+\mu=1\Rightarrow f(\lambda x_1+\mu x_2)\leq\lambda f(x_1)+\mu f(x_2)

即证:

(λ+μ)f(λx1+μx2)λf(x1)+μf(x2)λ[f(λx1+μx2)f(x1)]μ[f(x2)f(λx1+μx2)]拉格朗日中值定理f(ξ1)f(ξ2)\begin{aligned}(\lambda+\mu)f(\lambda x_1+\mu x_2)&\leq\lambda f(x_1)+\mu f(x_2)\\\\\Leftrightarrow\lambda[f(\lambda x_1+\mu x_2)-f(x_1)]&\leq\mu[f(x_2)-f(\lambda x_1+\mu x_2)]\\\\\text{拉格朗日中值定理}\Leftrightarrow f'(\xi_1)&\leq f'(\xi_2)\end{aligned}

由于f(x)>0f''(x)\gt0 所以f(x)f'(x)[x1,x2][x_1,x_2] 上单调递增。
x1<ξ1<λx1+μx2<ξ2<x2x_1\lt\xi_1\lt\lambda x_1+\mu x_2\lt\xi_2\lt x_2,故原不等式得证!


【例题2】f(x)f(x)[a,b][a,b] 单调递增,证明abxf(x)dxa+b2abf(x)dx\int_a^bxf(x)dx\geq\frac{a+b}{2}\int_a^bf(x)dx.

证明:根据题意,要证

abxf(x)dxa+b2abf(x)dx\int_a^bxf(x)dx\geq\frac{a+b}{2}\int_a^bf(x)dx

即证:

ab[(xa+b2)f(x)]dx=f(ξ1)a(a+b)/2(xa+b2)dx+f(ξ2)(a+b)/2b(xa+b2)dx=(ba)28[f(ξ1)f(ξ2)]0\begin{aligned}\int_a^b\left[\left(x-\frac{a+b}{2}\right)f(x)\right]dx&=f(\xi_1)\int_a^{(a+b)/2}\left(x-\frac{a+b}{2}\right)dx\\&\quad+f(\xi_2)\int_{(a+b)/2}^b\left(x-\frac{a+b}{2}\right)dx\\&=\frac{(b-a)^2}{8}[f(\xi_1)-f(\xi_2)]\\&\geq0\end{aligned}

由于f(x)f(x)[a,b][a,b] 单调递增,a<ξ1<(a+b)/2<ξ2<ba\lt\xi_1\lt(a+b)/2\lt\xi_2\lt b,故证毕!

本题可将bb 写为变量xx 构造变限积分求导解决

柯西中值放缩型

当不等式中含有反三角函数、对数函数等复合而成的函数时,如果出现了乘积,将会使得构造函数求导时导函数十分难以求解。
因此,我们可以将其化为除法形式,利用柯西中值定理将一次求导就能直接简化的函数简化(就如前面说的反三角函数等)。而通常我们需要找到特殊的临界点以定出ξ\xi 的区间。

【例题1】 证明ln(1+x)arcsinx>1x1+x(0<x<1)\begin{aligned}\frac{\ln(1+x)}{\arcsin x}\gt\sqrt{\frac{1-x}{1+x}}\quad(0\lt x\lt1)\end{aligned}

证明:利用柯西中值定理

LHS=ln(1+x)ln(1+0)arcsinxarcsin0=11+ξ11ξ2=1ξ1+ξ>1x1+x(0<ξ<x<1)LHS=\frac{\ln(1+x)-\ln(1+0)}{\arcsin x-\arcsin0}=\frac{\frac 1 {1+\xi}}{\frac1{\sqrt{1-\xi^2}}}=\sqrt{\frac{1-\xi}{1+\xi}}\gt\sqrt{\frac{1-x}{1+x}}\quad(0\lt \xi\lt x\lt1)

注:函数f(x)=(1x)/(1+x)f(x)=\sqrt{(1-x)/(1+x)} 显然单调递减

【例题2】 证明流行积分不等式

流行积分不等式

f(x)f(x)[0,1][0,1] 有一阶连续导数,0<f(x)1,f(0)=00\lt f'(x)\leq1,f(0)=0,证明:

(01f(x)dx)201f3(x)dx\left(\int_0^1f(x)dx\right)^2\geq\int_0^1f^3(x)dx

证明:根据题意,显然f(x)f(x) 不恒为0,于是待证结论可化为:

(01f(x)dx)201f3(x)dx1\frac{\left(\int_0^1f(x)dx\right)^2}{\int_0^1f^3(x)dx}\geq1

F(x)=(01f(t)dt)2,G(x)=0xf3(t)dtF(x)=(\int_0^1f(t)dt)^2,G(x)=\int_0^xf^3(t)dt,于是有:

F(1)F(0)G(1)G(0)=20ξf(t)dtf3(ξ)=20ξf(t)dtf(ξ)f2(ξ)\frac{F(1)-F(0)}{G(1)-G(0)}=\frac{2\int_0^\xi f(t)dt}{f^3(\xi)}=\frac{2\int_0^\xi f(t)dt·f(\xi)}{f^2(\xi)}

g(x)=0xf(t)dt,  h(x)=f2(x)g(x)=\int_0^xf(t)dt,\;h(x)=f^2(x),于是有:

2g(ξ)2g(0)h(ξ)h(0)=2f(η)2f(η)f(η)=1f(η)1\frac{2g(\xi)-2g(0)}{h(\xi)-h(0)}=\frac{2f(\eta)}{2f(\eta)f'(\eta)}=\frac{1}{f'(\eta)}\geq1

证毕!

本题可将积分上限11 变为xx 构造变限积分求导解决

【推广】设f(x)f(x)[a,b][a,b] 有一阶连续导数,0<f(x)2n+1,f(a)=00\lt f'(x)\leq\frac{2}{n+1},f(a)=0,则:

(01fn(x)dx)201f2n+1(x)dx\left(\int_0^1f^n(x)dx\right)^2\geq\int_0^1f^{2n+1}(x)dx

含多阶导数已知条件证明函数不等式

含多阶导数已知条件的不等式

经典例题 已知f(x)f(x) 二阶可导
且有:f(0)=1,f(0)=0,x>0  f(x)5f(x)+6f(x)0f(0)=1,f'(0)=0,\forall x\gt0\;f''(x)-5f'(x)+6f(x)\geq 0
证明f(x)3e2x2e3x(x0)f(x)\geq 3e^{2x}-2e^{3x}\quad(x\geq0)

【证明】由f(x)5f(x)+6f(x)=[f(x)2f(x)]3[f(x)2f(x)]f''(x)-5f'(x)+6f(x)=[f''(x)-2f'(x)]-3[f'(x)-2f(x)]
F(x)=e3x[f(x)2f(x)]F(x)=e^{-3x}[f'(x)-2f(x)],则有F(x)0F'(x)\geq0,从而F(x)F(0)=2F(x)\geq F(0)=-2
e3x[f(x)2f(x)]2  f(x)2f(x)+2e3x0e^{-3x}[f'(x)-2f(x)]\geq-2\;\Rightarrow f'(x)-2f(x)+2e^{3x}\geq0

又由f(x)2f(x)+2e3x=[f(x)+3×2e3x]2[f(x)+2e3x]f'(x)-2f(x)+2e^{3x}=[f'(x)+3\times2e^{3x}]-2[f(x)+2e^{3x}]
G(x)=e2x[f(x)+2e3x]G(x)=e^{-2x}[f(x)+2e^{3x}],则有G(x)0G'(x)\geq0,从而G(x)G(0)=3G(x)\geq G(0)=3
e2x[f(x)+2e3x]3  f(x)3e2x2e3xe^{-2x}[f(x)+2e^{3x}]\geq3\;\Rightarrow f(x)\geq3e^{2x}-2e^{3x}

证毕


解题模板

此类问题的标志一般是:给定f(x)f(x)x=x0x=x_0 处的初始条件,再给出关于f(x)f(x) 的多个导函数组成的多项式的不等式关系,最后要求证明关于f(x)f(x) 本身的不等式。

求解本问题的方法通常是将导函数的不等式利用【中值定理证明问题】的辅助函数构造方法建立辅助函数F(x)F(x),然后对其求导(通常结果F(x)F(x) 单调),利用F(x),F(x0)F(x),F(x_0) 的关系(初始条件)得到一个新的导函数不等式关系,实现对原条件的降阶。如此这般套娃下去,直到降阶到辅助函数只与f(x)f(x) 本身有关时,通常就证明完毕!


举一反三 已知f(0)0,f(0)0,f(x)f(x)f(0)\geq0,f'(0)\geq0,f''(x)\geq f(x),试证当x0x\geq 0 时,f(x)f(0)+f(0)xf(x)\geq f(0)+f'(0)x

事实上本问题看要证的结论就可以推测是泰勒公式相关的问题,其具体方法如下:

f(x)f(x) 由麦克劳林公式得:f(x)=f(0)+f(0)x+0.5f(ξ)x2f(x)=f(0)+f'(0)x+0.5f''(\xi)x^2
要证原不等式成立,只需证f(ξ)0f''(\xi)\geq0 即可。

利用求解中值定理证明问题的思想,考虑f(ξ)=f(ξ)f''(\xi)=f(\xi) 这类问题时,我们通常左右两边同时加或减f(ξ)f'(\xi) 作为中间变量,移项得到[f(ξ)+f(ξ)][f(ξ)+f(ξ)]=0[f'(\xi)+f''(\xi)]-[f(\xi)+f'(\xi)]=0
从而构造辅助函数F(x)=ex(f+f)F(x)=e^{-x}(f+f')

此处同理,我们构造F(x)F(x) 后不难得出F(x)0F'(x)\geq0 恒成立,从而F(x)F(0)0F(x)\geq F(0)\geq 0
f(x)+f(x)0f(x)+f'(x)\geq0 恒成立。此时我们把原问题的条件f(x)f(x)f''(x)\geq f(x) 降阶了!!

类似地,再令G(x)=exf(x)G(x)=e^xf(x),就得到G(x)0,G(x)G(0)0G'(x)\geq0,G(x)\geq G(0)\geq 0
f(x)0f(x)\geq0,结合原条件,就有f(x)f(x)0f(ξ)0f''(x)\geq f(x)\geq 0 \Rightarrow f''(\xi)\geq 0

所以f(x)=f(0)+f(0)x+0.5f(ξ)x2f(0)+f(0)xf(x)=f(0)+f'(0)x+0.5f''(\xi)x^2\geq f(0)+f'(0)x,证毕!

🎲事实上,本题如果没有考虑到利用泰勒公式,仍然可以使用上面的模板进行求解。
具体做法参阅:一道复习全书的改编不等式经典题|▷考研竞赛凯哥

变限积分与泰勒展开证明积分不等式

不断化简的函数构造之路

经典例题】证明2aa2+b2<lnblnaba<1ab(0<a<b)\begin{aligned}\frac{2a}{a^2+b^2}\lt\frac{\ln b-\ln a}{b-a}\lt\frac{1}{\sqrt{ab}}\quad(0\lt a\lt b)\end{aligned}

阿达马不等式| Hadamard

f(x)f(x) 为在[a,b][a,b] 上连续的凹函数,则有:

f(a+b2)1baabf(x)  dxf(a)+f(b)2f\left(\frac{a+b}2\right)\leq\frac{1}{b-a}\int_a^bf(x)\;\mathrm dx\leq\frac{f(a)+f(b)}{2}

实际上,阿达马不等式可以看作琴生不等式的积分形式,该结论找出了凹函数不等式结论的中间项。
中间的这一项称为积分平均值

积分平均值加强结论

f(x)f(x)[a,b][a,b] 上有f(x)>0f''(x)\gt0,证明:

abf[g(x)]  dxbaf(abg(x)  dxba)\frac{\int_a^bf[g(x)]\;dx}{b-a}\geq f\left(\frac{\int_a^bg(x)\;dx}{b-a}\right)

【证明】由凹函数性质,可得不等式f(x)f(x0)+f(x0)(xx0)f(x)\geq f(x_0)+f'(x_0)(x-x_0)
xg(x),x0abg(x)  dxba\begin{aligned}x\leftarrow g(x),x_0\leftarrow\frac{\int_a^bg(x)\;dx}{b-a}\end{aligned} ,两边同时积分即可得到结论。

不难发现,取xx,x0(a+b)/2x\leftarrow x,x_0\leftarrow(a+b)/2 即可证明阿达马不等式的左边。
不仅如此,本结论还是很多积分不等式问题的题源

带定积分项的证明

  • 构造变限积分函数再泰勒展开
  • 对被积函数进行泰勒展开
※利用柯西不等式直接证明积分不等式

柯西不等式| Cauchy

二维形式(a2+b2)(c2+d2)(ac+bd)2(a^2+b^2)(c^2+d^2)\geq (ac+bd)^2

积分形式
f(x),g(x)f(x),g(x)[a,b][a,b] 连续(可弱化为可积),则:

(abf(x)g(x)  dx)2abf2(x)  dxabg2(x)  dx\left(\int_a^bf(x)g(x)\;dx\right)^2\leq\int_a^bf^2(x)\;dx\int_a^bg^2(x)\;dx

其中,等号成立的条件是f(x),g(x)f(x),g(x) 线性相关。


证明:对于任意实函数和实数λ\lambda,总有[f(x)+λg(x)]20[f(x)+\lambda g(x)]^2\geq0 恒成立。
两边积分再展开,得:

ab[f(x)+λg(x)]2  dx0λ2abg2(x)  dx+2λabf(x)g(x)  dx+abf2(x)  dx0\begin{aligned}\int_a^b[f(x)+\lambda g(x)]^2\;dx&\geq0\\\lambda^2\int_a^bg^2(x)\;dx+2\lambda\int_a^bf(x)g(x)\;dx+\int_a^bf^2(x)\;dx&\geq0\end{aligned}

对于关于λ\lambda 的二次函数要大于等于零,当且仅当其判别式 Δ小于或等于0,即:

Δ=b24ac=4λ2(abf(x)g(x)  dx)24λ2abf2(x)  dxabg2(x)  dx0\Delta=b^2-4ac\\=4\lambda^2\left(\int_a^bf(x)g(x)\;dx\right)^2-4\lambda^2\int_a^bf^2(x)\;dx\int_a^bg^2(x)\;dx\leq0

证毕。

事实上,柯西不等式还可以通过二重积分、定积分的定义,以及前面我们学过的构造函数利用单调性的方法证明(取bb 为自变量),此处不再展开。


【例题1:给出两个积分乘积的情况】证明0πxasinxdx0π2acosxdxπ34(a>0)\begin{aligned}\int_0^\pi xa^{\sin x}dx\int_0^{\frac \pi 2}a^{-\cos x}dx\geq\frac{\pi^3}4\quad(a\gt0)\end{aligned}

利用区间再现的导出结论:[0→π]∫xf(sinx)dx=[0→π]/2∫πf(sinx)dx[0→π/2]∫f(cosx)dx=0→π/2∫f(sinx)dx 得:

LHS=π0π2asinxdx0π2asinxdxπ(0π2asinxasinxdx)2=π34LHS=\pi\int_0^{\frac \pi 2}a^{\sin x}dx\int_0^{\frac \pi 2}a^{-\sin x}dx\geq\pi\left(\int_0^{\frac \pi 2}\sqrt{a^{\sin x}a^{-\sin x}}dx\right)^2=\frac{\pi^3}4

【例题2:给出积分的平方的情况f(x)f(x)[a,b][a,b] 连续,f(x)0,abf(x)=1f(x)\geq0,\int_a^bf(x)=1,证明:

(abf(x)sinkx  dx)2(abf(x)coskx  dx)21\left(\int_a^bf(x)\sin kx\;dx\right)^2\left(\int_a^bf(x)\cos kx\;dx\right)^2\leq1

【例题3:给出一项函数平方的积分,需要自己配凑另一项的情况f(x)f(x)[a,b][a,b] 连续,f(a)=f(b)=0,abf2(x)=1f(a)=f(b)=0,\int_a^bf^2(x)=1,证明ab[xf(x)]2  dx>1/4\int_a^b[xf'(x)]^2\;dx\gt1/4

直接将已知条件作为需要的另一项:

ab[xf(x)]2  dx=ab[xf(x)]2  dxabf2(x)  dx(12abxdf2(x))2=14(abf2(x)dx)=14\begin{aligned}\int_a^b[xf'(x)]^2\;dx&=\int_a^b[xf'(x)]^2\;dx\int_a^bf^2(x)\;dx\\&\geq\left(\frac 1 2\int_a^bx\mathrm df^2(x)\right)^2=\frac 1 4\left(\int_a^bf^2(x)dx\right)\\&=\frac 1 4\end{aligned}

取等条件为kf(x)=xf(x)kf(x)=xf'(x),解此微分方程得:f(x)=Cxkf(x)=Cx^k.
f(a)=f(b)=0f(x)=0f(a)=f(b)=0\Rightarrow f(x)=0 恒成立,与已知条件(积分为1)矛盾。
故取不到等号。证毕!

🦄【例题4:逆用莱布尼茨公式+柯西不等式f(x)f(x)[0,1][0,1] 存在连续导函数,且有f(0)=0f(0)=0,试证01f2(x)dx1201[f(x)]2dx\int_0^1f^2(x)dx\leq\frac 1 2\int_0^1[f'(x)]^2dx

f(0)=0f(0)=0,逆用莱布尼茨公式得:

f(x)=0xf(t)dtf(0)=0xf(t)dtf(x)=\int_0^xf'(t)dt-f(0)=\int_0^xf'(t)dt

两边平方即可引入柯西不等式的积分平方项:

f2(x)=(0xf(t)1  dt)2x0x[f(t)]2dtx01[f(t)]2dt01f2(x)dx01[f(t)]2dt01x  dx=1201[f(t)]2dt\begin{aligned}f^2(x)&=\left(\int_0^xf'(t)·1\;dt\right)^2\leq x\int_0^x[f'(t)]^2dt\leq x\int_0^1[f'(t)]^2dt\\\Rightarrow\int_0^1f^2(x)dx&\leq\int_0^1[f'(t)]^2dt\int_0^1x\;dx=\frac 1 2\int_0^1[f'(t)]^2dt\end{aligned}

证毕!

🦄【例题5:对上一题缩小精度】f(x)f(x)[0,1][0,1] 存在连续导函数,且有f(0)=f(1)=0f(0)=f(1)=0,试证01f2(x)dx1801[f(x)]2dx\int_0^1f^2(x)dx\leq\frac 1 8\int_0^1[f'(x)]^2dx

证明:将区间拆分为[0,1/2],(1/2,1][0,1/2],(1/2,1] 两边分别放缩:

f(x)=0xf(t)dt,  x[0,12]f(x)=x1f(t)dt,  x(12,1]012f2(x)dx012[f(t)]2dt012x  dx=18012[f(t)]2dt121f2(x)dx121[f(t)]2dt121(1x)  dx=18012[f(t)]2dt01f2(x)dx1801[f(t)]2dt\begin{aligned}f(x)&=\int_0^xf'(t)dt,\;x\in[0,\frac1 2]\\f(x)&=\int_x^1f'(t)dt,\;x\in(\frac 1 2,1]\\\Rightarrow\int_0^{\frac 1 2}f^2(x)dx&\leq\int_0^{\frac 1 2}[f'(t)]^2dt\int_0^{\frac 1 2}x\;dx=\frac 1 8\int_0^{\frac 1 2}[f'(t)]^2dt\\\Rightarrow\int_{\frac 1 2}^1f^2(x)dx&\leq\int_{\frac 1 2}^1[f'(t)]^2dt\int_{\frac 1 2}^1(1-x)\;dx=\frac 1 8\int_0^{\frac 1 2}[f'(t)]^2dt\\\\\Rightarrow\int_0^1f^2(x)dx&\leq\frac 1 8\int_0^1[f'(t)]^2dt\end{aligned}

闵可夫斯基不等式| Minkowski

※利用切比雪夫不等式直接证明积分不等式

切比雪夫不等式| Chebyshev

证明张宇1000题,变下限积分
https://zhuanlan.zhihu.com/p/437399536

※利用无穷级数证明不等式
例题收件箱

问题f(x)=xx+1sin(et)dt\begin{aligned}f(x)=\int_x^{x+1}\sin(e^t)\mathrm dt\end{aligned},求证f(x)2ex\begin{aligned}|f(x)|\leq\frac{2}{e^x}\end{aligned}

答案

f(x)=xx+1etsin(et)etdt=xx+1etd(cos(et))=[etcos(et)]xx+1+xx+1cos(et)etdt=cos(ex+1)ex+1cos(ex)ex+xx+1cos(et)etdtcos(ex+1)ex+1+cos(ex)ex+xx+1cos(et)etdt1ex+1+1ex+xx+1cos(et)etdt1ex+1+1ex+xx+11etdt=2ex\begin{aligned}|f(x)|&=\bigg|\int_x^{x+1}\frac{e^t\sin(e^t)}{e^t}\mathrm dt\bigg|=\bigg|\int_x^{x+1}e^{-t}\mathrm d\left(\cos(e^t)\right)\bigg|\\\\&=\bigg|\left[e^{-t}\cos(e^t)\right]_x^{x+1}+\int_x^{x+1}\cos(e^t)e^{-t}\mathrm dt\bigg|\\\\&=\bigg|\frac{\cos(e^{x+1})}{e^{x+1}}-\frac{\cos(e^x)}{e^x}+\int_x^{x+1}\frac{\cos(e^t)}{e^t}\mathrm dt\bigg|\\\\&\leq\bigg|\frac{\cos(e^{x+1})}{e^{x+1}}\bigg|+\bigg|\frac{\cos(e^x)}{e^x}\bigg|+\bigg|\int_x^{x+1}\frac{\cos(e^t)}{e^t}\mathrm dt\bigg|\\\\&\leq\frac{1}{e^{x+1}}+\frac{1}{e^{x}}+\int_x^{x+1}\big|\frac{\cos(e^t)}{e^t}\big|\mathrm dt\\\\&\leq\frac{1}{e^{x+1}}+\frac{1}{e^{x}}+\int_x^{x+1}\big|\frac{1}{e^t}\big|\mathrm dt\\\\&=\frac{2}{e^x}\end{aligned}

积分概念及其相关性质

积分的定义与性质

一元函数y=f(x)y=f(x)[a,b][a,b] 上的定积分:

I=abf(x)dx=limλ0i=1nf(ξi)ΔxiI=\int_a^bf(x)\mathrm dx=\lim_{\lambda\to0}\sum_{i=1}^nf(\xi_i)\Delta x_i

其中,λ=max1inΔxi\lambda=\max\limits_{1\leq i\leq n}\Delta x_iΔxi\Delta x_i 是将区间[a,b][a,b] 任意分割为nn 个小区间后的第ii 个区间,同时亦以Δxi\Delta x_i 表示该小区间的长度。

显然,定积分要求积分区间有限且被积函数有界。

定积分的相关性质

比较定理

f(x)g(x)abf(x)dxabg(x)dx\begin{aligned}f(x)\leq g(x)\Rightarrow \int_a^b f(x)dx\leq\int_a^bg(x)dx\end{aligned}
主要推论:abf(x)dxabf(x)dx\begin{aligned}\bigg|\int_a^bf(x)dx\bigg|\leq\int_a^b\big|f(x)\big|dx\end{aligned} 其中要求aba\leq b.

✨若在[a,b][a,b]任意子区间[α,β][\alpha,\beta]总有αβf(x)dx=0\int_{\alpha}^{\beta}f(x)dx=0,则当x[a,b]x\in[a,b] 时,f(x)=0f(x)=0.

原函数存在定理

f(x)f(x)[a,b][a,b] 上连续,则变上限积分函数Φ(x)=axf(t)dt\begin{aligned}\Phi(x)=\int_a^xf(t)dt\end{aligned}f(x)f(x)[a,b][a,b] 上的一个原函数。将下限aa 换成任意一个定值c[a,b]c\in[a,b] 得到的积分函数仍然是一个原函数。

f(x)f(x)[a,b][a,b] 上有第一类间断点,则f(x)f(x)[a,b][a,b] 上不存在原函数。(这与在导函数专栏中我们讨论的导函数间断点问题相呼应)

🦄 存在有原函数不是初等函数,或者无法用初等函数表示的情况,下面这些最典型的例子应适当记忆:

ex2dx,  sinxxdx,  cosxxdx,  sin(x2)dx,  cos(x2)dx,  1lnxdx\int e^{-x^2}dx,\;\int\frac{\sin x}{x}dx,\;\int\frac{\cos x}{x}dx,\;\int\sin(x^2)dx,\;\int\cos(x^2)dx,\;\int\frac 1 {\ln x}dx

牛顿-莱布尼茨公式

f(x)f(x)[a,b][a,b] 上连续,F(x)F(x)f(x)f(x)[a,b][a,b] 上的原函数,则:

abf(x)  dx=F(x)ab=F(b)F(a)\int_a^bf(x)\;\mathrm dx=F(x)\bigg|_a^b=F(b)-F(a)

可以证明得出推论:

  1. F(x)F(x) 只在开区间(a,b)(a,b) 连续,则原定积分=F(b0)F(a+0)=F(b-0)-F(a+0)
  2. F(x)F(x)[a,b][a,b] 除去某个点c(a,b)c\in(a,b) 连续,则原定积分=[F(b)F(c+0)]+[F(c0)F(a)]=[F(b)-F(c+0)]+[F(c-0)-F(a)]

周期函数的积分

TT 为周期的周期函数f(x)f(x) 若在[0,T][0,T] 上可积,则:

  1. aa+Tf(x)dx=0Tf(x)dx\begin{aligned}\int_a^{a+T}f(x)dx=\int_0^Tf(x)dx\end{aligned}
  2. 函数F(x)=0xf(t)dtF(x)=\int_0^xf(t)dt 也是以TT 为周期的周期函数0Tf(t)dt=0\Leftrightarrow \int_0^Tf(t)dt=0
  3. f(x)f(x)RR 上连续,且0+f(x)dx\int_0^{+\infty}f(x)dx 收敛,则F(x)=0xf(t)dtF(x)=\int_0^xf(t)dt 也是以TT 为周期的周期函数。

关于 (3) 的证明:

limx0xf(t)dtlimn0nTf(t)dtlimxn0Tf(t)dt0Tf(t)dt=0\exists\lim_{x\to\infty}\int_0^xf(t)dt\Leftrightarrow\exists\lim_{n\to\infty}\int_0^{nT}f(t)dt\Leftrightarrow\exists\lim_{x\to\infty}n\int_0^Tf(t)dt\Leftrightarrow\int_0^Tf(t)dt=0

积分与求和的统一

积分与求和的统一

积分定义求极限

I=abf(x)dx=limλ0i=1nf(ξi)Δxi\begin{aligned}I=\int_a^bf(x)\mathrm dx=\lim_{\lambda\to0}\sum_{i=1}^nf(\xi_i)\Delta x_i\end{aligned} 中的ξi\xi_i 利用均匀选取的方法确定。从而可以得到如下情形:

abf(x)dx=limni=1nf(a+i(ba)n)ban\int_a^bf(x)\mathrm dx=\lim_{n\to\infty}\sum_{i=1}^nf\left(a+\frac{i(b-a)}{n}\right)\frac{b-a}{n}

更加特殊地,对上式取a=0,b=1a=0,b=1,我们就有:

limn1ni=1nf(in)=01f(x)dx\lim_{n\to\infty}\frac 1 n\sum_{i=1}^nf\left(\frac i n\right)=\int_0^1f(x)\mathrm dx

这是考研数学中常见的极限形式。遇见这类题型时,如果能成功化简为上述形式,且定积分好求时,极限问题就迎刃而解了。

形式统一比大小

事实上,积分就是求和,求和就是积分,而积分对应连续量、求和对应离散量。二者既是对立的又是统一的,并且在一定条件下可以相互转化。
当我们在求解问题时,遇见既有积分项又有求和项的问题时,通常我们需要将其化为统一的形式

例:设f(x)f(x)[0,1][0,1] 连续,x,yf(x)f(y)Mxy\forall x,y\quad |f(x)-f(y)|\leq M|x-y|
试证01f(x)dx1nk=1nf(kn)M2n\begin{aligned}\bigg|\int_0^1f(x)dx-\frac 1 n\sum_{k=1}^nf(\frac k n)\bigg|\leq\frac M {2n}\end{aligned}

【分析】为了完成积分与求和的统一,我们利用积分的可加性将积分项进行求和化:

01f(x)dx=k=1nk1nknf(x)dx\int_0^1f(x)dx=\sum_{k=1}^n\int_{\frac {k-1} n}^{\frac k n}f(x)dx

当然,求和符号的上下限应该与求和项保持一致。另一方面,我们再将求和项的通项积分化:

1nf(kn)=k1nknf(kn)dx\frac 1 nf(\frac k n)=\int_{\frac {k-1} n}^{\frac k n}f(\frac k n)dx

于是,待证不等式的左式(Left Hand Side)为:

LHS=k=1nk1nkn(f(x)f(kn))dxk=1nk1nkn(f(x)f(kn))dxk=1nk1nknf(x)f(kn)dxMk=1nk1nknknxdx=RHSQ.E.D\begin{aligned}LHS&=\bigg|\sum_{k=1}^n\int_{\frac {k-1} n}^{\frac k n}\left(f(x)-f(\frac k n)\right)dx\bigg|\leq\sum_{k=1}^n\bigg|\int_{\frac {k-1} n}^{\frac k n}\left(f(x)-f(\frac k n)\right)dx\bigg|\\&\leq\sum_{k=1}^n\int_{\frac {k-1} n}^{\frac k n}\bigg|f(x)-f(\frac k n)\bigg|dx\leq M\sum_{k=1}^n\int_{\frac {k-1} n}^{\frac k n}\bigg|\frac k n-x\bigg|dx\\&=RHS\\\\Q.E.D\end{aligned}

注:本题所给的已知条件实则被称之为 Lipschiz 条件。

对于一个函数 f(t,x)f(t,x) 如果对于点 (t0,x0)(t_0,x_0) 的某一邻域内的任意两点 (t,x)(t,x) 和 (t,y)(t,y) 满足不等式

f(t,x)f(t,y)Lxy||f(t,x)−f(t,y)||≤L||x−y||

这里 LL 是一个常数。则称这个不等式是利普希茨条件(Lipschitz condition),LL 是利普希茨常数(Lipschitz constant), f(t,x)f(t,x) 是关于 xx 的利普希茨函数。

对于积分求和的统一相关的不等式,不仅可以用于不等式的证明,还可以直接利用于和求和项有关的极限求解上(借助夹逼准则)。
事实上,本文在夹逼准则专栏给处的各类常用不等式中,也直接把这一类的不等式关系列举了出来,考生应当熟悉掌握。

重要题源

例:设f(x)f(x)[1,+)[1,+\infty) 单调递减,f(x)>0f(x)\gt0 且连续,an=k=1nf(k)1nf(x)dxa_n=\sum_{k=1}^nf(k)-\int_1^nf(x)dx
试证明,数列{an}\{a_n\} 收敛.(1999年真题)

【方法一】证明数列收敛,常规方法是通过证明数列单调有界。而单调有界的实质就是不等式,再结合通项的形式是求和项与积分项同时出现的情况,我们考虑进行拆分。

an=k=1nf(k)1nf(x)dx=k=1n1f(k)1nf(x)dx+f(n)=k=1n1kk+1f(k)k=1n1kk+1f(x)dx+f(n)=k=1n1kk+1[f(k)f(x)]dx    +f(n)>0an+1an=f(n+1)nn+1f(x)dx=f(n+1)f(ξ)0\begin{aligned}a_n&=\sum_{k=1}^nf(k)-\int_1^nf(x)dx=\sum_{k=1}^{n-1}f(k)-\int_1^nf(x)dx+f(n)\\&=\sum_{k=1}^{n-1}\int_k^{k+1}f(k)-\sum_{k=1}^{n-1}\int_k^{k+1}f(x)dx+f(n)\\&=\sum_{k=1}^{n-1}\int_k^{k+1}[f(k)-f(x)]dx\;\;+f(n)\gt0\\\\a_{n+1}-a_n&=f(n+1)-\int_n^{n+1}f(x)dx=f(n+1)-f(\xi)\leq0\end{aligned}

从而{an}\{a_n\} 单调递减有下界,由此数列收敛。

【方法二】利用数列收敛与级数收敛的关系,有:

{an} 收敛 limnan(a1+limnan)n=1(an+1an)n=1(an+1an) 收敛 \{a_n\}\text{ 收敛 }\Leftrightarrow\exists\lim_{n\to\infty}a_n\Leftrightarrow\exists\left(-a_1+\lim_{n\to\infty}a_n\right)\Leftrightarrow\exists\sum_{n=1}^\infty\left(a_{n+1}-a_n\right)\Leftrightarrow\sum_{n=1}^\infty\left(a_{n+1}-a_n\right)\text{ 收敛 }

从而考虑下面的一般项:

un=an+1an=f(n+1)nn+1f(x)dx=lagrangef(n+1)f(ξ)f(n)f(n+1)\begin{aligned}u_n=|a_{n+1}-a_n|&=\bigg|f(n+1)-\int_n^{n+1}f(x)dx\bigg|\xlongequal{lagrange}\big|f(n+1)-f(\xi)\big|\\&\leq f(n)-f(n+1)\end{aligned}

其中,n<ξ<n+1n\lt\xi\lt n+1. 下面再来考虑无穷级数

n=1[f(n)f(n+1)]=f(1)limnf(n)\sum_{n=1}^\infty[f(n)-f(n+1)]=f(1)-\lim_{n\to\infty}f(n)

显然是收敛的,于是根据比较判别法,一般项为unu_n 的无穷级数也收敛,进而原数列收敛.

可积性问题与变限积分

函数在区间上的可积性

必要条件f(x)f(x)[a,b][a,b] 上可积,则f(x)f(x)[a,b][a,b] 上有界

充分条件f(x)f(x)[a,b][a,b] 上满足下列条件中的一种,则f(x)f(x)[a,b][a,b] 上可积。

  1. f(x)f(x)[a,b][a,b] 上连续;
  2. f(x)f(x)[a,b][a,b] 上有界且只有有限个间断点
  3. f(x)f(x)[a,b][a,b] 上单调.

其他结论F(x)=0xf(t)dtF(x)=\int_0^xf(t)dt,则有:

  1. f(x)f(x) 可积,则F(x)F(x) 连续;
  2. f(x)f(x) 连续,则F(x)F(x) 可导;
  3. f(x)f(x) 仅有有限个可去间断点,其余点均连续 ,则F(x)F(x) 仍然可导;
  4. f(x)f(x) 存在跳跃间断点x=x0x=x_0,则F(x)F(x) 不可导,因为在x=x0x=x_0 处左右导数不相等;

关于变限积分的周期性和牛顿莱布尼茨公式等相关性质见上一节。

反常积分的敛散性判别

反常积分的敛散性

常见的广义积分

1.a+1xp  dx{is convergent, p>1is divergent, p1,a>02.ab1(xa)p  dx  ;ab1(bx)p  dx{is convergent, p<1is divergent, p13.011xplnqx  dx{is convergent, p<1  or  (p=1,q>1)is divergent, p>1  or  (p=1,q1)4.1+1xplnqx  dx{is convergent, p>1  or  (p=1,q>1)is divergent, p<1  or  (p=1,q1)\begin{aligned}1.&\int_a^{+\infty}\frac 1 {x^p}\;dx\begin{cases}\text{is convergent, }p\gt1\\\text{is divergent, }p\leq1\\\end{cases},a\gt0\\\\2.&\int_a^b\frac 1 {(x-a)^p}\;dx\;;\int_a^b\frac 1 {(b-x)^p}\;dx\begin{cases}\text{is convergent, }p\lt1\\\text{is divergent, }p\geq1\\\end{cases}\\\\3.&\int_0^1\frac 1 {x^p\ln^qx}\;dx\begin{cases}\text{is convergent, }p\lt1\;or\;(p=1,q\gt1)\\\text{is divergent, }p\gt1\;or\;(p=1,q\leq1)\\\end{cases}\\\\4.&\int_1^{+\infty}\frac 1 {x^p\ln^qx}\;dx\begin{cases}\text{is convergent, }p\gt1\;or\;(p=1,q\gt1)\\\text{is divergent, }p\lt1\;or\;(p=1,q\leq1)\\\end{cases}\\\\\end{aligned}

比较判别法

f(x),g(x)f(x),g(x)[a,+)[a,+\infty) 连续,且g(x)f(x)0  (ax+)g(x) \geq f(x) \geq 0\;(a\leq x\leq +\infty),则:

  1. a+g(x)dx\begin{aligned}\int_{a}^{+\infty}g(x)dx\end{aligned} 收敛,则a+f(x)dx\begin{aligned}\int_{a}^{+\infty}f(x)dx\end{aligned} 收敛;
  2. a+f(x)dx\begin{aligned}\int_{a}^{+\infty}f(x)dx\end{aligned} 发散,则a+g(x)dx\begin{aligned}\int_{a}^{+\infty}g(x)dx\end{aligned} 发散.

由上述比较原理,可衍生出其极限形式

  • f(x),g(x)f(x),g(x)[a,+)[a,+\infty) 连续且非负,若limx+f(x)g(x)=A\begin{aligned}\lim_{x\to +\infty}\frac{f(x)}{g(x)}=A\end{aligned},则有:
    • 0<A<+0 \lt A \lt +\infty 时,即f(x)g(x)  (x+)f(x)\sim g(x)\;(x\to +\infty) 时,a+f(x)dx,a+g(x)dx\begin{aligned}\int_{a}^{+\infty}f(x)dx\end{aligned},\begin{aligned}\int_{a}^{+\infty}g(x)dx\end{aligned} 同敛散;
    • A=0A=0 时,若a+g(x)dx\begin{aligned}\int_{a}^{+\infty}g(x)dx\end{aligned} 收敛,则a+f(x)dx\begin{aligned}\int_{a}^{+\infty}f(x)dx\end{aligned} 收敛;
    • A=A=\infty 时,若a+g(x)dx\begin{aligned}\int_{a}^{+\infty}g(x)dx\end{aligned} 发散,则a+f(x)dx\begin{aligned}\int_{a}^{+\infty}f(x)dx\end{aligned} 发散.

对于瑕积分,也有类似的结论。

  • f(x)f(x)(a,b](a,b] 连续,取g(x)=1/(xa)pg(x)=1/(x-a)^p,若limxa+f(x)g(x)=A\begin{aligned}\lim_{x\to a^+}\frac{f(x)}{g(x)}=A\end{aligned},则有:
    • 0<A<+0 \lt A \lt +\infty 时,即f(x)g(x)  (xa+)f(x)\sim g(x)\;(x\to a^+) 时,abf(x)dx,abg(x)dx\begin{aligned}\int_{a}^{b}f(x)dx\end{aligned},\begin{aligned}\int_{a}^{b}g(x)dx\end{aligned} 同敛散;
    • A=0,p<1A=0,p\lt1 时,则abf(x)dx\begin{aligned}\int_{a}^{b}f(x)dx\end{aligned} 收敛;
    • A=,p1A=\infty,p\geq1 时,则abf(x)dx\begin{aligned}\int_{a}^{b}f(x)dx\end{aligned} 发散.

其中,我们常常通过“等价无穷小/大”的方法(根本原理是其极限形式的第一种情况)将一个较为复杂的反常积分的敛散性等同于较为简单的反常积分的敛散性。
一般情况下,我们更愿意把后者化为常见的反常积分形式,从而直接判断。

解题步骤与例题

  1. 找到可能的瑕点以作拆分(在积分区间内被积函数没有定义的点,包括无穷远点)
  2. 找到等价量利用判别法进行同敛散判断(通常是等价无穷小量和等价无穷大量)
  3. 难以找到同敛散等价量时,考虑构造合适的g(x)g(x) 利用瑕积分的判别结论

【例题1. 2010年真题】设m,nm,n 均为正整数,则反常积分01ln2(1x)mxn  dx\begin{aligned}\int_0^1\frac{\sqrt[m]{\ln^2(1-x)}}{\sqrt[n]{x}}\;dx\end{aligned} 的收敛性是否与m,nm,n 有关,如果是请给出满足收敛的条件。

【分析】找到瑕点x=0,x=1x=0,x=1. 于是将原积分拆为两个积分,分别进行判断。

  1. 对于区间(0,a]  ,0<a<1(0,a]\;,0\lt a\lt1 有:ln2(1x)mxn1x1n2m\begin{aligned}\frac{\sqrt[m]{\ln^2(1-x)}}{\sqrt[n]{x}}\sim\frac 1 {x^{\frac 1 n-\frac 2 m}}\end{aligned}. 由于m,nm,n 均为正整数,所以1n2m<1\frac 1 n-\frac 2 m\lt1 恒成立,由pp 积分的结论可知,此部分收敛;
  2. 对于区间(a,1)(a,1),令g(x)=1/1xg(x)=1/\sqrt{1-x} ,则有:

limx1ln2(1x)mxn/11x=0\lim_{x\to1^-}\frac{\sqrt[m]{\ln^2(1-x)}}{\sqrt[n]{x}}\bigg/\frac{1}{\sqrt{1-x}}=0

g(x)g(x)(a,1)(a,1) 的积分收敛。

综上所述,原反常积分收敛,与m,nm,n 的取值无关。

不定积分计算手册

基本积分表与常见恒等式

《积分公式大全》网络版本科学空间-By BoJone

1.  arctanx+arctan(1x)={π2,x>0π2,x<02.  11+ex+11+ex=13.  a3+b3=(a+b)(a2ab+b2)4.  a3b3=(ab)(a2+ab+b2)5.  a4+b4=(a22ab+b2)(a2+2ab+b2)\begin{aligned}1.\;&\arctan x+\arctan (\frac 1 x)=\begin{cases}\frac \pi 2&,x\gt0\\-\frac \pi 2&,x\lt0\end{cases}\\2.\;&\frac 1 {1+e^x}+\frac 1 {1+e^{-x}}=1\\\\3.\;&a^3+b^3=(a+b)(a^2-ab+b^2)\\4.\;&a^3-b^3=(a-b)(a^2+ab+b^2)\\5.\;&a^4+b^4=(a^2-\sqrt 2ab+b^2)(a^2+\sqrt 2ab+b^2)\end{aligned}

特别注意,对于指数函数与三角函数乘积的积分,有如下公式:

ekxsinax  dx=(ekx)(sinax)ekxsinaxa2+k2+Cekxcosax  dx=(ekx)(cosax)ekxcosaxa2+k2+C\begin{aligned}\int e^{kx}\sin ax\;dx&=\frac{\begin{vmatrix}(e^{kx})'&(\sin ax)'\\e^{kx}&\sin ax\end{vmatrix}}{a^2+k^2}+C\\\\\int e^{kx}\cos ax\;dx&=\frac{\begin{vmatrix}(e^{kx})'&(\cos ax)'\\e^{kx}&\cos ax\end{vmatrix}}{a^2+k^2}+C\end{aligned}

拓展:分部积分表格法

分部积分表格法

例题(x3+2x+3)exdx\int(x^3+2x+3)e^x\text dx

步骤

  1. 将被积函数以乘号进行分割 并 绘制两行表格
  2. 将可以连续多次积分的一项放入第二行,可以连续求导最终求导为0的一项放入第一行
  3. 依次对上下两式求导和求积,如图所示,直到上一项导数为0
  4. 将表格从左到右交叉相乘,系数交错取++、-
  5. 最终合并,得到结果(别忘了+C+C

示例1

示例2

于是得到结果:

I=(x3+2x+3)ex(3x2+2)ex+6xex6ex+C=(x33x2+8x5)ex+C\begin{aligned}I&=(x^3+2x+3)e^x-(3x^2+2)e^x+6xe^x-6e^x+C\\&=(x^3-3x^2+8x-5)e^x+C\end{aligned}

证明和推导

ForI=uv(n+1)dx, There is I=ud(v(n))=uv(n)v(n)vdxletI=v(n)udx, thenI=ud(v(n1))=uv(n1)v(n1)udxI=uv(n)uv(n1)+\begin{aligned}\text{For} \quad I&=\int uv^{(n+1)}\text dx,\text{ There is }\\I&=\int u\text d(v^{(n)})=uv^{(n)}-\int v^{(n)}v'\text dx\\\text{let}\quad I'&=\int v^{(n)}u'\text dx,\text{ then}\\I'&=\int u'\text d(v^{(n-1)})=u'v^{(n-1)}-\int v^{(n-1)}u''\text dx\\I&=uv^{(n)}-u'v^{(n-1)}+\dots\end{aligned}

于是,可总结出如下表格:

uuuu'\dotsu(n)u^{(n)}\dots
v(n+1)v^{(n+1)}v(n)v^{(n)}\dotsvv\dots

参考分部积分如何避免出错–表格法

有理函数积分

有理函数不定积分

对于有理函数积分,通常我们可以将其划归为求解真分式积分,而真分式又可以通过分式分解化为有限个简单的有理函数积分
因此,我们的基本目标是对裂项方法、简单积分求解进行处理。

最后,我们再来讨论一些通过凑微分、换元的其他类方法。

  • 利用立方差公式进行变形化简
  • 利用部分分式分解进行拆项化简
  • 利用完全平方公式进行配方化简
  • 利用上下同除的恒等变换凑微分
  • 基于三角函数恒等式进行三角换元

部分分式分解

对于分式有理函数R(x)=Pn(x)Qm(x)(n<m)R(x)=\frac{P_n(x)}{Q_m(x)}\quad(n<m) 如果分母Qm(x)=0Q_m(x)=0 有根,则可将其进行分解。根据根的不同而不同。

当根均为单实根时,有Qm(x)=(xx1)(xx2)(xxm)Q_m(x)=(x-x_1)(x-x_2)·\cdots·(x-x_m),此时:

R(x)=A1xx1+A2xx2++AmxxmR(x)=\frac{A_1}{x-x_1}+\frac{A_2}{x-x_2}+\cdots+\frac{A_m}{x-x_m}

当根存在重实根时,有Qm(x)=(xx1)r(xx2)(xxk)Q_m(x)=(x-x_1)^r(x-x_2)·\cdots·(x-x_k),此时:

R(x)=A1xx1+A2(xx1)2++Ar(xx1)r+R(x)=\frac{A_1}{x-x_1}+\frac{A_2}{(x-x_1)^2}+\cdots+\frac{A_r}{(x-x_1)^r}+\cdots

当根存在共轭复根时,将此项写作(x+px+q)r(x+px+q)^r,则有:

R(x)=A1x+B1x+px+q+A2x+B2(x+px+q)2++Arx+Br(x+px+q)r+R(x)=\frac{A_1x+B_1}{x+px+q}+\frac{A_2x+B_2}{(x+px+q)^2}+\cdots+\frac{A_rx+B_r}{(x+px+q)^r}+\cdots


对系数的求解有多种方法,其中最简单的即是将拆分后的结果乘回去,利用待定系数法定出系数,这种方法我们不再阐述。下面引入复变函数中常用的留数法定系数。

  • 单根时,我们有Ai=Res[R(x),xi]=limxxi(xxi)R(x)=Pn(xi)Qm(xi)\begin{aligned}A_i=Res[R(x),x_i]=\lim_{x\to x_i}(x-x_i)R(x)=\frac{P_n(x_i)}{Q'_m(x_i)}\end{aligned},其中AiA_i 是所有分解项中,第ii 项的分子系数。
  • r重根 时,我们有

Ak=Res[(xx1)k1R(x),x1]=1(rk)!limxx1[(xxi)rR(x)](rk)\begin{aligned}A_k=Res[(x-x_1)^{k-1}R(x),x_1]=\frac1{(r-k)!}\lim_{x\to x_1}[(x-x_i)^rR(x)]^{(r-k)}\end{aligned}

其中,AkA_k 是重根x1x_1 的分解项中第kk 项的分子系数。

注:上面使用留数法时,并未限定必须是实根,即当根为复数时留数法规则不变,同样适用。我们只需求解之后再通分化为实数形式即可。

简单有理数积分

  1. A(xa)m  dx\begin{aligned}\int\frac{A}{(x-a)^m}\;\mathrm dx\end{aligned} 型(略)
  2. Ax+B(x2+px+q)m  dx(Δ=p24q<0)\begin{aligned}\int\frac{Ax+B}{(x^2+px+q)^m}\;\mathrm dx\quad(\Delta=p^2-4q\lt0)\end{aligned}

对于这类积分,我们通常的处理办法是:改造分子使其以分母中的二次函数的导函数形式现身,进而将积分拆分,最后逐个击破。

I=A2d(x2+px+q)(x2+px+q)m+(BAp2)dx(x2+px+q)mI=\frac A 2\int\frac{\mathrm d(x^2+px+q)}{(x^2+px+q)^m}+(B-\frac{Ap}{2})\int\frac{\mathrm dx}{(x^2+px+q)^m}

对分母进行配方:

(x2+px+q)m=[(x+p2)2+(qp24)2]m(x^2+px+q)^m=\left[(x+\frac p2)^2+\left(\sqrt{q-\frac{p^2}{4}}\right)^2\right]^m

从而,拆分之后的第一项积分可以直接利用第一类简单积分求得,第二项可以通过三角换元求得。

根据分母改造分子型

问题1+x41+x6  dx\begin{aligned}\int\frac{1+x^4}{1+x^6}\;\mathrm dx\end{aligned}

【分析】利用立方和公式将分母分解,然后根据分母凑分子:

I=(1+x2+x4)x2(1+x2)(1+x2+x4)=dx1+x2x21+x6dx=I1I2I=\int\frac {(1+x^2+x^4)-x^2}{(1+x^2)(1+x^2+x^4)}=\int\frac {dx}{1+x^2}-\int\frac{x^2}{1+x^6}dx=I_1-I_2

值得注意的是,通过凑分子拆开之后,需要把I2I_2 的分母再还原回最初的模样,此时即可发现I2I_2 可通过凑微分直接解决。

问题1x(x3+27)  dx\begin{aligned}\int\frac{1}{x(x^3+27)}\;\mathrm dx\end{aligned}.

【分析】本题分母极具迷惑性,很容易想到利用立方和公式对分母分解,最后利用常规方法裂项定系数求解。但事实上本题可以上下同时乘x2x^2d(x3)\mathrm d(x^3) 降次。

问题1x2n(1+x2)  dx\begin{aligned}\int\frac{1}{x^{2n}(1+x^2)}\;\mathrm dx\end{aligned}.

【分析】根据分母将分子凑出(1+x2)x2(1+x^2)-x^2,于是:

In=1x2ndx1x2n2(1+x2)dx=1x2ndxIn1I_n=\int\frac 1{x^{2n}}dx-\int\frac1{x^{2n-2}(1+x^2)}dx=\int\frac 1{x^{2n}}dx-I_{n-1}

同➗法凑微分型

【例题1】1+x21+x4  dx\begin{aligned}\int\frac{1+x^2}{1+x^4}\;\mathrm dx\end{aligned}

分析:求解该问题当然可以用有理函数通用解法利用四次方和公式进行拆项。但是本题还有一种更加巧妙的凑微分方法。即上下同时➗x2x^2

I=1x2+11x2+x2dx=d(x1x)(x1x)2+2I=\int\frac{\frac{1}{x^2}+1}{\frac{1}{x^2}+x^2}dx=\int\frac{d\left(x-\frac 1 x\right)}{(x-\frac 1 x)^2+2}

然而由于同时➗,所以导致原本在x=0x=0 处有定义的被积函数的积分(原函数)变得没有定义了。因此不能将结果直接得出。我们应该补充定义,使得F(x)F(x)x=0x=0 处连续!

F(x)={12arctan[12(x1x)]π22,x<00,x=012arctan[12(x1x)]+π22,x>0I=F(x)+C\begin{aligned}F(x)&=\begin{cases}\frac 1 {\sqrt{2}}\arctan[\frac 1{\sqrt2}(x-\frac{1}{x})]-\frac \pi{2\sqrt 2}&,x\lt0\\0&,x=0\\\frac 1 {\sqrt{2}}\arctan[\frac 1{\sqrt2}(x-\frac{1}{x})]+\frac \pi{2\sqrt 2}&,x\gt0\\\end{cases}\\\\I&=F(x)+C\end{aligned}

【例题2】1x21+x4  dx\begin{aligned}\int\frac{1-x^2}{1+x^4}\;\mathrm dx\end{aligned} . 事实上,求解方法如法炮制,一模一样!(本题可以不用补充定义)

事实上,根据”例题1“和”例题2“,我们可以更进一步推广得到对于形如这样的任意四次函数积分:1±x21+px2+x4  dx\begin{aligned}\int\frac{1\pm x^2}{1+px^2+x^4}\;\mathrm dx\end{aligned} 都能同时除以x2x^2 处理。

【例题3】11+x4  dx\begin{aligned}\int\frac{1}{1+x^4}\;\mathrm dx\end{aligned}. 由”例题1“和”例题2“可得:

I=11+x4dx=121+x21+x4dx+121x21+x4dxI=\int\frac{1}{1+x^4}\mathrm dx=\frac 1 2\int\frac{1+x^2}{1+x^4}\mathrm dx+\frac 1 2\int\frac{1-x^2}{1+x^4}\mathrm dx

可以更进一步推广得到对于11+px2+x4  dx\begin{aligned}\int\frac{1}{1+px^2+x^4}\;\mathrm dx\end{aligned} 都能这样处理。

【例题4】11+x6  dx\begin{aligned}\int\frac{1}{1+x^6}\;\mathrm dx\end{aligned}

利用立方和公式将分母分解,然后根据分母凑分子:

I=(1+x2)x2(1+x2)(1+x2+x4)=dx1+x2+x4x21+x6dx=I1I2I=\int\frac {(1+x^2)-x^2}{(1+x^2)(1+x^2+x^4)}=\int\frac {dx}{1+x^2+x^4}-\int\frac{x^2}{1+x^6}dx=I_1-I_2

其中I1I_1 就是”例题3“的推广,I2I_2 我们仿造上一节的分解并还原的思路,把分母还原回去之后,又可以凑微分d(x3)\mathrm d(x^3) . 最后本题迎刃而解!

【例题6】1tanx  dx\begin{aligned}\int\frac{1}{\sqrt{\tan x}}\;\mathrm dx\end{aligned}tanx  dx\begin{aligned}\int\sqrt{\tan x}\;\mathrm dx\end{aligned}. (换元法)

奥斯特罗格拉茨基方法

三角有理函数积分

三角有理函数积分

定义以R(u,v)R(u,v) 表示将函数u,vu,v 经过有限次的四则运算得到的二元函数。我们称R(sinx,cosx)R(\sin x,\cos x) 为三角有理函数。

理论上,任意三角有理函数都能通过 万能公式 将其化为有理函数,从而规避三角函数问题,直接求有理函数积分即可。
当然,并非转化之后的有理函数积分就易于求解。因此,我们还应该具体问题具体分析,深入挖掘每一类题巧妙的解法!

万能公式换元法

tan(x/2)=t\tan(x/2)=t ,得:

x=2arctant,  dx=21+t2dtThere are :sinx=2t1+t2,  sinx=1t21+t2\begin{aligned}x&=2\arctan t,\;\mathrm dx=\frac2{1+t^2}\mathrm dt\\\\&\text{There are }:\sin x=\frac{2t}{1+t^2},\;\sin x=\frac{1-t^2}{1+t^2}\end{aligned}


问题11+sinx+cosx  dx\begin{aligned}\int\frac{1}{1+\sin x+\cos x}\;\mathrm dx\end{aligned}. (答案:ln1+tanx2+C\begin{aligned}\ln\big|1+\tan\frac x 2\big|+C\end{aligned}

注:当被积函数中的三角函数项的次数太高时,不建议使用万能公式求解,因为转换后得到的有理函数次数会更高,难以求解!

化简分母法

  • 分母为(1±cosx)(1\pm\cos x)
    • 通过同时乘以共轭表达式进行化简.
    • 倍角公式1+cosx=2cos2x21+\cos x=2\cos^2\frac x 2 缩分母.
  • 分母为(sinx±cosx)(\sin x\pm\cos x)
    • 利用辅助角公式sinx±cosx=2sin(x±π/2)\sin x\pm\cos x=\sqrt2\sin(x\pm \pi/2) .
    • 同时乘以共轭表达式进行化简.

同乘共轭表达式举例:11+cosx  dx=1cosx(1+cosx)(1cosx)  dx\begin{aligned}\int\frac{1}{1+\cos x}\;\mathrm dx=\int\frac{1-\cos x}{(1+\cos x)(1-\cos x)}\;\mathrm dx\end{aligned}.

奇偶性凑微分

  • R(sinx,cosx)=R(sinx,cosx)R(\sin x,-\cos x)=-R(\sin x,\cos x),即三角有理函数R(sinx,cosx)R(\sin x,\cos x) 关于cosx\cos x奇函数,则应该想办法用cosxdx\cos x\mathrm dxdsinxd\sin x.
  • R(sinx,cosx)=R(sinx,cosx)R(-\sin x,\cos x)=-R(\sin x,\cos x),即三角有理函数R(sinx,cosx)R(\sin x,\cos x) 关于sinx\sin x奇函数,则应该想办法用sinxdx\sin x\mathrm dxdcosxd\cos x.
  • R(sinx,cosx)=R(sinx,cosx)R(-\sin x,-\cos x)=R(\sin x,\cos x),即三角有理函数R(sinx,cosx)R(\sin x,\cos x) 关于(sinx,cosx)(\sin x,\cos x) 对称,则应该想办法用sec2xdx\sec^2x\mathrm dxdtanxd\tan x.

问题1sin2xcosx  dx\begin{aligned}\int\frac{1}{\sin^2x\cos x}\;\mathrm dx\end{aligned}.

【分析】验证得出被积函数满足关于 cos(x) 为奇函数,从而为了凑微分,上下同时乘以一个 cos(x) .

问题5+4cosx(2+cosx)2sinx  dx\begin{aligned}\int\frac{5+4\cos x}{(2+\cos x)^2\sin x}\;\mathrm dx\end{aligned}.

【分析】验证得出被积函数满足关于 sin(x) 为奇函数,从而为了凑微分,上下同时乘以一个 sin(x) .

I=5+4cosx(2+cosx)2(1cos2x)  dcosxI=-\int\frac{5+4\cos x}{(2+\cos x)^2(1-\cos^2x)}\;\mathrm d\cos x

cosx=t\cos x = t 则有:

I=5+4t(2+t)2(1t2)dt=(2+t)2+(1t2)(2+t)2(1t2)dtI=-\int\frac{5+4t}{(2+t)^2(1-t^2)}dt=-\int\frac{(2+t)^2+(1-t^2)}{(2+t)^2(1-t^2)}dt

再裂项并回代即可。

问题1a2sin2x+b2cos2x  dx\begin{aligned}\int\frac{1}{a^2\sin^2x+b^2\cos^2x}\;\mathrm dx\end{aligned}.

【分析】验证得出被积函数满足关于 sin(x),cos(x) 对称,从而为了凑微分,上下同时除以一个cos2x\cos^2x

I=sec2xa2tan2x+b2dx=dtanxa2tan2x+b2I=\int\frac{\sec^2x}{a^2\tan^2x+b^2}\mathrm dx=\int\frac{\mathrm d\tan x}{a^2\tan^2x+b^2}

t=tanxt=\tan x 问题就化为求解有理函数积分 :

I=1a2t2+b2dt=1ad(at)(at)2+b2=1abarctanatb+CI=\int\frac{1}{a^2t^2+b^2}\mathrm dt=\frac 1 a\int\frac{\mathrm d(at)}{(at)^2+b^2}=\frac1{ab}\arctan\frac{at}{b}+C

注:事实上本题尚未结束。我们给出的情况是a0,b0a\neq0,b\neq0 时的积分结果。本题实际上理应分三种情况分类讨论才能得到完整答案。

♾️问题sec3x  dx\begin{aligned}\int\sec^3x\;\mathrm dx\end{aligned}.

【分析】验证得出被积函数满足关于 cos(x) 为奇函数,从而为了凑微分,上下同时乘以一个 cos(x) :

I=dxcos3x=dsinxcos4x=t=sinxdt(1t2)2I=\int\frac{dx}{\cos^3x}=\int\frac{d\sin x}{\cos^4x}\xlongequal{t=\sin x}\int\frac{dt}{(1-t^2)^2}

【另解】本题还可以通过分部积分+积分重现的方法快速求解:

I=(sec2x)secx  dx=secx  d(tanx)=secxtanxtan2xsecx  dx=secxtanxI+secx  dxI=12secxtanx+12lntanx+secx+C\begin{aligned}I&=\int(\sec^2x)\sec x\;dx=\int\sec x\;d(\tan x)\\&=\sec x\tan x-\int\tan^2x\sec x\;dx\\&=\sec x\tan x-I+\int\sec x\;dx\\\Rightarrow&I=\frac 1 2\sec x\tan x+\frac 1 2\ln|\tan x+\sec x|+C\end{aligned}

类似的思想可以指导我们计算secnx  dx\int\sec^nx\;dx 的递推公式。

♾️问题sin4xcos2x  dx\begin{aligned}\int\sin^4x\cos^2x\;\mathrm dx\end{aligned}.

【分析】验证得出被积函数满足关于 sin(x),cos(x) 对称,从而凑微分:

I=sin4xcos4xdtanx=sin4xcos4xsin2x+cos2xdtanxI=\int\sin^4x\cos^4x\mathrm d\tan x=\int\frac{\sin^4x\cos^4x}{\sin^2x+\cos^2x}\mathrm d\tan x

上下同时cos8x\cos^8x

I=tan4x(1+tanx)4dtanxI=\int\frac{\tan^4x}{(1+\tan x)^4}\mathrm d\tan x

【另解】本题可以利用倍角公式不断降阶化简。

I=sin2x(sin2xcos2x)dx=1cos2x2sin2x4dx=18sin2xsin4x2dx\begin{aligned}I=&\int\sin^2x(\sin^2x\cos^2x)\mathrm dx=\int\frac{1-\cos2x}{2}·\frac{\sin2x}{4}\mathrm dx\\=&\frac{1}{8}\int\sin2x-\frac{\sin4x}{2}\mathrm dx\end{aligned}

类似的思想可以指导我们计算sinnxcosmx  dx\int\sin^nx\cos^mx\;dx 的递推公式。我们有:

  • m,nm,n 有奇数时,直接凑微分;
  • m,nm,n 均为偶数时,反复利用倍角公式降阶。

组合性凑微分

对于形如Asinx+BcosxCsinx+Dcosx  dx:=PQ  dx\begin{aligned}\int\frac{A\sin x+B\cos x}{C\sin x+D\cos x}\;dx:=\int\frac{P}{Q}\;dx\end{aligned} 的三角函数积分,都可以通过将分子PP 凑成:P=pQ+qQP=pQ+qQ' 的形式。从而有:

I=pQ+qQQdx=px+lnQ+CI=\int\frac{pQ+qQ'}{Q}dx=px+\ln|Q|+C

其中,利用待定系数法可将p,qp,q 求出。

不断降阶化简

  • 尝试使用sin2x+cos2x=1,tan2x+1=sec2x\sin^2x+\cos^2x=1,\tan^2x+1=\sec^2x诱导公式、万能公式、倍角公式、积化和差、和差化积 等三角函数公式不断对高次项降阶,从而化为我们熟悉的低阶形式再积分。

如:

2sinxcosx=(sinx+cosx)212sinxcosx=1(sinxcosx)2sin4xcos4x=(sin2x+cos2x)(sin2xcos2x)=cos2xsin4x+cos4x=(sin2x+cos2x)22sin2xcos2x=112sin22x=14(3cos4x)sin6x+cos6x=(sin2x)3+(cos2x)3=(sin2x+cos2x)(sin4xsin2xcos2x+cos4x)=(sin2x+cos2x)23sin2xcos2x=134sin22x\begin{aligned}2\sin x\cos x&=(\sin x+\cos x)^2-1\\2\sin x\cos x&=1-(\sin x-\cos x)^2\\\\\sin^4x-\cos^4x&=(\sin^2x+\cos^2x)(\sin^2x-\cos^2x)\\&=-\cos2x\\\\\sin^4x+\cos^4x&=(\sin^2x+\cos^2x)^2-2\sin^2x\cos^2x\\&=1-\frac12\sin^22x=\frac14(3-\cos4x)\\\\\sin^6x+\cos^6x&=(\sin^2x)^3+(\cos^2x)^3\\&=(\sin^2x+\cos^2x)(\sin^4x-\sin^2x\cos^2x+\cos^4x)\\&=(\sin^2x+\cos^2x)^2-3\sin^2x\cos^2x\\&=1-\frac34\sin^22x\end{aligned}

♾️问题1sin3x+cos3x  dx\begin{aligned}\int\frac{1}{\sin^3x+\cos^3x}\;\mathrm dx\end{aligned}.

含三角函数的不定积分求解思路:

  • 尝试找寻整块可分部积分的特殊函数

三角有理式

sinnxsinxdx\begin{aligned}\int\frac{\sin nx}{\sin x}dx\end{aligned}

简单无理函数积分的求解

简单无理函数积分

所谓简单无理函数积分通常是指在被积函数中含有形如ax+bnax+bcx+dn\sqrt[n]{ax+b}、\sqrt[n]{\frac{ax+b}{cx+d}}ax2+bx+c\sqrt{ax^2+bx+c} 这样的根式的积分。这类积分通常我们直接利用换元法处理。

  • 对于第三种根式,我们通过配方的方法将其化为a2±x2x2a2\sqrt{a^2\pm x^2}、\sqrt{x^2-a^2} 的一种,然后使用三角换元解决。这将在后文中阐述。

事实上,即使是形如aex+bnaex+bcex+dn\begin{aligned}\sqrt[n]{ae^x+b}、\sqrt[n]{\frac{ae^x+b}{ce^x+d}}\end{aligned} 的根式也可以直接换元!!

问题xexex2  dx\begin{aligned}\int\frac{xe^x}{\sqrt{e^x-2}}\;\mathrm dx\end{aligned}. (答案略)

♾️问题1(x+1)2(x1)43  dx\begin{aligned}\int\frac1{\sqrt[3]{(x+1)^2(x-1)^4}}\;\mathrm dx\end{aligned}. (同济《高等数学》课后习题)

三角换元及其取值范围

在求解简单无理函数积分时,我们指出对于含根号下二次函数的情况应该利用三角换元。事实上,即使没有根号对于出现这种形式的式子也应该考虑是否可以使用三角换元使其变得更加简单。

下面我们先给出三角换元的具体情况:

三角换元的三种类型

对于x=sectx=\sec t 类的换元,由于其可能取负数,因此在根号下换元时应当考虑正负情况以分类讨论。

问题 计算不定积分1x1+x+x2dx\begin{aligned}\int \frac{1}{x\sqrt{1+x+x^2}}\mathrm dx\end{aligned}

解法一 利用三角函数换元:
分母根号项可以进行配方1+x+x2=(x+12)2+(32)2\sqrt{1+x+x^2}=\sqrt{(x+\frac 1 2)^2+(\frac {\sqrt 3} 2)^2}
x+12=32tantx+\frac 1 2=\frac {\sqrt 3} 2\tan t,则dx=32sec2tdt\mathrm dx=\frac{\sqrt 3} 2\sec^2t\mathrm dt

I=32sec2tdt32sect(32tant12)=132sint12costdt=sec(t+π3)dt\begin{aligned}I&=\int\frac{\frac{\sqrt 3} 2\sec^2t\mathrm dt}{\frac{\sqrt 3} 2\sec t(\frac{\sqrt 3} 2\tan t-\frac 1 2)}\\&=\int\frac 1 {\frac{\sqrt 3} 2\sin t-\frac 1 2\cos t}\mathrm dt\\&=-\int \sec(t+\frac {\pi} 3)\mathrm dt\end{aligned}

解法二 构造巧妙的换元方式:
1+x+x2=xt\sqrt{1+x+x^2}=x-t. 这种换元方法巧妙之处在于,将根号项换元为带x,tx,t 的式子,而非纯tt 的式子。
此时两边取平方时可以将含有x2x^2 项抵消。
于是得到:x=t212t+1x=\frac{t^2-1}{2t+1},从而:

I=d(t212t+1)t212t+1(t212t+1t)=2(t2+t+1)(2t+1)2dt(t2+t+1)(t21)(2t+1)2=2dtt21=(1t+11t1)dt\begin{aligned}I&=\int \frac{\mathrm d(\frac{t^2-1}{2t+1})}{\frac{t^2-1}{2t+1}·(\frac{t^2-1}{2t+1}-t)}\\&=-\int\frac{\frac{2(t^2+t+1)}{(2t+1)^2}\mathrm dt}{\frac{(t^2+t+1)(t^2-1)}{(2t+1)^2}}\\&=-2\int\frac{\mathrm dt}{t^2-1}=\int(\frac 1{t+1}-\frac 1 {t-1})\mathrm dt\end{aligned}

特殊无理积分的倒代换

当被积函数分母的最高次数高于分子最高次数时,可以考虑做倒代换。

对于形如1(x+d)kax2+bx+cdx\begin{aligned}\int\frac1{(x+d)^k\sqrt{ax^2+bx+c}}\mathrm dx\end{aligned} 其中k=1,2k=1,2 的积分,有一种特殊换元方法,即直接令x+d=1/tx+d=1/t

💦问题1x2x2+2x+1dx\begin{aligned}\int\frac1{x\sqrt{2x^2+2x+1}}\mathrm dx\end{aligned}. 本题同样可以利用常规思路,对根号下的二次函数配方,然后利用三角换元。而如果我们令x=1/tx=1/t 则有:

I=1t2+2t+2dt=1(t+1)2+1d(t+1)I=\int\frac1{\sqrt{t^2+2t+2}}\mathrm dt=\int\frac1{\sqrt{(t+1)^2+1}}\mathrm d(t+1)

后续直接利用公式即可!

巧用分部积分解决不定积分

分部积分的注意事项

谁怕谁先上

多个不同类型的初等函数乘积出现在被积函数中时,我们通常需要把最害怕被求导的一项想方设法让他被求导。因此,我们需要将除它之外的函数凑到后面。

下面我们总结出常见的可整块函数一起被凑的结论。(不含+C+C

  • 快速甄别可整块分部积分的常见函数:

x(lnx+1)=lnxdxx+1x=(11x2)dxsinx1+cosx=tan(x2)=11+cosxdxtan(x2π2)=11+sinxdxln(x+x2+1)=1x2+1dxln(x+x21)=1x21dx1x2+1=1(x2+1)3dx1xsinx+cosx=xcosx(xsinx+cosx)2dx12ex(sinxcosx)=exsinxdx\begin{aligned}x(\ln x+1)&=\int\ln x\mathrm dx\\x+\frac 1 x&=\int\left(1-\frac 1 {x^2}\right)\mathrm dx\\\\\frac {\sin x}{1+\cos x}=\tan(\frac x2)&=\int\frac 1 {1+\cos x}\mathrm dx\\\tan(\frac x2-\frac\pi2)&=\int\frac 1{1+\sin x}\mathrm dx\\\\\ln\left(x+\sqrt{x^2+1}\right)&=\int\frac 1 {\sqrt{x^2+1}}\mathrm dx\\\ln\left(x+\sqrt{x^2-1}\right)&=\int\frac 1 {\sqrt{x^2-1}}\mathrm dx\\\frac 1 {\sqrt{x^2+1}}&=\int\frac 1 {\sqrt{(x^2+1)^3}}\mathrm dx\\\\\frac 1{x\sin x+\cos x}&=\int\frac{x\cos x}{(x\sin x+\cos x)^2}\mathrm dx\\\\\frac12e^x(\sin x-\cos x)&=\int e^x\sin x\mathrm dx\end{aligned}

🦄要凑就凑巧

在分部积分时,我们可以通过多次凑微分、对凑出来的微分加项减项、利用exe^x 的特性等方法使得分部之后的结果更简洁。

❤️ 加个常数未雨绸缪凑微分

♾️问题xln(1+x2)arctanx  dx\begin{aligned}\int x\ln(1+x^2)\arctan x\;\mathrm dx\end{aligned}.
不难发现,对数函数与反三角函数求导之后的结果将会变得简单。为此,我们需要把xx 凑到后面。而凑的过程中,我们发现余下的函数求导会导致分母出现x2+1x^2+1 ,所以我们不妨凑的时候直接凑d(x2+1)\mathrm d(x^2+1)

I=12ln(1+x2)arctanx  d(1+x2)I=\frac12\int\ln(1+x^2)\arctan x\;\mathrm d(1+x^2)

利用分部积分,得到:

I=12(1+x2)ln(1+x2)arctanx12xarctanx  dx12ln(1+x2)  dxI=\frac12(1+x^2)\ln(1+x^2)\arctan x-\frac12\int x\arctan x\;dx-\frac12\ln(1+x^2)\;dx

而后两个积分,第一个也可以继续凑d(x2+1)\mathrm d(x^2+1) ,第二个直接分部积分即可。


❤️ 分母高次方凑微分

1xf(xn)  dx=xn1xnf(xn)  dx=t=xn1ndttf(t)\int\frac{1}{xf(x^n)}\;dx=\int\frac{x^{n-1}}{x^nf(x^n)}\;dx\xlongequal{t=x^n}\frac 1 n\int\frac{dt}{tf(t)}

♾️问题limt0(011t4+x44  dxlnt)\begin{aligned}\lim_{t\to0}\left(\int_0^1\frac 1 {\sqrt[4]{t^4+x^4}}\;dx-\ln t\right)\end{aligned}答案


❤️exe^x 相关的凑微分

  1. 当被积函数是exe^x 的函数时,通常可以凑dexde^x

f(ex)  dx=exf(ex)exdx=t=exf(t)tdt\int f(e^x)\;dx=\int\frac{e^xf(e^x)}{e^x}dx\xlongequal{t=e^x}\int\frac{f(t)}{t}dt

  1. 一类xef(x)xe^{f(x)} 题源:

I=1+xf(x)x(1+xef(x))dx=ef(x)[1+xf(x)]xef(x)(1+xef(x))dx=t=xef(x)dtt(1+t)I=\int\frac{1+xf'(x)}{x(1+xe^{f(x)})}\mathrm dx=\int\frac{e^{f(x)}[1+xf'(x)]}{xe^{f(x)}(1+xe^{f(x)})}\mathrm dx\xlongequal{t=xe^{f(x)}}\int\frac{dt}{t(1+t)}

问题1+xcosxx(1+xesinx)  dx\begin{aligned}\int\frac{1+x\cos x}{x(1+xe^{\sin x})}\;\mathrm dx\end{aligned}. 不难发现,上下同时乘以esinxe^{\sin x} 之后,分子正好就是xesinxxe^{\sin x} 的导函数。于是可以整体换元。


❤️ 强行凑微分

♾️问题x2(xcosx+sinx)2  dx\begin{aligned}\int\frac{x^2}{(x\cos x+\sin x)^2}\;\mathrm dx\end{aligned}.

首先我们考虑到1/u2du=d(1/u)1/u^2du=-d(1/u),因此,我们考虑求1/(xcosx+sinx)1/(x\cos x+\sin x) 求导。
如果结果正好和分子匹配那将皆大欢喜,否则如果我们可以改造分子强行凑微分。

I=xcosxxcosx(xcosx+sinx)2dx=xcosxd(1xcosx+sinx)I=\int\frac x{\cos x}·\frac{x\cos x}{(x\cos x+\sin x)^2}dx=\int\frac x{\cos x}d\left(\frac1{x\cos x+\sin x}\right)

接下来继续分部积分即可!!

♾️问题0+ex2(x2+12)2  dx\begin{aligned}\int_0^{+\infty}\frac{e^{-x^2}}{(x^2+\frac12)^2}\;\mathrm dx\end{aligned}.

https://www.bilibili.com/video/BV1Vv41117gf

切勿操之过急

在前面求解无理函数积分时,我们提出了换元的思想。而在换元过程中,我们未必需要把微分结果表出。
如:x=u(t)x=u(t),一般情况下我们会直接得出dx=u(t)dt\mathrm dx=u'(t)\mathrm dt 然后继续计算。
而有的时候,如果我们仅仅写作du(t)\mathrm du(t) 之后再计算过程中分部积分,则反而更利于计算。

总而言之,在遇到既需要换元又需要分部的题目时,我们先换元然后酌情分部。

♾️问题ln(1+1+xx)  dx\begin{aligned}\int\ln\left(1+\sqrt{\frac{1+x}{x}}\right)\;\mathrm dx\end{aligned}.

【分析】首先被积函数出现根式,我们整体换元:

t=1+xx,  x=1t21t=\sqrt{\frac{1+x}x},\;x=\frac1{t^2-1}

从而有:

I=ln(1+t)d(1t21)=ln(1+t)t21dt(t+1)2(t1)I=\int\ln(1+t)\mathrm d\left(\frac1{t^2-1}\right)=\frac{\ln(1+t)}{t^2-1}-\int\frac{\mathrm dt}{(t+1)^2(t-1)}

不难发现,我们如果将dxdx 写开,式子将十分复杂反而不利于我们遵循“谁怕谁先上”原则。

🦄积分抵消的来源

一般地,当一个题目出现拆项+分部积分实现积分抵消的效果时,其实际上来源于:
[h(x)g(x)+h(x)g(x)]eg(x)dx=h(x)eg(x)+C\begin{aligned}\int [h'(x)g(x)+h(x)g'(x)]e^{g(x)}dx=h(x)e^{g(x)}+C\end{aligned}
即被积函数中有指数函数。当然,也有例外,但基本规律都是求导法则。

对于这类问题,一般的求解思路是将积分拆成两项,然后分别(或者只变动一边)进行分部积分,使得分部积分后还原出可抵消的积分项,从而实现化简。


问题ex1+sinx1+cosxdx\begin{aligned}\int e^x \frac{1+\sin x}{1+\cos x}\mathrm d x\end{aligned}

【分析】直接利用exf(x)e^xf(x)构造函数方法处理:

(sinx1+cosx)=11+cosxex1+sinx1+cosxdx=ex[(sinx1+cosx)+(11+cosx)]dx=exsinx1+cosx+C=extanx2+C\begin{aligned}\because\quad(\frac {\sin x}{1+\cos x})'&=\frac 1 {1+\cos x}\\\\\therefore\quad\int e^x \frac{1+\sin x}{1+\cos x}\mathrm d x&=\int e^x[(\frac {\sin x}{1+\cos x})+(\frac 1 {1+\cos x})]\mathrm dx\\&=e^x\frac {\sin x}{1+\cos x}+C\\&=e^x\tan\frac x 2+C\end{aligned}

♾️问题x2ex22xdx\begin{aligned}\int x^2e^{\frac{x^2}{2}-x}\mathrm dx\end{aligned}

【分析】对于这个题而言,由于没有明显的可拆项,所以我们回顾这类问题的源头,既然会引入被积函数会引入ex22xe^{\frac{x^2}2-x} 的导数,所以我们应该要想到把[x22x][\frac{x^2}2-x]' 引入,从而想到应该出现(x1)(x-1),再结合已给定的x2x^2 进行改造。

I=x2ex22xdx=(x21)ex22xdx+ex22xdx=(x+1)d(ex22x)+ex22xdx=(x+1)ex22xex22xdx+ex22xdx=(x+1)ex22x+C\begin{aligned}I=&\int x^2e^{\frac{x^2}{2}-x}\mathrm dx\\=&\int (x^2-1)e^{\frac{x^2}{2}-x}\mathrm dx+\int e^{\frac{x^2}{2}-x}\mathrm dx\\=&\int(x+1)\mathrm d\left(e^{\frac{x^2}{2}-x}\right)+\int e^{\frac{x^2}{2}-x}\mathrm dx\\=&(x+1)e^{\frac{x^2}{2}-x}-\int e^{\frac{x^2}{2}-x}\mathrm dx+\int e^{\frac{x^2}{2}-x}\mathrm dx\\=&(x+1)e^{\frac{x^2}{2}-x}+C\end{aligned}

♾️问题1lnx(xlnx)2dx\begin{aligned}\int\frac{1-\ln x}{(x-\ln x)^2}\mathrm dx\end{aligned}

【分析】如果直接拆项将难以解决问题。我们应当将分母降次再拆分。

I=(xlnx)+1x(xlnx)2dx=1xlnxdx+1x(xlnx)2dx=xxlnxx  d(1xlnx)+1x(xlnx)2dx=xxlnx+C\begin{aligned}I&=\int\frac{(x-\ln x)+1-x}{(x-\ln x)^2}dx=\int\frac{1}{x-\ln x}dx+\int\frac{1-x}{(x-\ln x)^2}dx\\&=\frac x{x-\ln x}-\int x\;d\left(\frac1{x-\ln x}\right)+\int\frac{1-x}{(x-\ln x)^2}dx\\&=\frac x{x-\ln x}+C\end{aligned}

♾️问题(arctanxarctanxx)2dx\begin{aligned}\int\left(\frac{\arctan x}{\arctan x-x}\right)^2\mathrm dx\end{aligned}

【分析】参考之前的思路,我们为了对分母降次,对分子进行±x\pm x 操作。之后再打开平方和公式,将分母高阶项定住,对分母低阶项进行分部积分。
值得注意的是,本题对低阶项分部积分的时候如果不改写凑出来的微分±x\pm x)将抵消不了结果。
【答案】x+x2+1arctanxx+C\begin{aligned}x+\frac{x^2+1}{\arctan x-x}+C\end{aligned}

🦄复杂因子求导出奇迹

在前面的“要凑就凑巧”一节中,我们给出了以xef(x)xe^{f(x)} 的出题模板。事实上,这类出题方法可以不局限于指数函数。有:

I=f(x)g[1+f(x)]dxI=\int\frac{f'(x)}{g[1+f(x)]}\mathrm dx

如果是抽象函数,乍一看会十分简单。但是如果代入具体的函数,复杂程度就会增加。
如下面几道题:

问题1lnx(xlnx)2dx\begin{aligned}\int\frac{1-\ln x}{(x-\ln x)^2}\mathrm dx\end{aligned}

【分析】不难发现,本题在前面的“积分抵消的来源”一节中也出现过。现在我们利用这类的思想解决它。首先将分母提出一个x2x^2 于是有:

I=1lnxx2(1lnxx)2dxI=\int\frac{1-\ln x}{x^2(1-\frac{\ln x}{x})^2}dx

如果我们对lnxx\frac{\ln x}{x} 求导,会惊奇地发现其结果正好是1lnxx2\frac{1-\ln x}{x^2},于是本题直接凑微分即可解决。
显然,本题就是将模板中取:f(x)=lnxx,g(x)=x2f(x)=\frac{\ln x}{x},g(x)=x^2.

♾️问题x+sinxcosx(cosxxsinx)2dx\begin{aligned}\int\frac{x+\sin x\cos x}{(\cos x-x\sin x)^2}\mathrm dx\end{aligned}

【分析】分母提出一个cos2x\cos^2x 即可!

组合积分法+钓鱼积分题
  1. xsinx(x+cosx)2dx\begin{aligned} \int \frac{x\sin x}{(x+\cos x)^2}\mathrm dx\end{aligned} 不可积
  2. 15sinxx3(1+x)dx\begin{aligned}\int_1^5\frac{\sin x}{x^3(1+x)}\mathrm dx\end{aligned} 无初等解析解

https://math.stackexchange.com/questions/917833/evaluate-int-01-lnx-ln1-x-dx

组合积分法

待更

定积分计算手册

定积分奇偶性公式

llf(x)dx=0l[f(x)+f(x)]dx\int_{-l}^lf(x)dx=\int_0^l[f(x)+f(-x)]dx

区间再现导出结论汇总

区间再现导出结论汇总

【区间再现公式】abf(x)dx=abf(b+ax)dx\begin{aligned}\int_a^bf(x)\mathrm dx=\int_a^bf(b+a-x)\mathrm dx\end{aligned}

事实上,所谓的区间再现公式只不过是一个定积分的换元法,而熟记这个公式可以不需换元即可看出代换后的效果.


下面我们将给出区间再现常用的导出结论,这在求解定积分时可快速写出结果.

结论10π2f(sinx)dx=0π2f(cosx)dx\begin{aligned}\int_0^{\frac{\pi}{2}} f(\sin x)dx=\int_0^{\frac{\pi}{2}}f(\cos x)dx\end{aligned}

证明

0π2f(sinx)dx=t=π2x0π2f(sin(π2t))dt=0π2f(cost)dt\begin{aligned}\int_0^{\frac{\pi}{2}} f(\sin x)dx\xlongequal{t=\frac \pi 2-x}\int_0^{\frac{\pi}{2}}f\left(\sin (\frac \pi 2-t)\right)dt=\int_0^{\frac{\pi}{2}}f(\cos t)dt\end{aligned}

结论20πxf(sinx)dx=π20πf(sinx)dx\begin{aligned}\int_0^\pi xf(\sin x)dx=\frac \pi 2\int_0^\pi f(\sin x)dx\end{aligned}

证明

I=0πxf(sinx)dx=区间再现t=πxπ0πf(sint)dtx0πf(sint)dt2I=π0πf(sinx)dxI=π20πf(sinx)dx\begin{aligned}I&=\int_0^\pi xf(\sin x)dx\xlongequal[\text{区间再现}]{t=\pi-x}\pi \int_0^\pi f(\sin t)dt-x\int_0^\pi f(\sin t)dt\\\rightarrow2I&=\pi \int_0^\pi f(\sin x)dx\\\rightarrow I&=\frac \pi 2\int_0^\pi f(\sin x)dx\end{aligned}

结论30πxf(sinx)dx=π0π2f(sinx)dx\begin{aligned}\int_0^\pi xf(\sin x)dx=\pi \int_0^{\frac{\pi}{2}}f(\sin x)dx\end{aligned}

证明

I=0πxf(sinx)dx=0π2xf(sinx)dx+π2πxf(sinx)dx=对后一项t=πx0π2xf(sinx)dx+0π2(πt)f(sint)dt=π0π2f(sinx)dx\begin{aligned}I&=\int_0^\pi xf(\sin x)dx=\int_0^{\frac \pi 2} xf(\sin x)dx+\int_{\frac \pi 2}^\pi xf(\sin x)dx\\&\xlongequal[\text{对后一项}]{t=\pi-x}\int_0^{\frac \pi 2} xf(\sin x)dx+\int_0^{\frac \pi 2} (\pi-t)f(\sin t)dt\\&=\pi\int_0^{\frac \pi 2} f(\sin x)dx\end{aligned}

结论4 设函数f(x)f(x) 连续,且满足f(x)=f(Tx)f(x)=f(T-x),则有:

0Txf(x)dx=T20Tf(x)dx=T0T2f(x)dx\int_0^Txf(x)dx=\frac T 2\int_0^Tf(x)dx=T\int_0^{\frac T 2}f(x)dx

Wallis公式(点火公式)

华里士公式| Wallis

In=0π2sinnx  dx=0π2cosnx  dx={n1nn3n2    3412π2=(n1)!!n!!π2,n is evenn1nn3n2    45231=(n1)!!n!!,n is odd\begin{aligned}I_n&=\int_0^{\frac \pi 2}\sin^nx\;\mathrm dx=\int_0^{\frac \pi 2}\cos^nx\;\mathrm dx\\&=\begin{cases}\begin{aligned}&\frac{n-1}{n}\cdot\frac{n-3}{n-2}\cdot\;\cdots\;\cdot\frac 3 4\cdot\frac 1 2\cdot\frac\pi 2=\frac{(n-1)!!}{n!!}\cdot\frac{\pi}{2},&\quad n\text{ is even}\\&\frac{n-1}{n}\cdot\frac{n-3}{n-2}\cdot\;\cdots\;\cdot\frac 4 5\cdot\frac 2 3\cdot1=\frac{(n-1)!!}{n!!},&\quad n\text{ is odd}\end{aligned}\end{cases}\end{aligned}

此外特别地,通过t=xπ/2t=x-\pi/2 换元,还可以得到:

0πsinnx  dx=20π2cosnx  dx=20π2sinnx  dx\begin{aligned}\int_0^\pi\sin^nx\;\mathrm dx=2\int_0^{\frac{\pi}{2}}\cos^nx\;\mathrm dx=2\int_0^{\frac{\pi}{2}}\sin^nx\;\mathrm dx\end{aligned}

转化为二重积分求定积分

转化为二重积分求解定积分

常有一类一元函数定积分问题,其被积函数本身对应的不定积分或者说原函数是无法用初等函数表示的,或者即使能表示也十分复杂。这时我们明确我们的目标是求定积分而非不定积分,所以应该充分考虑到被积函数的形式,将其进行转化。

常用的方法有:

  • 转化为二重积分,并结合交换积分次序的方法简化计算
  • 将被积函数的一部分写成无穷级数的方式,利用无穷级数的符号可交换性处理
  • 对整个定积分引入参数α\alpha,通过含参积分求导得到原积分关于α\alpha 的函数关系,最后求解

问题f(x)f(x) 是在[0,π2][0,\frac\pi2] 上满足0π2f(tx)f(x)dt=1sin4x\begin{aligned}\int_0^{\frac\pi2}f(t-x)f(x)\mathrm dt=1-\sin^4x\end{aligned} 的连续正值函数,求其在该区间下的平均值。

待更

利用无穷级数求定积分

无穷级数求解定积分

待更

※利用含参积分求导解定积分

含参积分求导解定积分

待更

伽马函数和常见积分结论

伽马函数与泊松积分

考察 反常积分+ex2dx=π\begin{aligned}\int_{-\infty}^{+\infty}e^{-x^2}\mathrm dx=\sqrt\pi\end{aligned}.

该积分叫做 欧拉-泊松积分,是十分经典也十分重要的积分结果,在概率论中也常有使用。
在前面⌈利用二重积分计算定积分⌋部分,我们给出了其中一种证明方法。事实上该积分的计算方法还有多种,详见拓展知识。

泊松积分是伽马函数Γ(s)\Gamma(s)s=12s=\frac12 处取得的结果。下面我们来介绍伽马函数的相关定义与性质。

伽马函数 定义 关于变量s(s>0)s\quad(s\gt0) 的函数Γ(s)=0+exxs1dx\Gamma(s)=\begin{aligned}\int_0^{+\infty}e^{-x}x^{s-1}\mathrm dx\end{aligned}.

  1. 显然有Γ(1)=1\Gamma(1)=1
  2. s0+s\to0^+ 时,Γ(s)+\Gamma(s)\to+\infty
  3. 可以证明:Γ(s+1)=sΓ(s)\Gamma(s+1)=s\Gamma(s)
  4. 反复利用上述递推公式,有:Γ(n+1)=n!\Gamma(n+1)=n!
  5. 余元公式Γ(s)Γ(1s)=πsinπs(0<s<1)\begin{aligned}\Gamma(s)\Gamma(1-s)=\frac{\pi}{\sin \pi s}\quad(0\lt s\lt1)\end{aligned}
  6. 根据余元公式,令s=12s=\frac12 再适当换元即可得到泊松积分

参考:

  1. https://zhuanlan.zhihu.com/p/363468184
  2. https://www.cnblogs.com/Renascence-5/p/5432211.html

常见积分结论

  1. 欧拉积分(Euler)0π2ln(sinx)dx=π2ln2\begin{aligned}\int_{0}^{\frac\pi 2}\ln(\sin x)\mathrm dx=-\frac \pi 2\ln2\end{aligned}
  2. 泊松积分(Euler-Poisson)+ex2dx=π\begin{aligned}\int_{-\infty}^{+\infty}e^{-x^2}\mathrm dx=\sqrt{\pi}\end{aligned}
  3. 菲涅尔积分(Fresnel)0+sinx2dx=2π4\begin{aligned}\int_{0}^{+\infty}\sin{x^2}\mathrm dx=\frac{\sqrt{2\pi}}{4}\end{aligned}
  4. 狄利克雷积分(Dirichlet)0+sinxxdx=π2\begin{aligned}\int_{0}^{+\infty}\frac{\sin x}{x}\mathrm dx=\frac\pi 2\end{aligned}
例题收集

问题 求定积分π2π23exsin2x1+exdx\begin{aligned}\int_{-\frac \pi 2}^{\frac \pi 2}\frac{3e^x\sin^2x}{1+e^x}dx\end{aligned}.

答案

本题蕴含了 区间再现、奇偶性公式、点火公式等方法技巧,是一道很经典的例题。

**解**:$$\begin{aligned}I&=3\int_{-\frac \pi 2}^{\frac \pi 2}\frac{\sin^2x}{1+e^{-x}}dx\\&\xlongequal[]{\;\;\text{奇偶性公式}}3\int_0^{\frac \pi 2}[\frac 1 {1+e^x}+\frac 1 {1+e^{-x}}]\sin^2x\;dx\\&=3\int_0^{\frac \pi 2}\sin^2x\;dx\xlongequal[]{\;\;\text{Wallis公式}}3·\frac{1}{2}·\frac{\pi}{2}=\frac{3\pi}{4}\end{aligned}$$

问题 求定积分01x1lnxdx\begin{aligned}\int_0^1\frac {x-1}{\ln x}dx\end{aligned}.

答案

不定积分x1lnxdx\int\frac {x-1}{\ln x}dx无法直接求,于是考虑对被积函数f(x)=x1lnxf(x)=\frac {x-1}{\ln x}进行变换

f(x)=x1lnx=01xydyf(x)=\frac{x-1}{\ln x}=\int_0^1x^ydy

于是:

I=01x1lnxdx=01dx01xydy=  交换积分次序01dy01xydxI=\int_0^1\frac{x-1}{\ln x}dx=\int_0^1dx\int_0^1x^ydy\xlongequal{\quad\;\text{交换积分次序}}\int_0^1dy\int_0^1x^ydx

I=ln2I=\ln2


另解 |思路来源|Youtube

在处理被积函数中,发现因为lnx\ln x的出现,我们很难进行求解。而在回忆各类与ln()\ln(·)有关的导函数时,我们注意到如下规则:

(αx)=αxlnα(\alpha^x)'=\alpha^x·\ln \alpha

这里,事实上是对xx求导,而α\alpha作为一个ConstantConstant.此处我们换一种思考,将其变换为如下形式,并且对α\alpha求导,就能搞出一个lnx\ln x出来了。

d(xα)dα=xαlnxd[xα/lnx]dα=xαd[(xα1)/lnx]dα=xα\begin{aligned}\frac{d(x^\alpha)}{d\alpha}&=x^\alpha·\ln x\\\frac{d[x^\alpha/\ln x]}{d\alpha}&=x^\alpha\\\frac{d[(x^\alpha-1)/\ln x]}{d\alpha}&=x^\alpha\end{aligned}

于是,可构造函数I(α)I(\alpha)

I(α)=01xα1lnxdxI(\alpha)=\int_0^1\frac{x^\alpha-1}{\ln x}dx\\

显然有I(0)=0I(0)=0.

又:

I(α)=d[01xα1lnxdx]/dα=01[xα1lnx]dxI(α)=01xαdx=[xα+1α+1]0x=1=1α+1\begin{aligned}I'(\alpha)&=d[\int_0^1\frac{x^\alpha-1}{\ln x}dx]/d\alpha=\int_0^1[\frac{x^\alpha-1}{\ln x}]'dx\\I'(\alpha)&=\int_0^1x^\alpha dx=[\frac{x^{\alpha+1}}{\alpha+1}]^{x=1}_0=\frac 1 {\alpha +1}\end{aligned}

于是:

{I(α)=1α+1dα=lnα+1+CI(0)=0C=0.I(α)=lnα+1\begin{aligned}&\begin{cases}I(\alpha)=\int\frac{1}{\alpha+1}d\alpha=\ln|\alpha+1|+C\\I(0)=0\end{cases}\Rightarrow\quad C=0.\\\\&\therefore I(\alpha)=\ln|\alpha+1|\end{aligned}

于是

01x1lnxdx=I(1)=ln2\int_0^1\frac{x-1}{\ln x}dx=I(1)=\ln 2


问题实数集上的单调递增可微函数f(x)f(x) 满足:
(1)f(1)=1,12f(x)dx=54f(1)=1,\int_1^2f(x)\mathrm{d}x=\frac 5 4
(2)若f(x)f(x)反函数g(x)g(x),对于任意x1x\geq 1g(2x)=2f(x)g(2x)=2f(x).
如果18xf(x)dx=qp\int_1^8xf'(x)\mathrm dx = \frac q p,(p,qp, q互质),求p+qp+q的值。


问题f(x)=0xln(1+t)1+t2dt\begin{aligned}f(x)=\int_0^x\frac{\ln(1+t)}{1+t^2}\mathrm dt\end{aligned},计算定积分01xf(x)dx\int_0^1xf(x)\mathrm dx.

问题an=01xn1x2  dx,  bn=0π2sinnxcosnx  dx\begin{aligned}a_n=\int_0^1x^n\sqrt{1-x^2}\;\mathrm dx,\;b_n=\int_0^{\frac \pi 2} \sin^nx\cos^nx\;\mathrm dx\end{aligned},求limnbnan\begin{aligned}\lim_{n\to\infty}\frac{b_n}{a_n}\end{aligned}.

问题 计算反常积分0π2lnsinx  dx\begin{aligned}\int_0^{\frac{\pi}2}\ln\sin x \;\mathrm dx\end{aligned}

提示:令t=xπ/4t=x-\pi/4 做区间再现,使得原式复现

双线性变换方法

积分学的运用

几种常见曲线的表达式与图示

几种常见曲线

一元函数积分学的几何运用

在定积分的定义阐述中,我们利用曲边梯形面积进行分析和处理。这也揭示了极限与微元在求解具体问题时的方法论。根据这类知识(统称为 微元分析法),我们可以用来求解其他问题。包括:平面面积、曲线弧长甚至是物理学上的应用。

对平面面积的应用我们此处适当略过。只需注意,对于直角坐标系取微元时,我们视其为长条矩形,对于极坐标,视其为长条等腰三角形。

下面我们将介绍平面曲线的弧长,以及弧微分。这对我们之后要学的曲线积分有极大帮助!

平面曲线的弧长

设曲线弧由参数方程l:{x=φ(t),y=ψ(t)atbl:\begin{cases}x=\varphi(t),\\y=\psi(t)\end{cases}\quad a\leq t\leq b 给出,且函数有连续导数。在[a,b][a,b] 中任取一区间[t,t+Δt][t,t+\Delta t] 的小弧段,则其长度近似于弦长:Δt(Δx)2+(Δy)2\Delta t\approx\sqrt{(\Delta x)^2+(\Delta y)^2}.

于是有:

ds=φ2(t)+ψ2(t)dt\mathrm ds=\sqrt{\varphi'^2(t)+\psi'^2(t)}\mathrm dt

  • 若可以显式表示曲线y=f(x)y=f(x) 则有:ds=1+y2  dx\mathrm ds=\sqrt{1+y'^2}\;\mathrm dx,这也被称为 弧微分

极坐标方程也可以推导得出:

ds=r2(θ)+r2(θ)  dθ\mathrm ds=\sqrt{r^2(\theta)+r'^2(\theta)}\;\mathrm d\theta

平面曲线的曲率

平面曲线的曲率KK 及其曲率圆半径ρ\rho 表达式如下:

K=dαds=y(1+y2)3/2,  ρ=1KK=\left|\frac{d\alpha}{ds}\right|=\frac{|y''|}{(1+y'^2)^{3/2}},\;\rho=\frac1K

旋转体体积

xx

V=πabf2(x) dxV=\pi\int_a^bf^2(x)\ \text d x

yy

V=πcdφ2(y) dy   or   V=2πabxf(x) dxV=\pi\int_c^d\varphi^2(y)\ \text dy\;\text{ or }\; V=2\pi\int_a^bx|f(x)|\ \text dx

🦄 y=ax+by=ax+b 旋转

V=2πDr(x,y)dσV=2\pi\iint_Dr(x,y)\mathrm d\sigma

其中,r(x,y)r(x,y) 是任意一个区域内的点(x,y)(x,y) 到直线yaxb=0y-ax-b=0 的距离:

r(x,y)=yaxb1+a2r(x,y)=\frac{|y-ax-b|}{\sqrt{1+a^2}}

旋转面的侧面积

利用微元分析法,将每一份微元都视为圆台的侧面积
ab\overset{\frown}{ab} 为直线段,长ll,端点a,ba,b 的纵坐标分别为ya,yby_a,y_b 则有圆台侧面积大小:

S=πl(ya+yb)S=\pi l(y_a+y_b)

于是可得不同情况下的侧面积。

特别地,曲线y=y(x)y=y(x)xx 轴旋转得到的侧面积,有:

S=2πaby1+y2  dxS_{\text{侧}}=2\pi\int_a^by\sqrt{1+y'^2}\;\mathrm dx

下面我们将以上几种几何运用整理得到的表展示如下:

🦄 古尔丁定理

待更


问题 设曲线y=sinx  (0xnπ,n=1,2,)y=\sin x\ \ (0\leq x\leq n\pi,n=1,2,\dots)xx轴围成的区域为AAAAyy轴旋转得到旋转体体积为SnS_n.

(Ⅰ)求SnS_n

(Ⅱ)求极限limn[S1n3+13+S2n3+23++Snn3+n3]\lim_{n\to\infty}\left[\frac{S_1}{n^3+1^3}+\frac{S_2}{n^3+2^3}+\dots+\frac{S_n}{n^3+n^3}\right]

有:Sn=2π0nπxsinxdxS_n=2\pi\int_0^{n\pi}x|\sin x|\text d x,令x=nπtx=n\pi-t,得:

Sn=2π0nπ(nπt)sintdt=2nπ20nπsintdtSnSn=n2π20πsintdt=2n2π2S_n=2\pi\int_0^{n\pi}(n\pi-t)|\sin t|\text dt\\=2n\pi^2\int_0^{n\pi}|\sin t|\text dt-S_n\\S_n=n^2\pi^2\int_0^{\pi}\sin t\text dt=2n^2\pi^2

于是:

limnk=1nSkn3+k3=2π2limn1nk=1n11+(kn)3=2π20111+x3dx\lim_{n\to\infty}\sum_{k=1}^n\frac{S_k}{n^3+k^3}\\=2\pi^2\lim_{n\to\infty}\frac 1 n\sum_{k=1}^n\frac 1 {1+\left(\frac k n\right)^3}\\=2\pi^2\int_0^1\frac 1 {1+x^3}\text dx

一元函数积分学的物理运用

抽水做功问题:

dW=ρgxS(x)dx\mathrm dW=\rho gxS(x)\mathrm dx

空间解析几何

向量的乘积及其几何运用

向量的乘积

α,β\boldsymbol{\alpha,\beta} 为两个向量,不妨设其维数均为 3. 则这两个向量即为三维空间中的任意两个向量

我们以常用的右手坐标系为准,将所得OxyzO-xyz 坐标系的三个两两垂直的基向量 记为i,j,k\boldsymbol{i,j,k}.
从而可用 坐标 的形式表示这两个向量。下面给出其矩阵向量表示和坐标向量表示的对应关系:

α=[a1,a2,a3]T=a1i+b2j+a3k=(a1,a2,a3)β=[b1,b2,b3]T=b1i+b2j+b3k=(b1,b2,b3)\begin{aligned}\boldsymbol{\alpha}&=[a_1,a_2,a_3]^T=a_1\boldsymbol i+b_2\boldsymbol j+a_3\boldsymbol k=(a_1,a_2,a_3)\\\boldsymbol{\beta}&=[b_1,b_2,b_3]^T=b_1\boldsymbol i+b_2\boldsymbol j+b_3\boldsymbol k=(b_1,b_2,b_3)\\\end{aligned}

可见,向量xx 是通过对应维度的基向量线性组合而成。

  • 数量积

向量的数量积也称为向量的内积或点积,记为αβ\boldsymbol\alpha·\boldsymbol\beta(α,β)(\boldsymbol\alpha,\boldsymbol\beta). 规定:

αβ=αβcosθ\boldsymbol\alpha·\boldsymbol\beta=|\boldsymbol\alpha||\boldsymbol\beta|\cos \theta

其中,θ\theta 是两个向量之间的夹角;
x|x| 表示对向量xx 取模,也即是xx二范数,也是xx 与坐标原点OO欧氏距离

利用坐标计算的话,还有αβ=αTβ=aibi\boldsymbol\alpha·\boldsymbol\beta=\boldsymbol\alpha^T\boldsymbol\beta=\sum a_ib_i. 即对应元素相乘的累加。

几何运用上,我们可以通过数量积来判断两个向量α,β\boldsymbol\alpha,\boldsymbol\beta 的夹角θ:=<α,β>\theta:=<\boldsymbol\alpha,\boldsymbol\beta> 的大小:

cosθ=αβα  β=a1b1+a2b2+a3b3a12+a22+a32b12+b22+b32\cos\theta=\frac{\boldsymbol\alpha·\boldsymbol\beta}{|\boldsymbol\alpha|\;|\boldsymbol\beta|}=\frac{a_1b_1+a_2b_2+a_3b_3}{\sqrt{a_1^2+a_2^2+a_3^2}·\sqrt{b_1^2+b_2^2+b_3^2}}

特别地,有:αβ=0αβ\boldsymbol\alpha·\boldsymbol\beta = 0\Leftrightarrow \boldsymbol\alpha\perp\boldsymbol\beta
根据三角形的几何关系,还可以导出另一个右移结论:
α\boldsymbol\alphaβ\boldsymbol\beta 上的投影

Prβα=αββ=a1b1+a2b2+a3b3b12+b22+b32\Pr_{\boldsymbol\beta}\boldsymbol\alpha=\frac{\boldsymbol\alpha·\boldsymbol\beta}{|\boldsymbol\beta|}=\frac{a_1b_1+a_2b_2+a_3b_3}{\sqrt{b_1^2+b_2^2+b_3^2}}

  • 向量积

向量的向量积也称为向量的外积或叉积,记为α×β\boldsymbol\alpha\times\boldsymbol\beta. 规定:

α×β=ijka1a2a3b1b2b3\boldsymbol\alpha\times\boldsymbol\beta=\begin{vmatrix}\boldsymbol i&\boldsymbol j&\boldsymbol k\\a_1&a_2&a_3\\b_1&b_2&b_3\end{vmatrix}

求解上式一般对第一行按行展开即可。
其结果还可以进一步写成:

ijka1a2a3b1b2b3=(a2b3a3b2)i(a1b3a3b1)j+(a1b2a2b1)k=[a2b3a3b2,a1b3a3b1,a1b2a2b1]T=[0a3a2a30a1a2a10][b1b2b3]\begin{aligned}\begin{vmatrix}\boldsymbol i&\boldsymbol j&\boldsymbol k\\a_1&a_2&a_3\\b_1&b_2&b_3\end{vmatrix}&=(a_2b_3-a_3b_2)\boldsymbol i-(a_1b_3-a_3b_1)\boldsymbol j+(a_1b_2-a_2b_1)\boldsymbol k\\&=[a_2b_3-a_3b_2,a_1b_3-a_3b_1,a_1b_2-a_2b_1]^T\\\\&=\begin{bmatrix}0&-a_3&a_2\\a_3&0&-a_1\\-a_2&a_1&0\end{bmatrix}·\begin{bmatrix}b_1\\b_2\\b_3\end{bmatrix}\end{aligned}

将最后结果中左边的三阶矩阵称为 向量α\boldsymbol\alpha叉乘反对称矩阵.

与内积结果是一个不同,外积结果仍为一个向量
此外,还有外积的模满足:

α×β=αβsinθ|\boldsymbol\alpha\times\boldsymbol\beta|=|\boldsymbol\alpha||\boldsymbol\beta|\sin\theta

通过外积得到的向量也具有其空间几何上的意义
n=α×β\boldsymbol n = \boldsymbol\alpha\times\boldsymbol\beta 是与α,β\boldsymbol\alpha,\boldsymbol\beta 都垂直,且方向沿着α,β\boldsymbol\alpha,\boldsymbol\beta 满足右手定则的向量。
换句话说,n\boldsymbol n 向量 是 由α,β\boldsymbol\alpha,\boldsymbol\beta 所确定的平面Π\Pi法向量

更进一步,我们可以根据外积的模的特性,用于计算点到直线的距离

d=P0P1×SSd=\frac{|\overrightarrow{P_0P_1}\times\boldsymbol S|}{|\boldsymbol S|}

其中,S\boldsymbol S 是直线的方向向量P0P1\overrightarrow{P_0P_1} 是由目标点P0P_0 指向直线上的一点P1P_1 构成的向量。

向量积的运算不满足交换律,但是有:α×β=β×α\boldsymbol\alpha\times\boldsymbol\beta=-\boldsymbol\beta\times\boldsymbol\alpha. 负号在向量中表示方向相反。

不难发现,根据向量积的几何意义,交换次序之后得到的就是与原法向量方向相反的另一个法向量,因此结果自然只相差一个负号。

  • 混合积

规定(α×β)γ=(α,β,γ)(\boldsymbol\alpha\times\boldsymbol\beta)·\boldsymbol\gamma=(\boldsymbol\alpha,\boldsymbol\beta,\boldsymbol\gamma) 是三个向量α,β,γ\boldsymbol\alpha,\boldsymbol\beta,\boldsymbol\gamma混合积

(α,β,γ)=a1b1r1a2b2r2a3b3r3(\boldsymbol\alpha,\boldsymbol\beta,\boldsymbol\gamma)=\begin{vmatrix}a_1&b_1&r_1\\a_2&b_2&r_2\\a_3&b_3&r_3\end{vmatrix}

显然,在数值上混合积的大小就是列向量组(α,β,γ)(\boldsymbol\alpha,\boldsymbol\beta,\boldsymbol\gamma)行列式的值。因此,其各种特性与行列式的性质相同,可移步至本文章线性代数部分进行探究。

在几何运用上,混合积可用于判断三个向量是否共面共面可推出混合积为0.
即:

α,β,γ共面(α,β,γ)=0\boldsymbol\alpha,\boldsymbol\beta,\boldsymbol\gamma\text{共面}\Leftrightarrow(\boldsymbol\alpha,\boldsymbol\beta,\boldsymbol\gamma)=0

不仅如此,混合积的绝对值还可以用来表示以α,β,γ\boldsymbol\alpha,\boldsymbol\beta,\boldsymbol\gamma 为棱的六面体的体积!

平面、直线、平面束与公垂线

平面方程

名称方程形式说明
一般式Ax+By+Cz+D=0Ax+By+Cz+D=0n=(A,B,C)\boldsymbol n=(A,B,C) 是平面法向量
点法式A(xx0)+B(yy0)+C(zz0)=0A(x-x_0)+B(y-y_0)+C(z-z_0)=0法向量同上,且(x0,y0,z0)(x_0,y_0,z_0)是平面上一点
截距式xa+yb+zc=1\begin{aligned}\frac{x}{a}+\frac{y}{b}+\frac{z}{c}=1\end{aligned}a,b,ca,b,c均不为0,分别是在x,y,zx,y,z轴上的截距

利用混合积的知识,还可以通过如下方式确定一个平面:

  1. 已知平面Π\varPi 上一个点M0(x0,y0,z0)M_0(x_0,y_0,z_0)
  2. 已知与Π\varPi 平行的两个不共线(线性无关)的向量S1=(X1,Y1,Z1),S2=(X2,Y2,Z2)S_1=(X_1,Y_1,Z_1),S_2=(X_2,Y_2,Z_2)
  3. 如果 (2) 不成立,但是还知道平面Π\varPi 上另外两个点,则可以通过两两组合得到条件 (2)

那么,平面上任意一个点P(x,y,z)P(x,y,z)M0M_0 构成的向量与已知的两个向量必共面,从而可确定平面的方程为:

xx0yy0zz0X1Y1Z1X2Y2Z2=0\begin{vmatrix}x-x_0&y-y_0&z-z_0\\X_1&Y_1&Z_1\\X_2&Y_2&Z_2\end{vmatrix}=0

直线方程

名称方程形式说明
一般式/交面式{A1x+B1y+C1z+D1=0A2x+B2y+C2z+D2=0\begin{cases}A_1x+B_1y+C_1z+D_1=0\\A_2x+B_2y+C_2z+D_2=0\end{cases}直线由两个不平行的平面的交线构成,其方向向量可通过外积求得
点向式/对称式xx0l=yy0m=zz0n\begin{aligned}\frac{x-x_0}{l}=\frac{y-y_0}{m}=\frac{z-z_0}{n}\end{aligned}(x0,y0,z0)(x_0,y_0,z_0)是直线上一点,直线方向向量为S=(l,m,n)\boldsymbol{S}=(l,m,n)
参数式{x=x0+lty=y0+mtz=z0+nt\begin{cases}x=x_0+lt\\y=y_0+mt\\z=z_0+nt\end{cases}令点向式=t可得

平面束方程

过一条直线L:{A1x+B1y+C1z+D1=0A2x+B2y+C2z+D2=0L:\begin{cases}A_1x+B_1y+C_1z+D_1=0\\A_2x+B_2y+C_2z+D_2=0\end{cases} 的全体平面构成一个平面束。在根据各种几何关系求解一个平面方程时,往往可以通过平面束方程来建立联系,从而求出具体的平面方程。

平面束方程表示为:λ(A1x+B1y+C1z+D1)+μ(A2x+B2y+C2z+D2)=0\lambda(A_1x+B_1y+C_1z+D_1)+\mu(A_2x+B_2y+C_2z+D_2)=0
其中,λ,μ\lambda,\mu 是不同时为零的任意常数.

公垂线方程 🦄🦄

设直线L1,L2L_1,L_2 分别过 点P1,P2P_1,P_2 ,其方向向量分别是S1,S2S_1,S_2.
则,如果两直线异面,那么它们所过定点构成的向量P1P2\overrightarrow{P_1P_2} 与方向向量S1,S2S_1,S_2 三者必不公面,即有:

直线L1,L2异面(P1P2,S1,S2)=0\text{直线}L_1,L_2\text{异面}\Leftrightarrow (\overrightarrow{P_1P_2},S_1,S_2)=0

根据混合积的几何运用,(P1P2,S1,S2)|(\overrightarrow{P_1P_2},S_1,S_2)| 即是以三者为棱构成的六面体体积。根据几何关系可以判断出,该六面体如果以S1×S2|S_1\times S_2| 为底面积,那么它的高就是公垂线的长度dd 。于是:

d=(P1P2,S1,S2)S1×S2d=\frac{|(\overrightarrow{P_1P_2},S_1,S_2)|}{|S_1\times S_2|}

此外,公垂线LL 的方向向量S=S1×S2S=S_1\times S_2 ,根据几何关系,有S,S1S,S_1 构成的平面Π1\varPi_1S,S2S,S_2 构成的平面Π2\varPi_2 的交线正是公垂线!
从而根据平面方程的混合积构造法,可得到公垂线LL 的方程如下:

L:{(PP1,S1,S)=0(PP2,S2,S)=0L:\begin{cases}(\overrightarrow{PP_1},S_1,S)=0\\\\(\overrightarrow{PP_2},S_2,S)=0\end{cases}

空间曲面与曲线

常见曲面

二次曲面标准型

旋转曲面

  • 空间曲线表示

若两曲面F(x,y,z)=0,G(x,y,z)=0F(x,y,z)=0,G(x,y,z)=0 交成曲线LL,则曲线LL 可通过这两个曲面方程构成的方程组表示,这种表示法称为曲线的一般式或交面式(与直线方程是类似的)。

L:{F(x,y,z)=0G(x,y,z)=0L:\begin{cases}F(x,y,z)=0\\G(x,y,z)=0\end{cases}

  • 空间曲线的投影

将空间曲线LL 的一般式消去zz,得到f(x,y)=0f(x,y)=0,则LLxOyxOy 平面上的投影曲线方程可表示为:

{f(x,y)=0z=0\begin{cases}f(x,y)=0\\z=0\end{cases}

  • 旋转曲面 🦄🦄

如果将空间曲线Γ:{F(x,y,z)=0G(x,y,z)=0\varGamma:\begin{cases}F(x,y,z)=0\\G(x,y,z)=0\end{cases} 绕一直线L:xx0l=yy0m=zz0nL:\begin{aligned}\frac{x-x_0}{l}=\frac{y-y_0}{m}=\frac{z-z_0}{n}\end{aligned} 旋转而形成一个曲面,那么空间曲线Γ\Gamma ,即 母线 上一点M1(x1,y1,z1)M_1(x_1,y_1,z_1) 和 旋转曲面上任意一点P(x,y,z)P(x,y,z) 满足:

  1. M1PS\overrightarrow{M_1P}\perp SSS 是直线LL 的方向向量;
  2. M0P=M0M1\left|\overrightarrow{M_0P}\right|=\left|\overrightarrow{M_0M_1}\right|M0(x0,y0,z0)M_0(x_0,y_0,z_0) 是直线LL 上一点.

即是旋转曲面上的任意一点P(x,y,z)P(x,y,z) 都满足上述约束条件。因此我们可以通过联立构建方程组,将全体满足上式的(x,y,z)(x,y,z) “锁定”出来。
于是,旋转曲面的方程为:

{l(xx1)+m(yy1)+n(zz1)=0(xx0)2+(yy0)2+(zz0)2=(x1x0)2+(y1y0)2+(z1z0)2F(x1,y1,z1)=0G(x1,y1,z1)=0\begin{cases}l(x-x_1)+m(y-y_1)+n(z-z_1)=0\\(x-x_0)^2+(y-y_0)^2+(z-z_0)^2=(x_1-x_0)^2+(y_1-y_0)^2+(z_1-z_0)^2\\F(x_1,y_1,z_1)=0\\G(x_1,y_1,z_1)=0\end{cases}

特别地,当我们只考察 曲线绕zz 轴旋转 的旋转曲面时,其方向向量不妨取z=(0,0,1)\overrightarrow z=(0,0,1),直线上的参考点不妨取坐标原点O(0,0,0)O(0,0,0). 于是有:

{M1Pz=0OP=OM1{z=z1x2+y2+z2=x12+y12+z12\begin{cases}\overrightarrow{M_1P}· \overrightarrow z=0\\\left|\overrightarrow{OP}\right|=\left|\overrightarrow{OM_1}\right|\end{cases}\Rightarrow\begin{cases}z=z_1\\x^2+y^2+z^2=x_1^2+y_1^2+z_1^2\end{cases}

继而得到:x2+y2=f(z)x^2+y^2=f(z).
其中,f(z)=x12(z)+y12(z)f(z)=x_1^2(z)+y_1^2(z) 是由方程组{F(x1,y1,z1=z)=0G(x1,y1,z1=z)=0\begin{cases}F(x_1,y_1,z_1=z)=0\\G(x_1,y_1,z_1=z)=0\end{cases} 消去x1,y1x_1,y_1 得到的。

切平面与法线

注:该部分内容涉及到多元函数的偏导相关内容,具体定义和含义参见下一节.

空间曲面S:F(x,y,z)=0S:F(x,y,z)=0,其中P0(x0,y0,z0)P_0(x_0,y_0,z_0) 是曲面上一点,则SSP0P_0 点的切平面方程 为:

Fx(xx0)+Fy(yy0)+Fz(zz0)=0\frac{\partial F}{\partial x}(x-x_0)+\frac{\partial F}{\partial y}(y-y_0)+\frac{\partial F}{\partial z}(z-z_0)=0

曲面在该点处的法线方程xx0Fx=yy0Fy=zz0Fz\begin{aligned}\frac{x-x_0}{F'_x}=\frac{y-y_0}{F'_y}=\frac{z-z_0}{F'_z}\end{aligned}.
其中,要求函数F(x,y,z)F(x,y,z) 在点P0P_0 处的偏导数不全为零,即:

Fx2+Fy2+Fz20{F'_x}^2+{F'_y}^2+{F'_z}^2\neq 0

切线与法平面

如果是由参数方程确定了空间曲线Γ:{x=x(t)y=y(t)z=z(t),αtβ\varGamma:\begin{cases}x=x(t)\\y=y(t)\\z=z(t)\end{cases},\alpha\leq t\leq\beta,则有:

切线方程:xx0x(t0)=yy0y(t0)=zz0z(t0)切平面方程:x(t0)(xx0)+y(t0)(yy0)+z(t0)(zz0)=0\begin{aligned}\text{切线方程:}\quad& \frac{x-x_0}{x'(t_0)}=\frac{y-y_0}{y'(t_0)}=\frac{z-z_0}{z'(t_0)}\\\\\text{切平面方程:}\quad& x'(t_0)(x-x_0)+y'(t_0)(y-y_0)+z'(t_0)(z-z_0)=0\end{aligned}

如果空间曲线Γ:{F(x,y,z)=0G(x,y,z)=0\varGamma:\begin{cases}F(x,y,z)=0\\G(x,y,z)=0\end{cases} ,则其切线方程为:

{Fx(xx0)+Fy(yy0)+Fz(zz0)=0Gx(xx0)+Gy(yy0)+Gz(zz0)=0orxx0(F,G)(y,z)P0=yy0(F,G)(z,x)P0=zz0(F,G)(x,y)P0\begin{aligned}&\begin{cases}\frac{\partial F}{\partial x}(x-x_0)+\frac{\partial F}{\partial y}(y-y_0)+\frac{\partial F}{\partial z}(z-z_0)=0\\\\\frac{\partial G}{\partial x}(x-x_0)+\frac{\partial G}{\partial y}(y-y_0)+\frac{\partial G}{\partial z}(z-z_0)=0\end{cases}\\\\&\quad \quad or\\\\&\frac{x-x_0}{\frac{\partial (F,G)}{(y,z)}\bigg|_{P_0}}=\frac{y-y_0}{\frac{\partial (F,G)}{(z,x)}\bigg|_{P_0}}=\frac{z-z_0}{\frac{\partial (F,G)}{(x,y)}\bigg|_{P_0}}\end{aligned}

这是很显然的,因为曲线由两个曲面相交得来,那么两个曲面的切平面相交即可得到曲线的切线。而我们可以利用前面的知识,将 交面式方程 转化为 点向式方程

这是从数值上来理解,而接下来我们将解释点向式方程中这个方向向量,即切向量的由来:

切向量的确定逻辑是:曲面F=0F=0P0P_0 点的梯度方向 与 曲面G=0G=0 在同一点处的梯度方向 叉乘 得来。即切向量是一个同时垂直于两个曲面梯度方向的方向。

F(P0)×G(P0)=ijkFxFyFzGxGyGz=((F,G)(y,z),(F,G)(z,x),(F,G)(x,y))where (F,G)(u,v)=FuFvGuGv\begin{aligned}\nabla F(P_0)\times\nabla G(P_0)&=\begin{vmatrix}i&j&k\\\\\frac{\partial F}{\partial x}&\frac{\partial F}{\partial y}&\frac{\partial F}{\partial z}\\\\\frac{\partial G}{\partial x}&\frac{\partial G}{\partial y}&\frac{\partial G}{\partial z}\end{vmatrix}=\left(\frac{\partial (F,G)}{(y,z)},\frac{\partial (F,G)}{(z,x)},\frac{\partial (F,G)}{(x,y)}\right)\\\\\text{where }&\frac{\partial (F,G)}{(u,v)}=\begin{vmatrix}\frac{\partial F}{\partial u}&\frac{\partial F}{\partial v}\\\\\frac{\partial G}{\partial u}&\frac{\partial G}{\partial v}\end{vmatrix}\end{aligned}

以上公式为了书写方便,其中的偏导数都是指在指定点P0P_0 处的偏导数(是具体数值),而非偏导函数。

多元函数微分学

极限与连续性

二元函数的极限与连续

🦄 设函数f(x,y)f(x,y) 在区域DD 内有定义,点P0(x0,y0)DP_0(x_0,y_0)\in D 或为边界上的一点。
ε>0,δ>0\forall \varepsilon\gt0,\exists \delta\gt0 在点P(x,y)P(x,y) 满足0<PP0<δ0\lt|PP_0|\lt\delta 时都有:

f(x,y)A<ε|f(x,y)-A|\lt\varepsilon

则有limPP0f(P)=A\lim\limits_{P\to P_0}f(P)=Alim(x,y)(x0,y0)f(x,y)=A\lim\limits_{(x,y)\to(x_0,y_0)}f(x,y)=Alimxx0yy0f(x,y)=A\lim\limits_{x\to x_0\atop y\to y_0}f(x,y)=A

多元函数极限存在 要求 函数变量从任何方向,以任意路径趋于指定点时,其极限均存在且值相等
这是显然的,一元函数在一维空间中的路径只有±\pm ,而二元函数在平面中的路径更多,如果找到一条极限不存在,即可推出整个极限不存在。

lim(x,y)(x0,y0)f(x,y)=f(x0,y0)\lim\limits_{(x,y)\to(x_0,y_0)}f(x,y)=f(x_0,y_0) 则称f(x,y)f(x,y)P(x0,y0)P(x_0,y_0)连续
如果在DD 上的每一个点都连续,称f(x,y)f(x,y)DD 上连续。

  • 若函数在P0P_0 处不连续,则该点称为f(x,y)f(x,y)间断点,对于二元函数,间断点的全体集合可以是一条曲线,我们不讨论间断点的分类.
  • 与一元函数类似,在有界闭区域连续的二元函数,也有:有界性、最大值最小值定理、介值定理
偏导数、可微性与全微分

二元函数偏导数

偏导数的本质是一元函数的导数.

函数f(x,y)f(x,y) 在点(x0,y0)(x_0,y_0) 处对xx偏导数 有定义:

f(x0,y0)x=df(x,y0)dxx=x0=limΔx0f(x0+Δx,y0)f(x0,y0)Δx\frac{\partial f(x_0,y_0)}{\partial x}=\frac{\mathrm df(x,y_0)}{\mathrm dx}\bigg|_{x=x_0}=\lim_{\Delta x\to0}\frac{f(x_0+\Delta x,y_0)-f(x_0,y_0)}{\Delta x}

对于点(x0,y0)(x_0,y_0)处的混合偏导,有:

2fxy(x0,y0)=ddy(f(x,y)xx=x0)y=y0\frac{\partial^2f}{\partial x\partial y}\bigg|_{(x_0,y_0)}=\frac{\mathrm d}{\mathrm dy}\left(\\\frac{\partial f(x,y)}{\partial x}\bigg|_{x=x_0}\right)\bigg|_{y=y_0}

👉🏻若z=f(x,y)z=f(x,y)DD 上每一个点都有偏导数,那么其偏导数也是关于x,yx,y 的函数,我们称作 偏导函数,仍然简称为 偏导数。
👉🏻求对xx 的偏导函数时,只需暂时将yy 看作常数,用一元函数求导方式处理即可。

定理fxy(x,y)f''_{xy}(x,y)fyx(x,y)f''_{yx}(x,y) 在点(x,y)(x,y)连续,则它们的值与求导先后次序无关。即:fxy(x,y)=fyx(x,y)f''_{xy}(x,y)=f''_{yx}(x,y).

事实上二元函数的偏导数记号颇多,如fx(x,y),    f1(x,y),    zx    ...f'_x(x,y),\;\;f'_1(x,y),\;\;z'_x\;\;... 本文不再做过多介绍~

二元函数全微分

x=f(x,y)x=f(x,y) 在点P0(x0,y0)P_0(x_0,y_0)某领域内有定义,分别给x,yx,y 以增量Δx,Δy\Delta x,\Delta y,得到

Δz=f(x0+Δx,y0+Δy)f(x0,y0)\Delta z=f(x_0+\Delta x,y_0+\Delta y)-f(x_0,y_0)

称为z=f(x,y)z=f(x,y)P0P_0 处的全增量。如果全增量可以写成:

Δz=AΔx+BΔy+o(ρ)(ρ0)\Delta z=A\Delta x+B\Delta y+o(\rho)\quad(\rho\to0)

其中,A,BA,BΔx,Δy\Delta x,\Delta y 都无关,ρ=(Δx)2+(Δy)2\rho=\sqrt{(\Delta x)^2+(\Delta y)^2}.
则称z=f(x,y)z=f(x,y)P0P_0可微,将下式称为函数z=f(x,y)z=f(x,y)全微分

dz=AΔx+BΔy\mathrm dz=A\Delta x+B\Delta y

  • 可微的必要条件 若函数z=fz=fP0P_0 处可微,则fx,fyf'_x,f'_y 均存在,且有dz=fxΔx+fyΔy\mathrm dz=f'_x\Delta x+f'_y\Delta y.
  • 可微的充分条件 若函数z=fz=fP0P_0 处的两个一阶偏导数fx,fyf'_x,f'_y 都连续,则z=fz=f 在该点可微.

最终得到如下关系图:
偏导数、连续性、全微分的关系图

🦄证明可微的充分条件

证明极限不存在与求极限

证明极限不存在

对于二元函数的极限问题,由于其极限存在的条件苛刻,因此从路径选取的角度上就可以将一类函数的极限判断为不存在。

  1. 按某条路径求极限,其结果发散
  2. 按多条不同路径求极限,得到的结果不唯一
  3. 以某个参数kk 的函数假定x,yx,y 的关系,从这个路径上求极限,结果与kk 有关(这是第2条的一般化表示)

分母可以为零型

对于分式二元函数,如果分母存在等于0的情况,我们可以构造一种类似于泰勒展开的路径证明其极限不存在

如:limx0y0xyx+y\begin{aligned}\lim_{x\to0\atop y\to0}\frac{xy}{x+y}\end{aligned} ,分母为x+yx+y,当且仅当y=xy=-x 时,函数没有定义.

此时,我们可以取y=x+o(x)y=-x+o(x)y=x+x3y=-x+x^3 这样的路径求极限。不难得出此时极限不存在,进而得以证明二元函数的极限不存在。

特别注意的是,如果极限是x0+y0+x\to0^+\atop y\to0^+ ,则极限存在且为0.
这是因为:

limx0+y0+xyx+y=limx0+y0+xx+yy\begin{aligned}\lim_{x\to0^+\atop y\to0^+}\frac{xy}{x+y}=\lim_{x\to0^+\atop y\to0^+}\frac{x}{x+y}\cdot y\end{aligned}

前者有界,后者无穷小,因此结果是0.
此时不能取y=x+o(x)y=-x+o(x) 的原因是,当x0+x\to0^+ 时,x+o(x)0-x+o(x)\to0^-, 这与y0+y\to0^+ 矛盾.

这里用到一个结论,无穷小的正负只与其最低阶的符号有关,如xx50+(x0+)x-x^5\to0^+\quad(x\to0^+).

】 不同教材对 极限的定义 有所不同,例如有些定义中,指出因为分母为0无定义,所以立即推极限不存在。

而一般情况下我们并不采用这种定义方法,当一个极限问题给出时,我们默认在定义域内讨论其极限的存在性。

极坐标变换

👉🏻对于分式二元函数,如果函数形式可以通过极坐标变换化为较为简单的形式,那么可以加以利用从而证明其极限不存在

如:limx0y0xyx2+y2\begin{aligned}\lim_{x\to0\atop y\to0}\frac{xy}{x^2+y^2}\end{aligned}

作极坐标变换,则原式化为limr0r2sinθcosθr2=sinθcosθ\begin{aligned}\lim_{r\to0}\frac{r^2\sin\theta\cos\theta}{r^2}=\sin\theta\cos\theta\end{aligned} ,极限值随θ\theta 变化而变化,因此极限不存在。

更进一步地,我们可以得到如下结论:

limx0y0xaybx2+y2=limr0ra+b2sinaθcosbθ={0,a+b2>0∄,a+b20\begin{aligned}\lim_{x\to0\atop y\to0}\frac{x^ay^b}{x^2+y^2}&=\lim_{r\to0}r^{a+b-2}\sin^a\theta\cos^b\theta\\&=\begin{cases}0,&a+b-2\gt0\\\not\exists&,a+b-2\leq0\end{cases}\end{aligned}

】这里能够得出a+b2>0a+b-2\gt0 时极限为0,是因为 此处r,θr,\theta 是可分离的,且关于rr 的函数是一个无穷小量(rr非直线路径的情况下也是θ\theta 的函数),关于θ\theta 的函数是有界的,因此可以这样处理。并不是所有题都能这样做。

下面讨论一个进阶结论,及其结果:

limx0y0xaybxm+yn=?\begin{aligned}\lim_{x\to0\atop y\to0}\frac{x^ay^b}{x^m+y^n}=?\end{aligned}

其中,m,nm,n 为正偶数. 则:

L=limx0y0xaybxm+yn=limx0y0xayb(xm2)2+yn2)2=v=yn2u=xm2limu0v0u2amv2bnu2+v2={0,am+bn>1∄,am+bn1\begin{aligned}L=&\lim_{x\to0\atop y\to0}\frac{x^ay^b}{x^m+y^n}\\\\=&\lim_{x\to0\atop y\to0}\frac{x^ay^b}{(x^{\frac{m}{2}})^2+y^{\frac{n}{2}})^2}\xlongequal[v=y^{\frac{n}{2}}]{u=x^{\frac{m}{2}}}\lim_{u\to0\atop v\to0}\frac{u^{\frac{2a}{m}}\cdot v^{\frac{2b}{n}}}{u^2+v^2}\\\\=&\begin{cases}0,&\frac{a}{m}+\frac{b}{n}\gt1\\\not\exists&,\frac{a}{m}+\frac{b}{n}\leq1\end{cases}\end{aligned}

参考:二元函数极限常见题型|▷考研竞赛凯哥

👉🏻其他常用的选择路径y=kxy=kxy2=xy^2=x 等.
👉🏻对于分式二元函数,一般情况下,我们可以对分母通过重要不等式a2+b22aba^2+b^2\geq 2ab 对极限放缩从而利用夹逼准则。而取等号的条件是a=ba=b,所以我们还可以利用取等号的条件选择路径.

常用二元函数极限求法

👉🏻 均值不等式放缩法

👉🏻 极坐标变换法

经典反例与做题技巧

连续与偏导的经典反例

♾️ 二元函数连续但不一定可偏导 如 圆锥面:z=x2+y2z=\sqrt{x^2+y^2}

♾️ 二元函数可偏导但不一定连续

f(x,y)={xyx2+y2,(x,y)00,(x,y)=0\begin{aligned}f(x,y)=\begin{cases}\begin{aligned}\frac{xy}{x^2+y^2}\end{aligned},&(x,y)\neq0\\0,&(x,y)=0\end{cases}\end{aligned}

该函数在(0,0)(0,0) 处可偏导且偏导数为0,但是并不连续。证明如下:

fx(0,0)=limx0f(x,0)f(0,0)x=0f'_x(0,0)=\lim_{x\to0}\frac{f(x,0)-f(0,0)}{x}=0

而取路径y=kxy=kx 路径判断连续性:

limx0y0f(x,y)=limx0kx2(1+k2)x2\lim_{x\to0\atop y\to0}f(x,y)=\lim_{x\to0}\frac{kx^2}{(1+k^2)x^2}

kk 有关,因此极限不存在,即在(0,0)(0,0) 处不连续.

事实上,此处的极限还可以利用上面推导出来的结论,由12+12=10\frac 1 2+\frac 1 2=1\not\gt0 从而得出极限不存在

♾️ 混合偏导不一定相等

f(x,y)={xyx2y2x2+y2,(x,y)00,(x,y)=0\begin{aligned}f(x,y)=\begin{cases}\begin{aligned}xy\frac{x^2-y^2}{x^2+y^2}\end{aligned},&(x,y)\neq0\\0,&(x,y)=0\end{cases}\end{aligned}

计算(x,y)(0,0)(x,y)\neq(0,0) 时的一阶偏导:

fx(x,y)=y[x2y2x2+y2+x2x(x2+y2)2x(x2y2)(x2+y2)2]f'_x(x,y)=y\left[\frac{x^2-y^2}{x^2+y^2}+x\frac{2x(x^2+y^2)-2x(x^2-y^2)}{(x^2+y^2)^2}\right]

x=0x=0fx(0,y)=yf'_x(0,y)=-y,同理得fy(x,0)=xf'_y(x,0)=x.

根据混合偏导的定义,得:

fxy(0,0)=limy0fx(0,y)fx(0,0)y=limy0y0y=1f_{xy}''(0,0)=\lim_{y\to0}\frac{f'_x(0,y)-f_x(0,0)}{y}=\lim_{y\to0}\frac{-y-0}{y}=-1

fyx(0,0)=1f''_{yx}(0,0)=1 二者并不相等!!

♾️ 可微不一定偏导数连续

f(x,y)={(x2+y2)sin1x2+y2,(x,y)00,(x,y)=0\begin{aligned}f(x,y)=\begin{cases}\begin{aligned}(x^2+y^2)\sin\frac{1}{x^2+y^2}\end{aligned},&(x,y)\neq0\\0,&(x,y)=0\end{cases}\end{aligned}

计算点的偏导数

在求一个点的偏导数的时候,因为偏导数本质是一元函数的导数,因此我们可以直接将非变量的值直接代入再进行求解。见下例:

f(x,y)=x2+(y1)arctanyxf(x,y)=x^2+(y-1)\arctan\sqrt{\frac y x},求fx(2,1),  fy(2,1)f'_x(2,1),\;f'_y(2,1) 的值

fx(2,1)=df(x,1)dxx=2=(x2)x=2=4\begin{aligned}f'_x(2,1)=\frac{\mathrm df(x,1)}{\mathrm dx}\bigg|_{x=2}=(x^2)'\bigg|_{x=2}=4\end{aligned} 。这比先求出fx(x,y)f'_x(x,y) 再代值更加快捷!

fy(2,1)=df(2,y)dxy=1=limy1f(2,y)f(2,1)y1=limy1arctany2=π4\begin{aligned}f'_y(2,1)=\frac{\mathrm df(2,y)}{\mathrm dx}\bigg|_{y=1}=\lim_{y\to1}\frac{f(2,y)-f(2,1)}{y-1}=\lim_{y\to1}\arctan\sqrt{\frac y 2}=\frac \pi 4\end{aligned}
比起用上一种方法(直接令x=2x=2 再求yy 的偏导再然后再代值),这个方法直接回归定义,反而还极大化简了式子!

二元函数在某点处可微

  1. 利用可微的必要条件可以证明不可微,即证明f(x,y)f(x,y) 在点P0P_0不连续或不可偏导
  2. 利用可微的充分条件可以证明可微,即证明fx(x,y),fy(x,y)f'_x(x,y),f'_y(x,y)P0P_0连续
  3. 🦄利用可微的定义
    要证df\exists\mathrm df 只需证Δf=fxΔx+fyΔy+o(ρ)\Delta f=f'_x\Delta x+f'_y\Delta y+o(\rho) 成立。其中,ρ=Δx2+Δy2\rho=\sqrt{\Delta x^2+\Delta y^2}.
    进而可以分为以下步骤进行:
      1. 考察fxP0,fyP0\begin{aligned}f'_x\big|_{P_0},f'_y\big|_{P_0}\end{aligned} 是否存在,如果不存在就不可微,否则继续;
      1. 考察ΔffxΔx+fyΔy\Delta f-f'_x\Delta x+f'_y\Delta y 是不是ρ\rho高阶无穷小,成立即可微,否则不可微。

第二步的考察方法通常为计算下列极限是否成立

limΔx0Δy0ΔffxΔx+fyΔyρ=0\lim\limits_{\Delta x\to0\atop\Delta y\to0}\frac{\Delta f-f'_x\Delta x+f'_y\Delta y}{\rho}=0

基于可微定义的类题

(2007年)二元函数f(x,y)f(x,y) 在点(0,0)(0,0) 可微的充分条件 是:

  1. limx0y0[f(x,y)f(0,0)]=0\lim\limits_{x\to0\atop y\to0}[f(x,y)-f(0,0)]=0
  2. limx0y0f(x,0)f(0,0)x=0\lim\limits_{x\to0\atop y\to0}\frac{f(x,0)-f(0,0)}{x}=0
  3. limx0y0f(x,y)f(0,0)x2+y2=0\lim\limits_{x\to0\atop y\to0}\frac{f(x,y)-f(0,0)}{\sqrt{x^2+y^2}}=0
  4. limx0[fx(x,0)f(0,0)]=0\lim_{x\to0}[f_x'(x,0)-f(0,0)]=0limy0[fy(0,y)f(0,0)]=0\lim_{y\to0}[f_y'(0,y)-f(0,0)]=0
答案

选项(1)只能得出连续,推不出可微
选项(2)只能得出偏导数存在且为0推不出kew
选项(4)表示的意思是在固定某一变量为0时,一个偏导数在(0,0)(0,0)处连续,这与偏导数连续是不一样的。偏导数是关于x,yx,y 的二元函数,偏导数连续的符号表示应为:

limx0y0fx(x,y)=fx(0,0)\lim\limits_{x\to0\atop y\to0}f'_x(x,y)=f'_x(0,0)

选项(3)是正确的.

回顾全微分的定义,要求的是存在常数A,BA,B 使得Δz=AΔx+BΔy+o(ρ)\Delta z=A\Delta x+B\Delta y+o(\rho),因此选项(3)给出的是A=B=0A=B=0 的情况.

(2012年)设连续二元函数z=f(x,y)z=f(x,y) 满足limx0y1f(x,y)2x+y2x2+(y1)2=0\begin{aligned}\lim\limits_{x\to0\atop y\to1}\frac{f(x,y)-2x+y-2}{\sqrt{x^2+(y-1)^2}}=0\end{aligned},求dz(0,1)\mathrm dz\bigg|_{(0,1)}.

答案

由于极限存在且为0,不妨直接取分子恒为0,即f(x,y)=2xy+2f(x,y)=2x-y+2,从而秒杀本题.

下面从微分定义介绍完整解法.
由函数连续且极限存在,易知分子的极限存在且为0,即limx0y1f(x,y)2x+y2=0\lim\limits_{x\to0\atop y\to1}f(x,y)-2x+y-2=0.
从而f(0,1)=1f(0,1)=1.

于是:

limx0y1f(x,y)2x+y2x2+(y1)2=limx0y1[f(x,y)f(0,1)]2x(1)(y1)x2+(y1)2=0\begin{aligned}\lim\limits_{x\to0\atop y\to1}\frac{f(x,y)-2x+y-2}{\sqrt{x^2+(y-1)^2}}=\lim\limits_{x\to0\atop y\to1}\frac{[f(x,y)-f(0,1)]-2x-(-1)(y-1)}{\sqrt{x^2+(y-1)^2}}=0\end{aligned}

从而有fx(0,1)=2,fy(0,1)=1f'_x(0,1)=2,f'_y(0,1)=-1,因此:

dz(0,1)=2dxdy\mathrm dz\bigg|_{(0,1)}=2\mathrm dx-\mathrm dy

(2012年)若f(x,y)f(x,y)(0,0)(0,0) 处连续,则下列选项正确的是:

  1. limx0y0f(x,y)x+y\begin{aligned}\lim\limits_{x\to0\atop y\to0}\frac{f(x,y)}{|x|+|y|}\end{aligned} 存在,则f(x,y)f(x,y)(0,0)(0,0) 处可微
  2. limx0y0f(x,y)x2+y2\begin{aligned}\lim\limits_{x\to0\atop y\to0}\frac{f(x,y)}{x^2+y^2}\end{aligned} 存在,则f(x,y)f(x,y)(0,0)(0,0) 处可微
  3. f(x,y)f(x,y)(0,0)(0,0) 处可微,则limx0y0f(x,y)x+y\begin{aligned}\lim\limits_{x\to0\atop y\to0}\frac{f(x,y)}{|x|+|y|}\end{aligned} 存在
  4. f(x,y)f(x,y)(0,0)(0,0) 处可微,则limx0y0f(x,y)x2+y2\begin{aligned}\lim\limits_{x\to0\atop y\to0}\frac{f(x,y)}{x^2+y^2}\end{aligned} 存在
答案

对于选项(3)(4),不妨令f(x,y)1f(x,y)\equiv1,显然极限不存在,结论不成立
对于选项(1)(2),可以利用如下的无穷小极限性质:
limx0f(x)g(x)\begin{aligned}\lim\limits_{x\to0}\frac{f(x)}{g(x)}\end{aligned} 存在,则f(x)=o(g(x))f(x)=o(g(x))f(x)=Θ(g(x))f(x)=\Theta(g(x)).

因此,当分母变得比g(x)g(x) 低阶时,假设变为了h(x)h(x),则显然对应极限仍然存在,且为0;反之,当分母阶数升高时,则极限可能不存在。
总而言之,固定分子时,分母越低阶越好。

本题要求证明可微,而根据前提条件,知f(0,0)=0f(0,0)=0,从而凑微分定义时,需要以x2+y2\sqrt{x^2+y^2} 做分母的极限为0。
根据上述性质,如果选项(2)的极限存在,那么选项(2)的分母降阶为我们所需的x2+y2\sqrt{x^2+y^2} 时,极限一定存在其为0。

因此本题选(2)

多元函数为常数的条件

  1. (x,y)Dif fx=0,fy=0   then f=C\forall (x,y)\in D\quad \text{if }f'_x=0,f'_y=0\;\text{ then }f=C
  2. (x,y)Dif df(x,y)=0   then f=C\forall(x,y)\in D\quad\text{if }\mathrm df(x,y)=0\;\text{ then }f=C
  3. If fx=0, then f(x,y)=φ(y)\text{If } f'_x=0,\text{ then }f(x,y)=\varphi(y)

已知偏导数求二元函数

该类问题可以转化为求一元函数微分方程的问题,即:

zx=f(x)z(x,y)=f(x)dx+φ(y)\frac{\partial z}{\partial x}=f(x)\Rightarrow z(x,y)=\int f(x)\mathrm dx+\varphi(y)

类似地可得到偏yy 的结果。再结合其他已知条件即可解出原函数z=z(x,y)z=z(x,y).

在考研中通常不会单独考察这类指知识,而是结合后面的极值问题一并考察。不仅如此,有可能涉及变量替换而混淆做题者的思维,下面给出一例题:

【例题】 已知二元可微函数f(u,v)f(u,v) 满足f(u,v)uf(u,v)v=2(uv)e(u+v)\begin{aligned}\frac{\partial f(u,v)}{\partial u}-\frac{\partial f(u,v)}{\partial v}=2(u-v)e^{-(u+v)}\end{aligned} ,且f(u,0)=u2euf(u,0)=u^2e^{-u} ,设u=x,v=yxu=x,v=y-x 求:

  1. f(x,yx)x\begin{aligned}\frac{\partial f(x,y-x)}{\partial x}\end{aligned} 的表达式;
  2. f(u,v)f(u,v) 的表达式,并求其极值.

【分析】本问题并没有直接给出f(u,v)f(u,v) 的偏导函数,而是给出两个偏导函数之间的恒等关系,似乎无法入手。并且做变量替换的目的也并不清晰。好在第一问的做法比较清晰,我们可以走一步看一步。

f(x,yx)x=f(u,v)uux+f(u,v)vvx=f(u,v)uf(u,v)v=2(uv)e(u+v)=2(2xy)ey\begin{aligned}\frac{\partial f(x,y-x)}{\partial x}&=\frac{\partial f(u,v)}{\partial u}·\frac{\partial u}{\partial x}+\frac{\partial f(u,v)}{\partial v}·\frac{\partial v}{\partial x}\\&=\frac{\partial f(u,v)}{\partial u}-\frac{\partial f(u,v)}{\partial v}\\&=2(u-v)e^{-(u+v)}\\&=2(2x-y)e^{-y}\end{aligned}

自此,我们似乎发现问题的题眼,虽然已知偏导数的恒等关系不能反求原函数,但是如果做适当的变量替换,我们可以将其变换为其中一个变量的偏导数。于是,我们就可以用第一问的结果做积分反求原函数了!

f(x,yx)=2(2xy)eydx=2x(xy)ey+φ(y)f(x,y-x)=2\int(2x-y)e^{-y}\mathrm dx=2x(x-y)e^{-y}+\varphi(y)

这时我们发现,还有已知条件没用上,想来必然是用于给我们确定φ(y)\varphi(y) 的。

f(u,0)=u2euf(x,0)=φ(y)=x2exf(u,0)=u^2e^{-u}\Rightarrow f(x,0)=\varphi(y)=x^2e^{-x}

最终,我们再将变量代回,即可得到原函数。

f(u,v)=(u2+v2)e(u+v){fu=(u2+v22u)e(u+v)=0fu=(u2+v22v)e(u+v)=0  (0,0)  ,  (1,1)  为驻点ACB2>0   where (u,v)=(0,0)\begin{aligned}f(u,v)&=(u^2+v^2)e^{-(u+v)}\\\\\begin{cases}f_u'=(u^2+v^2-2u)e^{-(u+v)}=0\\f_u'=(u^2+v^2-2v)e^{-(u+v)}=0\end{cases}&\Rightarrow\;(0,0)\;,\;(1,1)\;\text{为驻点}\\\\

AC-B^2\gt0&;\text{ where }(u,v)=(0,0)
\end{aligned}

最后得到极值。

多元复合函数求导与泰勒公式

多元复合函数求导法则

全导数u=f(x,y,z)u=f(x,y,z) 在对应点(x=x(t),y=y(t),z=z(t))(x=x(t),y=y(t),z=z(t)) 处可微。在复合函数uutt 可导,有:

dudt=uxdxdt+uydydt+uzdzdt\frac{\mathrm du}{\mathrm dt}=\frac{\partial u}{\partial x}\frac{\mathrm dx}{\mathrm dt}+\frac{\partial u}{\partial y}\frac{\mathrm dy}{\mathrm dt}+\frac{\partial u}{\partial z}\frac{\mathrm dz}{\mathrm dt}

链式法则u=φ(x,y),v=ψ(x,y)u=\varphi(x,y),v=\psi(x,y) 在点(x,y)(x,y) 处有偏导数,而对于z=f(u,v)z=f(u,v) 在对应点可微。则zzx,yx,y 的偏导数为:

zx=fuux+fvvxzy=fuuy+fvvy\begin{aligned}\frac{\partial z}{\partial x}=\frac{\partial f}{\partial u}\frac{\partial u}{\partial x}+\frac{\partial f}{\partial v}\frac{\partial v}{\partial x}\\\\\frac{\partial z}{\partial y}=\frac{\partial f}{\partial u}\frac{\partial u}{\partial y}+\frac{\partial f}{\partial v}\frac{\partial v}{\partial y}\end{aligned}

一阶全微分形式不变性 对于z=f(u=φ(x,y),v=ψ(x,y))z=f(u=\varphi(x,y),v=\psi(x,y)) 有:

dz=zxdx+zydy\mathrm dz=\frac{\partial z}{\partial x}\mathrm dx+\frac{\partial z}{\partial y}\mathrm dy

这是显然的,因为对于可微二元函数z=g(x,y)z=g(x,y) 其全微分就是如此。而如果将该结果与上面的 链式法则 结论联立,即可得到:

dz=fu(uxdx+uydy)+fv(vxdx+vydy)=fudu+fvdv\begin{aligned}\mathrm dz&=\frac{\partial f}{\partial u}\left(\frac{\partial u}{\partial x}\mathrm dx+\frac{\partial u}{\partial y}\mathrm dy\right)+\frac{\partial f}{\partial v}\left(\frac{\partial v}{\partial x}\mathrm dx+\frac{\partial v}{\partial y}\mathrm dy\right)\\\\&=\frac{\partial f}{\partial u}\mathrm du+\frac{\partial f}{\partial v}\mathrm dv\end{aligned}

隐函数存在定理

隐函数存在定理 如果二元方程F(x,y)=0F(x,y)=0 满足以下条件:

  1. 函数F(x,y)F(x,y) 在点(x0,y0)(x_0,y_0) 某领域有连续偏导数
  2. F(x0,y0)=0F(x_0,y_0)=0
  3. F(x0,y0)0F'(x_0,y_0)\neq 0
    则方程F(x,y)=0F(x,y)=0 在点(x0,y0)(x_0,y_0) 某领域恒能唯一确定一个连续函数y=y(x)y=y(x),满足y0=y(x0)y_0=y(x_0),并且有连续导数,且:

y=dydx=FxFyy'=\frac{\mathrm dy}{\mathrm dx}=-\frac{F'_x}{F'_y}

该结论可推广到三元函数,用方程F(x,y,z)=0F(x,y,z)=0 确定一个函数z=z(x,y)z=z(x,y)

zx=FxFzzy=FyFz\frac{\partial z}{\partial x}=-\frac{F'_x}{F'_z}\quad\frac{\partial z}{\partial y}=-\frac{F'_y}{F'_z}

更进一步地,如果有{F(x,y,u,v)=0G(x,y,u,v)=0\begin{cases}F(x,y,u,v)=0\\G(x,y,u,v)=0\end{cases} 确定的隐函数u=u(x,y),v=v(x,y)u=u(x,y),v=v(x,y) 则可以通过两边分别对xxyy 求偏导,得到方程组(这里仅以对xx 为例):

{Fx+Fuux+Fvvx=0Gx+Guux+Gvvx=0ux=1JFxFvGxGv,ux=1JFuFxGuGxwhere J=FuFvGuGv\begin{aligned}&\begin{cases}F'_x+F'_u\frac{\partial u}{\partial x}+F'_v\frac{\partial v}{\partial x}=0\\G'_x+G'_u\frac{\partial u}{\partial x}+G'_v\frac{\partial v}{\partial x}=0\\\end{cases}\\\\\Rightarrow&\quad \frac{\partial u}{\partial x}=-\frac 1 J\begin{vmatrix}F'_x&F'_v\\G'_x&G'_v\end{vmatrix},\quad \frac{\partial u}{\partial x}=-\frac 1 J\begin{vmatrix}F'_u&F'_x\\G'_u&G'_x\end{vmatrix}\\\\&\text{where } J=\begin{vmatrix}F'_u&F'_v\\G'_u&G'_v\end{vmatrix}\end{aligned}

该过程利用了线性代数中的克莱姆法则,其中的JJ 又称 雅可比(Jacobian)行列式.

关于隐函数问题的求解,重在理解其求解思路(如两边求偏导),而不是硬套公式。

计算前,首先要明确函数关系:哪些是自变量哪些是因变量. 一般来说,有几个方程就确定几个因变量。
明确变量之间的关系后,推荐使用两边求全微分再搭配链式法则的方法求偏导

还有一类题型要求判断通过关于x,yx,y 的恒等式判断是否可以据此确定具有连续导数的隐函数。
此时,我们通常降恒等式移项,构造F(x,y)=0F(x,y)=0 ,通过验证dy/dxdy/dx 对应的Fy0F'_y\neq0 来判断是否存在隐函数y=y(x)y=y(x) 。同理,验证dx/dydx/dy 对应的分母Fx0F'_x\neq0 判断存在x=x(y)x=x(y) 与否。

例题{u=f(xut,yut,zut)g(x,y,z)=0\begin{cases}u=f(x-ut,y-ut,z-ut)\\g(x,y,z)=0\end{cases}ux,uy\frac{\partial u}{\partial x},\frac{\partial u}{\partial y}

答案

:根据题意,此处应该把x,yx,y 视为自变量,uu 为因变量。再由g(x,y,z)=0g(x,y,z)=0zz 应该是x,yx,y 的隐函数。目前未知数是x,y,z,u,tx,y,z,u,t 方程个数为 2 ,于是可确定tt 也是自变量。

对两个方程求全微分

du=f1d(xut)+f2d(yut)+f3d(zut)=f1(dxtduudt)+f2(dytduudt)+f3(dztduudt)dg=0=g1dx+g2dy+g3dz\begin{aligned}\mathrm du&=f'_1\mathrm d(x-ut)+f'_2\mathrm d(y-ut)+f'_3\mathrm d(z-ut)\\&=f'_1(\mathrm dx-t\mathrm du-u\mathrm dt)+f'_2(\mathrm dy-t\mathrm du-u\mathrm dt)+f'_3(\mathrm dz-t\mathrm du-u\mathrm dt)\\\\\mathrm dg&=0=g'_1\mathrm dx+g'_2\mathrm dy+g'_3\mathrm dz\end{aligned}

从而,有

[1+t(f1+f2+f3)]du=f1dx+f2dy+f3dzu(f1+f2+f3)dtdz=(g1dx+g2dy)/g3\begin{aligned}\left[1+t(f'_1+f'_2+f'_3)\right]\mathrm du&=f'_1\mathrm dx+f'_2\mathrm dy+f'_3\mathrm dz-u(f'_1+f'_2+f'_3)\mathrm dt\\\\\mathrm dz&=-(g'_1\mathrm dx+g'_2\mathrm dy)/g'_3\end{aligned}

联立两式,消去dz\mathrm dz 项,取题目所需的ux,uy\frac{\partial u}{\partial x},\frac{\partial u}{\partial y} . 得:

ux=f1g3f3g1g3[1+t(f1+f2+f3)],uy=f2g3f3g2g3[1+t(f1+f2+f3)]\frac{\partial u}{\partial x}=\frac{f'_1g'_3-f'_3g'_1}{g'_3\left[1+t(f'_1+f'_2+f'_3)\right]},\frac{\partial u}{\partial y}=\frac{f'_2g'_3-f'_3g'_2}{g'_3\left[1+t(f'_1+f'_2+f'_3)\right]}

二元函数的泰勒公式

z=f(x,y)z=f(x,y) 在点P0(x0,y0)P_0(x_0,y_0) 的某领域U(P0)U(P_0) 有直到nn连续偏导数,则:

f(x0+Δx,y0+Δy)=f(x0,y0)+(xΔx+yΔy)f(x0,y0)+12!(xΔx+yΔy)2f(x0,y0)++1n!(xΔx+yΔy)nf(x0,y0)+Rn+1\begin{aligned}f(x_0+\Delta x,y_0+\Delta y)&=f(x_0,y_0)\\&+\left(\frac\partial {\partial x}\Delta x+\frac\partial {\partial y}\Delta y\right)f(x_0,y_0)\\&+\frac 1 {2!}\left(\frac\partial {\partial x}\Delta x+\frac\partial {\partial y}\Delta y\right)^2f(x_0,y_0)\\&+\cdots\\&+\frac 1 {n!}\left(\frac\partial {\partial x}\Delta x+\frac\partial {\partial y}\Delta y\right)^nf(x_0,y_0)\\&+R_{n+1}\end{aligned}

其中的Rn+1R_{n+1} 为余项,称Rn+1=o(ρn+1)R_{n+1}=o(\rho^{n+1}) 称为 佩亚诺余项
另外,式中还有:

(xΔx+yΔy)n=k=1nCnknf(x0,y0)xkynkΔxkΔynk\left(\frac\partial {\partial x}\Delta x+\frac\partial {\partial y}\Delta y\right)^n=\sum\limits_{k=1}^{n}C_n^k\frac{\partial^nf(x_0,y_0)}{\partial x^k\partial y^{n-k}}\Delta x^k\Delta y^{n-k}

该记号针对偏微分时有其特殊含义,应区别于普通的幂次。

※ 多元k次齐次函数的性质及运用

定义nn 元函数f(x1,x2,...,xn)f(x_1,x_2,...,x_n) 满足f(tx1,tx2,...,txn)=tkf(x1,x2,...,xn)f(tx_1,tx_2,...,tx_n)=t^kf(x_1,x_2,...,x_n)
就称函数ffnnkk 次齐次函数.

例如:f(x,y)=x2+y2f(tx,ty)=t2(x2+y2)=t2f(x,y)f(x,y)=x^2+y^2\Rightarrow f(tx,ty)=t^2(x^2+y^2)=t^2f(x,y)

k次齐次函数的性质

【性质1】 二元kk 次齐次函数f(x,y)f(x,y) 可以化为z=xkg(yx)\begin{aligned}z=x^kg(\frac{y}{x})\end{aligned} 的形式,反之亦然.

证明:因为f(tx,ty)=tkf(x,y)\begin{aligned}f(tx,ty)=t^kf(x,y)\end{aligned},令t=1x\begin{aligned}t=\frac{1}{x}\end{aligned} 则:

f(tx,ty)=f(1,yx)=1xkf(x,y)f(x,y)=xkf(1,yx)\begin{aligned}f(tx,ty)=f(1,\frac{y}{x})=\frac{1}{x^k}f(x,y)\\\\\Rightarrow\quad f(x,y)=x^kf(1,\frac{y}{x})\end{aligned}

f(1,yx):=g(yx)\begin{aligned}f(1,\frac y x):=g(\frac y x)\end{aligned},则必要性证明完毕.

下面证明充分性,若f(x,y)=xkf(1,yx)\begin{aligned}f(x,y)=x^kf(1,\frac{y}{x})\end{aligned},则:

f(tx,ty)=(tx)kg(tytx)=tk[xkg(yx)]=tkf(x,y)f(tx,ty)=(tx)^kg(\frac{ty}{tx})=t^k[x^kg(\frac y x)]=t^kf(x,y)

证毕.


【性质2】kk 次齐次函数f(x,y,z)f(x,y,z) 一阶连续可偏导,求证xfx+yfy+zfz=kf(x,y,z)\begin{aligned}x\frac{\partial f}{\partial x}+y\frac{\partial f}{\partial y}+z\frac{\partial f}{\partial z}=kf(x,y,z)\end{aligned},反之亦然.

证明:已知f(tx,ty,tz)=tkf(x,y,z)f(tx,ty,tz)=t^kf(x,y,z),两边对tt 求导,得:

df(tx,ty,tz)dt=xf1+yf2+zf2d[tkf(x,y,z)]dt=ktk1f\begin{aligned}\frac{\mathrm df(tx,ty,tz)}{\mathrm dt}&=xf_1'+yf_2'+zf_2'\\\frac{\mathrm d[t^kf(x,y,z)]}{\mathrm dt}&=kt^{k-1}f\end{aligned}

从而:

xf(tx,ty,tz)(tx)+yf(tx,ty,tz)(ty)+zf(tx,ty,tz)(tz)=ktk1f(x,y,z)\begin{aligned}x\frac{\partial f(tx,ty,tz)}{\partial(tx)}+y\frac{\partial f(tx,ty,tz)}{\partial(ty)}+z\frac{\partial f(tx,ty,tz)}{\partial(tz)}=kt^{k-1}f(x,y,z)\end{aligned}

t=1t=1 则证明必要性.

下面证明充分性.
已知xfx+yfy+zfz=kf(x,y,z)\begin{aligned}x\frac{\partial f}{\partial x}+y\frac{\partial f}{\partial y}+z\frac{\partial f}{\partial z}=kf(x,y,z)\end{aligned},做变量替换:{x=txy=tyz=tz\begin{cases}x=tx\\y=ty\\z=tz\end{cases} 得:

txf(tx,ty,tz)(tx)+tyf(tx,ty,tz)(ty)+tzf(tx,ty,tz)(tz)=kf(tx,ty,tz)\begin{aligned}tx\frac{\partial f(tx,ty,tz)}{\partial(tx)}+ty\frac{\partial f(tx,ty,tz)}{\partial(ty)}+tz\frac{\partial f(tx,ty,tz)}{\partial(tz)}=kf(tx,ty,tz)\end{aligned}

显然左式是f(tx,ty,tz)f(tx,ty,tz)tt 求导再乘tt ,即:

tdf(tx,ty,tz)dt=kf(tx,ty,tz)\begin{aligned}t\frac{\mathrm df(tx,ty,tz)}{\mathrm dt}=kf(tx,ty,tz)\end{aligned}

解此微分方程:

df(tx,ty,tz)f(tx,ty,tz)=kdttlnf(tx,ty,tz)=lntk+Cf(tx,ty,tz)=Ctk(C=±eC)\begin{aligned}\frac{\mathrm df(tx,ty,tz)}{f(tx,ty,tz)}&=k\frac{\mathrm dt}{t}\\\ln f(tx,ty,tz)&=\ln t^k+C'\\f(tx,ty,tz)&=Ct^k\quad(C=\pm e^{C'})\end{aligned}

t=1t=1 解出C=f(x,y,z)C=f(x,y,z),从而有:f(tx,ty,tz)=tkf(x,y,z)f(tx,ty,tz)=t^kf(x,y,z),证毕.

此结论可拓展到nn 元函数.
x=[x1,x2,...,xn]T\boldsymbol x=[x_1,x_2,...,x_n]^T,一个nnkk 次齐次函数f(x)f(\boldsymbol x) 满足:

i=1nxifxi=kf(x)\sum_{i=1}^n x_i\frac{\partial f}{\partial x_i}=kf(\boldsymbol x)

该定理被称作 欧拉定理 !!

【常用技巧】 对一个二元kk 次齐次函数f(x,y)f(x,y) 做极坐标变换{x=rcosθy=rsinθ\begin{cases}x=r\cos\theta\\y=r\sin\theta\end{cases},可利用齐次函数的性质进行化简解题. 此时有:

f(x,y)=f(rcosθ,rsinθ)=rkf(cosθ,sinθ)f(x,y)=f(r\cos\theta,r\sin\theta)=r^kf(\cos\theta,\sin\theta)

例如:设f(x,y)f(x,y) 是定义在区域D={(x,y)1x1,1y1}D=\{(x,y)\big|-1\leq x\leq1,-1\leq y\leq1\} 上的有界函数,且为kk 次齐次函数,证明limx0y0f(x,y)=0\lim\limits_{x\to0\atop y\to0}f(x,y)=0.
此问题通过极坐标变换就可轻松证明。


【经典例题1】f(x,y)f(x,y) 是具有二阶连续偏导数的 1 次齐次函数,证明f11f22=(f12)2f''_{11}\cdot f''_{22}=(f''_{12})^2

解:由欧拉定理,xf1+yf2=fxf'_1+yf'_2=f,两边分别对x,yx,y 求偏导,得:

{f1+xf11+yf21=f1xf12+f2+yf22=f2{xf11=yf21yf22=xf12xyf11f22=xy(f12)2xy[f11f22(f12)2]=0\begin{aligned}&\begin{cases}f'_1+xf''_{11}+yf''_{21}=f'_1\\\\xf''_{12}+f'_2+yf''_{22}=f'_2\end{cases}\Rightarrow\begin{cases}xf''_{11}=-yf''_{21}\\\\yf''_{22}=-xf''_{12}\end{cases}\\\\&\Rightarrow xyf''_{11}\cdot f''_{22}=xy(f''_{12})^2\\\\&\Rightarrow xy[f''_{11}\cdot f''_{22}-(f''_{12})^2]=0\end{aligned}

xy0xy\neq0 时,显然结论成立,即f11f22=(f12)2f''_{11}\cdot f''_{22}=(f''_{12})^2
xy=0xy=0 时,由于f(x,y)f(x,y) 二阶偏导连续,所以此时也应该有f11f22(f12)2=0f''_{11}\cdot f''_{22}-(f''_{12})^2=0 ,否则xy=0xy=0 这个路径下二元函数f11f22(f12)2f''_{11}\cdot f''_{22}-(f''_{12})^2 存在间断点,与题设矛盾。

综上所述,f11f22=(f12)2f''_{11}\cdot f''_{22}=(f''_{12})^2.


【经典例题2】 证明函数f(x,y)f(x,y) 是二元零次齐次函数的充要条件是xfx+yfy=0\begin{aligned}x\frac{\partial f}{\partial x}+y\frac{\partial f}{\partial y}=0\end{aligned}

证明:由f(tx,ty)f(tx,ty)tt 求导再令t=1t=1 (即欧拉定理的推导过程)即可证明必要性。
下面尝试不用前面的推导,转而采用极坐标变换的方法来证明此问题的充分性。

fr=f(rcosθ,rsinθ)r=cosθfx+sinθfy\frac{\partial f}{\partial r}=\frac{\partial f(r\cos\theta,r\sin\theta)}{\partial r}=\cos\theta\frac{\partial f}{\partial x}+\sin\theta\frac{\partial f}{\partial y}

同时乘除rr 利用条件得到:

1r(rcosθfx+rsinθfy)=1r(xfx+yfy)=0\frac{1}{r}\left(r\cos\theta\frac{\partial f}{\partial x}+r\sin\theta\frac{\partial f}{\partial y}\right)=\frac{1}{r}\left(x\frac{\partial f}{\partial x}+y\frac{\partial f}{\partial y}\right)=0

即是fr=0\begin{aligned}\frac{\partial f}{\partial r}=0\end{aligned} ,说明ffrr 无关,仅与θ=arctan(yx)\theta=\arctan\left(\frac y x\right) 有关,即是f(x,y)=g(yx)f(x,y)=g(\frac y x).
通过齐次函数的【性质1】可得该函数为 零次齐次函数,证毕.


【经典例题3】 设在D={(x,y)y>0}D=\{(x,y)|y\gt0\}f(x,y)f(x,y) 的一阶偏导连续,t>0\forall t\gt0 都有f(tx,ty)=t2f(x,y)f(tx,ty)=t^{-2}f(x,y).
证明对DD 内任意分段光滑有向简单闭曲线LL 都有Lyfdxxfdy=0\begin{aligned}\oint_Lyfdx-xfdy=0\end{aligned}.

证明:要证Lyfdxxfdy=0\begin{aligned}\oint_Lyfdx-xfdy=0\end{aligned} 只需证D(QxPy)dxdy=0\begin{aligned}\iint_{D'}\left(\frac{\partial Q}{\partial x}-\frac{\partial P}{\partial y}\right)dxdy=0\end{aligned} (格林公式)
其中,DD'DD 上任意的曲面,并且被积函数连续因此要使等式成立,必有:

QxPy=0xfx+yfy=(2)f\frac{\partial Q}{\partial x}-\frac{\partial P}{\partial y}=0\rightarrow x\frac{\partial f}{\partial x}+y\frac{\partial f}{\partial y}=(-2)f

由欧拉定理可得此结论,从而原命题证毕.

根据充要条件,本问题的逆命题同样成立.

参考:多元微分重要考点-“k次齐次函数”|▷考研竞赛凯哥

多元函数的极值与最优化

多元函数的极值问题

如果P0\exists P_0 的某个领域U(P0,δ)U(P_0,\delta) 使得f(x,y)f(P0)f(x,y)\leq f(P_0) 那么称ffP0P_0 处取得极大值,极小值类似。称P0P_0极值点。(注:当且仅当在P0P_0 处取等)

  • 如果有fx=fy=0(x,y)f'_x=f'_y=0\bigg|_{(x,y)} 则称 点(x,y)(x,y)z=fz=f驻点
  • ff 在点P(x,y)P(x,y) 处有偏导,且在PP 处取得极值,那么fx=fy=0P=(x,y)f'_x=f'_y=0\bigg|_{P=(x,y)}
  • ff 在点P(x,y)P(x,y) 处偏导无定义,也有可能存在极值点,如z=x2+y2z=\sqrt{x^2+y^2}(0,0)(0,0) 处是极小值,同时也是最小值
  • ♾️具有偏导数的极值点一定是驻点,但驻点不一定是极值点。 反例如下:

经典反例:https://www.zhihu.com/question/20516102/answer/15353495


z=f(x,y)z=f(x,y) 在点(x0,y0)(x_0,y_0) 的某领域内连续且有一阶及二阶偏导数,且在该点处有:
fx=fy=0fxx=A,fxy=B,fyy=Cf'_x=f'_y=0\quad f''_{xx}=A,f''_{xy}=B,f''_{yy}=C,则:

  1. ACB2>0AC-B^2\gt0 时,在该点取得极值,且A>0A\gt0 取得极小值,A<0A\lt0 取得极大值;
  2. ACB2<0AC-B^2\lt0 时,该点不是极值点;
  3. ACB2=0AC-B^2=0 时,无法判断,需要另作讨论.

注:该结论为二元函数有极值点的充分条件,而非充要条件,并且只适用于二元函数!

♾️利用脱帽法判断极值点

🦄拉格朗日乘子法|Lagrange Multiplier

函数u=f(x,y,z)u=f(x,y,z) 在条件φ(x,y,z)=0,ψ(x,y,z)=0\varphi(x,y,z)=0,\psi(x,y,z)=0 的约束下的最值问题,可通过引入比例因子λ\lambda 构建拉格朗日函数,将有约束问题化为无约束问题。
即:

maxx,y,zf(x,y,z)s.t.{φ(x,y,z)=0ψ(x,y,z)=0maxx,y,z,λ,μL(x,y,z,λ,μ)L=f+λφ+μψ\begin{aligned}\max_{x,y,z}f(x,y,z)\\s.t. \begin{cases}\varphi(x,y,z)=0\\\psi(x,y,z)=0\end{cases}\end{aligned}\quad\Rightarrow\quad\begin{aligned}\max_{x,y,z,\lambda,\mu}{\cal{L}}(x,y,z,\lambda,\mu)\\{\cal{L}}=f+\lambda\varphi+\mu\psi\end{aligned}

而求解 Lagrangian Function 的最值问题,则可通过偏导数为零求得.

{Lx=fx+λφx+μψx=0Ly=fy+λφy+μψy=0Lz=fz+λφz+μψz=0Lλ=φ(x,y,z)=0Lμ=ψ(x,y,z)=0\begin{cases}\begin{aligned}\frac{\partial{\cal{L}}}{\partial x}&=\frac{\partial f}{\partial x}+\lambda\frac{\partial\varphi}{\partial x}+\mu\frac{\partial\psi}{\partial x}=0\\\frac{\partial{\cal{L}}}{\partial y}&=\frac{\partial f}{\partial y}+\lambda\frac{\partial\varphi}{\partial y}+\mu\frac{\partial\psi}{\partial y}=0\\\frac{\partial{\cal{L}}}{\partial z}&=\frac{\partial f}{\partial z}+\lambda\frac{\partial\varphi}{\partial z}+\mu\frac{\partial\psi}{\partial z}=0\\\frac{\partial{\cal{L}}}{\partial \lambda}&=\varphi(x,y,z)=0\\\frac{\partial{\cal{L}}}{\partial \mu}&=\psi(x,y,z)=0\end{aligned}\end{cases}

本内容涉及到了应用数学中最优化问题的重要分支,在数学建模、机器学习等方面有着重要运用。
本站也针对最优化问题发布了多篇文章(如KKT条件 等)。

🦄方向导数、梯度、散度与旋度

方向导数与梯度|Gradient

在给出方向导数的定义和公式前,我们先来思考偏导数的几何意义
我们可以类比一元函数,从一元函数的导数的意义出发,进而得到偏导数的几何意义如下:

来自:《高等数学》同济版

可见,偏导数描述了函数值沿着xx 轴 或yy 轴的变化情况。这事实上并不通用,因为现实中某个量变化的方向是有着多可能性的,即我们需要找寻沿着任意一个指定方向上函数值的变化情况

经过我们对 空间解析几何 的学习,我们可以考虑用一个方向向量表示一个空间的方向,在xOyxOy 坐标系下,可以以该方向相对x,yx,y 轴的夹角α,β\alpha,\beta 作为指标,用维度是2的向量表示之,不妨设为:l=(cosα,cosβ)\boldsymbol l=(\cos\alpha,\cos\beta).

更进一步地,我们考察函数z=f(x,y)z=f(x,y) 沿着该方向的变化率.
考虑增量Δx=tcosα,Δy=tcosβ\Delta x=t\cos\alpha,\Delta y=t\cos\beta,从而得到:

limt0f(x0+tcosα,y0+tcosβ)f(x0,y0)t\lim_{t\to0}\frac{f(x_0+t\cos\alpha,y_0+t\cos\beta)-f(x_0,y_0)}{t}

这个特征我们就称为方向导数,即函数ff 沿着l\boldsymbol l 方向上的“导数”,记为fl(x0,y0)\begin{aligned}\frac{\partial f}{\partial \boldsymbol l}\bigg|_{(x_0,y_0)}\end{aligned}.

如果函数ff(x0,y0)(x_0,y_0)可微的话,那么很幸运,我们还可以利用全微分对结果进行化简。

fl(x0,y0)=limt0fxtcosα+fytcosβt=fxcosα+fycosβ\begin{aligned}\frac{\partial f}{\partial \boldsymbol l}\bigg|_{(x_0,y_0)}=\lim_{t\to0}\frac{\frac{\partial f}{\partial x}·t\cos\alpha+\frac{\partial f}{\partial y}·t\cos\beta}{t}=\frac{\partial f}{\partial x}\cos\alpha+\frac{\partial f}{\partial y}\cos\beta\end{aligned}

该结论同样可以推广到多元.

【注】 如果方向导数在 360 度都存在也不能推出ff 关于x,yx,y 的偏导数存在!
只有方向导数关于x,yx,y 轴的正方向和负方向的方向导数均存在且互为相反数时,才能推出偏导数存在!
例如z=x2+y2z=\sqrt{x^2+y^2}(0,0)(0,0) 处的方向导数处处为1,但在此处偏导数不存在。


xOyxOy 平面下,我们之前定义的方向余弦,即cosα,cosβ\cos\alpha,\cos\beta 存在一定的取值,使得方向导数取得最值。即我们现在想要考察函数值变化最快的方向。

lmax=(α,β)=argmax(α,β)fl(x0,y0)\boldsymbol l_{\max}=(\alpha^*,\beta^*)=\arg\max_{(\alpha,\beta)}\frac{\partial f}{\partial \boldsymbol l}\bigg|_{(x_0,y_0)}

我们将方向导数写为向量形式,有:

fl(x0,y0)=[fxfy][cosαcosβ]\frac{\partial f}{\partial \boldsymbol l}\bigg|_{(x_0,y_0)}=\begin{bmatrix}\frac{\partial f}{\partial x}&\frac{\partial f}{\partial y}\end{bmatrix}·\begin{bmatrix}\cos\alpha\\\cos\beta\end{bmatrix}

我们就称 由偏导数组成的向量梯度(grad)。记为gradf,f\mathbf{grad}f,\nabla f. 有:

gradf(x,y)=f(x,y)=fxi+fyj=[fxfy]\mathbf{grad}f(x,y)=\nabla f(x,y)=\frac{\partial f}{\partial x}\boldsymbol i+\frac{\partial f}{\partial y}\boldsymbol j=\begin{bmatrix}\begin{aligned}\frac{\partial f}{\partial x}\\\\\frac{\partial f}{\partial y}\end{aligned}\end{bmatrix}

其中,=xi+yj\begin{aligned}\nabla=\frac{\partial}{\partial x}\boldsymbol i+\frac{\partial}{\partial y}\boldsymbol j\end{aligned} 称为 梯度算子,哈密顿算子,也叫 Nabla 算子.

显然,方向导数就是梯度与指定方向的内积大小,从而我们可以利用内积的知识更进一步地进行描述。

fl(x0,y0)=f(x0,y0)l=flcos<f,l>\frac{\partial f}{\partial \boldsymbol l}\bigg|_{(x_0,y_0)}=\nabla f(x_0,y_0)·\boldsymbol l=|\nabla f||\boldsymbol l|\cos<\nabla f,\boldsymbol l>

如果指定方向向量为单位向量,指定方向与梯度方向的夹角记为θ\theta ,那么方向导数的最值明显是在θ=0,θ=π\theta=0,\theta=\pi 时取得。进而我们得出结论:梯度方向是函数值变化最快的方向。其正方向是增加最快的方向,反方向则是减小最快的方向。

以上结论同样可推广到多维空间。
因为梯度有着如此含义,所以在考虑函数的最优化问题时,也有着 梯度下降法 这样迭代逼近函数最值的算法。关于梯度下降算法详见本站文章:

🦄散度与旋度|Divergence & Curl

我们看到,梯度其实是哈密顿算子对标量函数的点乘

而对于矢量函数F\boldsymbol F ,通常我们将其在空间中指示为 向量场,不妨设:

F(x,y,z)=P(x,y,z)i+Q(x,y,z)j+R(x,y,z)k\boldsymbol F(x,y,z)=P(x,y,z)\boldsymbol i+Q(x,y,z)\boldsymbol j+R(x,y,z)\boldsymbol k

对向量场与哈密顿算子求内积,则得到的是一个标量,称其为 散度(Divergence)

DivF=F=Px+Qy+Rz\mathrm{Div}\boldsymbol F=\nabla·\boldsymbol F=\frac{\partial P}{\partial x}+\frac{\partial Q}{\partial y}+\frac{\partial R}{\partial z}

散度是描述空气从周围汇合到某一处或从某一处散开来程度的量。它可用于表征空间各点矢量场发散的强弱程度 .
物理上,散度的意义是场的有源性

  • Divf>0\mathrm{Div} f \gt0,该点有散发通量的正源(发散源);
  • Divf<0\mathrm{Div} f \lt0,该点有吸收通量的负源(洞或汇);
  • Divf=0\mathrm{Div} f =0,该点无源。

高斯公式 为我们给出了 散度 与 通量 的关系:

ΩDivF  dV=SFn  dS\iint_{\Omega}\mathrm{Div}\boldsymbol F\;\mathrm dV=\iint_S\boldsymbol{F·n}\;\mathrm dS

通量是单位时间内通过某个曲面的量,散度是通量的强度


旋度(Curl, Rotation) 是哈密顿算子对矢量函数的外积,结果仍然是一个向量。
旋度是矢量分析中的一个矢量算子,可以表示三维矢量场对某一点附近的微元造成的旋转程度。该向量提供了向量场在这一点的旋转性质。

CurlF=RotF=×F=ijkxyzPQR\mathrm{Curl}\boldsymbol F=\mathrm{Rot}\boldsymbol F=\nabla\times\boldsymbol F=\begin{vmatrix}i&j&k\\\frac{\partial}{\partial x}&\frac{\partial}{\partial y}&\frac{\partial}{\partial z}\\P&Q&R\end{vmatrix}

斯托克斯公式 为我们给出了 旋度 与 环流量 的关系:

SRotFn  dS=ΓFτ  ds\iint_S\mathrm{Rot}\boldsymbol{F·n}\;\mathrm dS=\int_{\varGamma}\boldsymbol{F·\tau}\;\mathrm ds

环流量是单位时间内环绕的某个曲线的量,旋度是环流量强度


拉普拉斯算子|Laplace Operatornn 维欧几里得空间中的二阶微分算子,定义为 梯度的散度。

2f=f=[xyz][fxfyfz]=2fx2+2fy2+2fz2\nabla^2f=\nabla·\nabla f=\begin{bmatrix}\frac{\partial}{\partial x}&\frac{\partial}{\partial y}&\frac{\partial}{\partial z}\end{bmatrix}·\begin{bmatrix}\frac{\partial f}{\partial x}\\\frac{\partial f}{\partial y}\\\frac{\partial f}{\partial z}\end{bmatrix}=\frac{\partial^2 f}{\partial x^2}+\frac{\partial^2 f}{\partial y^2}+\frac{\partial^2 f}{\partial z^2}

详见本站文章:

多元函数积分学

各类多元函数积分的定义

多元函数积分的定义

二重积分

xOyxOy 平面的有界闭区域上一二元函数z=f(x,y)z=f(x,y) 的二重积分:

I=Df(x,y)dσ=limd0i=1nf(ξi,ηi)ΔσiI=\iint\limits_{D}f(x,y)\mathrm d\sigma=\lim_{d\to0}\sum_{i=1}^nf(\xi_i,\eta_i)\Delta\sigma_i

其中,d=maxidid=\max\limits_{i}d_idid_iDD 中一小区域Δσi\Delta\sigma_i 的直径,Δσi\Delta\sigma_i 是将区域DD 任意分割为nn 个区域后的第ii 个区域,同时亦以Δσi\Delta\sigma_i 表示该小区域的面积. 在直角坐标系中用平行于坐标轴的直线网划分Δσ\Delta\sigma 使得Δσi=ΔxjΔyk\Delta\sigma_i=\Delta x_j\Delta y_k 时,可用dxdy\mathrm dx\mathrm dy 代替上式中的dσ\mathrm d\sigma.

II 存在时,就称f(x,y)f(x,y)DD可积.
f(x,y)f(x,y)DD 上连续,则f(x,y)f(x,y)DD 上可积,即存在二重积分II.

定积分中的Δxi\Delta x_i 可正可负,下限大于或小于上限都可以,而二重积分中的Δσi\Delta\sigma_i 必须为正,因此将其化为累次积分时必须注意上下限!

几何意义f(x,y)0f(x,y)\geq 0 时,II 表示以区域DD 为底,曲面z=f(x,y)z=f(x,y) 为顶,侧面是以DD 的边界为准线,母线平行于zz 轴的柱面的曲顶柱体的体积
特别地,记底面区域DD 的面积为SDS_D ,则有:D1dσ=SD\begin{aligned}\iint_D1\mathrm d\sigma=S_D\end{aligned}.

三重积分

与二重积分类似,我们如下定义三重积分:
f(x,y,z)f(x,y,z) 在空间有界闭区域Ω\Omega 上有界,任意分割Ω\OmegannΔVi\Delta V_i (也用于表示体积)。则三重积分定义为在ΔVi\Delta V_i 上各取一点(ξi,ηi,ζi)(\xi_i,\eta_i,\zeta_i) 作和的极限:

J=Ωf(x,y,z)dV=limd0i=1nf(ξi,ηi,ζi)ΔViJ=\iiint\limits_{\Omega}f(x,y,z)\mathrm dV=\lim_{d\to0}\sum_{i=1}^nf(\xi_i,\eta_i,\zeta_i)\Delta V_i

其中,d=maxidid=\max\limits_{i}d_i,是ΔVi\Delta V_i 中直径的最大值,且极限值与ΔVi\Delta V_i 的分法,点(ξi,ηi,ζi)(\xi_i,\eta_i,\zeta_i) 的取法无关.

VΩV_{\Omega} 为空间闭区域Ω\Omega 的体积,则:Ω1dV=VΩ\begin{aligned}\iiint_{\Omega}1\mathrm dV=V_{\Omega}\end{aligned}.

对弧长的曲线积分

定义xOyxOy 平面上分段光滑曲线ll 上函数f(x,y)f(x,y)第一类曲线积分

lf(x,y)ds=limλ0i=1nf(ξi,ηi)Δsi where λ=maxi{Δsi}\int_lf(x,y)\mathrm ds=\lim_{\lambda\to0}\sum_{i=1}^nf(\xi_i,\eta_i)\Delta s_i\quad\text{ where } \lambda=\max_i\{\Delta s_i\}

其中,Δsi\Delta s_i 表示 第ii 个 小段的弧长,ff 称为被积函数,ll 称为积分弧段.

对坐标的曲线积分

定义F(x,y)={P(x,y),Q(x,y)}\boldsymbol F(x,y)=\{P(x,y),Q(x,y)\} 是定义在xOyxOy 平面上有向分段光滑线段ll 上的向量函数,则定义第二类曲线积分形式为:

lP(x,y)dx+Q(x,y)dy=lFds\int_lP(x,y)\mathrm dx+Q(x,y)\mathrm dy=\int_l\boldsymbol F·\mathrm ds

对面积的曲面积分

定义f(x,y,z)f(x,y,z)分段光滑曲面Σ\Sigma 上的第一类曲面积分的形式为:

Σf(x,y,z)dS\iint\limits_{\Sigma}f(x,y,z)\mathrm dS

注:曲面Σ:z=z(x,y),(x,y)D\varSigma:z=z(x,y),(x,y)\in D 光滑

即是曲面上各点有连续变化的切平面;

即是在DD 上有连续偏导数zx,zy\begin{aligned}\frac{\partial z}{\partial x},\frac{\partial z}{\partial y}\end{aligned}.

对坐标的曲面积分

定义F(x,y,z)={P(x,y,z),Q(x,y,z),R(x,y,z)}\boldsymbol F(x,y,z)=\{P(x,y,z),Q(x,y,z),R(x,y,z)\} 是定义在O-xyzO\text{-}xyz 空间上有向分片光滑曲面Σ\boldsymbol\Sigma 上的向量函数,则定义第二类曲面积分形式为:

ΣPdydz+Qdxdz+Rdxdy=ΣFdΣ\iint\limits_{\Sigma}P\mathrm dy\mathrm dz+Q\mathrm dx\mathrm dz+R\mathrm dx\mathrm dy=\iint\limits_{\Sigma}\boldsymbol F·\mathrm d\boldsymbol \Sigma

式中的dydz,dxdz,dxdy\mathrm dy\mathrm dz,\mathrm dx\mathrm dz,\mathrm dx\mathrm dy 分别表示有向曲面面积元素在三个坐标平面上的投影面积微元.

注:对双侧曲面,称取定法向量的曲面为有向曲面,通常由法向量的指向决定曲面的侧。

  1. 封闭曲面:外侧→法向量朝外;内测→法向量朝内
  2. 曲面z=z(x,y)z=z(x,y) :上侧→法向量与zz 轴成锐角;下侧→成钝角
两类曲线/曲面积分的关系

LabL_{\overset{\frown}{ab}} 为分段光滑曲线,则有:

LabPdx+Qdy=Lab(Pcosα+Qcosβ)ds\int_{L_{\overset{\frown}{ab}}}P\mathrm dx+Q\mathrm dy=\int_{L_{\overset{\frown}{ab}}}(P\cos\alpha+Q\cos\beta)\mathrm ds

式中,cosα,cosβ\cos\alpha,\cos\beta 是曲线弧LabL_{\overset{\frown}{ab}} 沿aabb 方向的切线的方向余弦.


Σ\Sigma 为分块光滑曲面,则有:

ΣPdydz+Qdxdz+Rdxdy=Σ(Pcosα+Qcosβ+Rcosγ)dS\iint_{\Sigma} P\mathrm dy\mathrm dz+Q\mathrm dx\mathrm dz+R\mathrm dx\mathrm dy=\iint_{\Sigma}(P\cos\alpha+Q\cos\beta+R\cos\gamma)\mathrm dS

式中,cosα,cosβ,cosγ\cos\alpha,\cos\beta,\cos\gamma 是曲面Σ\Sigma 在点(x,y,z)(x,y,z)法向量的方向余弦.


两类曲线/曲面积分的重要不同点就是第一类与方向无关。如果曲线L+,LL^+,L^- 是平面上同一条光滑曲线取相反方向,曲面S+,SS^+,S^- 是空间中同一块光滑曲面取相反侧,则有:

L+Pdx+Qdy=LPdx+QdyS+Pdydz+Qdxdz+Rdxdy=SPdydz+Qdxdz+Rdxdy\begin{aligned}\int_{L^+}Pdx+Qdy&=-\int_{L^-}Pdx+Qdy\\\iint_{S^+}Pdydz+Qdxdz+Rdxdy&=-\iint_{S^-}Pdydz+Qdxdz+Rdxdy\end{aligned}

多元函数积分的转化求解

多元函数积分化定积分

下面出现公式基本只涉及了在xOyxOy 平面的投影,实际处理中,可以灵活选取投影面,其核心思想不变!

二重积分化累次积分

直角坐标系下,如果被积区域DD 可以表示为如下方式:

表示方式图示
D:{φ1(x)yφ2(x)axbD:\begin{cases}\varphi_1(x)\leq y\leq\varphi_2(x)\\a\leq x\leq b\end{cases}
D:{ψ1(y)xψ2(y)cydD:\begin{cases}\psi_1(y)\leq x\leq\psi_2(y)\\c\leq y\leq d\end{cases}

那么,可以得出二重积分可分别转化为下面两种二次积分:

Df(x,y)dxdy=abdxφ1(x)φ2(x)f(x,y)dy(Case  1)=cddyψ1(y)ψ2(y)f(x,y)dx(Case  2)\begin{aligned}\iint_Df(x,y)\mathrm dx\mathrm dy&=\int_a^b\mathrm dx\int_{\varphi_1(x)}^{\varphi_2(x)}f(x,y)\mathrm dy\quad(Case\;1)\\\\&=\int_c^d\mathrm dy\int_{\psi_1(y)}^{\psi_2(y)}f(x,y)\mathrm dx\quad(Case\;2)\end{aligned}


极坐标系下,可通过极坐标变换x=rcosθ,y=rsinθx=r\cos\theta,y=r\sin\theta 将二重积分化为针对极坐标上一点(r,θ)(r,\theta) 的二重积分,通过微元思想得出dσ=rdrdθ\mathrm d\sigma=r·\mathrm dr\mathrm d\theta. 从而有:

Df(x,y)dσ=Df(rcosθ,rsinθ)rdrdθ\iint_Df(x,y)\mathrm d\sigma=\iint_Df(r\cos\theta,r\sin\theta)r\cdot\mathrm dr\mathrm d\theta

与直角坐标类似,如果被积区域DD 可以在极坐标系下利用r=r(θ)r=r(\theta)θ=θ(r)\theta=\theta(r) 的方式表示,那么可以进一步将其化为累次积分,此处不再赘述.

三重积分化累次积分

O-xyzO\text{-}xyz 空间坐标系下,若有界闭区域Ω\Omega 可以表示为如下方式:

表示名称图示求解方法
曲顶曲底柱形长条区域先一后二
截面已知区域先二后一
  • 曲顶曲底柱形长条区域

Ω={(x,y,z)z1(x,y)zz2(x,y),(x,y)Dxy}\varOmega=\{(x,y,z)|z_1(x,y)\leq z\leq z_2(x,y),(x,y)\in D_{xy}\}

其中,DxyD_{xy} 是 闭区域Ω\OmegaxOyxOy 平面上的投影区域.

于是,可先将x,yx,y 视作常数,对f(x,y,z)f(x,y,z)zz 上积分得到F(x,y)F(x,y),再将其作为被积函数求关于x,yx,y 的二重积分,用这种方法可以化简三重积分。即:

Ωf(x,y,z)dV=Ddxdyz1(x,y)z2(x,y)f(x,y,z)dz\iiint_{\Omega}f(x,y,z)\mathrm dV=\iint_D\mathrm dx\mathrm dy\int_{z_1(x,y)}^{z_2(x,y)}f(x,y,z)\mathrm dz

这种计算方法常被称作 先一后二法.

  • 截面已知区域

Ω={(x,y,z)z1zz2,(x,y)D(z)}\varOmega=\{(x,y,z)|z_1\leq z\leq z_2,(x,y)\in D(z)\}

即被积区域可表示为介于平面z=z1,z=z2(z1<z2)z=z_1,z=z_2\quad(z_1\lt z_2) 之间,且过zz 轴上区间[a,b][a,b] 中任一点zz 作垂直于zz 轴的平面截Ω\Omega 于平面D(z)D(z).

于是,可先将zz 视作常数,计算区域D(z)D(z) 的二重积分,再在zz 上计算定积分。即:

Ωf(x,y,z)dV=z1z2dzD(z)f(x,y,z)dxdy\iiint_{\Omega}f(x,y,z)\mathrm dV=\int_{z_1}^{z_2}\mathrm dz\iint_{D(z)}f(x,y,z)\mathrm dx\mathrm dy

特别地,当D(z)D(z) 的面积已知为S(z)S(z) ,被积函数只与zz 有关时,有:

Ωf(z)dV=z1z2f(z)S(z)dz\iiint_{\Omega}f(z)\mathrm dV=\int_{z_1}^{z_2}f(z)S(z)\mathrm dz

这种计算方法常被称作 先二后一法.


对于柱坐标变换x=rcosθ,y=rsinθ,z=zx=r\cos\theta,y=r\sin\theta,z=z,可将其视为被积空间区域在xOyxOy 平面上的投影面取极坐标变换的变换。

因此,无论是上面提到的先一后二,或是先二后一,都能把其中的“二”的部分转化为极坐标处理:

Ωf(x,y,z)dV=Ωf(rcosθ,rsinθ,z)rdrdθdz=Drdrdθz1(r,θ)z2(r,θ)fdz(Case  1)=abdθD(θ)frdrdz(Case  2)\begin{aligned}\iiint_\Omega f(x,y,z)\mathrm dV&=\iiint_{\Omega}f(r\cos\theta,r\sin\theta,z)r\cdot\mathrm dr\mathrm d\theta\mathrm dz\\\\&=\iint_Dr·\mathrm dr\mathrm d\theta\int_{z_1(r,\theta)}^{z_2(r,\theta)}f\mathrm dz\quad(Case\;1)\\\\&=\int_a^b\mathrm d\theta\iint_{D(\theta)}f·r·\mathrm dr\mathrm dz\quad(Case\;2)\end{aligned}


下面考虑球坐标系下求解三重积分的问题.

{x=rsinφcosθy=rsinφsinθz=rcosφ,0r<+,  0φπ,  0θ2πdV=r2sinφdrdθdφ\begin{aligned}\begin{cases}x=r\sin\varphi\cos\theta\\y=r\sin\varphi\sin\theta\\z=r\cos\varphi\end{cases},&\quad 0\leq r\lt+\infty,\;0\leq \varphi\leq\pi,\;0\leq\theta\leq2\pi\\&\mathrm dV=r^2\sin\varphi\mathrm dr\mathrm d\theta\mathrm d\varphi\end{aligned}

式中各含义如下图所示,而微元体积的结论这里不做推导已直接给出。

球坐标示意图

因此,我们得到在球坐标下三重积分表示为:

Ωf(x,y,z)dV=ΩF(r,φ,θ)r2sinφdrdθdφwhere F(r,φ,θ)=f(rsinφcosθ,rsinφsinθ,rcosφ)\begin{aligned}\iiint_{\Omega}f(x,y,z)\mathrm dV&=\iiint_{\Omega}F(r,\varphi,\theta)r^2\sin\varphi·\mathrm dr\mathrm d\theta\mathrm d\varphi\\\\\text{where }F(r,\varphi,\theta)&=f(r\sin\varphi\cos\theta,r\sin\varphi\sin\theta,r\cos\varphi)\end{aligned}

曲线积分化定积分

lf(x,y)  ds=abf(φ(t),ψ(t))φ2(t)+ψ2(t)  dt\int_lf(x,y)\;ds=\int_a^bf(\varphi(t),\psi(t))·\sqrt{\varphi'^2(t)+\psi'^2(t)}\;dt

其中,l:{x=φ(t),y=ψ(t)atbl:\begin{cases}x=\varphi(t),\\y=\psi(t)\end{cases}\quad a\leq t\leq b ,且要求φ,ψ\varphi,\psi[a,b][a,b] 有连续导数且不全为0. 不难发现,式中的根式即是对平面曲线弧的弧微分ds\mathrm ds 在参数方程下的结论.

易知,当曲线ll 可以表示为l:y=y(x),axbl:y=y(x),a\leq x \leq b 时,可以进一步写出:

lf(x,y)  ds=abf(x,y(x))1+yx2  dx\int_lf(x,y)\;ds=\int_a^bf(x,y(x))\sqrt{1+y'^2_x}\;dx

当曲线可由极坐标方程给出,即l:r=r(θ),aθbl:r=r(\theta),a\leq\theta\leq b 时,亦可根据弧微分的结论,换元得到:

lf(x,y)  ds=abf(rcosθ,rsinθ)r2+r2  dθ\int_lf(x,y)\;ds=\int_a^bf(r\cos\theta,r\sin\theta)\sqrt{r^2+r'^2}\;d\theta


对于第二类曲线积分,给出曲线的参数方程表示(参数tt 的取值与曲线方向有关)后,可以直接有:

l:{x=x(t)y=y(t)t:abdx=x(t)dtdy=y(t)dtlPdx+Qdy=ab(Px+Qy)dt\begin{aligned}l:\begin{cases}x=x(t)\\y=y(t)\end{cases}\quad t:a\to b\\\\\mathrm dx=x'(t)\mathrm dt\quad\mathrm dy=y'(t)\mathrm dt\\\\\Rightarrow \int_lPdx+Qdy=\int_a^b(Px'+Qy')\mathrm dt\end{aligned}

曲面积分化重积分

Σf(x,y,z)dS=Df(x,y,z(x,y))1+zx2+zy2dσ\iint_{\Sigma}f(x,y,z)\mathrm dS=\iint_Df(x,y,z(x,y))\sqrt{1+z'^2_x+z'^2_y}\mathrm d\sigma

其中,积分曲面Σ:z=z(x,y)\Sigma:z=z(x,y)DD 为曲面在xOyxOy 平面的投影。

下面简单阐述一下曲面微元面积dS\mathrm dS 是如何转化的. 对于曲面元dS\mathrm dS 在点(x,y,z)(x,y,z) 处的单位法向量n=(cosα,cosβ,cosγ)=±(zx,zy,1)1+zx2+zy2\boldsymbol n=(\cos\alpha,\cos\beta,\cos\gamma)=\pm\begin{aligned}\frac{(-z_x',-z_y',1)}{\sqrt{1+z'^2_x+z'^2_y}}\end{aligned}. 从而dS\mathrm dSxOyxOy 平面的投影为:

dσ=PrxOydS=cosγdS\mathrm d\sigma =\Pr_{xOy}\mathrm dS=|\cos\gamma|\mathrm dS

※ 如果积分曲面的表达式更加一般,由曲线Γ:{F(x,y,z)=0G(x,y,z)=0\varGamma:\begin{cases}F(x,y,z)=0\\G(x,y,z)=0\end{cases} 与坐标轴围成,那么:

Σf(x,y,z)dS=Df(x,y,z(x,y))ndσ\iint_{\Sigma}f(x,y,z)\mathrm dS=\iint_Df(x,y,z(x,y))|\boldsymbol n| d\sigma

其中,n=F(P0)×G(P0)\boldsymbol n=\nabla F(P_0)\times\nabla G(P_0).


对于 第二类曲面积分DijD_{ij}定向曲面iOjiOj 平面的投影,则:

ΣP  dydz=±DyzP(x(y,z),y,z)  dydzΣQ  dzdx=±DzxP(x,y(z,x),z)  dzdxΣR  dxdy=±DxyP(x,y,z(x,y))  dxdy\begin{aligned}\iint_{\Sigma}P\;dydz&=\pm\iint_{D_{yz}}P(x(y,z),y,z)\;dydz\\\iint_{\Sigma}Q\;dzdx&=\pm\iint_{D_{zx}}P(x,y(z,x),z)\;dzdx\\\iint_{\Sigma}R\;dxdy&=\pm\iint_{D_{xy}}P(x,y,z(x,y))\;dxdy\\\end{aligned}

其中,±\pm 的选取与Σ\Sigma 法向量n\boldsymbol n 的方向有关,如果法向量方向与xx 轴成锐角,则上式第一个等式取正,否则取负,其他式子同理。
通过这种转化,可以将第二类曲面积分问题转化为求三个二重积分之和的问题.

特别地,如果积分曲面可由显式函数Σ:z=z(x,y)\Sigma:z=z(x,y) 表示,DxyD_{xy} 为曲面在xOyxOy 平面的投影,那么有:

I=ΣPdydz+Qdzdx+Rdxdy=±Dxy[P(zx)+Q(zy)+R]dxdy\begin{aligned}I&=\iint_{\Sigma}Pdydz+Qdzdx+Rdxdy\\\\&=\pm\iint_{D_{xy}}\left[P\left(-\frac{\partial z}{\partial x}\right)+Q\left(-\frac{\partial z}{\partial y}\right)+R\right]dxdy\end{aligned}

法向量指向上侧取正.

事实上该结论是由 第二类曲面积分→第一类曲面积分→二重积分 演变而来.

拓展:重积分的坐标变换

二重积分的坐标变换

f(x,y)f(x,y)xOyxOy 平面的闭区域DD 上连续,若变换

T:x=x(u,v),y=y(u,v)T:x=x(u,v),\quad y=y(u,v)

uOvuOv 平面上的闭区域DD' 变为xOyxOy 平面上的DD,且:

  1. x(u,v),  y(u,v)x(u,v),\;y(u,v)DD' 上具有一阶连续偏导数;
  2. DD'雅可比式不为0;
  3. 变换T:DDT:D'\to D 是一一对应的.
    则有:

Df(x,y)dxdy=Dg(u,v)J(u,v)dudvwhere g(u,v)=f(x(u,v),y(u,v)),J(u,v)=(x,y)(u,v)=xuxvyuyv0\begin{aligned}\iint_Df(x,y)dxdy&=\iint_{D'}g(u,v)|J(u,v)|dudv\\\\\text{where }g(u,v)&=f(x(u,v),y(u,v)),\\\\J(u,v)&=\frac{\partial (x,y)}{(u,v)}=\begin{vmatrix}\frac{\partial x}{\partial u}&\frac{\partial x}{\partial v}\\\\\frac{\partial y}{\partial u}&\frac{\partial y}{\partial v}\end{vmatrix}\neq 0\end{aligned}

上式被称为 二重积分换元公式. 三重积分也类似。


下面我们来讨论简单情况,即只考虑平移变换情况下对二重积分的化简作用。
考虑变换T:{u=xav=ybT:\begin{cases}u=x-a\\v=y-b\end{cases},显然有J(u,v)=1J(u,v)=1,因此:

Df(x,y)dxdy=Df(u+a,v+b)dudv\iint_Df(x,y)dxdy=\iint_{D'}f(u+a,v+b)dudv

其中,DD' 是将被积区域DD' 平移后得到的,这使得可充分利用几何意义、对称性等方式对新的重积分进行化简,从而简化计算.

类似地,我们可以推导出将椭圆变换为圆,将椭球变换为球以化简复杂计算的结论。

以“椭球变球”为例,有着如下结论:

x2a2+y2b2+z2c2=1  {x=arcosθsinφy=brsinθsinφz=crcosφΩf(x,y,z)dV=02πdθ0πdφ01fabcr2sinφ  dr\begin{aligned}\frac{x^2}{a^2}+\frac{y^2}{b^2}+\frac{z^2}{c^2}=1\;\Rightarrow\begin{cases}x=ar\cos\theta\sin\varphi\\y=br\sin\theta\sin\varphi\\z=cr\cos\varphi\end{cases}\\\\\iiint_{\Omega}f(x,y,z)\mathrm dV=\int_{0}^{2\pi}d\theta\int_0^{\pi}d\varphi \int_0^1f·abc·r^2\sin\varphi\;\mathrm dr\end{aligned}

其中,对该变换得到的雅可比式为:J(r,θ,φ)=abcr2sinφ|J(r,\theta,\varphi)|=abc·r^2\sin\varphi

利用对称性进行简化计算

对称性对积分化简的作用

在二重积分情形下
f(x,y)f(x,y)DD 上连续. 若DD 关于xx 轴对称,则

  1. f(x,y)f(x,y)yy 为奇函数时,Df(x,y)dxdy=0\iint\limits_{D}f(x,y)dxdy=0
  2. f(x,y)f(x,y)yy 为偶函数时,Df(x,y)dxdy=2Df(x,y)dxdy\iint\limits_{D}f(x,y)dxdy=2\iint\limits_{D'}f(x,y)dxdy,其中D=D{y0}D'=D\cap \{y\geq0\}
  3. DD 关于yy 轴,关于原点对称,情况类似

考虑DD 关于y=xy=x 对称的情况,即(x,y)D(y,x)D(x,y)\in D\Leftrightarrow (y,x)\in D,则有:

Df(x,y)dxdy=Df(y,x)dxdy\iint_Df(x,y)dxdy=\iint_Df(y,x)dxdy

这个性质被称为 重积分的 轮换对称性 .


在三重积分情形下
f(x,y,z)f(x,y,z)Ω\Omega 连续. 若Ω\Omega 关于xOyxOy 平面对称,则

  1. f(x,y,z)f(x,y,z)zz 为奇函数时,Ωf(x,y,z)dV=0\iiint\limits_{\Omega}f(x,y,z)dV=0
  2. f(x,y,z)f(x,y,z)zz 为偶函数时,Ωf(x,y,z)dV=2Ωf(x,y,z)dV\iiint\limits_{\Omega}f(x,y,z)dV=2\iiint\limits_{\Omega'}f(x,y,z)dV,其中Ω=Ω{z0}\Omega'=\Omega\cap\{z\geq0\}
  3. Ω\Omega 关于zx,yzzx,yz 平面对称,情况类似

在第一类曲线/曲面积分情形下 也有类似的结论,但第二类则不尽相同。


在第二类曲线积分情形下
LL 为平面上分段光滑的定向曲线,P(x,y),Q(x,y)P(x,y),Q(x,y) 连续. 若LL 关于yy 轴对称,则:

LPdx={0,P 关于 x 为奇函数,2L1Pdx,P 关于 x 为偶函数.LQdy={0,Q 关于 x 为偶函数,2L1Qdy,Q 关于 x 为奇函数.\begin{aligned}\int_LP\mathrm dx=\begin{cases}0&,P\text{ 关于 }x\text{ 为奇函数,}\\2\int_{L_1}Pdx&,P\text{ 关于 }x\text{ 为偶函数.}\end{cases}\\\\\int_LQ\mathrm dy=\begin{cases}0&,Q\text{ 关于 }x\text{ 为偶函数,}\\2\int_{L_1}Qdy&,Q\text{ 关于 }x\text{ 为奇函数.}\end{cases}\end{aligned}

其中,L1L_1LL 在右半平面的部分.

LL 关于 平面xy=0x-y=0 对称,则:

Lf(x)ds=Lf(y)ds\int_Lf(x)ds=\int_Lf(y)ds


在第二类曲面积分情形下
SS 为分块光滑定向曲面,R(x,y,z)R(x,y,z)SS 上连续. 若SS 关于xyxy 平面对称,且上部分记为S1:z=z(x,y),(x,y)DxyS_1:z=z(x,y),(x,y)\in D_{xy} ,则:

SR(x,y,z)dxdy={0,R 关于 z 为偶函数,2S1R  dxdy,R 关于 z 为奇函数.\iint_SR(x,y,z)dxdy=\begin{cases}0,&R\text{ 关于 }z\text{ 为偶函数,}\\2\iint\limits_{S_1}R\;dxdy,&R\text{ 关于 }z\text{ 为奇函数.}\end{cases}

多元函数积分学的基本公式

多元函数积分学的基本公式

DD 为平面区域,如果DD 内任一闭曲线所围成的部分都属于DD ,则称DD平面单连通区域(不含孔的区域),否则为 复连通区域(含有孔的区域)。

LL 是平面区域DD 的边界,若观测者沿着某个方向观察时,DD 内的部分总在其左侧,这样的方向我们规定是LL 的正向。
于是对于复连通区域而言,就有外边界正向逆时针,内边界正向顺时针

格林公式

设平面上的有界闭区域DD 由分段光滑曲线LL 围成,函数P(x,y),Q(x,y)P(x,y),Q(x,y)DD 上有 一阶连续偏导数,则有:

D(QxPy)dxdy=LPdx+Qdy\iint_D\left(\frac{\partial Q}{\partial x}-\frac{\partial P}{\partial y}\right)\mathrm dx\mathrm dy=\oint_LP\mathrm dx+Q\mathrm dy

上式叫作 格林公式。对于复连通区域,只需把其余边界对正向求第二类曲线积分进行累加即可。
特别地,对于DD 的面积,我们有:

SD=1  dxdy=12D(xx(y)y)dxdy=12Lydx+xdyS_D=\iint1\;\mathrm dx\mathrm dy=\frac 1 2\iint_D\left(\frac{\partial x}{\partial x}-\frac{\partial (-y)}{\partial y}\right)\mathrm dx\mathrm dy=\frac 1 2\oint_L-y\mathrm dx+x\mathrm dy

在利用格林公式求解曲线积分时,一定要注意以下几个问题:

  1. 如果积分路径LL 不为封闭曲线,但化为二重积分好求时,可以考虑补一个或多个分段曲线LL' 使得L+LL+L' 构成闭曲线,然后计算此路径下的曲线积分。之后再单独计算LL' 反方向下的曲线积分,利用可加性就可得到原路径下的积分结果。
  2. 如果函数P,QP,Q 在区域内存在奇点,则必须用第(1)点提到的方法挖去奇点(可能区域因此变为复连通区域),然后再利用可加性处理。

高斯公式

设空间中的有界闭区域Ω\Omega 由分段光滑曲面Σ\Sigma 围成,函数P(x,y,z),Q(x,y,z),R(x,y,z)P(x,y,z),Q(x,y,z),R(x,y,z)Ω\Omega 上有 一阶连续偏导数,则有:

Ω(Px+Qy+Rz)dV=ΣPdydz+Qdzdx+Rdxdy\iiint_{\Omega}\left(\frac{\partial P}{\partial x}+\frac{\partial Q}{\partial y}+\frac{\partial R}{\partial z}\right)\mathrm dV=\iint_{\Sigma}P\mathrm dy\mathrm dz+Q\mathrm dz\mathrm dx+R\mathrm dx\mathrm dy

此处的Σ\SigmaΩ\Omega 取外侧(外法向)

在利用格林公式求解曲线积分时,也有和格林公式求解曲线积分类似思想的补曲面构造闭合区域的思想,此处不再赘述。

斯托克斯公式

Γ\varGamma 为光滑有向闭曲线,Σ\Sigma 是以Γ\varGamma 为边界的分块光滑有向曲面,Γ\varGamma 的正向与Σ\Sigma 的侧符合右手定则(右手四指按Γ\varGamma 方向绕行时,大拇指所指的方向就是Σ\Sigma 的正向法向),对应曲面的法向量为n=(cosα,cosβ,cosγ)\boldsymbol n=(\cos\alpha,\cos\beta,\cos\gamma). 若函数P(x,y,z),Q(x,y,z),R(x,y,z)P(x,y,z),Q(x,y,z),R(x,y,z) 在包含Σ\Sigma 的一个空间区域内有一阶连续偏导数,则有:

I=ΣdydzdzdxdxdyxyzPQR=ΓPdx+Qdy+Rdz=ΓPcosα+Qcosβ+RcosγdS=ΣcosαcosβcosγxyzPQRdS\begin{aligned}I&=\iint_{\Sigma}\begin{vmatrix}\mathrm dy\mathrm dz & \mathrm dz\mathrm dx & \mathrm dx\mathrm dy\\\frac{\partial}{\partial x} & \frac{\partial}{\partial y} & \frac{\partial}{\partial z}\\P & Q & R\end{vmatrix}=\int_{\varGamma}P\mathrm dx+Q\mathrm dy+R\mathrm dz\\&=\int_{\varGamma}P\cos\alpha+Q\cos\beta+R\cos\gamma\mathrm dS\\&=\iint_{\Sigma}\begin{vmatrix}\cos\alpha & \cos\beta & \cos\gamma\\\frac{\partial}{\partial x} & \frac{\partial}{\partial y} & \frac{\partial}{\partial z}\\P & Q & R\end{vmatrix}\mathrm dS\end{aligned}

注:使用斯托克斯公式后可将曲线积分转化为曲面积分,而这个曲面积分还可以再叠加高斯公式+补面进行化简。更进一步地,其实相当于对于曲面的选取是灵活的。

参考:https://www.bilibili.com/video/BV1w8411s78Z


曲线积分与路径无关问题

DD 是平面上的一个区域,L1,L2D\forall L_1,L_2\in D任意两条连接A,B(A,BD)A,B\quad(A,B\in D) 的分段光滑线段,若:

L1P  dx+Q  dy=L2P  dx+Q  dy\int_{L_1}P\;dx+Q\;dy=\int_{L_2}P\;dx+Q\;dy

恒成立,则称曲线积分LP  dx+Q  dy\begin{aligned}\int_{L}P\;dx+Q\;dy\end{aligned}DD与路径无关

根据上述定义,可以导出如下结论(证明见同济版《高等数学 第7版 下册》Page 211):

曲线积分在DD 内与路径无关
\LeftrightarrowDD 内任意光滑封闭曲线CC 都有CP  dx+Q  dy=0\begin{aligned}\oint_{C}P\;dx+Q\;dy=0\end{aligned}
\LeftrightarrowDD 内存在函数u(x,y)u(x,y) 使得du=Pdx+Qdydu=Pdx+Qdy

此外,结合格林公式,若DD单连通区域(复连通不行),且有QxPy\begin{aligned}\frac{\partial Q}{\partial x}\equiv\frac{\partial P}{\partial y}\end{aligned} 即可得知,此时的曲线积分的结果与积分路径无关,只与起点和终点有关,从而可以随意设立更好求的积分路径进行求解。

更进一步地,结合格林公式挖去奇点的思路,还可以得出若对于挖去奇点的连通域D=D0{M0}D=D_0-\{M_0\},若有QxPy\begin{aligned}\frac{\partial Q}{\partial x}\equiv\frac{\partial P}{\partial y}\end{aligned}围绕奇点M0M_0 的新分段光滑闭曲线C0C_0 能使曲线积分结果为0,此时的曲线积分的结果与积分路径无关,只与起点和终点有关

求 Pdx+Qdy 的原函数问题

Pdx+Qdy 的原函数

方法一 不定积分法
ux=P(x,y)\begin{aligned}\frac{\partial u}{\partial x}=P(x,y)\end{aligned} 两边对xx 积分(此时将yy 视为常数即可)可得:

u=P(x,y)  dx+C(y)u=\int P(x,y)\;\mathrm dx+C(y)

再由uy=Q(x,y)=y(Pdx)+C(y)\begin{aligned}\frac{\partial u}{\partial y}=Q(x,y)=\frac{\partial}{\partial y}\left(\int P\mathrm dx\right)+C'(y)\end{aligned} 由此反解出C(y)C'(y),再利用一元函数不定积分解出C(y)C(y) 即可。(此时又视xx 为常数)

方法二 特殊路径积分法
首先判断目标积分是否在单连通域内满足QxPy\begin{aligned}\frac{\partial Q}{\partial x}\equiv\frac{\partial P}{\partial y}\end{aligned} ,若是则说明与积分路径无关。
从而我们选取合适的定点(x0,y0)(x_0,y_0) 作为起点,以动点(x,y)(x,y) 作为终点,再选取合适的路径求此积分(通常为便于计算的折线):u(x,y)=(x0,y0)(x,y)Pdx+Qdy\begin{aligned}u(x,y)=\int_{(x_0,y_0)}^{(x,y)}Pdx+Qdy\end{aligned}
如果是复连通区域,则需要采取挖点的方式,单独计算围绕起点的某个路径下的积分是否为0,如果是则在该区域内存在原函数,否则原函数不存在。

方法三 凑微分法
顾名思义,是根据求导法则倒推的一种观察法。特别地,应当熟悉商的微分公式:

d(uv)=v  duu  dvv2\mathrm d\left(\frac u v\right)=\frac{v\;\mathrm du-u\;\mathrm dv}{v^2}

下面给出一实例加以说明。已知:

Pdx+Qdy=2x(1ey)(1+x2)2dx+ey1+x2dyPdx+Qdy=\frac{2x(1-e^y)}{(1+x^2)^2}dx+\frac{e^y}{1+x^2}dy

u(x,y)u(x,y) 使得du=Pdx+Qdydu=Pdx+Qdy

【方法一】由uy=ey1+x2\begin{aligned}\frac{\partial u}{\partial y}=\frac{e^y}{1+x^2}\end{aligned}yy 积分,得u=ey1+x2+C(x)\begin{aligned}u=\frac{e^y}{1+x^2}+C(x)\end{aligned},接下来再由
ux=2x(1ey)(1+x2)2=2xey(1+x2)2+C(x)\begin{aligned}\frac{\partial u}{\partial x}=\frac{2x(1-e^y)}{(1+x^2)^2}=-\frac{2xe^y}{(1+x^2)^2}+C'(x)\end{aligned} 得出C(x)=2x(1+x2)2C(x)=11+x2+C\begin{aligned}C'(x)=\frac{2x}{(1+x^2)^2}\Rightarrow C(x)=\frac{-1}{1+x^2}+C\end{aligned}
整理可得,原函数u=1ey1+x2+C\begin{aligned}u=-\frac{1-e^y}{1+x^2}+C\end{aligned}

【方法二】不难验证QxPy\begin{aligned}\frac{\partial Q}{\partial x}\equiv\frac{\partial P}{\partial y}\end{aligned} 在全平面恒成立。取起点为(0,0)(0,0),对于(0,0)(x,y)(0,0)\to(x,y) 的路径,取(0,0)(x,0)(x,y)(0,0)\to(x,0)\to(x,y),于是:

u(x,y)=(0,0)(x,y)Pdx+Qdy=0xP(x,0)dx+0yQ(x,y)dy=0x0dx+0yey1+x2dy=1ey1+x2+C\begin{aligned}u(x,y)&=\int_{(0,0)}^{(x,y)}Pdx+Qdy\\&=\int_0^xP(x,0)\mathrm dx+\int_0^yQ(x,y)\mathrm dy\\&=\int_0^x0dx+\int_0^y\frac{e^y}{1+x^2}dy\\&=-\frac{1-e^y}{1+x^2}+C\end{aligned}

【方法三】不难看出2x(1+x2)2dx=d(11+x2)  ,eydy=d(1ey)\begin{aligned}\frac{-2x}{(1+x^2)^2}dx=d\left(\frac{1}{1+x^2}\right)\;,-e^ydy=d(1-e^y)\end{aligned},从而有:

du=[(1ey)d(11+x2)+11+x2d(1ey)]u(x,y)=1ey1+x2+C\begin{aligned}du&=-\left[(1-e^y)d\left(\frac{1}{1+x^2}\right)+\frac{1}{1+x^2}d(1-e^y)\right]\\&\Rightarrow u(x,y)=-\frac{1-e^y}{1+x^2}+C\end{aligned}

多元函数积分学的物理应用

物理学应用

质心与转动惯量

以空间立体Ω\varOmega 为例,设其体密度为ρ(x,y,z)\rho(x,y,z) 则:

质量: M=Ωρ(x,y,z)dV质心: (x,y,z)=(ΩxρdV,ΩyρdV,ΩzρdV)/M\begin{aligned}\text{质量: }&M=\iiint\limits_{\varOmega}\rho(x,y,z)\mathrm dV\\\\\text{质心: }&(\overline{x},\overline{y},\overline{z})=\left(\iiint\limits_{\varOmega}x\rho\mathrm dV,\iiint\limits_{\varOmega}y\rho\mathrm dV,\iiint\limits_{\varOmega}z\rho\mathrm dV\right)\bigg/ M\end{aligned}

对于质量连续均匀分布的物体,其针对某转轴的转动惯量J=r2dm\begin{aligned}J=\int r^2\mathrm dm\end{aligned}.
式中,rr质量元dm\mathrm dm 到转轴的垂直距离.

于是,我们可以针对空间直角坐标系列出空间立体Ω\varOmega 的绕不同轴旋转的转动惯量:

绕 x 轴: Jx=Ω(y2+z2)ρ  dV绕 y 轴: Jy=Ω(z2+x2)ρ  dV绕 z 轴: Jz=Ω(x2+y2)ρ  dV对原点: Jo=Ω(x2+y2+z2)ρ  dV\begin{aligned}\text{绕 }x\text{ 轴: }J_x&=\iiint\limits_{\varOmega}(y^2+z^2)\rho\;\mathrm dV\\\\\text{绕 }y\text{ 轴: }J_y&=\iiint\limits_{\varOmega}(z^2+x^2)\rho\;\mathrm dV\\\\\text{绕 }z\text{ 轴: }J_z&=\iiint\limits_{\varOmega}(x^2+y^2)\rho\;\mathrm dV\\\\\text{对原点: }J_o&=\iiint\limits_{\varOmega}(x^2+y^2+z^2)\rho\;\mathrm dV\end{aligned}

引力计算

设质点M0(x0,y0,z0),M(x,y,z)M_0(x_0,y_0,z_0),M(x,y,z) 的质量分别为m0,mm_0,m. 则M0M_0MM 的引力为:

F=Gm0mri3ri\boldsymbol F=\frac{Gm_0m}{r_i^3}\boldsymbol r_i

其中,ri=M0M\boldsymbol r_i=\overrightarrow{M_0M}GG 为引力常数.

进而得到空间立体Ω\varOmegaΩ\varOmega 外一点P(x0,y0,z0)P(x_0,y_0,z_0) 的引力(以xx 方向的分力给出示例):

F=(Fx,Fy,Fz)Fx=ΩGρM(xx0)dV[(xx0)2+(yy0)2+(zz0)2]3/2\begin{aligned}\boldsymbol F&=(F_x,F_y,F_z)\\F_x&=\iiint\limits_{\varOmega}\frac{G\rho M(x-x_0)\mathrm dV}{\left[(x-x_0)^2+(y-y_0)^2+(z-z_0)^2\right]^{3/2}}\end{aligned}

功的计算

质点在常力F=(P,Q,R)\boldsymbol F=(P,Q,R) 作用下从AA 点沿直线运动到BB 点,则力做的功为:

W=FABW=\boldsymbol F\cdot\overset{\rightharpoonup}{AB}

质点在变力F=(P(x,y,z),Q(x,y,z),R(x,y,z))\boldsymbol F=(P(x,y,z),Q(x,y,z),R(x,y,z)) 作用下从AA 点沿曲线CC 移动到BB 点,则力做的功为:

W=abPdx+Qdy+RdzW=\int_{\overset{\frown}{ab}}Pdx+Qdy+Rdz

求解上式时,可考虑通过判断偏微分相等→积分结果与路径无关 从而简化计算。
特别地,应该尝试是否能通过凑全微分直接求出原函数。

由曲线积分求功时,首先要求出变力F\boldsymbol F 的表达式.
已知F|\boldsymbol F| 和其方向l\boldsymbol l (不一定是单位向量),则F=Fll\begin{aligned}\boldsymbol F=|\boldsymbol F|\frac{\boldsymbol l}{|\boldsymbol l|}\end{aligned}.

流量计算

流体在空间中流动,流速v\boldsymbol v 为常向量,则单位时间内流过指定平面上区域DD 的体积流量,即流体体积,简称流量,为:

q=vcosθS=vnSq=|\boldsymbol v|\cos\theta\cdot S=\boldsymbol v\cdot\boldsymbol nS

其中,n\boldsymbol n 是定向平面的单位法向量,θ\theta 是两向量夹角,SSDD 的面积。

若流速v=(P(x,y,z),Q(x,y,z),R(x,y,z))\boldsymbol v=(P(x,y,z),Q(x,y,z),R(x,y,z)),则:

q=Svn  dS=SPdydz+Qdzdx+Rdxdyq=\iint\limits_{S}\boldsymbol v\cdot\boldsymbol n\;\mathrm dS=\iint\limits_{S}Pdydz+Qdzdx+Rdxdy

微分方程的求解

一阶常微分方程常见类型

一阶常微分方程常见类型

可分离变量的微分方程

g(y)dy=f(x)dxg(y)\mathrm dy=f(x)\mathrm dx

通过两边同时积分得到该微分方程的 。(如果出现分母可能为0的情况,需要进行验证)

齐次方程

dydx=f(yx)\frac{\mathrm dy}{\mathrm dx}=f\left(\frac y x\right)

求解时,通过换元u=yx,y=u+xu\begin{aligned}u=\frac y x,y'=u+xu'\end{aligned} 化为其他微分方程的形式,从而再利用对应方法进行求解

一阶线性微分方程

dydx+P(x)y=Q(x)\frac{\mathrm dy}{\mathrm dx}+P(x)y=Q(x)

如果Q(x)0Q(x)\equiv 0 则称该微分方程 齐次,否则即为 非齐次

对于一阶齐次线性微分方程,我们可以利用 分离变量法 求得其通解如下:

y=CeP(x)dxy=Ce^{-\int P(x)dx}

该结论应当熟记。下面我们给出该齐次线性方程对应的非齐次方程的通解

y=eP(x)dx(Q(x)eP(x)dxdx+C)y=e^{-\int P(x)dx}\left(\int Q(x)e^{\int P(x)dx}dx+C\right)

该结论也应当牢记在心,可作为结论直接在解题时使用。
此通解的推导过程用到了 常数变易法。其具体推导过程见下:

常数变易法推导微分方程通解

将 一阶齐次线性微分方程的通解中的常数CC 视为关于xx 的函数u(x)u(x),继续进行求解。即:

y=u(x)eP(x)dxdydx=ueP(x)dxuP(x)eP(x)dx    ueP(x)dx=Q(x)    u=Q(x)eP(x)dx+C    y=CeP(x)dx(Q(x)eP(x)dx+C)\begin{aligned}y&=u(x)e^{-\int P(x)dx}\\\frac{\mathrm dy}{\mathrm dx}&=u'e^{-\int P(x)dx}-uP(x)e^{-\int P(x)dx}\\\\\Rightarrow\;\; &u'e^{-\int P(x)dx}=Q(x)\\\Rightarrow\;\; &u=\int Q(x)e^{\int P(x)dx}+C\\\Rightarrow\;\;&y=Ce^{-\int P(x)dx}\left(\int Q(x)e^{\int P(x)dx}+C\right)\end{aligned}

伯努利(Bernoulli)方程

dydx+P(x)y=Q(x)yn(n0,1)\frac{\mathrm dy}{\mathrm dx}+P(x)y=Q(x)y^n\quad(n\neq0,1)

将上式两边除以yny^n,得到:

yndydx+P(x)y1n=Q(x)y^{-n}\frac{\mathrm dy}{\mathrm dx}+P(x)y^{1-n}=Q(x)

进而,令z=y1nz=y^{1-n},有z=(1n)ynyz'=(1-n)y^{-n}·y',从而:

dzdx+(1n)P(x)z=(1n)Q(x)\frac{\mathrm dz}{\mathrm dx}+(1-n)P(x)z=(1-n)Q(x)

于是就化为了 一阶线性微分方程 的形式。

全微分方程

将一阶微分方程写成

P(x,y)dx+Q(x,y)dy=0P(x,y)\mathrm dx+Q(x,y)\mathrm dy=0

若存在二元(单值)函数u(x,y)u(x,y) 使得:

du(x,y)=P(x,y)dx+Q(x,y)dy\mathrm du(x,y)=P(x,y)\mathrm dx+Q(x,y)\mathrm dy

则称该微分方程为 全微分方程。若P,QP,Q 不同时为0,则通解即为u(x,y)=Cu(x,y)=C(隐式通解)

拥有此种形式的微分方程是全微分方程的充要条件QxPy(x,y)D\begin{aligned}\frac{\partial Q}{\partial x}\equiv \frac{\partial P}{\partial y}\quad (x,y)\in D\end{aligned}
其中,DDP,QP,Q 具有一阶连续偏导数的平面单连通域

此类微分方程的求解方法,其实主要是对多元函数uu 的求解方法,见本文多元函数专栏。

二阶及高阶微分方程

可降阶的二阶方程

  1. y=f(x)y''=f(x)
    • 通解:y=(f(x)dx)dx+C1x+C2\begin{aligned}y=\int\left(\int f(x)\mathrm dx\right)\mathrm dx+C_1x+C_2\end{aligned}
    • 可推广至y(n)=f(x)y^{(n)}=f(x)
  2. y=f(x,y)y''=f(x,y')
    • 特点:不含显yy
    • 求解:令p=yp=y',从而p=yp'=y'',化为一阶常微分方程
  3. y=f(y,y)y''=f(y,y')
    • 特点:不含显xx
    • 求解:令p=y=dy/dxp=y'=dy/dx,从而y=dpdx=pdpdy\begin{aligned}y''=\frac{dp}{dx}=p·\frac{dp}{dy}\end{aligned},化为以yy 为自变量的微分方程

例题yy(y)2=y4,y(0)=1,y(0)=1yy''-(y')^2=y^4,y(0)=1,y'(0)=1 的特解.

答案

y=p,y=pdpdy\begin{aligned}y'=p,y''=p\frac{dp}{dy}\end{aligned}.
原微分方程化为:ypdpdyp2=y4\begin{aligned}yp\frac{dp}{dy}-p^2=y^4\end{aligned}

两端同时除以ypyp,有dpdy=py+y2yp\begin{aligned}\frac{dp}{dy}=\frac{p}{y}+y^2\frac{y}{p}\end{aligned},从而令u=p/yu=p/y,得到:

dudy=yuu=py=±y2+C1,p=dydx=±yy2+C1\begin{aligned}\frac{du}{dy}=\frac{y}{u}\quad\Rightarrow\quad u=\frac{p}{y}=\pm\sqrt{y^2+C_1},\quad p=\frac{dy}{dx}=\pm y\sqrt{y^2+C_1}\end{aligned}

y(0)=1,y(0)=1C1=0,p=±y2y'(0)=1,y(0)=1 \Rightarrow C_1=0,p=\pm y^2,再分离变量得:

±dyy2=dxy=±1C2xy(0)=1,y(0)=1C2=1,y=11x\begin{aligned}\pm\int\frac{dy}{y^2}=\int dx\\y=\pm\frac{1}{C_2-x}\\\\y'(0)=1,y(0)=1\Rightarrow C_2=1,\\y=\frac 1 {1-x}\end{aligned}

事实上,在将原微分方程化为:ypdpdyp2=y4\begin{aligned}yp\frac{dp}{dy}-p^2=y^4\end{aligned} 后,还可以凑微分得出另一种解法,即:

pdpdy=12d(p2)dyd(p2)dy2y(p2)=2y3{p2=e2ydy(2y3e2ydy+C1)y(0)=1y(0)=1p=±y2=dydxy=11x\begin{aligned}\because\quad& p\frac{dp}{dy}=\frac 1 2·\frac{d(p^2)}{dy}\\\therefore\quad& \frac{d(p^2)}{dy}-\frac{2}{y}(p^2)=2y^3\\\therefore\quad&\begin{cases}p^2=e^{\int\frac 2 ydy}\left(\int2y^3·e^{-\int\frac 2 y dy}+C_1\right)\\y(0)=1\\y'(0)=1\end{cases}\Rightarrow p=\pm y^2=\frac{dy}{dx}\\\\\Rightarrow\quad& y=\frac 1 {1-x}\end{aligned}

在上面的例题中,给出了关于 凑微分 的思维方式,这在快速解题过程中很有益处,虽然本题并不明显,但是考生应当掌握。
此外,通过该例题还可以发现,尽早代入初始条件定出常数,可以在下一步积分时避免常数不确定的麻烦,十分有用。

二阶线性微分方程

二阶线性微分方程的一般形式为:

y+P(x)y+Q(x)y=f(x)y''+P(x)y'+Q(x)y=f(x)

与一阶线性微分方程相同,当f(x)0f(x)\equiv 0 时,称其为 二阶线性齐次微分方程。
如果式中的P,QP,Q 均为常数,则称其为 二阶常系数线性微分方程。

关于高阶线性微分方程的解的构造将在下一节详细介绍。这里仅给出二阶常系数线性微分方程的求解方法,以便查阅。

我们将二阶常系数齐次线性微分方程及其特征方程给出:

y+py+qy=0r2+pr+q=0\begin{aligned}y''+py'+qy=0\\r^2+pr+q=0\end{aligned}

可以证明y=erxy=e^{rx} 是齐次线性方程的通解。
于是求解这类问题可以转化为研究一元二次方程问题,从而得到下表:

Δ=p24q\Delta=p^2-4qr1,2=(p±Δ)/2r_{1,2}=(-p\pm\sqrt{\Delta})/2说明
>0\gt 0r1,r2R,r1r2\exists r_1,r_2\in R,r_1\neq r_2通解y=C1er1x+C2er2xy=C_1e^{r_1x}+C_2e^{r_2x}
=0=0r=r1=r2R\exists r=r_1=r_2\in R,二重根通解y=(C1+C2x)erxy=\left(C_1+C_2x\right)e^{rx}
<0\lt0r1,2=α±βi,β>0r_{1,2}=\alpha\pm\beta i,\beta\gt0为共轭复数通解y=eαx(C1cosβx+C2sinβx)y=e^{\alpha x}\left(C_1\cos\beta x+C_2\sin\beta x\right)

其中,α=p/2,β=4qp2/2\alpha=-p/2,\beta=\sqrt{4q-p^2}/2

推广到高阶,我们有:

  1. rrkk 重实根,则通解为:
    y=(C0+C1x+C2x2++Ck1xk1)erxy=(C_0+C_1x+C_2x^2+\cdots+C_{k-1}x^{k-1})e^{rx}
  2. r=α±βir=\alpha\pm\beta ikk 重共轭复根(此时特征方程的根一共有2k2k 个),则通解为:
    y=eαx[(C0+C1x++Ck1xk1)cosβx+(D0+D1x++Dk1xk1)sinβxy=e^{\alpha x}[(C_0+C_1x+\cdots+C_{k-1}x^{k-1})\cos\beta x+(D_0+D_1x+\cdots+D_{k-1}x^{k-1})\sin\beta x

有了齐次的通解,接下来我们可以根据解的结构,考虑非齐次的通解,即

y+py+qy=f(x)y''+py'+qy=f(x)

讨论以下几种类型:

  • f(x)=Pm(x)eλxf(x)=P_m(x)e^{\lambda x}
    其中,Pm(x)P_m(x) 表示关于xxmm多项式。其特解为:

y=xkRm(x)eλx,k={0,λ 不是特征根1,λ 为单重特征根s,λ 是 s 重特征根y^*=x^kR_m(x)e^{\lambda x},k=\begin{cases}0,&\lambda\text{ 不是特征根}\\1,&\lambda\text{ 为单重特征根}\\\vdots\\s,&\lambda\text{ 是 }s\text{ 重特征根}\end{cases}

  • f(x)=eλx[Pl(x)cosωx+Qn(x)sinωx]f(x)=e^{\lambda x}\left[P_l(x)\cos\omega x+Q_n(x)\sin\omega x\right]
    其中,Pl(x),Qn(x)P_l(x),Q_n(x) 分别表示关于xxll 次和nn多项式。其特解为:

y=xkeλx[Rm(1)(x)cosωx+Rm(2)(x)sinωx]y^*=x^ke^{\lambda x}\left[R_m^{(1)}(x)\cos\omega x+R_m^{(2)}(x)\sin\omega x\right]

特解中k:=1(λ+ωi=r)k:=1(\lambda+\omega i=r) 指示λ+ωi\lambda+\omega i 是否为特征根,是则取k=1k=1,否则取0。
Rm(1,2)(x)R_m^{(1,2)}(x) 表示xxmm 次多项式,其中,m=max{l,n}m=\max\{l,n\}.

注: 事实上第二种类型就是将第一种类型推广到复数。即f(x)=Pm(x)e(λ+ωi)xf(x)=P_m(x)e^{(\lambda+\omega i)x}.
借助欧拉公式eiθ=cosθ+isinθe^{i\theta}=\cos\theta+i\sin\theta 展开得来。

相关技巧与欧拉方程

特殊问题的求解技巧

♾️自变量因变量互换

在求解微分方程时,还需跳脱出惯性思维,考虑互换x,yx,y 的情况。将原微分方程化为形如:
x+P(y)x=Q(y);x=f(xy);\begin{aligned}x'+P(y)x=Q(y);\\x'=f(\frac x y);\\\cdots\end{aligned}
的形式,灵活求解。
不仅如此,很多时候如果遇见了不属于前面所介绍的形式时,还需考虑自行挖掘题目中的换元关系。


♾️含有变限积分的方程

求解此类方程一般的做法就是对等式两边同时求导,化为求解常微分方程。
值得注意的是,以积分方程给出的题,往往其 初始条件 已经蕴含其中,考生应当自行挖掘。


♾️已知通解反求微分方程问题

已知通解F(x,y,C)=0F(x,y,C)=0,一般可将通解两边对xx 求导,视yyxx 的函数:

Fx(x,y,C)+Fy(x,y,C)y=0F'_x(x,y,C)+F'_y(x,y,C)y'=0

将其与通解联立,消去任意常数CC 即可得到微分方程。
如果含有nn 个任意常数Ci,i=1,2,..,nC_i,i=1,2,..,n,则可求nn 次导,得到关于CiC_i 的方程组以进行消元


🦄欧拉(Euler)方程

一般变系数的微分方程都不宜求解,而一些特殊的变系数微分方程可以通过变量代换转化为常系数微分方程。欧拉方程就是这样一种微分方程形式。其一般形式为:

xny(n)+p1xn1y(n1)++pn1xy+pny=f(x)x^ny^{(n)}+p_1x^{n-1}y^{(n-1)}+\cdots+p_{n-1}xy'+p_ny=f(x)

做变换x=et(x>0)x=e^t(x\gt0),将自变量换成tt,用记号DD 表示对tt 的求导运算ddt\begin{aligned}\frac{\mathrm d}{\mathrm dt}\end{aligned}.
DD微分算子,对应地,有Dn:=dndtn\begin{aligned}D^n:=\frac{\mathrm d^n}{\mathrm dt^n}\end{aligned}
则有:

xky(k)=D(D1)(Dk+1)yx^ky^{(k)}=D(D-1)\cdots(D-k+1)y

于是就可将其化为常系数微分方程进行求解了。


好题发现

问题f(x)f(x)(,+)(-\infty,+\infty) 上有定义,f(x)0,f(0),x,yR,f(x+y)f(x)f(y)f(x)\neq0,\exists f'(0),\forall x,y\in R,f(x+y)\equiv f(x)·f(y).
f(x)f(x)

答案

已知条件并未给出f(x)f(x) 对任意x,yx,y 都可导。因此不能通过两边求导的方式化为微分方程进行求解。
考虑从f(0)f'(0) 存在入手,先设法求f(0)f(0),进而利用导数的定义寻找化为微分方程的途径。

x=0x=0 时,有:

f(y)=f(0)f(y)&&f(y)0f(0)=1Then,f(x+y)f(x)y=f(x)[f(y)f(0)]ylimy0f(x)[f(y)f(0)]y=f(x)f(0)=f(x)=limy0f(x+y)f(x)y\begin{aligned}&f(y)=f(0)f(y)\quad \&\&\quad f(y)\neq0\quad\Rightarrow f(0)=1\\\\\text{Then,}&\quad \frac{f(x+y)-f(x)}{y}=\frac{f(x)[f(y)-f(0)]}{y}\\\\\therefore&\quad\lim_{y\to0}\frac{f(x)[f(y)-f(0)]}{y}=f(x)f'(0)=f'(x)=\lim_{y\to0}\frac{f(x+y)-f(x)}{y}\end{aligned}

解此微分方程,得到:f(x)=Cef(0)xf(x)=Ce^{f'(0)x},再由f(0)=1f(0)=1C=1C=1.

从而解得所求函数:f(x)=ef(0)xf(x)=e^{f'(0)x}


问题sinydydx=(1xcosy)cosy\begin{aligned}\sin y·\frac{dy}{dx}=(1-x\cos y)\cos y\end{aligned} 的通解

答案

原微分方程可化为:d(cosy)dx=(1xcosy)cosy\begin{aligned}-\frac{d(\cos y)}{dx}=(1-x\cos y)\cos y\end{aligned}
不妨令u=cosyu=\cos y,则可进一步化为:

dudx+u=xu2\frac{du}{dx}+u=xu^2

这是一个uu 关于xx伯努利方程。从而可以做如下变换进行求解:

u2dudx+u1=xd(u1)dxu1=xu1=edx((x)edx+C)=x+1+Cex\begin{aligned}u^{-2}\frac{du}{dx}+u^{-1}&=x\\\frac{d(u^{-1})}{dx}-u^{-1}&=-x\\u^{-1}=e^{-\int-dx}\left(\int(-x)e^{\int-dx}+C\right)&=x+1+Ce^x\end{aligned}

即通解为:secy=Cex+x+1\begin{aligned}\sec y = Ce^x+x+1\end{aligned}.


问题 设曲线LL极坐标方程r=r(θ)r=r(\theta)M(r,θ)M(r,\theta)LL 上任意一点,M0(2,0)M_0(2,0)LL 的一定点。
极径OM,OM0OM,OM_0 与曲线LL 所围成的 曲边扇形 面积值等于LLM0,MM_0,M 两点间弧长的一半,求曲线LL 的极坐标方程。(取材自《 2023李正元数学复习全书 》)

答案rsin(π6±θ)=1\begin{aligned}r\sin\left(\frac \pi 6\pm\theta\right)=1\end{aligned}

※微分算子法及其应用

🦄微分算子法

在上面欧拉方程的求解中,我们引入了微分算子的概念,以此简化在换元过程中复杂的转化问题。事实上,微分算子不仅可以用于简化欧拉方程的计算,还可以对二元常系数线性微分方程求特解的问题有着奇效。

对于:

y+py+qy=f(x)y''+py'+qy=f(x)

引入微分算子,得(D2+pD+q)y=f(x)(D^2+pD+q)y=f(x).
我们令P(D)=D2+pD+qP(D)=D^2+pD+q ,称其为算子多项式,就有:

y=f(x)P(D)y=\frac{f(x)}{P(D)}

对于算子多项式,有着诸多性质可便于我们反解出特解,我们可以像之前一样,对不同的几种f(x)f(x) 类型分别利用算子多项式处理.

  • 逆算子位移原理

1P(D)eλxv(x)=eλx1P(D+λ)v(x)\begin{aligned}\frac1{P(D)}e^{\lambda x}v(x)=e^{\lambda x}\frac1{P(D+\lambda)}v(x)\end{aligned}

其中,λ\lambda 可以是复数,而v(x)v(x) 是关于xx实函数

通过该原理,我们可以推导出许多结论,这将在后续各种f(x)f(x) 类型中得出。

注:x/Dx/D 表示对xx 一次积分


  • f(x)=Pn(x)f(x)=P_n(x)
    对于二阶常系数非齐次线性微分方程,若f(x)f(x) 是关于xxnn 次多项式,则有:

y=Pn(x)D2+pD+qy^*=\frac{P_n(x)}{D^2+pD+q}

我们将分式1/(D2+pD+q)1/(D^2+pD+q) 展开成泰勒级数直到nn 项(因为超过nn 阶,则Pn(x)P_n(x) 的高阶导数的结果均为0),或利用多项式除法直到列出的商到nn 次。最后再逐一计算。

例题:求y+y=x2x+2y''+y=x^2-x+2 的特解。

(D2+1)y=x2x+2y=x2x+2D2+1  11+D2=11(D2)=1D2+(D2)2+y=(1D2)(x2x+2)=x2x+2(x2x+2)=x2x\begin{aligned}(D^2+1)y&=x^2-x+2\\\\y^*&=\frac{x^2-x+2}{D^2+1}\\\\\because\;\frac{1}{1+D^2}=\frac1{1-(-D^2)}&=1-D^2+(-D^2)^2+\cdots\\\\\therefore y^*=(1-D^2)(x^2-x+2)&=x^2-x+2-(x^2-x+2)''\\&=x^2-x\end{aligned}


  • f(x)=eλxf(x)=e^{\lambda x}

y=eλxP(D)={eλxP(λ),P(λ)0λ 不为特征根 xmeλxP(m)(λ),λ 为 m 重根y^*=\frac{e^{\lambda x}}{P(D)}=\begin{cases}\begin{aligned}\frac{e^{\lambda x}}{P(\lambda)}\end{aligned}&,P(\lambda)\neq0\Leftrightarrow\lambda\text{ 不为特征根 }\\\\\begin{aligned}\frac{x^me^{\lambda x}}{P^{(m)}(\lambda)}\end{aligned}&,\lambda\text{ 为 }m\text{ 重根}\end{cases}

其中,重根的情况可以利用移位定理,也可以用上式中的求mm 阶导的方法。事实上后者正是由前者推导得来的。

例题:求y5y+8y4y=e2xy'''-5y''+8y'-4y=e^{2x} 的特解。

P(D)=D35D2+8D4,  P(2)=0P(D)=3D210D+8,  P(2)=0P(D)=6D10,  P(2)=20    y=e2xP(D)=x2e2xP(2)=12x2e2x\begin{aligned}P(D)&=D^3-5D^2+8D-4,\;P(2)=0\\P'(D)&=3D^2-10D+8,\;P'(2)=0\\P''(D)&=6D-10,\;P''(2)=2\neq0\\\\\therefore \;\;y^*&=\frac{e^{2x}}{P(D)}=x^2\frac{e^{2x}}{P''(2)}=\frac12x^2e^{2x}\end{aligned}


  • f(x)=Pn(x)eλxf(x)=P_n(x)e^{\lambda x}

利用逆算子位移原理,将eλxe^{\lambda x} 项左移到算子左边,从而问题化为f(x)=Pn(x)f(x)=P_n(x) 型。

例题:求y4y+4y=x2e2xy''-4y'+4y=x^2e^{2x} 的特解。

P(D)=D24D+4=(D2)2y=1(D2)2x2e2x=e2x(1[(D+2)2]2x2)=e2x(1D2x2)=e2x112x4=112x4e2x\begin{aligned}P(D)&=D^2-4D+4=(D-2)^2\\y^*&=\frac1{(D-2)^2}x^2e^{2x}\\&=e^{2x}\left(\frac1{[(D+2)-2]^2}x^2\right)\\&=e^{2x}\left(\frac1{D^2}x^2\right)\\&=e^{2x}·\frac1{12}x^4\\&=\frac1{12}x^4e^{2x}\end{aligned}


  • f(x)=Pn(x)cosax,f(x)=Pn(x)sinaxf(x)=P_n(x)\cos ax,f(x)=P_n(x)\sin ax

此类问题可充分利用欧拉公式eiθ=cosθ+isinθe^{i\theta}=\cos\theta+i\sin\theta 直接把问题化为前面的类型。
微分算子法对复数同样适用

此外,还有对于三角函数,和eλxe^{\lambda x} 型类似的,可利用如下结论(sinx\sin x 同理):

1f(D2)cosax={1f(a2)cosax,f(a2)0xmf(m)(D2)cosax,a2 为 m 重根\frac1{f(D^2)}\cos ax=\begin{cases}\begin{aligned}\frac1{f(-a^2)}\cos ax\end{aligned}&,f(-a^2)\neq0\\\\\begin{aligned}\frac{x^m}{f^{(m)}(D^2)}\cos ax\end{aligned}&,-a^2\text{ 为 }m\text{ 重根}\end{cases}

一定要注意,此结论中的分母是关于D2D^2 这个整体的多项式函数。否则该公式不适用!!

👉🏻特别地,对于f(x)=xv(x)f(x)=xv(x) 型,其中v(x)v(x) 为关于xx 的实函数,还有:

1P(D)xv(x)=(xP(D)P(D))1P(D)v(x)\frac1{P(D)}xv(x)=\left(x-\frac{P'(D)}{P(D)}\right)\frac1{P(D)}v(x)

例题:求yy=xcosxy''-y=x\cos x 的特解。

y=1D21xcosx=(x2DD21)1D21cosx=xcosxD211(D21)2(2Dcosx)=xcosx(1)121[(1)1]2(sinx)=x2cosx+12sinx\begin{aligned}y^*&=\frac1{D^2-1}x\cos x\\&=\left(x-\frac{2D}{D^2-1}\right)\frac1{D^2-1}\cos x\\&=x·\frac{\cos x}{D^2-1}-\frac1{(D^2-1)^2}(2D\cos x)\\&=x\frac{\cos x}{(-1)-1}-2\frac{1}{[(-1)-1]^2}(-\sin x)\\&=-\frac x2\cos x+\frac12\sin x\end{aligned}


  • f(x)=eλxcosax,f(x)=eλxsinaxf(x)=e^{\lambda x}\cos ax,f(x)=e^{\lambda x}\sin ax

熟悉以上各类问题之后,剩下的问题都可以转化为上面的类型。如下面这道题:

例题:求y6y+13y=e3xsin2xy''-6y'+13y=e^{3x}\sin 2x 的特解。

P(D)=D26D+13P(D+3)=(D+3)26(D+3)+13=D2+4y=1P(D)e3xsin2x=e3x1P(D+3)sin2x=e3x1D2+4sin2x=e3x1D2+4Im(e2ix)=e3xIm(e2ix1(D2i)2+4)=e3xIm(e2ix1D(D+4i)1)\begin{aligned}P(D)&=D^2-6D+13\\P(D+3)&=(D+3)^2-6(D+3)+13=D^2+4\\\\y^*&=\frac1{P(D)}e^{3x}\sin 2x=e^{3x}\frac1{P(D+3)}\sin2x\\&=e^{3x}\frac1{D^2+4}\sin 2x=e^{3x}\frac1{D^2+4}\text{Im}(e^{2ix})\\&=e^{3x}·\text{Im}\left(e^{2ix}\frac1{(D-2i)^2+4}\right)\\&=e^{3x}·\text{Im}\left(e^{2ix}\frac1{D(D+4i)}1\right)\\\end{aligned}

式中,Im()\text{Im}(·) 表示取虚部,对应地,Re()\text{Re}(·) 表示取实部。

下面继续讨论虚部中的内容如何处理:

e2ix1D1D+4i1=e2ix1D(1D+4ie0)=e2ix1D(e010+4i)=e2ix(1Di4)=e2ixi4x=14(cos2x+isin2x)(ix)=14xcos2xi+14xsin2x\begin{aligned}e^{2ix}\frac1D\frac1{D+4i}1&=e^{2ix}\frac1D\left(\frac1{D+4i}e^0\right)\\&=e^{2ix}\frac1D\left(e^0\frac1{0+4i}\right)\\&=e^{2ix}\left(-\frac1D\frac{i}{4}\right)\\&=-e^{2ix}\frac{i}{4}x\\&=-\frac14(\cos2x+i\sin2x)(ix)\\&=-\frac14x\cos2x·i+\frac14x\sin2x\\\end{aligned}

于是有:Im(e2ix1D(D4i)1)=14xcos2x\begin{aligned}\text{Im}\left(e^{2ix}\frac1{D(D-4i)}1\right)=-\frac14x\cos2x\end{aligned},从而:

y=x4e3xcos2xy^*=-\frac{x}4e^{3x}\cos2x


  • f(x)=Pn(x)eλxcosax,f(x)=Pn(x)eλxsinaxf(x)=P_n(x)e^{\lambda x}\cos ax,f(x)=P_n(x)e^{\lambda x}\sin ax

类似地,利用欧拉公式将三角函数与eλxe^{\lambda x} 结合,然后利用位移原理处理。

例题:求y2y+2y=xexcosxy''-2y'+2y=xe^{x}\cos x 的特解。

先考虑复数情况的微分方程:

xexcosx=Re(xe(1+i)x)For   y2y+2y=xe(1+i)x,  P(D)=D22D+2P(D+(1+i))=(D+2i)D  y=e(1+i)x1D(D+2i)x=e(1+i)x1D[D2i(D+2i)(D2i)x]=e(1+i)x1D(D2iD2+4e0x)=e(1+i)x1D(e0D2i02+4x)=e(1+i)x1D(14i2x)=e(1+i)x(14x14ix2)\begin{aligned}xe^x\cos x&=\text{Re}\left(xe^{(1+i)x}\right)\\\\\text{For }\;y''-2y'+2y&=xe^{(1+i)x},\;P(D)=D^2-2D+2\\\\\Rightarrow P(D+(1+i))&=(D+2i)D\\\\\therefore\;y^*&=e^{(1+i)x}\frac1{D(D+2i)}x\\&=e^{(1+i)x}\frac1D\left[\frac{D-2i}{(D+2i)(D-2i)}x\right]\\&=e^{(1+i)x}\frac1D\left(\frac{D-2i}{D^2+4}·e^0x\right)\\&=e^{(1+i)x}\frac1D\left(e^0·\frac{D-2i}{0^2+4}x\right)\\&=e^{(1+i)x}\frac1D\left(\frac14-\frac{i}2x\right)\\&=e^{(1+i)x}\left(\frac14x-\frac1{4}ix^2\right)\\\end{aligned}

最后再对其取实部,即为所求:

y=14Re(e(1+i)x(xix2))=ex4(xcosx+x2sinx)y^*=\frac14\text{Re}\left(e^{(1+i)x}(x-ix^2)\right)=\frac{e^x}4(x\cos x+x^2\sin x)


参考:

  1. 微分方程算子法
  2. 微分算子法总结与例题|百度:高等数学吧
线性微分方程解的结构

线性微分方程解的结构

叠加原理 设非齐次微分方程的右端f(x)f(x) 是两个函数之和(如下)

y+P(x)y+Q(x)y=f1(x)+f2(x)y''+P(x)y'+Q(x)y=f_1(x)+f_2(x)

y1(x),y2(x)y^*_1(x),y^*_2(x) 分别是下列方程的特解

y+P(x)y+Q(x)y=f1(x)y+P(x)y+Q(x)y=f2(x)\begin{aligned}y''+P(x)y'+Q(x)y=f_1(x)\\y''+P(x)y'+Q(x)y=f_2(x)\end{aligned}

则,y1(x)+y2(x)y^*_1(x)+y^*_2(x) 就是原方程的特解。

解的性质

  • 如果y1(x),y2(x)y_1(x),y_2(x) 是二阶齐次线性微分方程的两个特解,则其线性组合C1y1(x)+C2y2(x)C_1y_1(x)+C_2y_2(x)也是该方程的解
  • ✨ 如果y1(x),y2(x)y_1(x),y_2(x) 是二阶非齐次线性微分方程的两个特解,则其y1(x)y2(x)y_1(x)-y_2(x) 是该方程对应的齐次方程的解
  • 如果y(x)y^*(x) 是二阶非齐次线性微分方程的特解,y(x)y(x) 是其对应的齐次方程任意特解,则二者之和y(x)+y(x)y^*(x)+y(x) 也是原非齐次线性微分方程的解。

解的结构
如果y1(x),y2(x)y_1(x),y_2(x) 是二阶齐次线性微分方程的两个线性无关的特解,而y(x)y^*(x) 是对应非齐次方程的特解。则该二阶非齐次线性微分方程的通解为:

y(x)=C1y1(x)+C2y2(x)+y(x)y(x)=C_1y_1(x)+C_2y_2(x)+y^*(x)

🎲 这些结论均可以推广到高阶线性微分方程中,并且与线性代数中线性方程组的解的性质十分类似,可以类比记忆。关于线性代数的部分可参考本文相关部分!

※解微分方程过程中是否省略绝对值问题

解微分方程时的绝对值

在求解微分方程时,特别是可分离变量型和一阶线性微分方程,通常会涉及到两边积分的情况。而由1/x  dx=lnx+C\int1/x\;dx=\ln|x|+C 可知,积分时会引入绝对值。
但是在各类参考书的参考答案中,缺乏对绝对值的讨论,甚至有些过程直接省略了绝对值直接得到答案。所以,我们接下来将引入幂函数的拓展,以及对两类方程求解时如何讨论绝对值问题做一个总结。

实数范围内的幂函数

对于幂函数y=xay=x^a,有:

  1. x>0x\gt0 时,aR\forall a\in R 都有yy 有意义;
  2. x<0x\lt0 时,若a=n2ma=\frac n {2m} (nn2m2m 均为整数且互质),或aa 为无理数时,yy 在实数范围内无意义。如:(1)π,2(-1)^\pi,\sqrt{-2}

可分离变量型的判定

结论:对于可分离变量微分方程,方程中没有无理常数因子的时候,可以直接不加绝对值。

如:xy=yxy'=y. 加上绝对值并通过分类讨论将正负含于常数CC中的结果,与直接无视绝对值的耍赖做法,结果一样。
2xy=y\sqrt{2}xy'=y 则不同。其正确做法如下:

2dyy=dxx2lny=lnx+C1y2=eC1x=±eC1x=Cx(C=±eC1)\begin{aligned}\sqrt{2}\frac{dy}{y}&=\frac{dx}{x}\\\sqrt{2}\ln|y|&=\ln|x|+C_1\\|y|^{\sqrt 2}&=e^{C_1}|x|=\pm e^{C_1}x\\&=Cx\quad(C=\pm e^{C_1})\end{aligned}

若采用不加绝对值的耍赖法,则只能得到y2=Cxy^{\sqrt 2}=Cx ,缩小了yy 的取值范围。

一阶常系数线性型的判定

结论:当P(x)P(x)没有无理常数因子也没有偶数的分母时,P(x)dx\int P(x)dx 积分中如果出现了对数函数 ,可以直接不写绝对值.

如:dydxyx+1=x+1\begin{aligned}\frac{dy}{dx}-\frac{y}{x+1}=x+1\end{aligned}. 加上绝对值并通过分类讨论将正负含于常数CC中的结果,与直接无视绝对值的耍赖做法,结果一样。
dydxy2x=x\begin{aligned}\frac{dy}{dx}-\frac{y}{2x}=x\end{aligned} 则不同。其正确做法如下:

y=e1/(2x)  dx(xe1/(2x)  dx  dx+C)=x(xx  dx+C)x>0:y=23x2+Cxx<0:y=23x2+Cxy=23x2+Cx\begin{aligned}y&=e^{\int1/(2x)\;dx}\left(\int xe^{\int-1/(2x)\;dx}\;dx+C\right)\\&=\sqrt{|x|}\left(\int\frac{x}{\sqrt{|x|}}\;dx+C\right)\\&\begin{aligned}x\gt0:y&=\frac{2}{3}x^2+C\sqrt{x}\\x\lt0:y&=\frac{2}{3}x^2+C\sqrt{-x}\end{aligned}\Rightarrow y=\frac{2}{3}x^2+C\sqrt{|x|}\end{aligned}

参考:【微分方程绝对值】|▷心一学长

级数敛散性判别

概念、性质与常见的重要级数

定义

如果常数项级数n=1un\sum_{n=1}^{\infty}u_n 的部分和数列{Sn}\{S_n\} 的极限存在,则称该级数收敛,否则就发散
称极限值SS 为级数的 rn=SSnr_n=S-S_n 为级数的余项

基本性质

对于常数项级数n=1un\sum_{n=1}^{\infty}u_n ,其一般项unu_n 满足:
如果 级数n=1un\sum_{n=1}^{\infty}u_n 收敛,则limnun=0\lim_{n\to \infty} u_n = 0.

注意,该结论是级数收敛的必要条件,而非充分条件。
limnun=0\lim_{n\to \infty} u_n = 0 \nRightarrow 收敛
但是反过来,如果limnun0\lim_{n\to \infty} u_n \neq 0 \Rightarrow 发散.

常见的重要级数

  1. 几何级数n=0qn\begin{aligned}\sum_{n=0}^\infty q^n\end{aligned},当q<1|q|\lt 1 时收敛,此时S=11q\begin{aligned}S=\frac 1 {1-q}\end{aligned},反之发散
  2. p 级数n=01/np\begin{aligned}\sum_{n=0}^\infty 1/n^p\end{aligned},当p>1p \gt 1 时收敛,反之发散
  3. n=21np(lnn)q\begin{aligned}\sum_{n=2}^\infty\frac 1 {n^p(\ln n)^q}\end{aligned} ,当p>1p\gt 1p=1  &  q>1p=1\;\&\;q \gt 1 时收敛,p<0p\lt 0p=1  &  q1p=1\;\&\;q\leq 1 时发散

问题 判断级数n=21np(lnn)q\sum\limits_{n=2}^{\infty}\frac 1 {n^p(\ln n)^q} 的敛散性.

证明过程

p1p\neq 1 时,与1/nα1/n^\alpha 运用比值判别法:

limn(1np(lnn)q/1nα)=limn1npα(lnn)q={0,p>α,p<α\lim_{n\to\infty}\left(\frac 1 {n^p(\ln n)^q}\big/\frac 1 {n^\alpha}\right)=\lim_{n\to\infty}\frac 1 {n^{p-\alpha}(\ln n)^q}=\begin{cases}0,&p\gt\alpha\\\infty,&p\lt\alpha\end{cases}

从而,当p>α>1p\gt\alpha\gt 1 时,级数收敛;当p<α<1p\lt\alpha\lt 1 时,级数发散.

p=1p=1 时,分两种情况。

  1. q0q\leq 0 时,原级数的一般项变为了lnqn/n>1/nα\ln^{-q} n/n \gt 1/n^\alpha,则原级数发散;
  2. q>0q\gt 0 时,引入函数f(x)=1/(x(lnx)q)f(x)=1/(x(\ln x)^q) ,该函数在[3,+)[3,+\infty)为正值且单调递减,从而有:

3+f(x)dx=3+1xlnqxdx=ln3+dttq{q>1,convergence,0<q1,divergence.\int_3^{+\infty}f(x)\mathrm dx=\int_3^{+\infty}\frac 1 {x\ln^qx}\mathrm dx=\int_{\ln 3}^{+\infty}\frac {\mathrm dt}{t^q}\quad\begin{cases}q\gt 1,&\text{convergence,}\\0 \lt q\leq 1,&\text{divergence.}\end{cases}

综上所述,当p>1p\gt 1p=1  &  q>1p=1\;\&\;q \gt 1 时收敛,p<0p\lt 0p=1  &  q1p=1\;\&\;q\leq 1 时发散.

常用的各类判别方法

比较判别法

对于正项级数,设cvnun0  (nN),  c>0cv_n \geq u_n \geq 0\;(n \geq N),\;c\gt 0,若n=1vn\sum\limits_{n=1}^{\infty}v_n 收敛,则n=1un\sum\limits_{n=1}^{\infty}u_n 收敛;若n=1un\sum\limits_{n=1}^{\infty}u_n 发散,则n=1vn\sum\limits_{n=1}^{\infty}v_n 发散.

由上述比较原理,可衍生出其极限形式

  • un>0,vn>0u_n \gt 0,v_n \gt 0,若limnvnun=A\begin{aligned}\lim_{n\to \infty}\frac{v_n}{u_n}=A\end{aligned},则有:
    • 0<A<+0 \lt A \lt +\infty 时,即unAvn(n)u_n\sim Av_n(n\to \infty) 时,n=1vn,n=1un\sum\limits_{n=1}^{\infty}v_n, \sum\limits_{n=1}^{\infty}u_n 同敛散;
    • A=0A=0 时,即un=o(vn)u_n = o(v_n) 时,n=1vn\sum\limits_{n=1}^{\infty}v_n 收敛可推出n=1un\sum\limits_{n=1}^{\infty}u_n 收敛;
    • A=A=\infty 时,即un=ω(vn)u_n=\omega(v_n), 也就是vn=o(un)v_n=o(u_n) 时,n=1un\sum\limits_{n=1}^{\infty}u_n 发散可推出n=1vn\sum\limits_{n=1}^{\infty}v_n 发散.

特别地,与pp 级数比较,即如果limnun1np=A\begin{aligned}\lim_{n\to\infty}\frac{u_n}{\frac 1 {n^p}}=A\end{aligned}

  1. 0A<+,  p>10\leq A \lt +\infty,\; p\gt 1,则级数收敛;
  2. 0A+,  p10\leq A \leq +\infty,\; p\leq 1,则级数发散.

比值判别法(达朗贝尔判别法)|D’Alembert’s test

正项级数(Positive Series)的发散与收敛(Divergence and Convergence)还有一种判别方法叫做比值判别法,又叫检比法,达朗贝尔判别法,D’Alembert’s test.

其内容如下:

if limnun+1un=ρ{<1,Series n=1un is convergent>1,Series n=1un is divergent, and limnun=+=1,invalid\text{if }\lim_{n\to \infty}\frac{u_{n+1}}{u_n}=\rho\begin{cases}\lt 1,&\text{Series } \sum\limits_{n=1}^{\infty}u_n\text{ is convergent}\\\gt1,&\text{Series } \sum\limits_{n=1}^{\infty}u_n\text{ is divergent, and }\lim_{n\to\infty}u_n=+\infty\\=1,&\text{invalid}\end{cases}

根值判别法

与比值判别法类似地,我们还有根值判别法:

if limnunn=ρ{<1,Series n=1un is convergent>1,Series n=1un is divergent, and limnun=+=1,invalid\text{if }\lim_{n\to \infty}\sqrt[n]{u_n}=\rho\begin{cases}\lt 1,&\text{Series } \sum\limits_{n=1}^{\infty}u_n\text{ is convergent}\\\gt1,&\text{Series } \sum\limits_{n=1}^{\infty}u_n\text{ is divergent, and }\lim_{n\to\infty}u_n=+\infty\\=1,&\text{invalid}\end{cases}

李正元复习全书 》中给出了一个很有用的结论:

limn2n1n=1\lim_{n\to\infty}\sqrt[n]{2n-1}=1

该结论可以结合根值判别法充分利用在求级数敛散性,乃至函数项级数的收敛域问题上。

积分判别法

如果f(x),x1\exists f(x),x\geq 1f(x)f(x)单调下降的正值函数,使得对于正项级数n=1un\sum\limits_{n=1}^{\infty}u_nun=f(n),n=1,2,3,u_n=f(n),n=1,2,3,\cdots,则:

n=1un=n=1f(n) is convergent1+f(x)dx is convergent.\sum_{n=1}^{\infty}u_n=\sum_{n=1}^{\infty}f(n) \text{ is convergent} \Leftrightarrow \int_1^{+\infty}f(x)\mathrm dx \text{ is convergent.}

莱布尼茨判别法|Leibniz Theorem

un>0u_n \gt 0,则称n=1(1)n1un=u1u2+u3\sum\limits_{n=1}^{\infty}(-1)^{n-1}u_n=u_1-u_2+u_3-\cdots交错级数

区别于上述三种只针对正项级数的判别方法,Leibniz Theorem给出了交错级数的敛散性判别:

如果交错级数n=1(1)n1un\sum\limits_{n=1}^{\infty}(-1)^{n-1}u_n 满足下列条件:
(I)unun+1,  nN1u_n \geq u_{n+1},\;n\geq N\geq 1
(II)limnun=0\lim\limits_{n\to\infty}u_n=0.
则该交错级数收敛,且其和s<u1,  rn<un+1s\lt u_1,\; |r_n|\lt u_{n+1}.

绝对收敛和条件收敛

下面讨论unu_n 为任意项的级数n=1un\sum\limits_{n=1}^{\infty}u_n

  • 绝对收敛|absolutely convergent:对于n=1un\sum\limits_{n=1}^{\infty}u_n,如果级数n=1un\sum\limits_{n=1}^{\infty}|u_n| 收敛则称其绝对收敛。
  • 条件收敛|conditionally convergent:如果级数n=1un\sum\limits_{n=1}^{\infty}u_n 收敛但是n=1un\sum\limits_{n=1}^{\infty}|u_n| 发散,则称其条件收敛。

此外还有以下结论:

  1. 绝对收敛的级数一定收敛:n=1un is convergentn=1un is convergent.\sum\limits_{n=1}^{\infty}|u_n|\text{ is convergent} \Leftrightarrow \sum\limits_{n=1}^{\infty}u_n \text{ is convergent.}
  2. 条件收敛的级数其全部正项(或负项)构成的级数一定发散:
    if n=1un is conditionally convergent, then n=112(un+un)  &n=112(unun) are both divergent.\text{if } \sum\limits_{n=1}^{\infty}u_n\text{ is conditionally convergent, then }\\ \sum\limits_{n=1}^{\infty}\frac 1 2 (u_n+|u_n|) \;\& \sum\limits_{n=1}^{\infty}\frac 1 2(u_n-|u_n|) \text{ are both divergent.}

Tips:

  1. 如果由比值或根值判别法得出n=1un\sum\limits_{n=1}^{\infty}|u_n| 发散,则n=1un\sum\limits_{n=1}^{\infty}u_n 必发散
  2. 绝对收敛级数 + 条件收敛级数 = 条件收敛级数
拓展:对数判别法

待更

🦄常数项级数的审敛与选择题考点

常数项级数的审敛通用步骤

Step1. 先判断常数项级数的类型:正项级数、交错级数、任意项级数

Step2. 如果是正项级数,首先考察limnun={0,divergence,=0,turn Step3.\lim\limits_{n\to\infty}u_n=\begin{cases}\neq0,\text{divergence,}\\=0,\text{turn Step3.}\end{cases},否则 转到 Step4

Step3. 根据unu_n 特点,选择合适的判别法:

  1. n!n!nn 的乘积,通常选择比值判别法
  2. 含以nn 为指数幂因子的项,通常选择根值判别法
  3. 含形如nαn^\alpha (α\alpha 可以不是整数),通常选择 比较判别法
  4. 一般项可以利用已知的正项单调减函数f(x)f(x) 表示,并且函数敛散性可判断时,选择积分判别法
  5. 以上方法行不通时,根据定义判断{Sn}\{S_n\} 是否有上界.

Step4. 如果是交错级数,利用莱布尼茨判别法判断敛散性。若不满足条件,或不是交错级数,转 Step5

Step5. 根据 Step2 和 Step3 判断一般项为un|u_n| 的级数的敛散性,如果收敛,则原级数绝对收敛,否则其根据比值/根值判别法计算得到ρ>1\rho \gt 1,则原级数发散,若不然,转 Step6

Step6. 讨论{S2n},{S2n+1}\{S_{2n}\},\{S_{2n+1}\} 的敛散性,以及其他能判断敛散性的方法,如加括号构造新级数等.

对于 Step6,有如下实用的结论:

 If there is limnun=0, and limnS2n=S, then limnSn=S\text{ If there is } \lim_{n\to\infty}u_n=0,\text{ and }\lim_{n\to\infty}S_{2n}=S,\text{ then }\lim_{n\to\infty}S_n=S

上面的S2nS_{2n} 也可以换成S2n+1S_{2n+1},结论依然成立。

证明也很简单,当unu_n极限为0,S2nS_{2n}极限存在并且为SS 时,S2n+1S_{2n+1} 的极限就是S2n+u2n+1S_{2n}+u_{2n+1} 的极限,仍为SS。因此,原级数收敛。

类似地,两两加括号、奇偶变换都能转化为S2nS_{2n} 的形式,从而进行敛散性判别。

对一般项的各类操作

  1. 如果由比值或根值判别法得出n=1un\sum\limits_{n=1}^{\infty}|u_n| 发散,则n=1un\sum\limits_{n=1}^{\infty}u_n 必发散
  2. 收敛+收敛=收敛;发散+收敛=发散;发散+发散=?
  3. 绝对+绝对=绝对;条件+绝对=条件;条件+条件=?
  4. 💦一般项加绝对值会提高发散的可能性
  5. 💦正项级数变为交错级数会提高收敛的可能性
  6. 💦加括号会提高收敛的可能性
  7. 💦提高一般项的阶数会提高收敛的可能性

其中,4~7 所给出的内容可以帮助我们判断级数审敛选择题的正确与否,从而构筑反例

加括号类反例

  1. an\sum a_n 收敛,(a2n1+a2n)\sum(a_{2n-1}+a_{2n}) 也收敛
  2. an\sum a_n 收敛,(a2n1a2n)\sum(a_{2n-1}-a_{2n}) 可能发散,如an=(1)n1/na_n=(-1)^{n-1}/n 使得后者变为1/n\sum 1/n
  3. (a2n1+a2n)\sum(a_{2n-1}+a_{2n}) 收敛,当且仅当limnan=0\lim\limits_{n\to\infty}a_n=0 时,an\sum a_n 收敛
  4. (un+vn)\sum (u_n+v_n) 收敛,则取un=vn=1u_n=-v_n=1 时,un,vn\sum u_n,\sum v_n 发散

乘积类反例

  1. bn\sum b_n 发散,anbn\sum a_nb_n 不一定发散,当an=0a_n=0 时必收敛
  2. bn\sum b_n 收敛,anbn\sum a_nb_n 不一定收敛,当二者都是交错级数时,乘积得到的正项可能不收敛,比如当an=bn=(1)n/na_n=b_n=(-1)^{n}/\sqrt n 时必发散
  3. 若存在常数p,  p>1,  s.t.limnnpanp,\;p\gt1,\;s.t.\lim\limits_{n\to\infty}n^pa_n 存在,则an1/npa_n\sim 1/n^pan=o(1/np)a_n=o(1/n^p) 因此,an\sum a_n 收敛
  4. 反过来,若an\sum a_n 收敛,不能反推limnpan\lim n^pa_n 存在。因为 级数的收敛与发散没有分界线,没有一种标尺分辨级数的收敛和发散(除非限定是pp 级数)。当然,对于该问题还可以举反例:an=1/(nln2n)a_n=1/(n\ln^2n).

奇偶类反例

  1. an\sum a_n 收敛,则{Sn},{S2n},{S2n1}\{S_n\},\{S_{2n}\},\{S_{2n-1}\} 均收敛,且收敛于同一个值
  2. an\sum a_n 收敛,但a2n,a2n1\sum a_{2n},\sum a_{2n-1} 可能发散(拎出收敛级数的奇数项、偶数项)
  3. a2n,a2n1\sum a_{2n},\sum a_{2n-1} 均收敛,则an\sum a_n 收敛(收+收=收)
  4. {S2n},{S2n1}\{S_{2n}\},\{S_{2n-1}\} 均收敛,则an\sum a_n 不一定收敛(已知条件不一定收敛于同一值。反例:an=(1)na_n=(-1)^n

最大最小类反例

an,bn\sum a_n,\sum b_n 收敛,则max{an,bn},min{an,bn}\sum \max\{a_n,b_n\},\sum \min\{a_n,b_n\} 不一定收敛

取反例:an=(1)n1n=bn\begin{aligned}a_n=\frac{(-1)^{n-1}}{n}=-b_n\end{aligned} 此时,n=1max{an,bn}=n=11/n=n=1min{an,bn}\sum\limits_{n=1}^{\infty}\max\{a_n,b_n\}=\sum\limits_{n=1}^{\infty}1/n=-\sum\limits_{n=1}^{\infty}\min\{a_n,b_n\}
二者均发散。

敛散性无法判断

【经典例题】设级数n=1anxn\sum\limits_{n=1}^{\infty}a_nx^nx=2x=2 处条件收敛,则n=1ann+1(x1)n\sum\limits_{n=1}^{\infty}\frac{a_n}{n+1}(x-1)^nx=1x=-1 处的敛散性:
只能视{an}\{a_n\} 的情况确定。

保号性证明敛散性

收敛半径与收敛域的求解

收敛半径与收敛域

对于幂级数n=1an(xx0)n\sum\limits_{n=1}^{\infty}a_n(x-x_0)^n,我们可以令t=xx0t=x-x_0 将其化简为n=1anxn\sum\limits_{n=1}^{\infty}a_nx^n的形式进行求解,二者收敛半径下同,收敛域相差x0x_0.

收敛域 是由所有的收敛点构成的集合,而收敛点如x0x_0 是指:使得常数项级数n=1un(x0)\sum\limits_{n=1}^{\infty}u_n(x_0) 收敛的点,并且x0x_0 需要满足x0{i=1DiDi is domain of function ui(x)}x_0\in\{\bigcap_{i=1}^{\infty}D_i|D_i\text{ is domain of function }u_i(x)\}.

收敛半径 是幂级数级数收敛与发散的临界点与零点的距离RR.
即有n=1anxn is convergent where x(R,R)\sum\limits_{n=1}^{\infty}a_nx^n \text{ is convergent where } x\in (-R,R)

下面给出幂级数的收敛半径与收敛域的求解步骤:

  1. 使用比值法或根值法计算收敛半径limnan+1an=1/R or limnann=1/R\lim\limits_{n\to\infty}\left|\frac{a_{n+1}}{a_n}\right|=1/R\text{ or }\lim\limits_{n\to\infty}\sqrt[n]{|a_n|}=1/R
  2. 0<R<+0\lt R \lt +\infty 时,讨论x=±Rx=\pm Rn=1anxn\sum\limits_{n=1}^{\infty}a_nx^n 的敛散性;
  3. 根据第 2 步确定收敛域.

注意 上面给出的只是幂级数的求解方略,因此该方法的适用条件必须是幂级数。
对于其他类的函数项级数则需要回到级数审敛法的一般思路。
例如:n=1unx2n\sum\limits_{n=1}^{\infty} u_nx^{2n} 就需要作变量替换t=x2t=x^2 化为幂级数,抑或直接对这个级数整体进行分析,甚至可能需要对xx 分类讨论(如n=1ln(1+n)/nx\sum\limits_{n=1}^{\infty} \ln(1+n)/n^x),从而得到收敛域.

含三角函数的级数审敛

通过±nπ\pm n\pi 的方法提取出形如(1)n1(-1)^{n-1} 项,将原问题化为 交错级数 的问题。充分利用放缩方法,通过比较判别法证明敛散性。


问题{xn}\{x_n\} 是单调递增且有界的正数数列,试判断级数n=1(1xnxn+1)\sum_{n=1}^\infty\left(1-\frac {x_n}{x_{n+1}}\right) 的敛散性.

答案

由题意可知:

01xnxn+1=xn+1xnxn+1xn+1xnx10\leq 1-\frac{x_n}{x_{n+1}}=\frac{x_{n+1}-x_n}{x_{n+1}}\leq\frac{x_{n+1}-x_n}{x_1}

从而,原级数的部分和SnS_n 为:

Sn=n=1(1xnxn+1)n=1(xn+1xnx1)=xn+1x1x1S_n=\sum_{n=1}^\infty\left(1-\frac{x_n}{x_{n+1}}\right)\leq\sum_{n=1}^\infty\left(\frac{x_{n+1}-x_n}{x_1}\right)=\frac{x_{n+1}-x_1}{x_1}

因为{xn}\{x_n\} 有界,不妨设xnM|x_n|\leq M,则有:

Sn=xn+1x1x1Mx1+1|S_n|=|\frac{x_{n+1}-x_1}{x_1}|\leq \frac M {|x_1|}+1

{Sn}\{S_n\} 也有界,可得SnS_n 的极限存在,即原级数收敛.


问题 判断(1+12)(1+122)(1+123)(1+12n)(1+\frac 1 2)(1+\frac 1 {2^2})(1+\frac 1 {2^3})·\cdots·(1+\frac 1 {2^n}) 的敛散性

答案

注:本题常在各大论坛套以求极限的形式进行出题钓鱼。即要求计算:

limn(1+12)(1+122)(1+123)(1+12n)=?\lim_{n\to\infty}(1+\frac 1 2)(1+\frac 1 {2^2})(1+\frac 1 {2^3})·\cdots·(1+\frac 1 {2^n})=?

而事实上,这个极限虽然我们上面能证明收敛,却得不到具体值
不过如果将之改为如下形式,就可以做了。

limn(1+12)(1+122)(1+124)(1+122n)=?\lim_{n\to\infty}(1+\frac 1 2)(1+\frac 1 {2^2})(1+\frac 1 {2^4})·\cdots·(1+\frac 1 {2^{2n}})=?

求解方法为左乘一个(112)(1-\frac 1 2) 逐步利用平方差公式进行化简。

❤️根据本题的求解不难总结出,对于无穷乘积,我们有推论:
无穷乘积n=1(1+an)\prod_{n=1}^\infty(1+a_n) 与 无穷级数n=1an\sum_{n=1}^\infty a_n 同敛散

对于其他无穷乘积的内容,可参阅下列文章进行扩展:https://www.docin.com/p-2011398918.html

和函数问题

和函数的性质

和函数的连续性收敛域DD 上的和函数S(x)S(x) 必连续,所以其在收敛区间端点处的极限存在。若不存在,则说明收敛域不包含该点。

逐项求导公式 如果和函数S(x)S(x)(R,R)(-R,R) 可导,则有如下公式,且收敛半径仍为RR

S(x)=(n=0anxn)=n=0(anxn)=n=1nanxn1S'(x)=\left(\sum_{n=0}^\infty a_nx^n\right)'=\sum_{n=0}^\infty(a_nx^n)'=\sum_{n=1}^\infty na_nx^{n-1}

逐项积分公式 如果和函数S(x)S(x) 在收敛域DD 上连续,则有如下公式,且收敛半径不变

0xS(t)dt=0x(n=0antn)dt=n=0(0xantndt)=n=0ann+1xn+1(xD)\int_0^xS(t)\mathrm dt=\int_0^x\left(\sum_{n=0}^\infty a_nt^n\right)\mathrm dt=\sum_{n=0}^\infty\left(\int_0^xa_nt^n\mathrm dt\right)=\sum_{n=0}^\infty\frac{a_n}{n+1}x^{n+1}\quad(x\in D)

注意

  1. 逐项求kk 次导后级数下标变成了n=kn=k
  2. 逐项求导后的级数在x=R(R)x=R(-R)可能发散
  3. 若原级数在x=R(R)x=R(-R) 发散,则其逐项求导后的级数在此处一定发散
  4. 若原级数在x=R(R)x=R(-R) 发散,则其逐项积分后的级数在此处也有可能收敛.

牛顿-莱布尼茨公式的正确运用

对于有些不便求幂级数展开的函数,可以先对其求导,再对其导数进行展开,之后再利用逐项积分的方式还原。
值得注意的是,这个“积回去”的过程实质是牛顿-莱布尼茨公式的运用,从而正确的写法是:

S(x)S(a)=axS(x)dxS(x)-S(a)=\int_a^xS'(x)\mathrm dx

特别地,当x=0x=0 时,我们有:S(x)=S(0)+0xS(x)dxS(x)=S(0)+\int_0^xS'(x)\mathrm dx.
很多时候往往有S(0)=0S(0)=0 ,但这并不具备一般性,做题时理应特别关注,避免忽略计算S(0)S(0).

有些辅导教材甚至对这个部分有一些错误的解释,如规定有:

S(x)=S(x)dx=φ(x)+CS(x)=\int S'(x)\mathrm dx=\varphi(x)+C

然后要求做题者代入0对常数项 C 进行计算从而得出结果。
虽然结果是对的,但是概念上是完全错误的!!!

参考:https://www.docin.com/p-66971719.html

和函数的求解技巧

对于和函数的求解,通常我们需要将所求级数进行代数运算,特别地,要充分利用以下几种方式进行拆分或化简,尽量将原式化为我们熟知的形式(即常见泰勒级数形式):

  • 合并整体换元
  • 加项再减项化简
  • 乘项再除项化简
  • 分母有理化
  • 部分分式分解
  • 先求导再积分,先积分再求导

❤️ 一类换元技巧:n=0xnan+b\begin{aligned}\sum_{n=0}^{\infty}\frac{x^n}{an+b}\end{aligned}

n=0xnan+b=n=0(xa)anan+b=t=xa1tbn=0tan+ban+b\sum_{n=0}^{\infty}\frac{x^n}{an+b}=\sum_{n=0}^{\infty}\frac{(\sqrt[a]{|x|})^{an}}{an+b}\xlongequal{t=\sqrt[a]{|x|}}\frac 1 {t^b}\sum_{n=0}^\infty\frac{t^{an+b}}{an+b}

❤️ 一种常见的已知幂级数复合结论:

ln(1+x)=x12x2+13x3+ln(1x)=x+12x213x312ln(1+x1x)=x+13x3++12n1x2n1+12ln(1x1+x)=12x2+14x4++12nx2n+\begin{aligned}\ln(1+x)&=x-\frac 1 2x^2+\frac1{3}x^3+\cdots\quad&①\\\ln(1-x)&=-x+\frac 1 2x^2-\frac1{3}x^3-\cdots\quad&②\\\\①-②\Rightarrow\frac 1 2\ln\left(\frac{1+x}{1-x}\right)&=x+\frac1{3}x^3+\cdots+\frac1{2n-1}x^{2n-1}+\cdots\\②-①\Rightarrow\frac 1 2\ln\left(\frac{1-x}{1+x}\right)&=\frac1{2}x^2+\frac1{4}x^4+\cdots+\frac1{2n}x^{2n}+\cdots\\\end{aligned}

其中,对于级数的展开要特别注意xx 的收敛区间。有的时候,我们可以通过先求得收敛区间,根据收敛区间的形式指导我们如何对式子进行化简。

类似地,对于函数的幂级数展开也同样利用上述思想进行化简和求解。

此外,对于常数项级数的和的求解有如下策略供参考:

  1. 直接计算部分和的极限limSn\lim S_n
  2. 转化为求幂级数和函数在x=x0x=x_0 处的函数值问题;
  3. 借助傅里叶级数展开式求解,具体见下一个板块.
与阶乘和双阶乘有关的和函数

阶乘变换的和函数→解微分方程

纵观我们已经所学过的各种常用的泰勒级数展开式,分母都没有例如(2n)!(3n)!(2n+1)!!(2n)!、(3n)!、(2n+1)!! 这类对阶乘做变换的式子。如n=0x2n(2n)!\begin{aligned}\sum_{n=0}^\infty\frac{x^{2n}}{(2n)!}\end{aligned}.
不难发现,如果对其求一次导,就可以化为n=1x2n1(2n1)!\begin{aligned}\sum_{n=1}^\infty\frac{x^{2n-1}}{(2n-1)!}\end{aligned},这与原式有着密不可分的关系。
因此,这类需要借助求导找关系得出结果的和函数问题,常常可以转化为一个 微分方程问题

【例题1】求S(x)=n=0x2n(2n)!\begin{aligned}S(x)=\sum_{n=0}^\infty\frac{x^{2n}}{(2n)!}\end{aligned}

【解】省略收敛域的验证,对其依次求导以寻求关系:

S(x)=1+x22!+x44!+S(x)=0+x1+x33!+S(x)=1+x22!+x44!+\begin{aligned}S(x)&=1+\frac{x^2}{2!}+\frac{x^4}{4!}+\cdots\\S'(x)&=0+\frac x 1+\frac{x^3}{3!}+\cdots\\S''(x)&=1+\frac{x^2}{2!}+\frac{x^4}{4!}+\cdots\\\end{aligned}

不难发现,当求到二阶导时,我们发现和函数的形式出现了轮回。从而我们可以得到微分方程:S(x)=S(x)S(x)=S''(x).
这是一个二阶常系数齐次线性微分方程。其特征方程为:r21=0r^2-1=0 ,所以我们得到:

S(x)=C1ex+C2exS(x)=C_1e^{-x}+C_2e^x

再由已知条件S(0)=1,S(0)=0S(0)=1,S'(0)=0 定出C1,C2C_1,C_2。最终得到S(x)=ex+ex2S(x)=\frac{e^x+e^{-x}}{2}.

【例题2】求S(x)=n=0x3n(3n)!\begin{aligned}S(x)=\sum_{n=0}^\infty\frac{x^{3n}}{(3n)!}\end{aligned}

【解】省略收敛域的验证,对其依次求导以寻求关系:

S(x)=1+x33!+x66!+x99!S(x)=0+x22!+x55!+x88!+S(x)=0+x11!+x44!+x77!+\begin{aligned}S(x)&=1+\frac{x^3}{3!}+\frac{x^6}{6!}+\frac{x^9}{9!}\cdots\\S'(x)&=0+\frac{x^2}{2!}+\frac{x^5}{5!}+\frac{x^8}{8!}+\cdots\\S''(x)&=0+\frac{x^1}{1!}+\frac{x^4}{4!}+\frac{x^7}{7!}+\cdots\\\end{aligned}

不难发现,当求到二阶导时,我们发现和函数的形式把n!n! 遍历了一遍。从而我们可以得到微分方程:S(x)+S(x)+S(x)=n=0xn/n!=exS(x)+S'(x)+S''(x)=\sum\limits_{n=0}^{\infty}x^n/n!=e^x.

显然有特解:y=S=S=S=ex/3y^*=S=S'=S''=e^x/3

再解此微分方程的齐次解,最后代入初始条件,最终得到:

S(x)=23ex2cos32x+ex3S(x)=\frac 2 3e^{-\frac x 2}\cos\frac{\sqrt 3}{2}x+\frac{e^x}{3}

事实上,对于【例题2】,如果我们求导到三阶,就能得到微分方程S(x)=S(x)S(x)=S'''(x),但这并非考研范围内能处理的微分方程,因此并没有这样处理。
如果硬是要求该微分方程,可以得到特征方程:r3=1r^3=1,解得r=1,1/2±3/2r=1,-1/2\pm\sqrt 3/2.
进而设S(x)=C1e1x+e1/2(C2cos32x+C3sin32x)\begin{aligned}S(x)=C_1e^{1x}+e^{-1/2}(C_2\cos\frac{\sqrt 3}{2}x+C_3\sin\frac{\sqrt 3}{2}x)\end{aligned},从而求出结果。

其实对于【例题1】,也可以用类似【例题2】的“累加前几次求导结果”的方法,得出S(x)+S(x)=exS(x)+S'(x)=e^x.

【例题3】求S(x)=n=0(2n)!!(2n+1)!!x2n+1\begin{aligned}S(x)=\sum_{n=0}^\infty\frac{(2n)!!}{(2n+1)!!}x^{2n+1}\end{aligned}

【解】省略收敛域的验证,对其依次求导以寻求关系:

S(x)=x+23x3+4×25×3x5+6×4×27×5×3x7+S(x)=1+2x2+4×23x4+6×4×25×3x6+\begin{aligned}S(x)&=x+\frac{2}{3}x^3+\frac{4\times2}{5\times3}x^5+\frac{6\times4\times2}{7\times5\times3}x^7+\cdots\\S'(x)&=1+2x^2+\frac{4\times2}{3}x^4+\frac{6\times4\times2}{5\times3}x^6+\cdots\\\end{aligned}

到此我们发现,xx 的系数分子分母项数不平衡了,因此继续求导反而会将问题复杂化,我们不妨寻求已知式子的关系。
由于S(x)S'(x) 都是偶数项,S(x)S(x) 都是奇数项,考虑对S(x)S(x) 升阶并求导,这就发现了二者之间的关系:

[xS(x)]=S(x)1[xS(x)]'=S'(x)-1

最后解此微分方程,并结合S(0)=0S(0)=0,得:

S(x)=arcsinx1+x2S(x)=\frac{\arcsin x}{\sqrt{1+x^2}}

或许你已经注意到,本题的结果正是我们在本文高阶导数的求解专栏中出现的难题。
所以,我们可以既而利用泰勒展开公式与幂级数一一对应的性质将结果给出。


动手试一试 求幂级数n=111×3×5××(2n1)\sum_{n=1}^\infty\frac 1 {1\times3\times5\times\cdots\times(2n-1)} 的和

答案

设如下函数项幂级数的和函数:

S(x)=n=1x2n11×3×5××(2n1)S(x)=\sum_{n=1}^\infty\frac {x^{2n-1}}{1\times3\times5\times\cdots\times(2n-1)}

现对其逐项求导,得

S(x)=n=1x2n21×3×5××(2n3)=1+xn=1x2n11×3×5××(2n1)S'(x)=\sum_{n=1}^\infty\frac{x^{2n-2}}{1\times3\times5\times\cdots\times(2n-3)}=1+x\sum_{n=1}^\infty\frac {x^{2n-1}}{1\times3\times5\times\cdots\times(2n-1)}

S(x)=1+xS(x)S'(x)=1+xS(x),又S(0)=0S(0)=0,从而解此微分方程得到:

S(x)=e12x20xe12t2dtS(x)=e^{\frac 1 2x^2}\int_0^xe^{-\frac 1 2t^2}\mathrm dt

构造本身和函数方程的问题

双求和变换的和函数→解方程

【例题】求S(x)=n=0(1+12++1n)xn\begin{aligned}S(x)=\sum_{n=0}^\infty\left(1+\frac 1 2+\cdots+\frac 1 n\right)x^{n}\end{aligned}

斐波那契数列

🦄柯西乘积

下面,我们利用柯西乘积公式对上面的例题进行求解:

S(x)=n=0(k=1n1k)xn=n=0(k=1n1kxn)=n=0(k=1n1kxkxnk)=(n=0xnn)(n=0xn)=ln(1x)1x\begin{aligned}S(x)&=\sum_{n=0}^\infty\left(\sum_{k=1}^n\frac 1 k\right)x^n=\sum_{n=0}^\infty\left(\sum_{k=1}^n\frac 1 kx^n\right)\\&=\sum_{n=0}^\infty\left(\sum_{k=1}^n\frac 1 kx^k·x^{n-k}\right)\\&=\left(\sum_{n=0}^\infty\frac {x^n}{n}\right)\left(\sum_{n=0}^\infty x^n\right)\\&=-\frac{\ln(1-x)}{1-x}\end{aligned}

傅里叶级数

Dirichlet收敛定理

设函数f(x)f(x) 在区间[l,l][-l,l] 满足:
i. 连续,或只有有限个间断点,且都是第一类间断点;
ii. 只有有限个极值点.
f(x)f(x)[l,l][-l,l] 上的傅里叶级数收敛,此时有:

a02+n=1(ancosnπlx+bnsinnπlx)={f(x),x(l,l) and x is continuity point,12[f(x)+f(x+)],x(l,l) and x is discontinuity point of the first kind ,12[f(l)+f(l+)],x=±l\frac{a_0} 2+\sum_{n=1}^\infty\left(a_n\cos\frac{n\pi}{l}x+b_n\sin\frac{n\pi}{l}x\right)\\=\begin{cases}f(x)&,x\in(-l,l)\text{ and }x \text{ is continuity point,}\\\frac 1 2[f(x^-)+f(x^+)]&,x\in(-l,l)\text{ and }x \text{ is discontinuity point of the first kind ,}\\\frac 1 2[f(l^-)+f(-l^+)]&,x=\pm l\end{cases}

上述定理被称为狄利克雷收敛定理,是傅里叶级数收敛的充分条件
其中,有:

{an=1lllf(x)cosnπlxdx,n=0,1,2,..bn=1lllf(x)sinnπlxdx,n=1,2,3,..\begin{cases}a_n=\frac 1 l\int_{-l}^lf(x)\cos\frac{n\pi} lx\mathrm dx,&n=0,1,2,..\\b_n=\frac 1 l\int_{-l}^lf(x)\sin\frac{n\pi} lx\mathrm dx,&n=1,2,3,..\end{cases}

函数奇偶延拓

傅里叶级数指出能对任何周期函数进行傅里叶级数展开,而对于非周期函数可以通过对其进行奇延拓或者偶延拓的方式,将其看作周期函数再继续级数展开。

特别地,奇函数的傅里叶级数只由正弦函数组成,即为正弦级数,此时an=0a_n=0

f(x)n=1bnsinnπlx,    bn=1lllf(x)sinnπlxdx    n=1,2,3,..f(x)\sim\sum_{n=1}^\infty b_n\sin\frac{n\pi} lx,\;\; b_n=\frac 1 l\int_{-l}^lf(x)\sin\frac{n\pi} lx\mathrm dx\;\;n=1,2,3,..

类似的,偶函数的傅里叶级数是余弦级数bn=0b_n=0

f(x)a02+n=1ancosnπlx,    an=1lllf(x)cosnπlxdx    n=0,1,2,..f(x)\sim\frac {a_0} 2+\sum_{n=1}^\infty a_n\cos\frac{n\pi} lx,\;\; a_n=\frac 1 l\int_{-l}^lf(x)\cos\frac{n\pi} lx\mathrm dx\;\;n=0,1,2,..

式中,因为没有判断收敛性,所以用“\sim”。

问题a0=1,a1=2,a2=72,an+1=(1+1n+1)an(n2)a_0=1,a_1=-2,a_2=\frac 7 2,\quad a_{n+1}=-(1+\frac 1 {n+1})a_n(n\geq2).

证明:当x<1|x|<1时,n=0anxn\sum_{n=0}^{\infty}a_nx^n收敛,并求其和函数S(x)S(x).

答案
  • 先证明收敛性. 由比值判别法:

limnan+1an=limn(1+1n+1)=1\lim_{n\to\infty}\left|\frac{a_{n+1}}{a_n}\right|=\lim_{n\to\infty}(1+\frac 1 {n+1})=1

得:n=0anxn\sum_{n=0}^{\infty}a_nx^n收敛,且收敛半径R=1R=1.

  • 再求数列通项.

an+1=(1+1n+1)an\quad a_{n+1}=-(1+\frac 1 {n+1})a_n\quad, 知an+1/an=(1+1n+1)\quad a_{n+1}/a_n=-(1+\frac 1 {n+1})

an+1=a2(an+1/a2)=a2(1)n+1i=2n(n+2n+1)=(1)n+17(n+2)6a_{n+1}=a_2(a_{n+1}/a_2)=a_2(-1)^{n+1}\prod_{i=2}^n(\frac{n+2}{n+1})=(-1)^{n+1}\frac{7(n+2)}{6}

所以an=(1)n7(n+1)6,n2\quad a_n=(-1)^n\frac{7(n+1)}{6}\quad,n\geq2

  • 最后求解和函数.

S(x)=12x+76n=2(x)n(n+1)=12x+76[1+2x+n=0(x)n(n+1)]=16+13x+76S~(x)where S~(x)=n=0(x)n(n+1)then 0xS~(x)dx=n=00x(x)n(n+1)dx=n=0(1)nxn+1=xn=0(1)nxn=x1+xS~(x)=1(1+x)2S(x)=16+13x+761(1+x)2,1<x<1\begin{aligned}\quad S(x)&=1-2x+\frac 7 6\sum_{n=2}^{\infty}(-x)^n(n+1)\\&=1-2x+\frac 7 6\left[-1+2x+\sum_{n=0}^{\infty}(-x)^n(n+1)\right]\\&=-\frac 1 6+\frac 1 3x+\frac 7 6\tilde{S}(x)\\\\\text{where }&\tilde{S}(x)=\sum_{n=0}^{\infty}(-x)^n(n+1)\\\text{then }&\quad \int_0^x\tilde{S}(x)dx\\&=\sum_{n=0}^{\infty}\int_0^x(-x)^n(n+1)dx\\&=\sum_{n=0}^{\infty}(-1)^nx^{n+1}\\&=x\sum_{n=0}^{\infty}(-1)^nx^n=\frac x {1+x}\\\\&\therefore\quad \tilde{S}(x)=\frac 1 {(1+x)^2}\\\\&\therefore\quad S(x)=-\frac 1 6+\frac 1 3x+\frac 7 6\frac 1 {(1+x)^2}\quad ,-1<x<1\end{aligned}


问题f(x)=arcsinxf(x) = \arcsin xx=0x=0 处展开成幂级数。

答案

线性代数

行列式的相关性质

行列式的定义和性质

完全展开式

基本性质

  1. AT=A|A^T|=|A|
  2. 第一类初等变换| 交换两行(或两列) 行列式的值变号
  3. cA=cnA|cA|=c^n|A|
  4. 第三类初等变换| 某行(列)kk 倍加到另一行(列) 不改变行列式的值
  5. 对一行或一列可分解,例如:α,β1+β2,γ=α,β1,γ+α,β2,γ\boldsymbol{|\alpha,\beta_1+\beta_2,\gamma|=|\alpha,\beta_1,\gamma|+|\alpha,\beta_2,\gamma|}
  6. 如果一个行(列)向量是另一个行(列)向量的倍数,则行列式值为0 (此时矩阵的秩有r(A)n1r(A)\leq n-1

代数余子式

nn阶行列式的第ii行第jj列划去后得到的n1n-1阶行列式称为(i,j)(i,j)位元素aija_{ij}余子式,记作MijM_{ij}.
(1)i+jMij=Aij(-1)^{i+j}M_{ij}=A_{ij}为元素aija_{ij}代数余子式。有:

  • (对某一行或某一列展开的)行列式的值 = 该行(列)各元素与其代数余子式的乘积之和
    以第一行为例,则:

A=a11A11+a12A12++a1nA1n|A|=a_{11}A_{11}+a_{12}A_{12}+\cdots+a_{1n}A_{1n}

  • ii行(列)的各元素与第jj行(列)对应元素的代数余子式乘积之和 = 0 (iji \neq j
    事实上,这相当于求第jj行(列)对应元素完全替换为 第ii行(列) 之后形成的行列式的值。
    结合基本性质6可知,结果为0.
    同时我们还可以得到下面这个推论……

  • b1Ai1+b2Ai2++bnAinb_{1}A_{i1}+b_{2}A_{i2}+\cdots+b_{n}A_{in} = 将行列式A|A|的第ii行所有元素替换为[b1,b2,...,bn][b_1,b_2,...,b_n]之后形成的新行列式B|B|的值。

克拉默法则

在线性方程组的未知数个数等于方程个数,即其系数矩阵nn阶方阵时:
A0|A|\neq 0 是方程组AX=β\boldsymbol{AX=\beta} 有唯一解的充分必要条件。解为:

[D1A,D2A,,DnA]T\left[\frac{D_1}{|A|},\frac{D_2}{|A|},\cdots,\frac{D_n}{|A|}\right]^T

式中,DiD_i是把AA的第ii个列向量替换成常数列向量β\boldsymbol\beta之后所得的行列式的值。

此外还有:A0|A|\neq 0 是方程组AX=0\boldsymbol{AX=0} 只有零解充分必要条件


求解上述方程组的解时,还有一种更方便也更常用的方法:
✨求方程组AX=β\boldsymbol{AX=\beta} 的解时,可对增广矩阵(Aβ)\boldsymbol{(A|\beta)}初等行变换,使得AA化为单位矩阵

(Aβ)(Eη)\boldsymbol{(A|\beta)}\rightarrow \boldsymbol{(E|\eta)}

η\boldsymbol\eta就是解。

✨类似地,如果是方程组XA=β\boldsymbol{XA=\beta},可以对两边转置,得到ATXT=βT\boldsymbol{A}^T\boldsymbol{X}^T=\boldsymbol{\beta}^T

(ATβT)(EηT)\boldsymbol{(A^T|\beta^T)}\rightarrow \boldsymbol{(E|\eta^T)}

注意,此时η\boldsymbol\eta才是解。

行列式的计算

特殊行列式的计算

特殊分块矩阵

范德蒙行列式| Vandermonde

111a1a2ana12a22an2a1n1a2n1ann1=ni>j1(aiaj)\begin{vmatrix} 1&1&\cdots&1\\a_1&a_2&\cdots&a_n\\ a_1^2&a_2^2&\cdots&a_n^2\\\vdots&\vdots&&\vdots\\a_1^{n-1}&a_2^{n-1}&\cdots&a_n^{n-1}\\\end{vmatrix}=\prod_{n\geq i>j\geq 1}(a_i-a_j)

三斜线行列式

爪形行列式


问题 计算行列式:2+x12+x122+x1n2+x22+x222+x2n2+xn2+xn22+xnn\begin{vmatrix}2+x_1&2+x_1^2&\cdots&2+x_1^n\\2+x_2&2+x_2^2&\cdots&2+x_2^n\\ \vdots&\vdots&&\vdots\\2+x_n&2+x_n^2&\cdots&2+x_n^n\end{vmatrix}

矩阵及其可逆性

矩阵乘法和初等矩阵

矩阵乘法

  1. 转置律(AB)T=BTAT,(A+B)T=AT+BT(AB)^T=B^TA^T, (A+B)^T=A^T+B^T
  2. 单位律EA=AE=AEA=AE=A
  3. 当 A, B 都是同阶方阵时:AB=A  B|AB|=|A|\;|B|
  4. 矩阵乘法无交换律,即一般情况下ABBAAB\neq BA
  5. 矩阵乘法无消去律,即一般情况下:
    • AB=0A=0  or  B=0AB=0 \nRightarrow A=0\;\text{or}\;B=0
    • AC=BC  and  C0A=BAC=BC\;\text{and}\;C \neq 0 \nRightarrow A=B(无右消去律)
    • CA=CB  and  C0A=BCA=CB\;\text{and}\;C \neq 0 \nRightarrow A=B(无左消去律)
  6. 方幂运算中,(AB)kAkBk(AB)^k \neq A^kB^k
  7. 关于同一个方阵AA的多项式的乘积可交换:f(A)g(A)=g(A)f(A)f(A)g(A)=g(A)f(A)
  8. 常用的方幂公式EA3=(EA)(E+A+A2)E-A^3=(E-A)(E+A+A^2)
  9. 同阶的两个对角矩阵的乘积就是对角元素对应相乘后得到的新对角矩阵

初等矩阵

对单位矩阵做一次初等行(列)变换所得到的矩阵就是 初等矩阵
三类初等矩阵就对应三类初等变换

  • E(i,j)E(i,j):交换 E 的 i, j 两行(列)所得到的矩阵。iji \neq j
  • E(i(c))E(i(c)):用非0数 c 乘 E 的第 i 行(列)所得到的矩阵。c0c \neq 0
  • E(i,j(c))E(i,j(c)):把 E 第 j 行(列)的 c 倍加到第 i 行(列)所得到的矩阵。iji \neq j

对矩阵做一次初等行变换,相当于对该矩阵左乘一个相应的初等矩阵;
对矩阵做一次初等列变换,相当于对该矩阵右乘一个相应的初等矩阵。

伴随矩阵和可逆矩阵

可逆矩阵

定义AAnn 阶矩阵,如果存在nn 阶矩阵BB,使得AB=E,BA=EAB=E,BA=E,则称AA 为可逆矩阵,BBAA 的逆矩阵,并且唯一,记作A1A^{-1}.

♾️判别nn 阶矩阵AA 可逆\LeftrightarrowA0|A| \neq 0\Leftrightarrowr(A)=nr(A)=n\Leftrightarrow 0不是A的特征值\LeftrightarrowAX=βAX=\beta 有唯一解\LeftrightarrowAX=0AX=0 只有零解

性质
♾️AB=EBA=EAB=E \Leftrightarrow BA=E
该结论可用于很多矩阵表达式是否可逆的证明上,方便快捷

  • (AB)1=B1A1(AB)^{-1}=B^{-1}A^{-1}
  • (AT)1=(A1)T,    (cA)1=c1A1,    (Ak)1=(A1)k(A^T)^{-1}=(A^{-1})^T,\;\;(cA)^{-1}=c^{-1}A^{-1},\;\;(A^k)^{-1}=(A^{-1})^k
  • E(i,j)1=E(i,j)    E(i(c))1=E(i(c1)),    E(i,j(c))1=E(i,j(c))E(i,j)^{-1}=E(i,j)\;\; E(i(c))^{-1}=E(i(c^{-1})),\;\; E(i,j(c))^{-1}=E(i,j(-c))
    即:初等矩阵都是可逆矩阵,且其逆仍是初等矩阵

♾️ 分块对角矩阵的逆

[A1A2An]1=[A11A21An1]\begin{bmatrix}A_{1}&& \\& A_{2}&\\& & \ddots \\& & & A_{n} \end{bmatrix}^{-1}=\begin{bmatrix}A_{1}^{-1}&& \\& A_{2}^{-1}&\\& & \ddots \\& & & A_{n}^{-1} \end{bmatrix}

[A1A2An]1=[An1A21A11]\begin{bmatrix}&&&A_{1}\\&&A_{2}\\& \dots \\ A_{n} \end{bmatrix}^{-1}=\begin{bmatrix}&&&A_{n}^{-1}\\&& \dots \\\\&A_{2}^{-1}\\ A_{1} ^{-1}\end{bmatrix}

♾️计算

  1. 初等变换法:(AE)(EA1)\boldsymbol{(A|E)\rightarrow (E|A^{-1})}
  2. 伴随矩阵法:A1=AA\boldsymbol{A^{-1}=\frac{A^*}{|A|}}
  3. 对角矩阵的逆也是对角矩阵,且对角线元素取倒数

伴随矩阵

定义AAnn阶矩阵,记AijA_{ij}(i,j)(i,j)位元素aija_{ij}代数余子式,规定AA 的伴随矩阵为:

A=(Aij)T=[A11A21An1A12A22An2A1nA2nAnn]\boldsymbol{A^*}=(A_{ij})^T=\begin{bmatrix}A_{11} & A_{21}&\cdots & A_{n1} \\A_{12} & A_{22}&\cdots & A_{n2}\\\vdots & \vdots & \ddots& \vdots \\A_{1n} & A_{2n}& \cdots & A_{nn} \end{bmatrix}

♾️基本公式AA=AA=AEAA^*=A^*A=|A|E
♾️由此基本公式+分块矩阵的行列式与逆的公式可推导出:分块伴随矩阵的结果

性质

  1. A=AA1A^*=|A|A^{-1}
  2. (A)1=A/A=(A1)(A^*)^{-1}=A/|A|=(A^{-1})^*
  3. A=An1|A^*|=|A|^{n-1}
  4. (AB)=BA(AB)^*=B^*A^*
  5. (AT)=(A)T,    (cA)=cn1A,    (Ak)=(A)k,    (A)=An2A(A^T)^{*}=(A^{*})^T,\;\; (cA)^{*}=c^{n-1}A^{*},\;\; (A^k)^{*}=(A^{*})^k,\;\; (A^*)^*=|A|^{n-2}A

若已知nn 阶非零实矩阵AA 有:A=ATA^*=A^T ,则有A=1|A|=1AAT=EAA^T=E.

【证明】首先由A=An1=AT=A|A^*|=|A|^{n-1}=|A^T|=|A|A=0   or   1   or   1|A|=0\;\text{ or }\;1\;\text{ or }\;-1

再由对应元素相等得aij=Aija_{ij}=A_{ij} 其中,AijA_{ij} 为对应的代数余子式

于是:A=a11A11+a12A12++a1nA1n=a112+a122++a1n2>0|A|=a_{11}A_{11}+a_{12}A_{12}+\cdots+a_{1n}A_{1n}=a_{11}^2+a_{12}^2+\cdots+a_{1n}^2\gt 0

从而确定A=1|A|=1

再由AAT=AA=A(AA1)=AE=EAA^T=AA^*=A(|A|A^{-1})=|A|E=E,证毕.

抽象矩阵的可逆证明与计算

配多项式凑定义

已知AA,且有f(A)=0f(A)=0(A+aE)1(A+aE)^{-1} 的问题,一般要回归可逆的定义,对原式进行配凑,期望由f(A)=0(A+aE)B=Ef(A)=0\Rightarrow(A+aE)B=E,进而得出结果。
(当然,除了配凑也可以利用多项式除法/长除法求出BB

【例题1】设AAnn 阶非零矩阵,且A3=0A^3=0,试讨论A+E,EAA+E,E-A 的可逆性。

解:两边同时加上EE,利用立法和公式得:(A+E)(A2A+E)=E(A+E)(A^2-A+E)=E,同理借助立方差公式可得另一个情况。

♾️另解:由f(A)=0λ,f(λ)=0f(A)=0\Rightarrow \forall \lambda,f(\lambda)=0 ,代入本题,有:AA 的全体特征值都满足λ3=0\lambda^3=0。因此,AA 对应的特征值只能是0,即它的所有特征值均为0。
因此,A+EA+E 对应的特征值全为 0+1=1,EAE-A 的特征值全为 1-0=1。
因此,A+E=1n=10|A+E|=1^n=1\neq0 ,所以A+EA+E 可逆,另一情况同理。

【例题2】α,β\alpha,\beta 是三维列向量,(α,β)=1,  A=EαβT(\alpha,\beta)=-1,\;A=E-\alpha\beta^T,求(A+E)1(A+E)^{-1}.

♾️ 分析:遇见两个列向量中αTβ=βTααβT\alpha^T\beta=\beta^T\alpha、\alpha\beta^T 同时出现时,应当敏锐察觉到其特征值、秩等信息(这将在本文 “秩一矩阵” 专栏详细阐述),此外还应该考虑出现乘积复现。即想办法凑出(αβT)2=α(βTα)βT(\alpha\beta^T)^2=\alpha(\beta^T\alpha)\beta^T

解:A=EαβTA2=(EαβT)2=E2αβT+α(βTα)βTA=E-\alpha\beta^T\Rightarrow A^2=(E-\alpha\beta^T)^2=E-2\alpha\beta^T+\alpha(\beta^T\alpha)\beta^T
A2=E3αβT=2A2EA^2=E-3\alpha\beta^T=2A-2E.
从而本问题转化为我们熟悉的配多项式型问题。

恒等式变形推导

已知AA,且有矩阵AABB 的一个或多个已知恒等式条件,求特殊表达式的逆或行列式的问题。这种题一般要充分利用可逆条件,对原式进行恒等式变形,期望能够将待求表达式由已知表达式表示。
这类题的求解我们可以归结为“想办法提公因式”。但下面我将提供一种取巧的方法就是“化矩为数”法。

【例题1】A,BA,B33 阶矩阵,且A=3,B=2,A1+B=2|A|=3,|B|=2,|A^{-1}+B|=2,求A+B1|A+B^{-1}|

♾️ 分析:为了得到结果,我们应该想办法让A+B1A+B^{-1}A,B,A1+BA,B,A^{-1}+B 表示出来,而且还是乘积形式,因为矩阵相加的行列式并没有公式。

解:【提公因式法】将待求表达式左边提出一个AA,右边提出一个B1B^{-1}

A+B1=A(E+A1B1)=A(B+A1)B1A+B^{-1}=A(E+A^{-1}B^{-1})=A(B+A^{-1})B^{-1}

从而得到A+B1=3|A+B^{-1}|=3.

解:【化矩为数法】问题转化成已知:a,b,1a+b\begin{aligned}a,b,\frac1a+b\end{aligned},求a+1b\begin{aligned}a+\frac1b\end{aligned}.
从而得到:

a+1b=ab+1b=abab+1a=ab(1a+b)a+\frac1b=\frac{ab+1}b=\frac ab·\frac{ab+1}{a}=\frac ab·\left(\frac1a+b\right)

之后再代回即可。

【例题2】 已知矩阵AA,且B=(E+A)1(EA)B=(E+A)^{-1}(E-A),求(E+B)1(E+B)^{-1}

采用化矩为数法,有:b=1+a1a\begin{aligned}b=\frac{1+a}{1-a}\end{aligned},求11+b\begin{aligned}\frac{1}{1+b}\end{aligned}.

1+b=1+1+a1a=21a11+b=1a21+b=1+\frac{1+a}{1-a}=\frac{2}{1-a}\Rightarrow\frac1{1+b}=\frac{1-a}2

于是,(E+B)1=0.5(EA)(E+B)^{-1}=0.5(E-A).

不难发现,我们在求解过程中出现了 “通分” 操作,而这在矩阵里相当于把EE 分解为两个互逆矩阵相乘。这也是提公因式法的精髓所在。
因此,我们在真正写步骤的时候可以根据化矩为数法对应的矩阵形式写回去。

乘积可交换的证明

乘积可交换问题,一般是要求证明矩阵A,BA,B 满足AB=BAAB=BA.
这类问题一般通过已知条件构造形如(aA+bE)(cB+dE)=fE(aA+bE)(cB+dE)=fE 的式子解决。事实上这与我们求矩阵可逆的配凑法是一脉相承的。

根据可逆矩阵的可交换性,我们可以通过将我们构造的(aA+bE)(cB+dE)(aA+bE)、(cB+dE) 交换,得到:

  • (cB+dE)(aA+bE)=fE(cB+dE)(aA+bE)=fE
    去括号再与原式作比较即可得到 AB的交换问题。

【例题1】 证明当出现如下情况时,有AB=BAAB=BA

  1. AB=aA+bB,  a,b0AB=aA+bB,\;a,b\neq0
  2. A2+aAB=E,  a0A^2+aAB=E,\;a\neq0

证明:利用配凑法。
(1)由AB=aA+bBABaAbB=0AB=aA+bB\Rightarrow AB-aA-bB=0 更进一步进行配凑,
有:(AbE)(BaE)=abE(BaE)(AbE)=E(A-bE)(B-aE)=abE\Rightarrow (B-aE)(A-bE)=E
从而:ABaAbB=BAaAbBAB=BAAB-aA-bB=BA-aA-bB\Rightarrow AB=BA 证毕。

(2)由A2+aAB=EA(A+aB)=EA^2+aAB=E\Rightarrow A(A+aB)=E 进行交换,
有:(A+aB)A=EA2+aBA=E(A+aB)A=E\Rightarrow A^2+aBA=E 与原式对比即可得到:AB=BAAB=BA,证毕。

【例题2】AAnn 阶可逆矩阵,有ABA=2BA1+EABA^*=2BA^{-1}+E (张宇八套卷)
试证下列结论:

  1. AB=BAAB=BA
  2. BBAA 有完全相同的特征向量;
  3. BBAA 是否相似?请说明理由。

证明:由A=AA1A^*=|A|A^{-1} 代入已知条件,得:

AABA1=2BA1+EAABA2B=0|A|ABA^{-1}=2BA^{-1}+E\Rightarrow|A|AB-A-2B=0

利用 ⌈例题1⌋ 的思想,两边同时乘以 A 的行列式,有:

A2ABAA2AB=0(AA2E)(ABE)=3E|A|^2AB-|A|A-2|A|B=0\Rightarrow(|A|A-2E)(|A|B-E)=3E

将其交换,得:

(ABE)(AA2E)=3EABAA2B=0(|A|B-E)(|A|A-2E)=3E\Rightarrow|A|BA-A-2B=0

从而有AB=BAAB=BA.


下面证明BBAA 有完全相同的特征向量。
由第(1)问可知,r(ABE)0r(|A|B-E)\neq0 ,即1/A1/|A| 不会是BB 的特征值。
下面设BB 的任意特征值为λ\lambda 的特征向量为η\eta ,则:Bη=ληB\eta=\lambda\eta。代入AABA2B=0|A|AB-A-2B=0 得:

λAAηAη2λη=0Aη=2λλA1η\lambda|A|A\eta-A\eta-2\lambda\eta=0\Rightarrow A\eta=\frac{2\lambda}{\lambda|A|-1}\eta

η\eta 也是AA 的特征向量。

同理可以证明AA 的任意特征向量也都是BB 的。因此BBAA 有完全相同的特征向量。


BBAA 不一定相似。有一反例:

A=(1101)B=(A2E)1A=(1201)A=\begin{pmatrix}1&1\\0&1\end{pmatrix}\Rightarrow B=(A-2E)^{-1}A=\begin{pmatrix}-1&-2\\0&-1\end{pmatrix}

AA 的特征值全为1,BB 的特征值全为 -1,显然二者不相似。

齐次方程只有零解证明

🦄问题A\boldsymbol{A}nn 阶实反对称矩阵,证明E+A\boldsymbol{E+A} 可逆. (李正元复习全书)

反对称矩阵|Skew-symmetric Matrix 的定义:

A=(aij)n×nA=(a_{ij})_{n\times n},若满足aij=aji,aii=0,i,j=1,2,,na_{ij}=-a_{ji},a_{ii}=0,i,j=1,2,\dots,n,则AA是反对称矩阵。

因为A\boldsymbol{A}是一个抽象矩阵,利用行列式证明较为困难,因此考虑通过(E+A)X=0\boldsymbol{(E+A)X=0}只有零解 证明

答案

证明η\boldsymbol{\eta}是一个nn维实向量,要证明结论成立只需证明满足(E+A)η=0\boldsymbol{(E+A)\eta=0}时,η=0\boldsymbol{\eta=0}

于是,对上式两边同时左乘ηT\boldsymbol{\eta^{T}},得:

ηT(E+A)η=0ηTη=ηTAη\boldsymbol{\eta^T(E+A)\eta=0}\\\boldsymbol{\eta^T\eta=-\eta^TA\eta}

易知,η1×nTAn×nηn×1\boldsymbol{\eta^T_{1\times n}A_{n\times n}\eta_{n\times1}}这一项的结果是一个,而a=aT,aa=a^T,a是一个数。因此:

ηTAη=(ηTAη)T=ηTAT(ηT)T=ηTAη(AT=A,A是反对称矩阵)\boldsymbol{\eta^TA\eta=(\eta^TA\eta)^T=\eta^TA^T(\eta^T)^T=-\eta^TA\eta}\\(\boldsymbol{A^T}=\boldsymbol{-A},A是反对称矩阵)

联立上式,有ηTη=ηTη\boldsymbol{\eta^T\eta=-\eta^T\eta},又因为η\boldsymbol \eta是实向量,所以等式成立时只有ηTη=0\boldsymbol{\eta^T\eta}=0.

如果令η=(η1,η2,,ηn)T\boldsymbol{\eta}=(\eta_1,\eta_2,\dots,\eta_n)^T,则ηTη=η12+η22++ηn2=0\eta^T\eta=\eta_1^2+\eta_2^2+\dots+\eta_n^2=0

要使得等式成立,当且仅当η1=η2==ηn=0\eta_1=\eta_2=\dots=\eta_n=0成立,即实向量η\boldsymbol{\eta}为零向量。

证毕.

线性关系与秩

线性相关性与秩的概念

线性表示

α1,α2,,αs\boldsymbol{\alpha_1,\alpha_2,\cdots,\alpha_s}是一个nn向量组(I)

  • 如果nn 维向量β\boldsymbol{\beta} 可以用 (I) 线性表示,说明存在数组c1,c2,,csc_1,c_2,\cdots,c_s 使得:

c1α1+c2α2++csαs=βc_1\boldsymbol{\alpha}_1+c_2\boldsymbol{\alpha}_2+\cdots+c_s\boldsymbol{\alpha}_s=\boldsymbol{\beta}

如果令向量c=[c1,c2,,cs]T\boldsymbol{c}=[c_1,c_2,\cdots,c_s]^T,上式可以转化为:Ac=β\boldsymbol{Ac=\beta} 有解。

  • 如果nn 维向量组β1,β2,,βt\boldsymbol{\beta_1,\beta_2,\cdots,\beta_t} 记作 (II) 可以用 (I) 线性表示,说明 (II) 中任意一个向量βi\boldsymbol{\beta}_i 都能被 (I) 线性表示。
    进一步地,存在矩阵 C 使得:AC=B\boldsymbol{AC=B}C 不一定是唯一的,C 唯一的充要条件是 (I) 线性无关

  • 传递性 如果α1,α2,,αs\boldsymbol{\alpha_1,\alpha_2,\cdots,\alpha_s}可以用γ1,γ2,,γr\boldsymbol{\gamma_1,\gamma_2,\cdots,\gamma_r}线性表示,β1,β2,,βt\boldsymbol{\beta_1,\beta_2,\cdots,\beta_t} 又能用α1,α2,,αs\boldsymbol{\alpha_1,\alpha_2,\cdots,\alpha_s}线性表示,那么β1,β2,,βt\boldsymbol{\beta_1,\beta_2,\cdots,\beta_t}就能用γ1,γ2,,γr\boldsymbol{\gamma_1,\gamma_2,\cdots,\gamma_r}线性表示。

  • 等价性 如果β1,β2,,βt\boldsymbol{\beta_1,\beta_2,\cdots,\beta_t} 能用α1,α2,,αs\boldsymbol{\alpha_1,\alpha_2,\cdots,\alpha_s} 线性表示,并且α1,α2,,αs\boldsymbol{\alpha_1,\alpha_2,\cdots,\alpha_s} 又能用β1,β2,,βt\boldsymbol{\beta_1,\beta_2,\cdots,\beta_t}线性表示,那么就说他们等价
    记作:{α1,α2,,αs}{β1,β2,,βt}\{\boldsymbol{\alpha_1,\alpha_2,\cdots,\alpha_s}\} \cong \{\boldsymbol{\beta_1,\beta_2,\cdots,\beta_t}\}
    等价关系也有传递性。

线性相关性

如果向量组α1,α2,,αs\boldsymbol{\alpha_1,\alpha_2,\cdots,\alpha_s} 中任何一个向量都不能用其他s1s-1个向量线性表示,那么称该向量组线性无关,否则就 线性相关

定义 如果存在 不全为0 的一组数c1,c2,,csc_1,c_2,\cdots,c_s使得

c1α1+c2α2++csαs=0c_1\boldsymbol{\alpha}_1+c_2\boldsymbol{\alpha}_2+\cdots+c_s\boldsymbol{\alpha}_s=\boldsymbol{0}

则说明α1,α2,,αs\boldsymbol{\alpha_1,\alpha_2,\cdots,\alpha_s} 线性相关。如果要使等式成立,cic_i必须全为零,则 线性无关.

定理1A=(α1,α2,,αs)\boldsymbol{A=(\alpha_1,\alpha_2,\cdots,\alpha_s)},则:
α1,α2,,αs\boldsymbol{\alpha_1,\alpha_2,\cdots,\alpha_s}线性相关(无关)\LeftrightarrowAX=0\boldsymbol{AX=0}有非零解(无非零解)

定理2 如果α1,α2,,αs\boldsymbol{\alpha_1,\alpha_2,\cdots,\alpha_s}线性无关,则:
α1,α2,,αs,β\boldsymbol{\alpha_1,\alpha_2,\cdots,\alpha_s,\beta}线性相关(无关)\Leftrightarrowβ\boldsymbol{\beta}(不)可以用α1,α2,,αs\boldsymbol{\alpha_1,\alpha_2,\cdots,\alpha_s}线性表示

定理3 如果β\boldsymbol{\beta} 可以用α1,α2,,αs\boldsymbol{\alpha_1,\alpha_2,\cdots,\alpha_s}线性表示,则:
表示方式唯一\Leftrightarrowα1,α2,,αs,β\boldsymbol{\alpha_1,\alpha_2,\cdots,\alpha_s,\beta}线性无关

定理4 向量组整体线性无关\Rightarrow 向量组部分无关;向量组部分相关\Rightarrow 向量组整体相关

定理5 如果α1,α2,,αs\boldsymbol{\alpha_1,\alpha_2,\cdots,\alpha_s}线性无关,将其维度nn 延长到t(tn)t\quad(t\geq n) ,则这个新的向量组α1,α2,,αs\boldsymbol{\alpha'_1,\alpha'_2,\cdots,\alpha'_s} 也线性无关 (即:本身无关,延长也无关)

定理6 如果α1,α2,,αs\boldsymbol{\alpha_1,\alpha_2,\cdots,\alpha_s} 是两两正交非零向量,则该向量组一定线性无关

相同的线性关系

若向量组 (I)(II) 向量个数相同,且二者均线性无关或二者内部有相同的线性表示关系,则称他们有相同的线性关系。

当且仅当:Ax=0,Bx=0Ax=0,Bx=0 同解时,A,BA,B 的列向量组有相同的线性关系。

🎲Tips🎲

  1. 如果向量组只有一个向量,并且该向量组线性有关,说明该向量是零向量
  2. 如果一个向量组中含有零向量,则该向量组必定线性有关
  3. 向量个数ss 大于向量维度nn 时,向量组必线性有关

向量组的秩

α1,α2,,αs\boldsymbol{\alpha_1,\alpha_2,\cdots,\alpha_s}是一个nn向量组(I)
如果 (I) 有线性无关的部分组,那么定义 (I)秩就是其线性无关部分组中,包含的向量个数的最大值,这个向量个数最大的线性无关部分组则称为该向量组的最大无关组
如果没有线性无关部分组(此时向量组内所有向量全为零向量)记秩为0.

根据定义,显然有:0r(α1,α2,,αs)min{s,n}0\leq r(\boldsymbol{\alpha_1,\alpha_2,\cdots,\alpha_s}) \leq \min{\{s,n\}}

向量组等价

  • 向量组等价 如果β1,β2,,βt\boldsymbol{\beta_1,\beta_2,\cdots,\beta_t} 能用α1,α2,,αs\boldsymbol{\alpha_1,\alpha_2,\cdots,\alpha_s} 线性表示,并且α1,α2,,αs\boldsymbol{\alpha_1,\alpha_2,\cdots,\alpha_s} 又能用β1,β2,,βt\boldsymbol{\beta_1,\beta_2,\cdots,\beta_t}线性表示,那么就说他们等价
    记作:{α1,α2,,αs}{β1,β2,,βt}\{\boldsymbol{\alpha_1,\alpha_2,\cdots,\alpha_s}\} \cong \{\boldsymbol{\beta_1,\beta_2,\cdots,\beta_t}\}
    注意:向量个数不一定相等

注:下面各式的 (I)(II) 表示两个向量组.

  1. (I) 可由 (II) 表示,则r(I)r(II)r(I)\leq r(II)(即,被表示的秩不大)
  2. (I) 可由 (II) 表示,但 (II) 不可由 (I) 表示,则r(I)<r(II)r(I)\lt r(II)
  3. (I) 可由 (II) 表示,且 (II) 也可由 (I) 表示,即 (I)(II) 等价时,则有:r(I)=r(II)=r(I,II)r(I)=r(II)=r(I,II)
    注:这个性质是证明两个向量组等价的常用方法,也是两个向量组等价充要条件
  4. (I) 可由 (II) 表示,且 (I) 中向量个数大于 (II) 中向量个数,则 (I) 必定线性相关 (即,以少表多,多必相关)

对于矩阵Am×nA_{m\times n} 若存在可逆阵Pm×mP_{m\times m} 和可逆阵Qn×nQ_{n\times n} 使得PAQ=BPAQ=BAA 等价于矩阵Bm×nB_{m\times n},二者同形。

  • 如果两个矩阵能够用初等变换相互转换,说明两个矩阵等价
  • 矩阵等价的充要条件是他们类型相同且秩相等,即行数列数对应相等并且秩相等
  • 向量组等价不一定推出矩阵等价,因为不一定同形
  • 矩阵等价不一定推出向量组等价,向量组是针对列向量或行向量,矩阵是针对整体

请注意,矩阵等价不等同于向量组等价!!!

矩阵的秩及相关不等式

矩阵的秩

定义 矩阵的秩r(A)r(A) 就是矩阵非零子式的最高阶数,也是其对应行(列)向量的秩。

🎲 n阶矩阵A满秩\Leftrightarrow r(A)=n\Leftrightarrow A的行(列)向量组无关\Leftrightarrow |A|≠0\Leftrightarrow A可逆

🎲 矩阵秩的计算时,可以行列初等变换交替使用

矩阵秩的性质

  1. r(AT)=r(A)=r(AAT)=r(ATA)r(A^T)=r(A)=r(AA^T)=r(A^TA)
  2. r(cA)=r(A),c0r(cA)=r(A),c\neq 0
  3. r(A±B)r(A)+r(B)r(A\pm B)\leq r(A)+r(B)
  4. max{r(A),r(B)}r(A,B)r(A)+r(B)\max \{r(A),r(B)\} \leq r(A,B) \leq r(A)+r(B)
  5. r(AB)min{r(A),r(B)}r(AB)\leq \min{\{r(A),r(B)\}}
  6. A可逆时,r(AB)=r(B)r(AB)=r(B);B可逆时,r(AB)=r(A)r(AB)=r(A)
  7. ♾️A列满秩(A可以不是方阵)时,r(AB)=r(B)r(AB)=r(B);B 行满秩时,r(AB)=r(A)r(AB)=r(A)
  8. 如果AB=0AB=0,那么r(A)+r(B)nr(A)+r(B)\leq n (其中nnAA 的列数BB 的行数)
  9. Sylvester 不等式:r(AB)r(A)+r(B)nr(AB)\geq r(A)+r(B)-n 。可见它是性质 8 的一般形式(其中nnAA 的列数BB 的行数)
  10. r(An+1)=r(An)r(A^{n+1})=r(A^n) 其中,nn 是矩阵AA阶数,而不能任意取

更多秩的不等式及其证明:https://blog.csdn.net/Infinity_07/article/details/109624414

AAm×nm\times n 矩阵,m<n,r(A)=mm\lt n,r(A)=m 。则:存在nn 阶可逆阵PP 使得AP=(Em,O)AP=(E_m,O)

试想,行满秩的长条“平躺”矩阵我们可以通过列变换将其相关的部分全部堆叠到左边

分块矩阵的秩

1.  r(AOOB)=r(A)+r(B)2.  r(ACOB)r(A)+r(B)r(AOCB)r(A)+r(B)(当且仅当 A  or  B 可逆时取等)\begin{aligned}1.\;r\begin{pmatrix}A&O\\O&B\end{pmatrix}&=r(A)+r(B)\\\\2.\;r\begin{pmatrix}A&C\\O&B\end{pmatrix}&\geq r(A)+r(B)\\r\begin{pmatrix}A&O\\C&B\end{pmatrix}&\geq r(A)+r(B)\quad(\text{当且仅当 }A\;or\;B\text{ 可逆时取等})\\\\\end{aligned}

伴随矩阵的秩

r(A)={n,r(A)=n1,r(A)=n10,r(A)<n1r(A^*)=\begin{cases}n,&r(A)=n\\1,&r(A)=n-1\\0,&r(A)\lt n-1\end{cases}

抽象向量组相关性证明

相关性证明问题

  • 当题目给出一个向量组线性无关时,通常会要求判断其线性组合之后的向量组是否线性无关。例如下面这道题:

例题 已知(α1,α2,α3)(\alpha_1,\alpha_2,\alpha_3) 为 3 维列向量,则:
“对k,l\forall k,lα1+kα3\alpha_1+k\alpha_3α2+lα3\alpha_2+l\alpha_3 无关” 是 “(α1,α2,α3)(\alpha_1,\alpha_2,\alpha_3) 无关” 的什么条件?(2014年)

解:若(α1,α2,α3)(\alpha_1,\alpha_2,\alpha_3) 线性无关,则有:

(α1+kα3,α2+lα3)=(α1,α2,α3)(1001kl)(\alpha_1+k\alpha_3,\alpha_2+l\alpha_3)=(\alpha_1,\alpha_2,\alpha_3)\begin{pmatrix}1&0\\0&1\\k&l\end{pmatrix}

记为B=PAB=PA,显然PP 矩阵列满秩,而对于AA 矩阵由 “本身无关,延长必无关”可得,r(A)=2r(A)=2,于是r(B)=2r(B)=2BB 也列满秩,故“对k,l\forall k,lα1+kα3\alpha_1+k\alpha_3α2+lα3\alpha_2+l\alpha_3 无关” 。

必要性得证。

而当α3=0,(α1,α2)\alpha_3=0,(\alpha_1,\alpha_2) 无关 时,显然满足“对k,l\forall k,lα1+kα3\alpha_1+k\alpha_3α2+lα3\alpha_2+l\alpha_3 无关” ,但是(α1,α2,α3)(\alpha_1,\alpha_2,\alpha_3) 线性相关。

综上所述,前者为后者的必要不充分条件。

通过上述例子,我们可以总结出对于这类问题,除了直接利用定义法证明外,我们还可以逆用矩阵乘法,将问题转化为矩阵的秩的问题。


  • 当题目只给出某种线性关系,以及诸如向量α0\alpha\neq0 这样的有限个条件时,通常要求证明其中的向量组线性无关,此时我们应该考虑回归定义,利用定义法,有时还需结合反证法进行证明。

例题AAnn 阶方阵,α\alphann 维列向量,Ak1α0A^{k-1}\alpha\neq0Akα=0A^k\alpha=0 ,证明:α,Aα,,Ak1α\alpha,A\alpha,\cdots,A^{k-1}\alpha 线性无关。

证明:设存在li,    i=0,1,...,k1l_i,\;\;i=0,1,...,k-1 有:

l0α+l1Aα++lk1Ak1α=0l_0\alpha+l_1A\alpha+\cdots+l_{k-1}A^{k-1}\alpha=0

Akα=0Ak+1α=0,Ak+2α=0,A^k\alpha=0\Rightarrow A^{k+1}\alpha=0,A^{k+2}\alpha=0,\cdots,从而对上式两端同时乘以Ak1A^{k-1} 得:

l0Ak1α+l1Akα++lk1A2k2α=0l0Ak1α=0l0=0\begin{aligned}l_0A^{k-1}\alpha&+l_1A^k\alpha+\cdots+l_{k-1}A^{2k-2}\alpha=0\\l_0A^{k-1}\alpha&=0\Rightarrow l_0=0\end{aligned}

同理,同时左乘Ak2A^{k-2} 之后有l1=0l_1=0,以此类推,得到:

l0=l1==lk1=0l_0=l_1=\cdots=l_{k-1}=0

证毕!

该结论可以证明:若nn 阶方阵AAAn+1x=0A^{n+1}x=0 ,则必有Anx=0A^nx=0. 这是因为,如果Anx0A^nx\neq0,就可以推出x,Ax,,Anxx,Ax,\cdots,A^n x 这个n+1n+1 个向量线性无关。这与“nn 维列向量组最多只有nn 个线性无关向量”矛盾。

正因如此,有An+1x=0A^{n+1}x=0 的解必是Anx=0A^nx=0 的解,反过来,不难推出Anx=0A^{n}x=0 的解必是An+1x=0A^{n+1}x=0 的解. 二者同解,即可推出我们矩阵秩的性质10:

r(An)=r(An+1)r(A^n)=r(A^{n+1})


  • 当题目中涉及正交向量时,一定充分利用正交结果为0的特性进行证明。(相当于并没有像上面这类题一样给出已知条件,但其实隐含了条件)

例题nn 维列向量α1,α2\alpha_1,\alpha_2 无关,β1,β2\beta_1,\beta_2 也无关,且对i,j\forall i,j,均有αi\alpha_iβj\beta_j 正交. 证明:α1,α2,β1,β2\alpha_1,\alpha_2,\beta_1,\beta_2 线性无关.(n3n\geq3

证明:设k1α1+k2α2+l1β1+l2β2=0k_1\alpha_1+k_2\alpha_2+l_1\beta_1+l_2\beta_2=0
根据题意,对等式两边同时对k1α1+k2α2k_1\alpha_1+k_2\alpha2 作内积,从而有:k1α1+k2α22=0||k_1\alpha_1+k_2\alpha2||^2=0
k1α1+k2α2k_1\alpha_1+k_2\alpha2 为零向量,所以k1=k2=0k_1=k_2=0。代入原式,得:l1β1+l2β2=0l_1\beta_1+l_2\beta2=0
因为β1,β2\beta_1,\beta_2 线性无关,所以l1=l2=0l_1=l_2=0 ,证毕!

该结论可以拓展到多个向量组。
如果向量组 (Ⅰ) 线性无关,向量组 (Ⅱ) 线性无关,并且 (Ⅰ) (Ⅱ) 相互正交,则两个向量组拼接起来的新向量组也线性无关,当且仅当向量个数小于或等于维数。

该结论可以用于证明:

r(AOOB)=r(A)+r(B)r\begin{pmatrix}A&O\\O&B\end{pmatrix}=r(A)+r(B)

秩的相关计算及其最大无关组

计算秩与最大无关组

将向量组作为列向量组构造出矩阵A=(α1,α2,,αs)\boldsymbol{A=(\alpha_1,\alpha_2,\cdots,\alpha_s)}
对其进行 初等行变换 化为阶梯形的矩阵 B,则 B非0行数就是向量组的秩
B各台角所在的列号对应的部分组就是一个最大无关组
根据 B 中其他列的元素值可以得出 对应列号的向量如何用最大无关组的向量线性表示。

示例:

正交矩阵与向量空间

正交矩阵与向量空间

正交矩阵

定义 如果一个nn实矩阵QQ 满足QQT=EQQ^T=E ,则称其为正交矩阵。

♾️有:QQ 是正交矩阵\LeftrightarrowQQ 的行(列)向量组是单位正交向量组

QQ 是正交矩阵\RightarrowQQ 的实特征值只可能是1  or  1-1\;or\;1
QQ 是正交矩阵\RightarrowQ=1  or  1|Q|=-1\;or\;1
QQ 是正交矩阵QT,Q,Q1\Rightarrow Q^T,Q^*,Q^{-1} 也是正交阵
A,BA,B 是正交矩阵AB,BA\Rightarrow AB,BA 也是正交阵

  • 单位正交向量组:如果向量组中每个向量两两正交,则称该向量组为单位正交向量组
  • 正交:如果(α,β)=0\boldsymbol{(\alpha,\beta)}=0,则说α\alphaβ\beta正交
  • (α,β)=αTβ\boldsymbol{(\alpha,\beta)=\alpha^T\beta} 称为两个实向量的内积
  • 定义实向量α\boldsymbol{\alpha}长度α=(α,α)||\boldsymbol{\alpha}||=\sqrt{(\boldsymbol{\alpha},\boldsymbol{\alpha})}
  • 单位向量:长度为1的向量。
  • 如果α\boldsymbol{\alpha}不是零向量,则α/α\boldsymbol{\alpha}/||\boldsymbol{\alpha}||是单位向量,该操作称为α\boldsymbol{\alpha}的单位化

关于正交矩阵的行列式,通常我们要反复逆用E=AATE=AA^T 以及(A+B)T=AT+BT(A+B)^T=A^T+B^T进行化简。

问题AA 是实反对称矩阵,证明:(EA)(E+A)1(E-A)(E+A)^{-1} 是正交矩阵.

🔔选自 《大学生数学竞赛线代版》陈启浩

答案

这道题再次拿实反对称矩阵出题,希望你还记得它的性质。

证明:要证(EA)(E+A)1(E-A)(E+A)^{-1} 是正交矩阵 只需证:

(EA)(E+A)1[(EA)(E+A)1]T=E(E-A)(E+A)^{-1}[(E-A)(E+A)^{-1}]^T=E

而:

LHS=(EA)(E+A)1[(EA)(E+A)1]T=(EA)(E+A)1[(E+A)T]1(EA)T=(EA)(E+A)1(EA)1(E+A)(AT=A)=(EA)(EA)1(E+A)1(E+A)=E2=E=RHSQ.E.D\begin{aligned}LHS&=(E-A)(E+A)^{-1}[(E-A)(E+A)^{-1}]^T\\&=(E-A)(E+A)^{-1}[(E+A)^T]^{-1}(E-A)^T\\&=(E-A)(E+A)^{-1}(E-A)^{-1}(E+A)\quad(\because A^T=-A)\\&=(E-A)(E-A)^{-1}(E+A)^{-1}(E+A)\\&=E^2=E=RHS\\Q.E.D\end{aligned}

注:(EA)(E+A)=(E+A)(EA)(E-A)(E+A)=(E+A)(E-A) 是可交换的!

问题A,BA,B 为 3 阶正交矩阵,且满足A+B=0|A|+|B|=0,证明:r[(A+B)]1r[(A+B)^*]\leq1

答案

分析待证结论,结合伴随矩阵的秩可以将问题转化为证明(A+B)(A+B) 不满秩,即证A+B=0|A+B|=0.

A,BA,B 为正交阵,因此A=±1,B=±1|A|=\pm1,|B|=\pm1 ,而A+B=0|A|+|B|=0 ,当且仅当二者的行列式互为相反数,不妨设A=1,B=1|A|=1,|B|=-1,于是:

A+B=A  E+ATB=A  BT+AT  B=A+B|A+B|=|A|\;|E+A^TB|=|A|\;|B^T+A^T|\;|B|=-|A+B|

证毕!

施密特正交化

施密特正交化是把线性无关的向量组改造为单位正交向量组的方法。
以 3 个线性无关的向量组α1,α2,α3\alpha_1,\alpha_2,\alpha_3 为例。有:

β1=α1β2=α2(α2,β1)(β1,β1)β1β3=α3(α3,β1)(β1,β1)β1(α3,β2)(β2,β2)β2\begin{aligned}\beta_1&=\alpha_1\\\beta_2&=\alpha_2-\frac{(\alpha_2,\beta_1)}{(\beta_1,\beta_1)}\beta_1\\\beta_3&=\alpha_3-\frac{(\alpha_3,\beta_1)}{(\beta_1,\beta_1)}\beta_1-\frac{(\alpha_3,\beta_2)}{(\beta_2,\beta_2)}\beta_2\end{aligned}

此时得到的β1,β2,β3\beta_1,\beta_2,\beta_3 就是和α1,α2,α3\alpha_1,\alpha_2,\alpha_3 等价的正交非零向量组。
接下来对其单位化:

e1=β1β1e2=β2β2e3=β3β3e_1=\frac{\beta_1}{||\beta_1||}\quad e_2=\frac{\beta_2}{||\beta_2||}\quad e_3=\frac{\beta_3}{||\beta_3||}

e1,e2,e3e_1,e_2,e_3 就是和α1,α2,α3\alpha_1,\alpha_2,\alpha_3 等价的单位正交向量组。

向量空间

Rn\boldsymbol{R}^n 为由全部nn实向量构成的集合,其中规定了加法和数乘两种线性运算,称其为nn维向量空间

V\boldsymbol{V}Rn\boldsymbol{R}^n 的一个子集,如果V\boldsymbol{V} 中任意一组元素的任意线性组合都在V\boldsymbol{V} 中,则称V\boldsymbol{V}Rn\boldsymbol{R}^n 的一个子空间

V\boldsymbol{V}Rn\boldsymbol{R}^n 的一个非0子空间,称V\boldsymbol{V} 的秩为 维数,记作dimV\dim \boldsymbol{V}. 称V\boldsymbol{V} 排了次序的极大无关组为V\boldsymbol{V}

V\boldsymbol{V} 中的每一个元素α\boldsymbol{\alpha} 都能用其 η1,η2,,ηk\boldsymbol{\eta_1,\eta_2,\cdots,\eta_k} 线性表示:

α=c1η1+c2η2++ckηk\boldsymbol{\alpha} = c_1\boldsymbol{\eta}_1+c_2\boldsymbol{\eta}_2+\cdots+c_k\boldsymbol{\eta}_k

系数cic_i 组成的向量(c1,c2,,ck)(c_1,c_2,\cdots,c_k) 就是α\boldsymbol{\alpha}关于这一组基的坐标

如果某个子空间的 基是单位正交向量组,则称其为 规范正交基。
于是,两个向量的内积就等于他们在规范正交基中坐标的内积。

设 某个子空间的其中两个分别为:η1,η2,,ηk,ξ1,ξ2,,ξk\boldsymbol{\eta_1,\eta_2,\cdots,\eta_k},\quad\boldsymbol{\xi_1,\xi_2,\cdots,\xi_k}
显然,后者能够用前者线性表示,前者也能够用后者线性表示。我们还可以说这两组基都是等价的

从而,我们可以构造可逆矩阵CC,使得:(η1,η2,,ηk)C=(ξ1,ξ2,,ξk)(\boldsymbol{\eta_1,\eta_2,\cdots,\eta_k})C=(\boldsymbol{\xi_1,\xi_2,\cdots,\xi_k}).

称为CC 为从Rk\boldsymbol{R^k} 空间中的基η1,η2,,ηk\boldsymbol{\eta_1,\eta_2,\cdots,\eta_k}ξ1,ξ2,,ξk\boldsymbol{\xi_1,\xi_2,\cdots,\xi_k}过渡矩阵

🎲两个 规范正交基 之间的过渡矩阵 是 正交矩阵

线性方程组的解

方程组解的情况与通解的构造

解的情况

对于一般方程组Ax=βAx=\beta,记方程的个数为mm,未知数的个数为nn,(即Am×nA_{m\times n} )有:

  • r(A)<r(Aβ)r(A)\lt r(A|\beta)\Leftrightarrow 无解
  • r(A)=r(Aβ)=nr(A)=r(A|\beta)=n\Leftrightarrow 有唯一解
  • r(A)=r(Aβ)<nr(A)=r(A|\beta)\lt n\Leftrightarrow 有无穷多解
  • 方程的的个数mmr(A),r(Aβ)r(A),r(A|\beta) 的上界;
  • r(A)=mr(A)=m 时,方程组一定有解;
  • m<nm\lt n 时,若有解那也一定不是唯一解。

对于齐次方程组Ax=0Ax=0,有:r(A)=nr(A)=n\Leftrightarrow 只有零解,此时矩阵列满秩。

  • ♾️推论:当AA 列满秩时,AA 在矩阵乘法中有左消去律AB=ACB=CAB=AC\Rightarrow B=C

通解的构造

齐次线性方程组

  1. ξ1,ξ2,,ξs\xi_1,\xi_2,\cdots,\xi_s 是齐次方程组Ax=0Ax=0 的解,则它们的任意线性组合也是该方程组的解。
  2. Ax=0Ax=0 有非零解,则它的解集JJ 是无穷集,称JJ 的最大无关组为Ax=0Ax=0基础解系
  3. ξ1,ξ2,,ξs\xi_1,\xi_2,\cdots,\xi_sAx=0Ax=0 的基础解系,则η\eta 也是它的解η\Leftrightarrow \eta 能用ξ1,ξ2,,ξs\xi_1,\xi_2,\cdots,\xi_s 线性表出。
  4. 由 (3) 可知,Ax=0Ax=0 的通解为:若k1ξ1+k2ξ2++ksξsk_1\xi_1+k_2\xi_2+\cdots+k_s\xi_s ,其中kik_i 为任意实数。
  5. Ax=0Ax=0nn 个未知数,则r(J)=nr(A)r(J)=n-r(A),即基础解系的个数s=nr(A)s=n-r(A)
  6. ♾️由 (5) 可知,若AB=0AB=0,则r(B)r(J)=nr(A)r(A)+r(B)nr(B)\leq r(J)=n-r(A)\Rightarrow r(A)+r(B)\leq n

非齐次线性方程组

  1. Ax=βAx=\beta 的两个解η1,η2\eta_1,\eta_2 之差 一定是Ax=0Ax=0 的解,即A(η1η2)=0A(\eta_1-\eta_2)=0
  2. η\etaAx=βAx=\beta 的一个解,ξ\xiAx=0Ax=0 的一个解,则ξ+η\xi+\eta 也是Ax=βAx=\beta 的解。
  3. 对 (1) 推广:若ξ1,ξ2,,ξs\xi_1,\xi_2,\cdots,\xi_s 是非齐次方程组Ax=βAx=\beta 的解,则
    • ki=1\sum k_i=1 时,k1ξ1+k2ξ2++ksξsk_1\xi_1+k_2\xi_2+\cdots+k_s\xi_s 也是Ax=βAx=\beta 的解;
    • ki=0\sum k_i=0 时,k1ξ1+k2ξ2++ksξsk_1\xi_1+k_2\xi_2+\cdots+k_s\xi_s 是齐次方程组Ax=0Ax=0 的解.
  4. 对 (2) 推广:Ax=βAx=\beta通解ξ0+k1η1+k2η2++ksηs\xi_0+k_1\eta_1+k_2\eta_2+\cdots+k_s\eta_s,其中,ηi\eta_i 是对应齐次方程组Ax=0Ax=0 的基础解系,ξ0\xi_0 是非齐次方程组的一个特解。
  5. 由 (4) 知,非齐次方程组的解集的秩为nr(A)+1n-r(A)+1.

下面给出一道对抽象线性方程组通解求解的例题。虽然看似题中给出了实际的方程组,但是因为参数众多,所以直接求出各个参数并不现实,应该用抽象方程组的思路求解。

例题1


有些时候,系数矩阵的秩往往要通过其通解的结构看出,其次,抽象方程组的特解和通解还需要根据已知的线性条件进行配凑判断。所以应该充分抓紧并利用所给条件。

例题2

公共解和同解问题

方程组公共解

已知Ax=0,Bx=0Ax=0,Bx=0 的表达式,求公共解.

直接联立两个方程组,即求(AB)x=0\begin{pmatrix}A\\B\end{pmatrix}x=0 的通解,该通解即为公共解。

已知Ax=0Ax=0 的表达式,只知道Bx=0Bx=0 的通解(若为)k1ξ1+k2ξ2k_1\xi_1+k_2\xi_2,求公共解.

公共解在满足Bx=0Bx=0 的通解k1ξ1+k2ξ2k_1\xi_1+k_2\xi_2 条件下,还有约束条件A(k1ξ1+k2ξ2)=0A(k_1\xi_1+k_2\xi_2)=0,由此解出k1,k2k_1,k_2 还需满足的其他等量关系,最后代回原通解中即可得到公共解。

只知道Ax=0,Bx=0Ax=0,Bx=0 的通解(若为)k1ξ1+k2ξ2,  l1η1+l2η2k_1\xi_1+k_2\xi_2,\;l_1\eta_1+l_2\eta_2,求公共解.

公共解使得k1ξ1+k2ξ2=l1η1+l2η2k_1\xi_1+k_2\xi_2=l_1\eta_1+l_2\eta_2 成立,由此需要确定此时的k1,k2,l1,l2k_1,k_2,l_1,-l_2 的取值,即有:

(ξ1,ξ2,η1,η2)(k1k2l1l2)=0(\xi_1,\xi_2,\eta_1,\eta_2)\begin{pmatrix}k_1\\k_2\\-l_1\\-l_2\\\end{pmatrix}=0

记作Cx=0Cx=0 ,因此只需求出Cx=0Cx=0 的通解得到具体的k1,k2,l1,l2k_1,k_2,l_1,-l_2 的取值或等量关系后,代回原式即可。

对于非齐次方程组Ax=β,Bx=γAx=\beta,Bx=\gamma 的公共解问题

类似地,如果只知道两个非齐次方程组的通解ξ0+k1ξ1+k2ξ2,  η0+l1η1+l2η2\xi_0+k_1\xi_1+k_2\xi_2,\;\eta_0+l_1\eta_1+l_2\eta_2,也可以直接令ξ0+k1ξ1+k2ξ2=η0+l1η1+l2η2\xi_0+k_1\xi_1+k_2\xi_2=\eta_0+l_1\eta_1+l_2\eta_2,移向得:

(ξ0+k1ξ1+k2ξ2η0)l1η1l2η2=0(ξ0+k1ξ1+k2ξ2η0  ,η1,  η2)(1l1l2)=0\begin{aligned}(\xi_0+k_1\xi_1+k_2\xi_2-\eta_0)-l_1\eta_1-l_2\eta_2=0\\\\(\xi_0+k_1\xi_1+k_2\xi_2-\eta_0\;,\eta_1,\;\eta_2)\begin{pmatrix}1\\-l_1\\-l_2\end{pmatrix}=0\end{aligned}

记为Cx=0Cx=0 。有公共解说明矩阵CC非零解,当且仅当r(C)=r(η1,η2)r(C)=r(\eta_1,\eta_2) ,由此可以得出关于k1,k2k_1,k_2 的等量关系,从而代回原式得出公共解。

方程组的同解

  1. Ax=0,Bx=0Ax=0,Bx=0 同解Ax=0,Bx=0\Leftrightarrow Ax=0,Bx=0 有相同或等价的基础解系
  2. Ax=0,Bx=0Ax=0,Bx=0 同解r(A)=r(B)\Rightarrow r(A)=r(B) (基础解系的个数相等,注意此处是单箭头)
  3. Ax=0Ax=0 的解是Bx=0Bx=0 的解Ax=0\Leftrightarrow Ax=0{Ax=0Bx=0\begin{cases}Ax=0\\Bx=0\end{cases} 同解r(A)=r(AB)\Leftrightarrow r(A)=r\begin{pmatrix}A\\B \end{pmatrix}
  4. ♾️Ax=0,Bx=0Ax=0,Bx=0 同解r(A)=r(B)=r(AB)\Leftrightarrow r(A)=r(B)=r\begin{pmatrix}A\\B \end{pmatrix}A,BA,B行向量组等价
  5. ♾️由 结论(3,4) 可知,当Ax=0Ax=0 的解是Bx=0Bx=0 的解(或相反)并且r(A)=r(B)r(A)=r(B) 时,它们同解

由结论 (2) 可知,当我们想证明列数相同的两个矩阵AABB 的秩相等时,可以尝试去证明Ax=0Ax=0Bx=0Bx=0 是同解方程组。

我们有:Ax=0Ax=0ATAx=0A^TAx=0 同解;An×nnx=0A_{n\times n}^nx=0An×nn+1x=0A_{n\times n}^{n+1}x=0 同解。(后者的证明之前我们已给出,前者的证明见下一栏)

方程组同解的相关证明见:http://www.doc88.com/p-0542618799889.html

非齐次方程组的同解问题

非齐次线性方程组Ax=β,Bx=γAx=\beta,Bx=\gamma 同解 当且仅当 对应的齐次方程组Ax=0,Bx=0Ax=0,Bx=0 同解(“三秩相同”) 并且Ax=β,Bx=γAx=\beta,Bx=\gamma 有公共解。得出的公共解即为二者公用的非齐次特解

转置、伴随阵相关的同解问题

转置相关的同解

对于转置矩阵,有一个依据概念即可证明的经典例题:

证明:方程组AX=0AX=0ATAX=0A^TAX=0 同解

  1. 先证明AX=0AX=0 的解是ATAX=0A^TAX=0 的解。
    AX=0AX=0 两边左乘ATA^TATAX=0\Rightarrow A^TAX=0.
  2. 再证明ATAX=0A^TAX=0 的解是AX=0AX=0 的解。
    ATX=0A^TX=0 两边左乘XTX^TXTATAX=(AX)T(AX)=AX=0\Rightarrow X^TA^TAX=(AX)^T(AX)=||AX||=0 当且仅当AX=0AX=0.

该结论根据方程组同解的充要条件,顺便证明了矩阵秩的性质:r(A)=r(ATA)r(A)=r(A^TA).

伴随矩阵的齐次方程组

对于Ax=0A^*x=0,我们可以通过重要公式AA=AA=AEA^*A=AA^*=|A|E 进行判断。
Ax=0Ax=0 有非零解时,此时有r(A)<n,A=0r(A)\lt n,|A|=0。从而得到AA=OA^*A=O,将A,OA,O 按列分块:

A(ξ1,ξ2,,ξn)=(0,0,,0)A^*(\xi_1,\xi_2,\cdots,\xi_n)=(0,0,\cdots,0)

从而,AA 的每一列都是A=0A^*=0 的解。

由于r(A)<nr(A)\lt n,根据伴随矩阵的秩公式,可以得出r(A)1r(A^*)\leq1.
我若伴随矩阵的秩为零,则伴随矩阵为零矩阵,讨论其对应齐次方程组的解没有什么意义,因此我们考虑伴随矩阵的秩为 1 的情况。而此时只有r(A)=n1r(A)=n-1.
更进一步地,伴随矩阵的齐次方程组基础解系的个数为s=nr(A)=n1s=n-r(A^*)=n-1. 结合上述分析,我们可以得出:

Ax=0A^*x=0 的基础解系就是AA最大无关组(内含AAn1n-1 列线性无关的列向量)。

线性方程组与平面空间关系

线性方程组与平面空间关系

设三维空间中的三个平面方程分别如下:

{a1x+b1y+c1z=d1a2x+b2y+c2z=d1a3x+b3y+c3z=d3\begin{cases}a_1x+b_1y+c_1z=d_1\\a_2x+b_2y+c_2z=d_1\\a_3x+b_3y+c_3z=d_3\end{cases}

自然,我们可以在代数上以线性方程组的形式看待它们。
记对应的系数矩阵AA增广矩阵Aˉ\bar{A}

不难得出,系数矩阵AA 的一个行向量为其对应平面的法向量,平面相交意味着平面对应的法向量不成比例,即系数矩阵行向量线性无关,由此得到系数矩阵非零行的情况,即系数矩阵秩的情况。

根据线性方程组的解的情况,结合空间平面的关系,我们可以得出:

  1. 方程组有解,即r(A)=r(Aˉ)r(A)=r(\bar{A}),即三个平面有(公共点)交点,或有公共交线,或直接重叠。
  2. 方程组无解,即r(A)<r(Aˉ)r(A)\lt r(\bar{A}),即三个平面平行,或任意两个平面有交线。

根据方程组解的情况,划分为以上两种类型之后,我们还根据秩的关系具体划分:

  • r(A)r(A) 说明几个平面相交
  • r(Aˉ)r(\bar{A}) 说明几个平面互异
  1. 若三个平面相交于一点,则r(A)=3r ( A ) = 3
  2. 若至少有两个平面相交于一线,则r(A)=2r ( A ) = 2
  3. 若三平面平行,则r(A)=1r ( A ) = 1
  4. 方程组无解时,r(Aˉ)=r(A)+1r(\bar{A})=r(A)+1

最终,我们可以得到如下的分类图:

矩阵秩判断平面位置关系


线性方程组与直线空间关系

问题 已知平面上三条直线的方程为:
l1:ax+2by+3c=0l_1:ax+2by+3c=0
l2:bx+2cy+3a=0l_2:bx+2cy+3a=0
l3:cx+2ay+3b=0l_3:cx+2ay+3b=0
求证 这三条直线交于一点的 充分必要条件a+b+c=0a+b+c=0. (2003 数一)

答案

证明 三条直线交于一点,即 方程组:

{ax+2by=3cbx+2cy=3acx+2ay=3b\begin{cases}ax+2by=-3c\\bx+2cy=-3a\\cx+2ay=-3b\end{cases}

唯一解
如果记系数矩阵AA增广矩阵BB,则原问题可转化为 证明r(A)=r(B)=2r(A)=r(B)=2(未知数为x,y 共两个).因为矩阵存在明显的倍数关系,我们可以将A, B改写成下列形式:

A=[abbcca],B=[abcbcacab]A=\begin{bmatrix}a&b\\b&c\\c&a\end{bmatrix},B=\begin{bmatrix}a&b&c\\b&c&a\\c&a&b\end{bmatrix}

  • 必要性

因为r(B)=2r(B)=2, 所以对于3×33\times 3的矩阵BB来说,一定有B=0|B|=0. 即:

B=(a+b+c)(a2+b2+c2abbcac)=12(a+b+c)[(ab)2+(bc)2+(ca)2]=0\begin{aligned}|B|&=-(a+b+c)(a^2+b^2+c^2-ab-bc-ac)\\&=-\frac 1 2(a+b+c)[(a-b)^2+(b-c)^2+(c-a)^2]\\&=0\end{aligned}

而要使r(A)=2r(A)=2,显然a=b=ca=b=c并不成立(否则r(A)=1r(A)=1,或者另一个角度上来看,此时三条直线的解析式相同,本质属于同一条直线,就不会只相交于一个点,而是无数个点)。
于是得到a+b+c=0a+b+c=0.

  • 充分性

a+b+c=0a+b+c=0时,有B=0|B|=0,从而有r(A)r(B)2r(A) \leq r(B)\leq 2这里利用矩阵的秩的性质r(A)r(AB)r(A) \leq r(A|B).
因此,要证明原命题只需再证明r(A)=2r(A)=2即可。

反证法.
如果r(A)<2r(A) \lt 2,那么AA的两个列向量一定线性相关(对应成比例)。不妨设比例系数为λ\lambda.
则有:λa=b,λb=c,λc=a\lambda a=b, \lambda b=c, \lambda c=a,从而得到λ3a=a,λ3b=b,λ3c=c\lambda^3 a=a, \lambda^3 b=b, \lambda^3 c=c.

a,b,ca,b,c均不能为0(否则r(A)=0r(A)=0,或者另一个角度上来看,此时方程组是一个恒等式,就不能用于表达一条直线的解析式了),从而得到λ3=λ,λ=1\lambda^3=\lambda,\lambda=1.
于是,就有a=b=ca=b=c,结合a+b+c=0a+b+c=0得到a=b=c=0a=b=c=0矛盾

综上,a+b+c=0r(A)=r(B)=2a+b+c=0 \Leftrightarrow r(A)=r(B)=2 \Leftrightarrow 三条直线交于一点
证毕

特征值与特征向量

定义与性质

定义

  • 定义 对于nn方阵AA ,如果存在非零向量η\eta 和一个数λ\lambda,使得:

Aη=ληA\eta=\lambda\eta

则称,λ\lambdaAA 的一个特征值η\etaAA 的一个特征向量

对上式变形,容易得到(AλE)η=0(A-\lambda E)\eta=0。于是我们可以得出结论:

  • η\eta齐次方程组(AλE)X=0(A-\lambda E)X=0 的一个非零解。进而得到下面一系列的结论:
  • λ\lambdaAA 的特征值AλE=0\Leftrightarrow |A-\lambda E|=0
  • λ\lambda 不是AA 的特征值AλE0\Leftrightarrow |A-\lambda E|\neq 0,该结论是上一个结论的逆否,并且我们可以充分利用这个结论来证明矩阵可逆。要证明AλEA-\lambda E 可逆,只需证λ\lambda 不是AA 的特征值即可。

⚠️ 特征值全为0的矩阵不一定就是零矩阵,反例如下:A=[1,1;11]A=[1,1;-1-1]

另外,我们称λEA|\lambda E-A|AA特征多项式,显然特征值就是它特征多项式的
进而我们得到计算矩阵的特征值和特征向量的一般方法:

  1. 计算特征多项式
  2. 求特征多项式的根λi\lambda_i (共有nn个根,其中包含有重根乃至虚数的可能)
  3. 代入齐次方程组(AλiE)X=0(A-\lambda_i E)X = 0,解出非零解ηi\eta_i.

定理与性质

  • 定理1 如果η\etaAA 的特征向量,λ\lambda 是特征值,则:
  1. η\eta 也是多项式f(A)f(A) 的特征向量,特征值为f(λ)f(\lambda)
  2. 如果AA 可逆,则λ0\lambda \neq 0,并且η\eta 也是A1,AA^{-1}, A^* 的特征向量,特征值分别为1/λ,A/λ1/\lambda, |A|/\lambda
  3. 具体见下一节《与A相关的矩阵的特征值对应》中给出的表格
  4. 特别地,A+cEA+cE 的特征值为λ+c\lambda+c。该结论可用于快速求解特殊矩阵的特征值
    • ♾️推论 如果f(A)=0f(A)=0,则AA 的全体特征值λ\lambda 都满足f(λ)=0f(\lambda)=0.
    • ▷注:根据推论,我们可以根据多项式确定所有满足的λ\lambda取值,即在未知特征值的情况下,我们可以确保特征值可以在哪些值里面去取得。
  • 定理2λ1,λ2,,λn\lambda_1,\lambda_2,\cdots,\lambda_nnn 阶方阵AA 的全体特征值,则:
  1. i=1nλi=A\prod_{i=1}^n \lambda_i = |A|
  2. i=1nλi=tr(A)=i=1naii\sum_{i=1}^n \lambda_i = \mathrm {tr} (A)=\sum_{i=1}^n a_{ii}
    其中,tr(A)\mathrm {tr} (A)称为AAtrace
  • 定理3λ\lambdann 阶方阵AA 的特征值,则它的重数nr(AλE)\geq n-r(A-\lambda E)
    该定理还有一个更加学术性的说法:特征值的几何重数不大于代数重数,本文出现的只带有“重数”的概念均属于代数重数。

  • 代数重数: 特征多项式的根的重数;

  • 几何重数: 特征矩阵AλEA-\lambda E 零空间的维数.

对于这部分的证明需要运用到 矩阵论 中 Jordan矩阵相似性的相关知识。

  • 定理4 如果AA的一组特征向量η1,η2,,ηs\eta_1,\eta_2,\cdots,\eta_s 的特征值两两不同,则由这些特征向量组成的向量组线性无关

  • 定理5 如果nn 阶矩阵AA 的秩r(A)1,(n1)r(A)\leq 1,(n\geq 1),则AA 的特征值为0,0,,tr(A)0,0,\cdots,tr(A).

  • 推论 对于nn 维列向量α,β\alpha,\beta 有:αβT\alpha\beta^T的特征值为0,0,,βTα0,0,\cdots,\beta^T\alpha

相关证明见秩一矩阵板块

  • 定理6 实对称矩阵的特征值一定是实数,实反对称矩阵的特征值一定是零或纯虚数!
  • 定理7f(λ)f(\lambda)AA特征多项式,则f(A)=Of(A)=O (哈密顿凯莱定理)
与 A 相关的矩阵的特征值对应

在矩阵特征值的性质时,我们给出了关于对AA 做变换的一系列特征值及特征向量的对应关系。这里我们铺开来,具体如下表所示:

AAaA+bEaA+bEAkA^kf(A)f(A)A1A^{-1}AA^*P1APP^{-1}AP
λ\lambdaaλ+ba\lambda+bλk\lambda^kf(λ)f(\lambda)1/λ1/\lambdadet(A)/λ\det(A)/\lambdaλ\lambda
ξ\xiξ\xiξ\xiξ\xiξ\xiξ\xiP1ξP^{-1}\xi

我们指出,在AA 可逆,即AA 的特征值中不含零时,λ\lambda 就可以做分母,从而得出伴随矩阵的特征值。

几何意义

详见本站文章:

伴随矩阵的特征值

事实上,即使存在AA 的特征值为零,我们也可以得出伴随矩阵的特征值。

引入变量t,  t0t,\;t\neq0,有(A+tE)x=(λ1+tE)x(A+tE)x=(\lambda_1+tE)x,此时,AA^* 对应于λ1\lambda_1 的特征值为:

limt0A+tEλ1+t=i1λi\lim_{t\to0}\frac{|A+tE|}{\lambda_1+t}=\prod_{i\neq1}\lambda_i

即,AA 关于λi\lambda_i 的特征向量为ξi\xi_i 时,AA^* 关于ξi\xi_i 的特征值是AAλi\lambda_i 以外所有特征值的乘积。

特别地,当AA 为三阶方阵时,有:

A:  λ1,λ2,λ3A:  λ2λ3,λ1λ3,λ1λ2\begin{aligned}A:&\;&\lambda_1,&&\lambda_2,&&\lambda_3\\A^*:&\;&\lambda_2\lambda_3,&&\lambda_1\lambda_3,&&\lambda_1\lambda_2\end{aligned}

上述结论的另一种常见证明涉及到矩阵论中的若当标准型,此处不给出。
相关内容:伴随矩阵的特征值| CSDN

🦄特别地,当AA三阶非零矩阵,且仅存在单重特征值λ=0\lambda=0 时,根据上述定理,AA^* 必含有二重特征值00 和一个非零特征值。
由于阶数是三,所以AA^*非零特征值λ=tr(A)=0+0+λ=A11+A22+A33\lambda=tr(A^*)=0+0+\lambda=A_{11}+A_{22}+A_{33} ,其中AijA_{ij} 是矩阵AA代数余子式

转置矩阵的特征值

在上表中,我们并没有列出 A 转置的特征值情况。但事实上,二者的特征值是相同的,不过特征向量却完全没有联系。这是因为:

f(λ)=AλE=(AλE)T=ATλEf(\lambda)=|A-\lambda E|=|(A-\lambda E)^T|=|A^T-\lambda E|

但是(AλE)x=0(A-\lambda E)x=0(ATλE)x=0(A^T-\lambda E)x=0 的解却没有联系。

🦄解决抽象矩阵问题的技巧数则

转换抽象矩阵的研究对象

若已知α1,α2,α3\alpha_1,\alpha_2,\alpha_3 线性无关,且有{Aα1=k11α1+k12α2+k13α3Aα2=k21α1+k22α2+k23α3Aα3=k31α1+k32α2+k33α3\begin{cases}A\alpha_1=k_{11}\alpha_1+k_{12}\alpha_2+k_{13}\alpha_3\\A\alpha_2=k_{21}\alpha_1+k_{22}\alpha_2+k_{23}\alpha_3\\A\alpha_3=k_{31}\alpha_1+k_{32}\alpha_2+k_{33}\alpha_3\end{cases},则可将给定等式写为矩阵乘法的形式。
P=(α1,α2,α3)P=(\alpha_1,\alpha_2,\alpha_3) 则有A(α1,α2,α3)=AP=PBA(\alpha_1,\alpha_2,\alpha_3)=AP=PB,其中BB 是由kijk_{ij} 组成的矩阵 .
最终就能得到ABA\sim B。建立得到抽象/未知矩阵AA 与矩阵BB 的桥梁。

类似地,若已知α,Aα,A2α\alpha,A\alpha,A^2\alpha 线性无关,通常会给出A3αA^3\alpha的线性关系,这样的问题同样也有类似的处理办法:令P=(α,Aα,A2α)P=(\alpha,A\alpha,A^2\alpha)AP=PBAP=PB 即可。

秩一矩阵的性质与解结构

秩一矩阵的性质

设矩阵AAnn 阶方阵,且r(A)=1r(A)=1 ,这样的矩阵就是秩一矩阵

分解定理 秩一矩阵可以被分解为两个列向量的乘积,即:  n\exists\;n 维列向量α,β\alpha,\beta 使得A=αβTA=\alpha\beta^T. 并且,不难发现,还有tr(A)=αTβtr(A)=\alpha^T\beta.

我们有如下定理:

  1. 秩一矩阵AA 的特征值为:λ1=λ2==λn1=0,  λn=tr(A)\lambda_1=\lambda_2=\cdots=\lambda_{n-1}=0,\;\lambda_n=tr(A)
  2. 迹不为零的秩一矩阵一定可以相似对角化
  3. 秩一矩阵的高次幂有:An=[tr(A)]n1AA^n=[tr(A)]^{n-1}A (可利用分解定理推出)

🔔定理1 的证明A2=(αβT)(αβT)=tr(A)(αβT)=tr(A)AA^2=(\alpha\beta^T)(\alpha\beta^T)=tr(A)(\alpha\beta^T)=tr(A)A 知,秩一矩阵对应的特征值满足:

λ2=tr(A)λ\lambda^2=tr(A)\lambda

解得λ=0  or  tr(A)\lambda=0\;or\;tr(A) .
从而AA 的特征值的取值仅限这两种。

一方面,对A=αβTA=\alpha\beta^T 两边右乘α\alpha 得:Aα=tr(A)αA\alpha=tr(A)\alpha 。可见秩一矩阵的迹恰好就是一个特征值,且对应的特征向量为α\alpha.

另一方面,由于r(A)=1A=0r(A)=1\Rightarrow |A|=000 必然也是AA 的特征值。又r(A)=r(A0E)=1r(A)=r(A-0·E)=1Ax=0Ax=0 基础解系的个数s=nr(A0E)=n1s=n-r(A-0·E)=n-1 ,即秩一矩阵属于零的特征向量有n1n-1 个,由于矩阵几何重数不大于代数重数,所以AA 为 0 的特征值的个数至少为n1n-1.

结合以上两点,则有秩一矩阵AA 的特征值为:λ1=λ2==λn1=0,  λn=tr(A)\lambda_1=\lambda_2=\cdots=\lambda_{n-1}=0,\;\lambda_n=tr(A)

证毕!


🔔定理2 的证明 由定理1知,秩一矩阵在tr(A)0tr(A)\neq0 时有n1n-1 重零特征值,从定理1的证明中我们知道,此时矩阵的几何重数等于代数重数,故可相似对角化。而tr(A)=0tr(A)=0 时,零特征值有nn 重,大于AA 对应零的n1n-1 个特征向量的数目,从而此时不能相似对角化。


秩一矩阵与二次型

相似与相似对角化

相似及其对角化

相似矩阵

定义A,BA, B 是两个nn 阶矩阵,如果存在nn 阶可逆矩阵PP ,使得P1AP=BP^{-1}AP=B ,则称AABB 相似,记作:ABA\sim B

与等价类似,相似矩阵也具有对称性传递性
即:ABBAA\sim B \Leftrightarrow B\sim AAB  and  BCACA\sim B\;\text{and}\; B\sim C \Rightarrow A\sim C

  • 定理ABA\sim B,且P1AP=BP^{-1}AP=B 时:
    • f(A)f(B)f(A)\sim f(B),且P1f(A)P=f(B)P^{-1}f(A)P=f(B).f(x)f(x) 为多项式函数
    • AA可逆时,BB也一定可逆,并且ATBT,  A1B1,  ABA^T\sim B^T,\;A^{-1}\sim B^{-1},\;A^*\sim B^*
    • A=B|A|=|B|
    • r(A)=r(B)r(A)=r(B)
    • tr(A)=tr(B)tr(A)=tr(B)
    • η\etaAA 的特征向量\LeftrightarrowP1ηP^{-1}\etaBB 的特征向量
    • AABB 的特征值完全相同。注意,特征值完全相同不能推出A和B相似,除非A, B 都可以相似对角化

注:上述定理几乎都是必要条件,只有在 A,B相似时才成立(除了ATBTA^T\sim B^T ,这是充要的)。
例如:A=(1001),B=(1101)A=\begin{pmatrix}1&0\\0&1\end{pmatrix},B=\begin{pmatrix}1&1\\0&1\end{pmatrix} 它们的特征多项式、特征值、迹、秩以及行列式都完全相同,但是却并不相似!

相似对角化

定义 如果一个矩阵相似于一个对角矩阵,则称该矩阵可以 相似对角化 (简称:可对角化

令可逆矩阵P=(η1,η2,,ηn)P=(\eta_1,\eta_2,\cdots,\eta_n),显然因为可逆所以 P 的列向量组之间线性无关。
如果AA可对角化,则说明AP=PΛAP=P\Lambda,其中Λ\Lambda表示一个对角矩阵,即:

A(η1,η2,,ηn)=(η1,η2,,ηn)[λ1λ2λn]A(\eta_1,\eta_2,\cdots,\eta_n)=(\eta_1,\eta_2,\cdots,\eta_n)\begin{bmatrix}\lambda_1&&&\\&\lambda_2&&\\&&\ddots&\\&&&\lambda_n\end{bmatrix}

不难看出,有Aη1=λ1η1,Aη2=λ2η2,A\eta_1=\lambda_1\eta_1,\quad A\eta_2=\lambda_2\eta_2,\cdots
因此可以得到判断矩阵能否可以相似对角化的方法。

对角化的判别

nn 阶矩阵AA 可对角化\Leftrightarrow 矩阵AAnn 个线性无关的特征向量
nn 阶矩阵AA 可对角化\LeftrightarrowAA的每个特征值λi\lambda_i的重数ki=nr(AλiE)k_i=n-r(A-\lambda_iE)
nn 阶矩阵AAtr(A)0,r(A)=1  Atr(A)\neq0,r(A)=1 \Leftrightarrow\;A 可相似对角化

nn 阶矩阵AAnn 个互不相同的特征值  A\Rightarrow\;A 可相似对角化(注意是单箭头)
nn 阶矩阵AA 是实对称矩阵  A\Rightarrow\;A 可相似对角化(注意是单箭头)

♾️若A2(a+b)A+abE=O,  abA^2-(a+b)A+abE=O,\;a\neq b ,则AA 可相似对角化(注意是单箭头)

实对称矩阵对角化

  • 定理 如果AAnn实对称矩阵,则:
    • AA 的特征值都是实数
    • AA 相似于 实对角矩阵
    • AA 的不同特征值对应的特征向量之间相互正交
    • 存在正交矩阵QQ,使得Q1AQ=QTAQ=ΛQ^{-1}AQ=Q^TAQ=\Lambda
    • 上述结论的逆命题也成立:
      如果对于实矩阵AAQTAQ=ΛQ^TAQ=\LambdaQQ 是正交阵),则AA 为实对称矩阵

🎲求形如ax1+bx2+cx3=0ax_1+bx_2+cx_3=0正交基础解系 时,可以先求出其中一个解η1\eta_1 ,然后利用正交性,对第二个解η2\eta_2 设未知数使得η1η2=0\eta_1·\eta_2=0。该方法避免了施密特正交化的过程,更方便。


相似对角化证明

证明AAATA^T 相似

待更

问题AAnn 阶矩阵,数aba\neq b . 证明下列三个断言相互等价:

  1. (AaE)(AbE)=0(A-aE)(A-bE)=0
  2. r(AaE)+r(AbE)=nr(A-aE)+r(A-bE)=n
  3. AA 相似于对角阵,并且特征值满足(λa)(λb)=0(\lambda-a)(\lambda-b)=0
答案

如果aa 不是AA 的特征值,则AaEA-aE 可逆,从而:
由(1)可得,AbE=0,A=bEA-bE=0,A=bE.
由(2)可得,r(AbE)=0,A=bEr(A-bE)=0,A=bE.
由(3)可得,λb=0,λ=b\lambda-b=0,\lambda=b,所以λ=b\lambda=b 是唯一特征值。而因为可对角化,所以重数kb=nr(AbE)=nk_b=n-r(A-bE)=n,与(2)一样,能够得到A=bEA=bE.

同理,bb 不是AA 的特征值时,三者均能得到同样的结论:A=aEA=aE.
即此时三个断言互相等价。

下面考虑a,ba, b 均是AA 的特征值的情况:

  • (1)(2)(1) \Rightarrow (2)
    (AaE)(AbE)=0r(AaE)+r(AbE)n(A-aE)(A-bE)=0 \Rightarrow r(A-aE)+r(A-bE)\leq n
    又由r(AaE)+r(AbE)r((AaE)(AbE))=r((ba)E)=nr(A-aE)+r(A-bE)\geq r((A-aE)-(A-bE))=r((b-a)E)=n.
    所以,r(AaE)+r(AbE)=nr(A-aE)+r(A-bE)=n.

  • (2)(3)(2) \Rightarrow (3)
    a,ba, b 的重数分别为ka,kbk_a, k_b。则:
    kanr(AaE)k_a\geq n-r(A-aE)
    kbnr(AbE)k_b\geq n-r(A-bE)
    由特征值个数为nn ,所以:nka+kb2n[r(AaE)+r(AbE)]=nn \geq k_a+k_b\geq 2n-[r(A-aE)+r(A-bE)]=n.
    所以ka+kb=nk_a+k_b=n,说明AA 的特征值只有a,ba,b,所以满足(λa)(λb)=0(\lambda-a)(\lambda-b)=0.
    此外,因为上式的 “\geq”全部取“==”,所以ka=nr(AaE),kbnr(AbE)k_a = n-r(A-aE),k_b- n-r(A-bE)
    所以,AA 可以相似对角化。

  • (3)(1)(3) \Rightarrow (1)
    因为 特征值满足(λa)(λb)=0(\lambda-a)(\lambda-b)=0,所以AA 的特征值只有a,ba,b
    BB 是与AA 相似的对角矩阵,则BB 对角上的元素只有a,ba,b.
    于是(BaE)(BbE)=0(B-aE)(B-bE)=0
    从而(BaE)(BbE)(AaE)(AbE)=0(B-aE)(B-bE)\sim (A-aE)(A-bE)=0

证毕.

证明 对于nn 阶方阵AAA2=AA^2=A ,证明AA 可相似对角化

确定特征值的可取值有哪些:λ2=λλ=1  or  0\lambda^2=\lambda\Rightarrow\lambda=1\;or\;0.

分析重数:由A(AE)=Or(A)+r(AE)nA(A-E)=O\Rightarrow r(A)+r(A-E)\leq n
而由r(A±B)r(A)+r(B)n=r(E)=r[A(AE)]r(A)+r(AE)r(A\pm B)\leq r(A)+r(B)\Rightarrow n=r(E)=r[A-(A-E)]\leq r(A)+r(A-E)
夹逼可得,r(A)+r(AE)=nr(A)+r(A-E)=n

AA 关于特征值0的线性无关的特征向量个数是nr(A)n-r(A)
AA 关于特征值1的无关特征向量个数是nr(AE)n-r(A-E) .
二者之和正好为nn ,是阶数。

故,AA 的几何重数等于代数重数,即无关特征向量个数等于特征值重数,故可对角化。

注:该证明其实是前面我们给出的对角化判别中最后一个的特例。对其证明也是与此处类似的。之所以要求aba\neq b 自然就是构造反向不等式时能够出现(ab)E(a-b)E 的秩。

两矩阵相似与否的证明问题

证明矩阵不相似

  1. 利用必要条件
    • 比如因为“若 ,则: ; ; ; ”,那么我们就可以通过 、 等来说明 和 不相似——这种思想在选择题中很常见;
  2. 利用相似的传递性
    • 若我们能证明出 能相似对角化,但 不能相似对角化,则 和 一定不相似.
  3. 利用特征向量个数
    • 若两个矩阵相似,则不仅特征值相同,且重特征值对应的线性无关特征向量个数也相同. 所以,我们可以 利用这一点去证明两个矩阵不相似. 比如,当我们发现 时,说明 和 的特征多项式相同,故 的特征值、行列式、秩、 迹全部都对应相等,很难判断出是否相似. 此时我们可以判断每一个特征值 对应的 和 . 若存在某个 ,使得 ,则说明 和 一定不相似(其实这也算是利用了结论“若 , 则 ”的逆否命题)
三阶实对称矩阵的重根特征向量

正交性求重根特征向量

一道综合性好题

待更

矩阵对角化的各类变形问题

同时对角化问题

Q\exists Q 使得QTAQ=EQ^TAQ=E,即找到可逆变换y=Qxy=Qx 将二次型f=xTAxf=x^TAx 作标准化(一般通过配方法)。
那么,利用单位正交矩阵PP 作如下处理时,结果不变:

PT(QTAQ)P=PTEP=PTP=EP^T(Q^TAQ)P=P^TEP=P^TP=E

C=QPC=QP,则CC 也是一个使得AA 得以标准化的可逆矩阵。

另一边,我们需要使得BB 在相同的变换下转为规范形,即CTBC=ΛC^TBC=\varLambda. 有:

PTQTBQP=ΛP^TQ^TBQP=\varLambda

若令B1=QTBQB_1=Q^TBQ ,则对B1B_1 作正交变换使其对角化就相当于找到单位正交矩阵PP.

总结一下就是,对AA 作标准化得到QQ ,对QTBQQ^TBQ 作规范化得到PP,则C=QPC=QP 即为一个可逆矩阵,使得A,BA,B 同时对角化。

待更

矩阵相似“开方”问题

矩阵与伴随一起对角化

二次型

二次型与变量替换

二次型的定义

定义 关于nn 个变量x1,x2,,xnx_1,x_2,\cdots,x_n 的齐二次多项式函数 称作nn二次型,其一般形式如下:

f(x1,x2,,xn)=a11x12+2a12x1x2+2a13x1x3++2a1nx1xn+a22x22+2a23x2x3++2a2nx2xn++annxn2=i=1naiixi2+2ijaijxixj\begin{aligned}f(x_1,x_2,\cdots,x_n)&=a_{11}x_1^2&+2a_{12}x_1x_2+2a_{13}x_1x_3+\cdots+2a_{1n}x_1x_n+\\&&a_{22}x_2^2+2a_{23}x_2x_3+\cdots+2a_{2n}x_2x_n+\\&&\cdots\cdots\cdots\\&&+a_{nn}x_n^2\\&=&\sum_{i=1}^na_{ii}x_i^2+2\sum_{i \neq j}a_{ij}x_ix_j\end{aligned}

称其中含有xi2x_i^2 的项为 平方项,称含有xixj(ij)x_ix_j(i \neq j) 的项为 交叉项

二次型可以用矩阵乘法的形式表示:

let X=[x1x2xn],A=[a11a12a1na12a22a2na1na2nann]then f(x1,x2,,xn)=XTAX\begin{aligned}\text{let }&X=\begin{bmatrix}x_1\\x_2\\ \vdots\\x_n\end{bmatrix},A=\begin{bmatrix}a_{11}&a_{12}&\cdots&a_{1n}\\a_{12}&a_{22}&\cdots&a_{2n}\\\vdots&\vdots&\ddots&\vdots\\a_{1n}&a_{2n}&\cdots&a_{nn}\end{bmatrix}\\\\\text{then }&f(x_1,x_2,\cdots,x_n)=X^TAX\end{aligned}

注意 二次型的矩阵AA 是对称矩阵,这是和二次型相互决定的。

变量替换

引进新的变量y1,y2,,yny_1,y_2,\cdots,y_n 使得原变量x1,x2,,xnx_1,x_2,\cdots,x_n 可以用新变量的齐一次线性函数表示,即:

X=CY,C=[c11c12c1nc21c22c2ncn1cn2cnn]X=CY,C=\begin{bmatrix}c_{11}&c_{12}&\cdots&c_{1n}\\c_{21}&c_{22}&\cdots&c_{2n}\\\vdots&\vdots&\ddots&\vdots\\c_{n1}&c_{n2}&\cdots&c_{nn}\end{bmatrix}

然后将其代入f(x1,x2,,xn)f(x_1,x_2,\cdots,x_n) 从而得到关于新变量的二次型g(y1,y2,,yn)g(y_1,y_2,\cdots,y_n).
该过程称之为 对二次型f(x1,x2,,xn)f(x_1,x_2,\cdots,x_n)线性变量替换
如果CC 可逆,则称为 可逆线性变量替换。此时有:

g(y1,y2,,yn)=f(x1,x2,,xn)=XTAX=YTCTACYg(y_1,y_2,\cdots,y_n)=f(x_1,x_2,\cdots,x_n)=X^TAX=Y^TC^TACY

即 二次型g(y1,y2,,yn)g(y_1,y_2,\cdots,y_n) 的矩阵B=CTACB=C^TAC.

矩阵合同与惯性指数

矩阵合同

定义 对于两个nn 阶实矩阵A,BA,B,如果存在可逆阵CC 使得B=CTACB=C^TAC,则称AABB 合同
记作ABA\simeq B.

一定要注意,在矩阵合同的定义中,我们并没有要求两个矩阵都是实对称矩阵。即,非对称矩阵也可以相互合同!

显然,根据上面我们对变量替换的推导,可以得出:
对称矩阵AABB 合同\Leftrightarrow 它们的二次型可通过 可逆线性变量变换 进行相互转化

此外,还有:对称矩阵相似(特征值相同)→ 对称矩阵合同

因此,要想求使得ABA\simeq B 的矩阵CC 只需将 实对称矩阵 的特征向量正交单位化,拼成CC 即可。


对于对称矩阵AA 确定的二次型f=xTAxf=x^TAx,不难将其化为我们熟悉的形式f=xTAsxf=x^TA_sx,其中AsA_s 为实对称阵,且必有:

As=A+AT2A_s=\frac{A+A^T}{2}

可以举出各种反例证明,A,AsA,A_s 之间除了上述等式关系外,再无其他关系(如秩是否相等,是否相似、合同等)

🦄有结论:ABAsBs,AwBwA\simeq B\Rightarrow A_s\simeq B_s,A_w\simeq B_w 其中,我们记Aw=(AAT)/2A_w=(A-A^T)/2,是反对称阵.

证明方法直接由A=As+Aw,B=Bs+BwA=A_s+A_w,B=B_s+B_w 代入即可。(注:反对称矩阵对角元素全0)

由此可知,判断矩阵不合同可以通过判断其对应的对称矩阵不合同得出.

由于,对称矩阵BB 对应的反对称阵必为零矩阵Bw=0B_w=0,而非对称矩阵AA 对应的反对称阵Aw0A_w\neq0 ,所以必有Bw≄AwB≄AB_w\not\simeq A_w\Rightarrow B\not\simeq A.
即:对称阵与非对称阵必定不合同!!

更多相关内容可参考:非对称矩阵合同判断方法|心一学长

综上所述,如果我们想要判断非实对称矩阵是否不合同,只需验证As,BsA_s,B_s 不合同即可。但是反过来,即使AsBsA_s\simeq B_s 我们却不能反推ABA\simeq B !!!

要想证明ABA\simeq B 只能从定义出发。如下面这道题:

已知A=(123231321),B=(123132321)A=\begin{pmatrix}1&2&3\\2&3&1\\3&2&1\end{pmatrix},B=\begin{pmatrix}1&2&3\\1&3&2\\3&2&1\end{pmatrix},请判断A,BA,B 是否相似,是否合同。

【解】经计算,A,BA,B 的特征值均为:2,1,6-2,1,6 各特征值互不相同,因此ABA\sim B
下面验证合同关系。
不难发现,AA 可以通过有限次行列变换化为BB
AA 的第一行和第三行交换,再将AA 的第一列与第三列交换即可。
即有:

(111)A(111)=B:=CTAC\begin{pmatrix}&&1\\&1\\1\end{pmatrix}A\begin{pmatrix}&&1\\&1\\1\end{pmatrix}=B:=C^TAC

满足合同定义,故ABA\simeq B

惯性指数

惯性定理:一个二次型ff 虽然标准化结果不唯一,但是它们的平方项系数中,正的个数和负的个数由ff 唯一确定。因此ff 的规范型在形式上唯一,称其为 规范形

我们称平方项系数(矩阵的特征值)中正的个数为 正惯性指数,负的个数为 负惯性指数
正负惯性指数之和 =ff 的秩

有:两个二次型有相同的规范形\Leftrightarrow 对应的两个实对称阵合同\Leftrightarrow 它们的正(负)特征值个数都相同

二次型的标准化与规范化

标准化与规范化

  • 实二次型 系数和变量都限定为实数的二次型
  • 标准二次型 交叉项的系数都为0,即矩阵是对角矩阵的二次型
  • 规范二次型 形如x12++xp2xp+12xp+q2x_1^2+\cdots+x_p^2-x_{p+1}^2-\cdots-x_{p+q}^2 的二次型,其矩阵是规范对角矩阵

A=[EpEq0]A=\begin{bmatrix}E_p&&\\&-E_q&&\\&&0&\end{bmatrix}

它由阶数nn正负惯性指数p,qp,q 决定。

  • 标准化 通过可逆线性变量替换把给定二次型化为标准二次型(构造可逆阵CC 使得CTACC^TAC 为对角阵)
  • 规范化 通过可逆线性变量替换把给定二次型化为规范二次型(构造可逆阵CC 使得CTACC^TAC 为规范对角阵)

变换方法

  1. 正交变化法 只能用于标准化,构造方法与实对称阵的相似对角化类似,略
  2. 配方法
    1. 选定一个系数非0的平方项xi2x_i^2,将其与含有xix_i 的交叉项整合并配方。剩下的式子则可以看作新一组二次型,用同样的方法配方,直到所有项配方完成;
    2. 引进变量y1,y2,,yny_1,y_2,\cdots,y_n 对上一步中配方的每一项进行替换。(个数少于nn 时要保证替换可逆);
    3. 写出用新变量表示x1,x2,,xnx_1,x_2,\cdots,x_n 的一次线性变换函数以及变换矩阵。
    • 注:如果第一步没有平方项,可以先构造一次变量替换(未必已经标准化),然后将新的二次型进行处理即可。

一道经典例题:只能配方不能正交变换

问题 已知矩阵A=[120242022]A=\begin{bmatrix}1&-2&0\\-2&4&2\\0&2&-2\end{bmatrix},求作一个 3 阶可逆矩阵PP 使得PTAPP^TAP 为对角阵.

答案

很容易想到的做法是:根据实对称阵的特点,企图利用矩阵的特征值和特征向量构造一个正交矩阵QQ,使得QTAQ=ΛQ^TAQ=\Lambda,并且对角阵元素是特征值。

然而实际上这道题的矩阵AA 的特征值难以求解。
λEA=0|\lambda E-A|=0,会得到:λ33λ214λ+4=0\lambda^3-3\lambda^2-14\lambda+4=0
该方程只能通过一元三次方程的求根公式暴力求解,或者利用 牛顿-拉弗森方法| Newton-Raphson method 求出近似解
借助 MATLAB 计算得到的结果为:λ1=2.7102,λ2=0.27136,λ3=5.4389\lambda_1=-2.7102,\lambda_2=0.27136,\lambda_3=5.4389.
如果再利用这个近似解去求特征向量,显然无法得到正确的答案。
由此不考虑该做法。

注意到问题只要求有可逆阵PP,而并没有要求PP 是正交阵,因此我们可以利用二次型与矩阵的对应关系对本题进行求解。

矩阵AA 对应的二次型为:f(x1,x2,x3)=x12+4x222x324x1x2+4x2x3f(x_1,x_2,x_3)=x_1^2+4x_2^2-2x_3^2-4x_1x_2+4x_2x_3
对其进行配方:f(x1,x2,x3)=(x12x2)22(x2x3)2+2x22f(x_1,x_2,x_3)=(x_1-2x_2)^2-2(x_2-x_3)^2+2x_2^2.
于是可作线性变量替换:

{y1=x12x2y2=x2x3y3=x2\begin{cases}y_1=x_1-2x_2\\y_2=x_2-x_3\\y_3=x_2\end{cases}

则原二次型化为:f(x1,x2,x3)=y122y22+2y32f(x_1,x_2,x_3)=y_1^2-2y_2^2+2y_3^2.
变换公式即为:

{x1=y1+2y3x2=y3x3=y2+y3\begin{cases}x_1=y_1+2y_3\\x_2=y_3\\x_3=-y_2+y_3\end{cases}

于是所求变换矩阵为:

P=[102001011],PTAP=[100020002]P=\begin{bmatrix}1&0&2\\0&0&1\\0&-1&1\end{bmatrix},P^TAP=\begin{bmatrix}1&0&0\\0&-2&0\\0&0&2\end{bmatrix}

利用配方法快速求解惯性指数

利用秩快速求解惯性指数

正定矩阵的概念与性质

正定矩阵

x1,x2,,xnx_1,x_2,\cdots,x_n 不全为0时,如果f(x1,x2,,xn)>0f(x_1,x_2,\cdots,x_n) \gt 0,则称二次型ff正定二次型.
正定二次型对应的矩阵就是 正定矩阵. 与正定矩阵 合同 的实对称矩阵 也 正定.

有:实对称矩阵AA 正定
\Leftrightarrownn 维列向量η0\eta\neq 0 时,恒有ηTAη>0\eta^TA\eta\gt 0
\Leftrightarrow 正惯性指数等于阶数nn
\LeftrightarrowAA 的特征值都是正数
\LeftrightarrowAA 的所有顺序主子式均大于0
\LeftrightarrowAA 合同于单位阵,与AA 合同的实对称矩阵B=PTAPB=P^TAP 也正定
\Leftrightarrow 存在可逆阵CC 使得A=CTEC=CTCA=C^TEC=C^TC

  • 顺序主子式:把实对称阵左上角rr 阶子矩阵的行列式称为其rr 阶 顺序主子式.

由此可以总结出判断矩阵正定的一般方法:顺序主子式法、特征值法、定义法等.


问题A,BA,B 都是m×nm\times n 实矩阵,其中r(B)=nr(B)=n. 证明ATA+(A+B)T(A+B)A^TA+(A+B)^T(A+B) 是正定矩阵

答案

B=CAB=C-A,则r(CA)=r(B)=nr(C-A)=r(B)=n,所以nn 元齐次线性方程组(CA)X=0(C-A)X=0 没有非零解。

从而对于一个nn非零列向量α\alpha(CA)α0(C-A)\alpha\neq 0. 则Cα,AαC\alpha,A\alpha 不全为零向量

于是,对于实对称矩阵ATA+(A+B)T(A+B)=ATA+CTCA^TA+(A+B)^T(A+B)=A^TA+C^TC 有:

α(ATA+CTC)α=Aα2+Cα2>0\alpha(A^TA+C^TC)\alpha=||A\alpha||^2+||C\alpha||^2 \gt 0

根据正定矩阵的定义知,ATA+(A+B)T(A+B)A^TA+(A+B)^T(A+B) 为正定矩阵。


问题 证明对任何m×nm\times n 实矩阵A,ATAA,A^TA 的负惯性指数为 0. 如果r(A)=nr(A)=n , 则ATAA^TA 是正定矩阵。

答案

证明λ\lambdaATAA^TAATAA^TA是实对称矩阵)的一个特征值,η\eta 是属于它的一个特征向量(因此η\eta 非零)。
从而ATAη=ληA^TA\eta=\lambda\etaηTATAη=(Aη)T(Aη)=ληTη\eta^TA^TA\eta=(A\eta)^T(A\eta)=\lambda\eta^T\eta.
所以λ=(Aη,Aη)/ηTη0\lambda=(A\eta,A\eta)/\eta^T\eta \geq 0 恒成立。
因此,ATAA^TA 的负惯性指数为 0 ,其特征值都是非负实数。

又由r(A)=nr(A)=n 知,AX=0AX=0 没有非零解。即对任何非零列向量η\eta 都有Aη0A\eta\neq 0.
于是(Aη,Aη)>0,λ>0(A\eta,A\eta)\gt 0,\lambda\gt 0
从而得到ATAA^TA 为正定矩阵,证毕.

二次型的非零解、最值问题

二次型的非零解

二次型的不等式

设实二次型f=xTAxf=x^TAx 中,二次型矩阵AA 的特征值为λ1,λ2,λ3\lambda_1,\lambda_2,\lambda_3,且

λ1λ2λ3\lambda_1\leq\lambda_2\leq\lambda_3

证明:对任意三维实列向量xx ,有:

λ1xTxλ2xTxλ3xTx\lambda_1x^Tx\leq\lambda_2x^Tx\leq\lambda_3x^Tx

ffxTx=1x^Tx=1 的情况下,最大值是λ3\lambda_3,最小值是λ1\lambda_1.

选自《张宇线性代数 9 讲

实对称矩阵规避正交化

规避斯密特正交化

待更

※谱分解定理

谱分解定理

矩阵的高次幂

求解矩阵的高次幂问题

相似对角化法

哈密顿-凯莱定理| Hamilton-Cayley

f(λ)f(\lambda)AA特征多项式,则f(A)=Of(A)=O

数学归纳法

主对角线及其下方元素全为零,循环矩阵
幂零矩阵

https://blog.csdn.net/weixin_35505731/article/details/104541127

牛顿二项展开法

秩一矩阵分解法

初等矩阵变换法

概率论

随机事件和概率

基本公式
  • 乘法公式

P(AB)={P(A)P(BA),P(A)>0P(B)P(AB),P(B)>0P(A)P(B),A,B independent of each other0,A,B mutually exclusiveP(B),BAP(A)+P(B)P(AB)P(AB)=\begin{cases}P(A)P(B|A)&,P(A)\gt 0\\P(B)P(A|B)&,P(B)\gt 0\\P(A)P(B)&,A,B\text{ independent of each other}\\0&,A,B\text{ mutually exclusive}\\P(B)&,B\subset A\\P(A)+P(B)-P(A\cup B)\end{cases}

  • 全概率公式

P(B)=i=1nP(BAi)=i=1nP(Ai)P(BAi)P(B)=\sum_{i=1}^nP(BA_i)=\sum_{i=1}^nP(A_i)P(B|A_i)

  • 贝叶斯公式

P(AmB)=P(AmB)P(B)=P(Am)P(BAm)i=1nP(Ai)P(BAi)P(A_m|B)=\frac{P(A_mB)}{P(B)}=\frac{P(A_m)P(B|A_m)}{\sum_{i=1}^nP(A_i)P(B|A_i)}

伯努利公式

独立性的性质

  • A 与 B 相互独立
    P(AB)=P(A)P(B)\Leftrightarrow\quad P(AB)=P(A)P(B)
    P(AB)=P(AB)=P(A),(0<P(B)<1)\Leftrightarrow\quad P(A|B)=P(A|\overline{B})=P(A),\quad (0 \lt P(B)\lt 1) 无论事件 B 发生与否,A 发生的概率不变
    P(BA)=P(BA)=P(B),(0<P(A)<1)\Leftrightarrow\quad P(B|A)=P(B|\overline{A})=P(B),\quad(0\lt P(A)\lt 1)
    P(BA)+P(BA)=1,(0<P(A)<1)\Leftrightarrow\quad P(B|A)+P(\overline{B}|\overline{A})=1,\quad(0\lt P(A)\lt 1)

  • Ω,\Omega,\varnothing 与任意事件独立

  • A, B, C 两两独立\nRightarrow A, B, C 相互独立

伯努利试验|Bernoulli experiment

如果在两个或多个试验中与各试验相联系的事件之间相互独立,且同一事件在各个试验中出现的概率相同,则称这些试验是 独立重复试验
如果某独立重复试验只有两种对立结果,则称其为 伯努利试验
将一个伯努利试验重复 n 次,则称这样的试验模型为 n 重伯努利概型

设在每次试验中,P(A)=p,(0<p<1)P(A)=p,(0\lt p\lt 1),则在nn 重伯努利试验中的事件AA 发生kk 次的概率为:

P(Ak)=Cnkpkqnk(k=0,1,2,...,n,q=1p)P(A_k)=C_n^kp^kq^{n-k}\quad(k=0,1,2,...,n,q=1-p)

上述公式称为 伯努利公式,也是二项概率公式
如果XXnn 重伯努利概型中,事件AA 发生的次数,则说XX 服从于参数为n,pn,p二项分布,记作XB(n,p)X\sim B(n,p).

问题 重复独立掷两个均匀的骰子。求问两个骰子的点数之和为 4 的结果出现在 点数之和为 7 的结果之前的概率。

答案

给出如下事件标记:

事件标记事件内容事件的概率备注
AA点数和4 出现在 点数和7 之前待求-
BB第一次试验出现了点数和43/363/36总样本数62=366^2=36,“点数和4”的可能为(1,3),(2,2),(3,1)(1,3),(2,2),(3,1)
CC第一次试验出现了点数和76/366/36
DD第一次试验没有出现 点数和4点数和71P(B)P(C)=27/361-P(B)-P(C)=27/36

于是,有:

P(AB)=1,P(AC)=0,P(AD)=P(A)P(A|B)=1,\quad P(A|C)=0,\quad P(A|D)=P(A)

由全概率公式,得:

P(A)=336+2736P(A)P(A)=13P(A)=\frac 3 {36}+\frac{27}{36}P(A)\\P(A)=\frac 1 3


问题 一条自动生产线连续生产nn 件产品不出故障的概率为λnn!eλ,  n=0,1,2,...\frac{\lambda^n}{n!}e^{-\lambda},\;n=0,1,2,... 假设产品的优质品率为p,(0<p<1)p,(0\lt p \lt 1),且各个产品之间的优质性相互独立。

  1. 计算该生产线从头开始连续无故障地生产了kk优质品 的概率;
  2. 若已知无故障生产了kk 件优质品,求该过程中生产了mm 件产品的概率.
答案

给出如下事件标记:

事件标记事件内容备注
AnA_n生产线无故障连续生产了nn 件产品A0,A1,A2,...A_0,A_1,A_2,... 构成了完备的事件组
BkB_k生产线无故障连续生产了kk 件优质品kk 的个数与nn 的个数相关,即A,BA,B 相关

显然,k>nk \gt n 的情况,即该过程中,总的优质品个数超过总的生产个数的情况并不存在。即:
P(BkAn)=0,where k>nP(B_k|A_n)=0,\quad \text{where } k \gt n

利用全概率公式,得

P(Bk)=n=0P(An)P(BkAn)=n=kP(An)P(BkAn)=n=kλnn!eλCnkpkqnk=n=kλnn!eλn!k!(nk)!pkqnk=(λp)kk!eλpn=k((λq)nk(nk)!)eλq=(λp)kk!eλp(eλqeλq)=(λp)kk!eλp\begin{aligned}P(B_k)&=\sum_{n=0}^\infty P(A_n)P(B_k|A_n)=\sum_{n=k}^\infty P(A_n)P(B_k|A_n)\\&=\sum_{n=k}^\infty\frac{\lambda^n}{n!}e^{-\lambda}·C_n^kp^kq^{n-k}\\&=\sum_{n=k}^\infty\frac{\lambda^n}{n!}e^{-\lambda}·\frac{n!}{k!(n-k)!}p^kq^{n-k}=\frac{(\lambda p)^k}{k!}e^{-\lambda p}\sum_{n=k}^\infty\left(\frac{(\lambda q)^{n-k}}{(n-k)!}\right)e^{-\lambda q}\\&=\frac{(\lambda p)^k}{k!}e^{-\lambda p}·(e^{\lambda q}·e^{-\lambda q})=\frac{(\lambda p)^k}{k!}e^{-\lambda p}\end{aligned}

对于问题 2 ,显然当产品数m<km \lt k 时,概率为0.
下面考虑mkm \geq k 的情况:

P(AmBk)=P(Am)P(BkAm)P(Bk)=λmm!eλCmkpkqmkk!eλp(λp)k=(λq)mk(mk)!eλqP(A_m|B_k)=\frac{P(A_m)P(B_k|A_m)}{P(B_k)}=\frac{\lambda^m}{m!}e^{-\lambda}·C_m^kp^kq^{m-k}·\frac{k!e^{-\lambda p}}{(\lambda p)^k}\\=\frac{(\lambda q)^{m-k}}{(m-k)!}e^{-\lambda q}

随机变量和分布

分布函数

定义XX 是一个随机变量,则XX 的分布函数F(x)F(x) 满足:F(x)=P{Xx<x<+}F(x)=P\{X\leq x|-\infty \lt x \lt+\infty\}.

分布函数的性质

  1. x1,x2,x1<x2, there is F(x1)F(x2)\forall x_1,x_2,\quad x_1\lt x_2,\text{ there is } F(x_1)\leq F(x_2) | 单调非减函数
  2. 0F(x)1,limxF(x)=0,limx+F(x)=10\leq F(x) \leq 1,\quad \lim_{x\to-\infty}F(x)=0,\lim_{x\to+\infty}F(x)=1
  3. F(x)=F+(x)F(x)=F_+(x) | 分布函数右连续
  4. x1,x2,x1<x2, there is P{x1<Xx2}=F(x2)F(x1)\forall x_1,x_2,\quad x_1\lt x_2,\text{ there is } P\{x_1 \lt X \leq x_2\}=F(x_2)-F(x_1)
  5. x,P{X=x}=F(x)F(x)\forall x,\quad P\{X=x\}=F(x)-F_-(x)

其中,前3条是一个函数是分布函数的充分必要条件。考试时,可用于判断某个函数是否为分布函数。

离散型随机变量

离散型随机变量XX 只能取有限或无穷可列个值,通常可以用表格的形式给出其概率分布或分布律。表格的内容由随机变量的取值xix_i 和其概率P{X=xi}=pi0P\{X=x_i\}=p_i\geq 0 构成。

离散型随机变量的分布律满足pi0,pi=1p_i\leq 0,\sum p_i=1.
其分布函数有:

F(x)=P{Xx}=xixP{X=xi}=ipiF(x)=P\{X\leq x\}=\sum_{x_i\leq x}P\{X=x_i\}=\sum_ip_i

对于已知分布律求分布函数的题目,需要将(,+)(-\infty,+\infty) 分成若干个小区间,除了第一个小区间外,其他区间均采用 左闭右开 的写法。此外,切记P{a<Xb}=F(b)F(a)P\{a \lt X \leq b\}=F(b)-F(a) 中又是左开右闭,做题时应该明确等于号的位置,以免求错。

连续型随机变量

如果存在非负可积函数f(x)f(x) 使得x,F(x)=xf(t)dt\forall x,F(x)=\int_{-\infty}^xf(t)\mathrm dt,则称其对应的随机变量XX连续型随机变量。而函数f(x)f(x) 称为XX概率密度函数

概率密度函数具有如下性质:

  1. f(x)0f(x)\geq 0
  2. +f(x)dx=1\int_{-\infty}^{+\infty}f(x)\mathrm dx=1
  3. x1,x2,x1x2, there is P{x1Xx2}=F(x2)F(x1)=x1x2f(x)dx\forall x_1,x_2,\quad x_1\leq x_2,\text{ there is } P\{x_1\leq X \leq x_2\}=F(x_2)-F(x_1)=\int_{x_1}^{x_2}f(x)\mathrm dx
  4. P{X=c}=0P\{X=c\}=0cc 是连续型随机变量XX 所取的任意定值
  5. 如果f(x)f(x)xx 处连续,则有F(x)=f(x)F'(x)=f(x)

其中,前 2 条是一个函数是概率密度函数的充分必要条件,考试时,可用于判断某个函数是否为概率密度函数。

此外,由函数f(x)f(x) 可积可以确定一个分布函数,且F(x)F(x) 必连续,并且改变f(x)f(x) 在区间内有限个点的函数值,其积分大小不变,即在该区间内取值的概率不发生改变。

常见离散型随机变量
  • 0-1分布(两点分布):P{X=k}=pkq1k,k=0,1P\{X=k\}=p^kq^{1-k},\quad k=0,1,是n=1n=1 情况下的二项分布。
  • 二项分布P{X=k}=Cnkpkqnk(k=0,1,2,...,n,q=1p)P\{X=k\}=C_n^kp^kq^{n-k}\quad(k=0,1,2,...,n,q=1-p)
  • 几何分布P{X=n}=pqn1(q=1p)P\{X=n\}=pq^{n-1}\quad(q=1-p),描述连续独立重复nn 次试验后首次成功的概率。
  • 超几何分布

P{X=k}=CMkCNMnkCNn(k=0,1,2,...,min{n,M})P\{X=k\}=\frac{C_M^kC_{N-M}^{n-k}}{C_N^n}\quad(k=0,1,2,...,\min\{n,M\})

参数为n,N,Mn,N,M 均为正数,且nN,MNn\leq N,M\leq N.

  • 泊松分布

P{X=k}=λkk!eλ(k=0,1,2,...)P\{X=k\}=\frac{\lambda^k}{k!}e^{-\lambda}\quad(k=0,1,2,...)

其中,λ>0\lambda\gt 0,则称XX 服从于参数为λ\lambda 的泊松分布,记作:XP(λ)X\sim P(\lambda).

泊松定理 若随机变量XB(n,p)X\sim B(n,p),则当pp 充分小,而nn 充分大,且np=λnp=\lambda 适中时,XX 近似服从于泊松分布,即:

Cnkpk(1p)nkλkk!eλ(λ=np)C_n^kp^k(1-p)^{n-k}\approx \frac{\lambda^k}{k!}e^{-\lambda}\quad(\lambda=np)

实际运用中,当n100,p0.10n\geq 100,p\leq 0.10 即可使用上式进行近似,不过nn 应该尽量地大,否则近似效果不佳。

常见连续型随机变量
  • 均匀分布XU(a,b)X\sim U(a,b) 表示随机变量XX 在区间[a,b][a,b] 上服从均匀分布,在该区间内的任意子区间取值的概率只与其长度有关。即P{cXd}=(dc)/(ba)P\{c\leq X\leq d\}=(d-c)/(b-a).

f(x)={1ba,a<x<b0,elseF(x)={0,x<axaba,ax<b1,x>bf(x)=\begin{cases}\frac 1 {b-a}&,a\lt x\lt b\\0&,else\end{cases}\\F(x)=\begin{cases}0&,x \lt a\\\frac{x-a}{b-a}&,a\leq x\lt b\\1&,x\gt b\end{cases}

  • 指数分布XE(λ),λ>0X\sim E(\lambda),\lambda \gt 0. 具有“无记忆性”,即P{X>s+tX>s}=P{X>s}P\{X\gt s+t|X\gt s\}=P\{X\gt s\}

f(x)={λeλx,x>00,x0F(x)={1eλx,x>00,x0f(x)=\begin{cases}\lambda e^{-\lambda x}&,x\gt 0\\0&,x\leq 0\end{cases}\\F(x)=\begin{cases}1-e^{-\lambda x}&,x\gt 0\\0&,x\leq 0\end{cases}

正态分布|Normal distribution

若随机变量XX 的概率密度为:

f(x)=12πσe(xμ)22σ2,(<x<+)f(x)=\frac 1 {\sqrt{2\pi}·\sigma}e^{-\frac{(x-\mu)^2}{2\sigma^2}},\quad(-\infty \lt x\lt +\infty)

其中,μ,σ\mu,\sigma 均为常数,且σ>0\sigma\gt 0,则称XX 服从于参数为μ,σ2\mu,\sigma^2正态分布,记为XN(μ,σ2)X\sim N(\mu,\sigma^2).
其分布函数为:

F(x)=12πσxe(tμ)22σ2dtF(x)=\frac{1}{\sqrt{2\pi}\sigma}\int_{-\infty}^xe^{-\frac{(t-\mu)^2}{2\sigma^2}}\mathrm dt

特别地,当μ=0,σ=1\mu=0,\sigma=1 时,称XX 服从 标准正态分布,记为XN(0,1)X\sim N(0,1). 我们将其对应的概率密度和分布函数分别记为φ(x),Φ(x)\varphi(x),\varPhi(x).

事实上,根据函数的平移伸缩变换,可以将普通的正态分布用标准正态分布表示。即:

F(x)=P{Xx}=Φ(xμσ),xRF(x)=P\{X\leq x\}=\varPhi\left(\frac{x-\mu}{\sigma}\right),x\in R

此外,如果利用函数分析的方法对正态分布的概率密度函数分析,还可以得到以下结论:

  1. y=f(x)y=f(x) 关于x=μx=\mu 对称,即对h>0,P{μh<Xμ}=P{μ<Xμ+h}\forall h\gt 0,P\{\mu-h\lt X \leq\mu\}=P\{\mu\lt X\leq \mu+h\}
  2. fmax(x)=f(μ)=1/2πσf_{\max}(x)=f(\mu)=1/\sqrt{2\pi}\sigma
  3. x=μx=\muf(x)f(x) 的驻点
  4. x=μ±σx=\mu\pm\sigma 处的曲线f(x)f(x) 有拐点
  5. 曲线y=f(x)y=f(x)OxOx 轴为渐近线
  6. Φ(x)=1Φ(x),Φ(0)=0.5\varPhi(x)=1-\varPhi(-x),\varPhi(0)=0.5
  7. P{Xa}=2Φ(a)1,XN(0,1),a>0P\{|X|\leq a\}=2\varPhi(a)-1,\quad X\sim N(0,1),a\gt 0
  8. φ(x)=xφ(x)\varphi'(x)=x\varphi(x)

既然正态分布(也叫高斯分布|Gaussian distribution)的函数是概率密度函数,那么它一定满足概率密度的必要条件:+f(x)dx=1\int_{-\infty}^{+\infty}f(x)\mathrm dx=1.
下面我们根据这个结论看看标准正态分布的概率密度:

+φ(x)dx=12π+e(x/2)2dx=1π+e(x/2)2d(x/2)=1\int_{-\infty}^{+\infty}\varphi(x)\mathrm dx=\frac 1 {\sqrt{2\pi}}\int_{-\infty}^{+\infty}e^{-(x/\sqrt 2)^2}\mathrm dx=\frac 1 {\sqrt{\pi}}\int_{-\infty}^{+\infty}e^{-(x/\sqrt 2)^2}\mathrm d(x/\sqrt 2)=1

即是:

+ex2dx=π\int_{-\infty}^{+\infty}e^{-x^2}\mathrm dx=\sqrt \pi

这个公式就是 欧拉泊松积分。是一个十分常用的定积分结论,其不定积分是不能用初等函数表示的。
而这个积分的证明我们也在本文章高等数学的Part给出。

随机变量的函数也是随机变量

随机变量的函数

对于随机变量XX 其函数Y=g(X)Y=g(X) 也是随机变量。对YY 的概率密度以及分布函数的求解,往往可以借助已知的XX 来计算。

  • 离散型
    已知P{X=xi}=piP\{X=x_i\}=p_i ,则有P{Y=g(xi)}=piP\{Y=g(x_i)\}=p_i. 特别的,在函数g(x)g(x) 不是一一映射的情况下,有:

P{Y=yj}=g(xi)=yjpiP\{Y=y_j\}=\sum_{g(x_i)=y_j}p_i

  • 连续型

连续型有两种思路。
第一种是直接根据定义,求出YY 的分布函数:

FY(y)=P{g(X)y}=g(x)yf(x)dxF_Y(y)=P\{g(X)\leq y\}=\int_{g(x)\leq y}f(x)\mathrm dx

然后再对其求导得到概率密度。
第二种则对函数有要求,需要在函数g(x)g(x) 严格单调 的情况下,设其反函数为h(y)h(y),从而有概率密度:

fY(y)={fX[h(y)]h(y),α<y<β0,elsef_Y(y)=\begin{cases}f_X[h(y)]·|h'(y)|&,\alpha\lt y\lt\beta\\0&,else\end{cases}

其中,(α,β)(\alpha,\beta)g(x)g(x)XX 可能取值的区间上的值域。

特别地,我们有:随机变量YYXX 的分布函数(连续)时,即Y=F(X)Y=F(X) 时,YY 服从于均匀分布。YU(0,1)Y\sim U(0,1)

推导如下:

FY(y)=P{Yy}=P{F(X)y}=P{XF1(y)}=F[F1(y)]=y\begin{aligned}F_Y(y)&=P\{Y\leq y\}=P\{F(X)\leq y\}\\&=P\{X\leq F^{-1}(y)\}\\&=F[F^{-1}(y)]\\&=y\end{aligned}

多维随机变量及联合分布

联合分布函数与边缘分布函数

(X,Y)(X,Y) 是二维随机变量,x,y,F(x,y)=P(Xx,Yy)\forall x,y,\quad F(x,y)=P(X\leq x,Y\leq y) 称为该二维随机变量的联合分布函数

联合分布函数的性质

  1. 0F(x,y)10\leq F(x,y)\leq 1,且关于每一自变量均单调不减
  2. F(x,y)F(x,y) 分别是x,yx,y右连续函数
  3. F(+,+)=limx+y+F(x,y)=1;F(,)=limxyF(x,y)=0\begin{aligned}F(+\infty,+\infty)=\lim_{x\to+\infty\atop y\to+\infty}F(x,y)=1;\quad F(-\infty,-\infty)=\lim_{x\to-\infty\atop y\to-\infty}F(x,y)=0\end{aligned}
  4. F(,y)=limxF(x,y)=0;F(x,)=limyF(x,y)=0\begin{aligned}F(-\infty,y)=\lim_{x\to-\infty}F(x,y)=0;\quad F(x,-\infty)=\lim_{y\to-\infty}F(x,y)=0\end{aligned}
  5. 随机点(X,Y)(X,Y) 落在矩形域G={(x,y)x1<Xx2,y1<Yy2}G=\{(x,y)|x_1\lt X\leq x_2,y_1\lt Y\leq y_2\} 上的概率为:

P{(X,Y)G}=P{(x,y)x1<Xx2,y1<Yy2}=F(x2,y2)F(x2,y1)[F(x1,y2)F(x1,y1)]\begin{aligned}\boldsymbol P\{(X,Y)\in G\}&=\boldsymbol P\{(x,y)|x_1\lt X\leq x_2,y_1\lt Y\leq y_2\}\\&=F(x_2,y_2)-F(x_2,y_1)-\left[F(x_1,y_2)-F(x_1,y_1)\right]\end{aligned}

特别地,对于第 5 点,如果恒有x1<x2,y1<y2x_1\lt x_2,y_1\lt y_2,即区域GG 的面积大于0,那么:

P{(X,Y)G}0\boldsymbol P\{(X,Y)\in G\}\geq0

也应该恒成立。这一点可作为联合分布函数的重要性质,对于判断一个二元函数是否为联合发布函数十分有用。

例如F(x,y)={1,x+y0,0,x+y<0F(x,y)=\begin{cases}1,&x+y\geq 0,\\0,&x+y\lt0\end{cases} 并不是分布函数。
虽然可以证明F(x,y)F(x,y) 满足单调不减,右连续 等等性质。但是:
P{2<x2,2<y2}=F(2,2)F(2,2)F(2,2)+F(2,2)=1<0P\{-2\lt x\leq 2,-2\lt y\leq 2\}=F(2,2)-F(2,-2)-F(-2,2)+F(-2,-2)=-1\lt0
从而可以说明它本非分布函数。

  • 边缘分布函数

设二维随机变量(X,Y)(X,Y) 的分布函数为F(x,y)F(x,y)
分别称FX(x)=P{Xx},FY(y)=P{Yy}F_X(x)=P\{X\leq x\},F_Y(y)=P\{Y\leq y\} 为关于XXYY边缘分布函数
有:

FX(x)=P{Xx}=P{Xx,Y<+}=F(x,+)=limy+F(x,y)FY(y)=P{Yy}=P{X<+,Yy}=F(+,y)=limx+F(x,y)\begin{aligned}F_X(x)=P\{X\leq x\}=P\{X\leq x,Y\lt+\infty\}=F(x,+\infty)=\lim_{y\to+\infty}F(x,y)\\F_Y(y)=P\{Y\leq y\}=P\{X\lt+\infty,Y\leq y\}=F(+\infty,y)=\lim_{x\to+\infty}F(x,y)\end{aligned}

一般地,由(X,Y)(X,Y) 的分布函数可以确定其对应的边缘分布函数,反正则不然。由边缘分布函数并不能反过来确定分布函数。
只有当XYX、Y 相互独立时,可以由边缘分布得到联合发布。

对比项离散型连续型
表现形式联合概率分布, 表格联合概率密度, 满足F(x,y)=xyf(u,v)dudvF(x,y)=\int_{-\infty}^x\int_{-\infty}^yf(u,v)\mathrm du\mathrm dv
特征pij>0,i=1j=1pij=1p_{ij}\gt0,\sum\limits_{i=1}^{\infty}\sum\limits_{j=1}^{\infty}p_{ij}=1f(x,y)>0,(x,y)R2f(x,y)=1f(x,y)\gt0,\iint_{(x,y)\in R^2}f(x,y)=1
分布函数分段函数, 与一维离散类似f(x,y)f(x,y)(x,y)(x,y) 处连续时,f(x,y)=2F(x,y)xyf(x,y)=\frac{\partial^2F(x,y)}{\partial x\partial y}
边缘概率pi=j=1pijp_{i·}=\sum\limits_{j=1}^{\infty}p_{ij}fX(x)=+f(x,y)dyf_X(x)=\int_{-\infty}^{+\infty}f(x,y)\mathrm dy
条件概率P{X=xiY=yj}=pijpj\begin{aligned}P\{X=x_i▏Y=y_j\}=\frac{p_{ij}}{p_{·j}}\end{aligned}fXY(xy)=f(x,y)fY(y)\begin{aligned}f_{X▏Y}(x▏y)=\frac{f(x,y)}{f_Y(y)}\end{aligned}

上表中给出了离散型和连续型涉及到概率分布的计算时所运用到的公式。其中,边缘概率和条件概率分别都只举了一例,另一例可类比得出。

在求条件概率密度时应当注意 定义域,确保作为条件的边缘概率密度大于0,因此要去除边缘概率为0的点。

由连续型条件概率密度(此处以在条件Y=yY=yXX 的条件概率密度 为例)可得出对应的条件分布函数

FXY(xy)=xf(t,y)fY(y)dtF_{X|Y}(x|y)=\int_{-\infty}^x\frac{f(t,y)}{f_Y(y)}\mathrm dt

此外,对于条件密度函数,也有和乘法公式类似的结论:

f(x,y)=fX(x)fYX(yx)(fX(x)>0)=fY(y)fXY(xy)(fY(y)>0)\begin{aligned}f(x,y)&=f_X(x)f_{Y|X}(y|x)\quad(f_X(x)\gt0)\\&=f_Y(y)f_{X|Y}(x|y)\quad(f_Y(y)\gt0)\\\end{aligned}

二维独立性

x,y\forall x,y 有:P{Xx,Yy}=P{Xx}P{Yy}P\{X\leq x,Y\leq y\}=P\{X\leq x\}P\{Y\leq y\},即:

F(x,y)=FX(x)FY(y)F(x,y)=F_X(x)F_Y(y)

则称 随机变量X,YX,Y 相互独立.

  • 充要条件
    • 离散型:P{X=xi,Y=yj}=P{X=xi}P{Y=yj}P\{X=x_i,Y=y_j\}=P\{X=x_i\}P\{Y=y_j\}pij=pipjp_{ij}=p_{i·}p_{·j}
    • 连续型:f(x,y)=fX(x)fY(y)f(x,y)=f_X(x)f_Y(y),其中(X,Y)(X,Y) 是连续点
  • 性质
    • 若随机变量X1,X2,...,XmX_1,X_2,...,X_m 相互独立,则它们的函数g(X1),g(X2),...,g(Xm)g(X_1),g(X_2),...,g(X_m) 也相互独立
    • X,YX,Y 相互独立,那么XX 关于YY 的条件分布就是其无条件分布(边缘概率分布)
装箱问题与几何相关的概率问题

经典例题之装箱问题

示例题:求边缘密度和条件密度

二维随机分布与变量函数的分布

二维连续型随机分布

  • 均匀分布

f(x,y)={1A,(x,y)G0,otherwisef(x,y)=\begin{cases}\frac{1}{A},&(x,y)\in G\\\\0,&\text{otherwise}\end{cases}

其中,AA 是平面上一有界区域GG 的面积.

  • 二维正态分布 记为:(X,Y)N(μ1,μ2;σ12,σ22;ρ)(X,Y)\sim N(\mu_1,\mu_2;\sigma_1^2,\sigma_2^2;\rho)

f(x,y)=exp{12(1ρ2)[(xμ1σ1)22ρxμ1σ1yμ2σ2+(yμ2σ2)2]}2πσ1σ21ρ2\begin{aligned}f(x,y)=\frac{\exp\left\{-\frac{1}{2(1-\rho^2)}\left[\left(\frac{x-\mu_1}{\sigma_1}\right)^2-2\rho·\frac{x-\mu_1}{\sigma_1}·\frac{y-\mu_2}{\sigma_2}+\left(\frac{y-\mu_2}{\sigma_2}\right)^2\right]\right\}}{2\pi\sigma_1\sigma_2\sqrt{1-\rho^2}}\\\end{aligned}

其中,σ1,σ2>0,ρ<1\sigma_1,\sigma_2\gt0,|\rho|\lt1X,YX,Y相关系数,即ρ=Cov(X,Y)σ1σ2\begin{aligned}\rho=\frac{\mathrm{Cov}(X,Y)}{\sigma_1\sigma_2}\end{aligned}

有如下性质:

  1. (X,Y)N(μ1,μ2;σ12,σ22;ρ)XN(μ1,σ12),YN(μ2,σ22)(X,Y)\sim N(\mu_1,\mu_2;\sigma_1^2,\sigma_2^2;\rho)\Rightarrow X\sim N(\mu_1,\sigma_1^2),Y\sim N(\mu_2,\sigma_2^2)
  2. X,YX,Y 相互独立ρ=0\Leftrightarrow \rho=0
  3. (X,Y)N(μ1,μ2;σ12,σ22;ρ)aX+bYN(aμ1+bμ2,a2σ12+b2σ22+2abρσ1σ2)(X,Y)\sim N(\mu_1,\mu_2;\sigma_1^2,\sigma_2^2;\rho)\Rightarrow aX+bY\sim N(a\mu_1+b\mu_2,a^2\sigma_1^2+b^2\sigma_2^2+2ab\rho\sigma_1\sigma_2)

二维随机变量函数的分布

对于离散型二维随机变量的函数,其分布较为简单,只需逐一列出即可,因此此处直接略过。
我们将重点放在二维连续型随机变量的函数中来。

设二维连续型随机变量(X,Y)(X,Y) 有联合概率密度f(x,y)f(x,y)。随机变量函数可作如下几种分类:

  1. g(x,y)g(x,y) 二元连续可微时,Z=g(X,Y)Z=g(X,Y) 仍是连续型随机变量。其概率密度的计算方法是:
    • 先计算FZ(z)=P{Zz}=Dzf(x,y)dxdy,Dz={(x,y)g(x,y)z}F_Z(z)=P\{Z\leq z\}=\iint_{D_z}f(x,y)dxdy,\quad D_z=\{(x,y)|g(x,y)\leq z\}
    • 对分布函数求导得到概率密度fZ(z)=ddzFZ(z)f_Z(z)=\frac{d}{dz}F_Z(z)
  2. Z=g(X,Y)Z=g(X,Y)离散型随机变量时,有:
    • P{Z=zi}=Dzif(x,y)dxdy,Dzi={(x,y)g(x,y)=zi}P\{Z=z_i\}=\iint_{D_{z_i}}f(x,y)dxdy,\quad D_{z_i}=\{(x,y)|g(x,y)=z_i\}

更加特别地,如果Z=g(X,Y)Z=g(X,Y) 中,XX 是离散型 而YY 是连续型。则一般对离散型变量XX 的各种可能取值用 全概率公式 展开。即:

FZ(z)=P{g(X,Y)z}=iP{X=xi}P{g(X,Y)zX=xi}=ipiP{g(X,Y)zX=xi}\begin{aligned}F_Z(z)=P\{g(X,Y)\leq z\}&=\sum_iP\{X=x_i\}P\{g(X,Y)\leq z|X=x_i\}\\&=\sum_i p_i·P\{g(X,Y)\leq z|X=x_i\}\end{aligned}

上表格.

随机变量函数概率分布/密度备注
Z=X+YZ=X+YfZ(z)=+f(x,zx)dxf_Z(z)=\int_{-\infty}^{+\infty}f(x,z-x)dx相互独立时,FZ=fXfYF_Z=f_X*f_Y
Z=XYZ=X-YfZ(z)=+f(x,xz)dxf_Z(z)=\int_{-\infty}^{+\infty}f(x,x-z)dx-
Z=XYZ=XYfZ(z)=+1abs(x)f(x,zx)dxf_Z(z)=\int_{-\infty}^{+\infty}\frac 1 {abs(x)}f(x,\frac z x)dx-
Z=max{X,Y}Z=\max\{X,Y\}Fmax(z)=F(z,z)F_{\max}(z)=F(z,z)独立时,可推广到nn
Z=min{X,Y}Z=\min\{X,Y\}Fmin(z)=FX(z)+FY(z)F(z,z)F_{\min}(z)=F_X(z)+F_Y(z)-F(z,z)同上

上表仅列举了一般情况,均可以从原理出发推导而来,也建议考生多尝试从原理进行推导,以便应对上表中没有的随机变量函数。

此外,上述结论在当X,YX,Y 独立时,还可以进一步化为边缘密度函数的乘积形式。
注:第一行中的符号 * 表示的是卷积,第三行的 abs(·) 表示绝对值

其他重要结论
X,YX,Y 相互独立时,其二项分布、泊松分布、正态分布和卡方分布有:

  1. XB(n,p),YB(m,p)X+YB(n+m,p);X\sim B(n,p),Y\sim B(m,p)\Rightarrow X+Y\sim B(n+m,p);
  2. XP(λ1),YP(λ2)X+YP(λ1+λ2);X\sim P(\lambda_1),Y\sim P(\lambda_2)\Rightarrow X+Y\sim P(\lambda_1+\lambda_2);
  3. XN(μ1,σ12),YN(μ2,σ22)X+YN(μ1+μ2,σ12+σ22);X\sim N(\mu_1,\sigma_1^2),Y\sim N(\mu_2,\sigma_2^2)\Rightarrow X+Y\sim N(\mu_1+\mu_2,\sigma_1^2+\sigma_2^2);
  4. Xχ2(n),Yχ2(m)X+Yχ2(n+m).X\sim \chi^2(n),Y\sim \chi^2(m)\Rightarrow X+Y\sim \chi^2(n+m).

上述结果对nn 个相互独立同分布的随机变量也成立。

随机变量的数字特征

数学期望与方差

数学期望与方差

  • 离散型随机变量X,P{X=xi}=piX,P\{X=x_i\}=p_i ,若 无穷级数i=1xipi\sum\limits_{i=1}^{\infty}x_ip_i 绝对收敛,则称其和为XX数学期望 或 均值。记作E(X)E(X).
  • 连续型随机变量XX,其概率密度为f(x)f(x),若 反常积分+xf(x)dx\int\limits_{-\infty}^{+\infty}xf(x)dx 绝对收敛,则称其值为XX 的数学期望.
  • 若反常积分或无穷级数并不绝对收敛,则称该随机变量的数学期望不存在

当离散变量的取值xix_i 取极限时,即xixx_i\to x
其概率大小为pi=F(xi)F(xi1)=ΔFdF(x)p_i=F(x_i)-F(x_{i-1})=\Delta F\to\mathrm d F(x).
从而,E(x)=xipixdF(x)=xf(x)dxE(x)=\sum x_ip_i\to\int xdF(x)=\int xf(x)dx

  • 考虑随机变量XX 的函数Y=g(X)Y=g(X)
    • 离散型E(Y)=i=1g(xi)piE(Y)=\sum\limits_{i=1}^{\infty}g(x_i)p_i(同样要求绝对收敛)
    • 连续型E(Y)=+g(x)f(x)dxE(Y)=\int\limits_{-\infty}^{+\infty}g(x)f(x)dx (同样要求绝对收敛)
  • 二维随机变量(X,Y)(X,Y) 及其函数X=g(X,Y)X=g(X,Y) 可同样类比,此处略过.

💦下面总结一些常用公式:

  1. E(C)=CE(C)=CCC 是常数;
  2. E(aX+b)=aE(x)+bE(aX+b)=aE(x)+ba,ba,b 是常数;
  3. E(X±Y)=E(X)±E(Y)E(X\pm Y)=E(X)\pm E(Y)
  4. X,YX,Y相互独立X,Y\Rightarrow X,Y 不相关E(XY)=E(X)E(Y)\Leftrightarrow E(XY)=E(X)E(Y)
  5. If XY, then E(X)E(Y)\text{If }X\geq Y,\text{ then } E(X)\geq E(Y) .

  • E[XE(X)]2E[X-E(X)]^2 存在,则称其为随机变量XX方差,记作D(X),Var(X)D(X),Var(X).
  • 方差的算术平方根D(X)\sqrt{D(X)} 称为XX标准差(或均方差),记作σ(X)\sigma(X).
  • X=XE(X)D(X)\begin{aligned}X^*=\frac{X-E(X)}{\sqrt{D(X)}}\end{aligned}XX标准化随机变量,有E(X)=0,D(X)=1E(X^*)=0,D(X^*)=1.

D(X)=E{[XE(X)]2}=E{X22XE(X)+E2(X)}=E(X2)2E(X)E(X)+E2(X)=E(X2)E2(X)\begin{aligned}D(X)&=E\{[X-E(X)]^2\}=E\{X^2-2XE(X)+E^2(X)\}\\&=E(X^2)-2E(X)·E(X)+E^2(X)\\&=E(X^2)-E^2(X)\end{aligned}

该结论应当熟记。

💦下面总结一些常用公式:

  1. D(C)=0D(C)=0CC 是常数;
  2. D(aX+b)=a2D(x)D(aX+b)=a^2D(x)a,ba,b 是常数;
  3. D(X±Y)=D(X)+D(Y)±2Cov(X,Y)D(X\pm Y)=D(X)+D(Y)\pm2Cov(X,Y) .

常见一元随机变量的数字特征:

分布数学期望方差
二项分布XB(n,p)X\sim B(n,p)npnpnp(1p)np(1-p)
泊松分布XP(λ)X\sim P(\lambda)λ\lambdaλ\lambda
几何分布P{X=k}=p(1p)k1P\{X=k\}=p(1-p)^{k-1}1/p1/p(1p)/p2(1-p)/p^2
指数分布XE(λ)X\sim E(\lambda)1/λ1/\lambda1/λ21/\lambda^2
均匀分布XU(a,b)X\sim U(a,b)(a+b)/2(a+b)/2(ba)2/12(b-a)^2/12
正态分布XN(μ,σ2)X\sim N(\mu,\sigma^2)μ\muσ2\sigma^2

之所以前面一直强调期望存在,以及很多题目中都给出期望存在的前提条件,自然是因为存在没有期望和方差的概率分布。例如:柯西分布
(物理学家也将之称为洛伦兹分布或者 Breit-Wigner 分布)

对于随机变量XC(γ,x0)X\sim C(\gamma,x_0) ,取γ=1,x0=0\gamma=1,x_0=0 时,有:

f(x)=1π(1+x2)f(x)=\frac{1}{\pi(1+x^2)}

可以自行验证其不存在数学期望、方差、高阶矩。

协方差与相关系数

协方差与相关系数

  • 二维随机变量(X,Y)(X,Y)E[(XEX)(YEY)]E[(X-EX)(Y-EY)] 存在,则称其为(X,Y)(X,Y)协方差,记为Cov(X,Y)Cov(X,Y).
  • 在上述基础上若有D(X),D(Y)D(X),D(Y) 均存在,则定义X,YX,Y相关系数ρXY\rho_{XY}

ρXY=Cov(X,Y)D(X)D(Y)\rho_{XY}=\frac{Cov(X,Y)}{\sqrt{D(X)}\sqrt{D(Y)}}

  • 相关性质
    • ρXY1|\rho_{XY}|\leq 1
    • ρXY=0\rho_{XY}=0 时,称 随机变量X,YX,Y 不相关
    • ρXY=1a,bP{Y=aX+b}=1|\rho_{XY}=1|\Leftrightarrow \exists a,b\quad P\{Y=aX+b\}=1

💦下面总结一些常用公式:

  1. Cov(X,Y)=E(XY)E(X)E(Y)Cov(X,Y)=E(XY)-E(X)E(Y)
  2. D(X±Y)=D(X)+D(Y)±2Cov(X,Y)D(X\pm Y)=D(X)+D(Y)\pm2Cov(X,Y)
  3. Cov(aX+b,cY+d)=acCov(X,Y)Cov(aX+b,cY+d)=acCov(X,Y)
  4. Cov(aX1+bX2+c,Y)=aCov(X1,Y)+bCov(X2,Y)Cov(aX_1+bX_2+c,Y)=aCov(X_1,Y)+bCov(X_2,Y)
  5. X,YX,Y 相互独立Cov(X,Y)=0\Rightarrow Cov(X,Y)=0 ,反之不一定成立.
    • 🦄二维正态分布中 不相关能反推出独立
    • 🦄0-1分布中 不相关能反推出独立

事实上,相关系数ρ\rho 是用于衡量以aX+baX+b 近似YY 的好坏程度的。

以均方误差e=E[(aX+b)Y]2e=E[(aX+b)-Y]^2 作为目标函数,将问题化为最优化问题。该凸函数可通过偏导数为0求得其最小值。

最终得到:mina,be=(1ρ2)D(Y)\min\limits_{a,b} e=(1-\rho^2)D(Y)

  • (X,Y)(X,Y) 是二维随机变量,现定义其关于的数字特征如下表:
数字特征表达式备注
kk 阶原点矩E(Xk)E(X^k)简称:kk 阶矩,数学期望就是其一阶矩
kk 阶中心矩E[XE(X)]kE[X-E(X)]^k方差D(X)D(X) 就是其二阶中心矩
k+lk+l 阶混合矩E(XkYl)E(X^kY^l)-
k+lk+l 阶混合中心距E[(XE(X))k(YE(Y))l]E[(X-E(X))^k(Y-E(Y))^l]协方差Cov(X,Y)Cov(X,Y) 就是二阶混合中心距
※重期望及其相关应用

重期望与条件期望

待更

大数定律与中心极限定理

概念速览

依概率收敛

X1,X2,,Xn,X_1,X_2,\cdots,X_n,\cdots 是一个随机变量序列,aa 是一个常数,若ε>0\forall \varepsilon\gt0,有:

limnP{Xna<ε}=1\lim_{n\to\infty}\boldsymbol P\{|X_n-a|\lt\varepsilon\}=1

则称该 随机变量序列依概率收敛于aa,记作:XnPaX_n\stackrel{P}{\longrightarrow}a

XnPa,YnPbX_n\stackrel{P}{\longrightarrow}a,Y_n\stackrel{P}{\longrightarrow}b,又设函数g(x,y)g(x,y) 在点(a,b)(a,b) 处连续,则:g(Xn,Yn)Pg(a,b)g(X_n,Y_n)\stackrel{P}{\longrightarrow}g(a,b)

切比雪夫不等式

设随机变量XX 存在数学期望μ\mu 和方差σ2\sigma^2 ,则ε>0\forall \varepsilon \gt0,有:

P{Xμε}σ2ε2\boldsymbol P\{|X-\mu|\geq\varepsilon\}\leq\frac{\sigma^2}{\varepsilon^2}

切比雪夫不等式给出了随机变量分布未知但期望和方差已知时事件{Xμε}\{|X-\mu|\geq\varepsilon\} 的概率界限

切比雪夫大数定律

X1,X2,,Xn,X_1,X_2,\cdots,X_n,\cdots 相互独立,且EXi=μi,DXi=σi2EX_i=\mu_i,DX_i=\sigma^2_i. 若c>0\exists c\gt0 使得σi2c\sigma_i^2\leq c 对任意ii 都成立,即是这一随机变量序列的方差有公共上界,则:

ε>0limnP{1ni=1nXi1ni=1nμi<ε}=11ni=1n(Xiμi)P0\begin{aligned}\forall\varepsilon\gt0\quad&\lim_{n\to\infty}\boldsymbol P\left\{\left|\frac 1 n\sum_{i=1}^nX_i-\frac 1 n\sum_{i=1}^n\mu_i\right|\lt\varepsilon\right\}=1\\\\\sim\quad&\frac 1 n\sum_{i=1}^n(X_i-\mu_i)\stackrel{P}\longrightarrow0\end{aligned}

伯努利大数定律

μn\mu_nnn 次伯努利试验中,事件AA 发生的次数,每次AA 发生的概率都是pp,则有:

ε>0limnP{μnnp<ε}=1μnnpP0μnnPp\begin{aligned}\forall\varepsilon\gt0\quad&\lim_{n\to\infty}\boldsymbol P\left\{\left|\frac{\mu_n}{n}-p\right|\lt\varepsilon\right\}=1\\\\\sim\quad&\frac{\mu_n}{n}-p\stackrel{P}\longrightarrow0\\\sim\quad&\frac{\mu_n}{n}\stackrel{P}\longrightarrow p\end{aligned}

这表明伯努利试验次数nn 很大时,事件发生次数/试验次数,即频数依概率趋于事件发生的概率。

辛钦大数定律

X1,X2,,Xn,X_1,X_2,\cdots,X_n,\cdots 相互独立同分布,且EXi=μEX_i=\mu,则有:

ε>0limnP{1ni=1nXiμ<ε}=11ni=1nXiμP01ni=1nXiPμ\begin{aligned}\forall\varepsilon\gt0\quad&\lim_{n\to\infty}\boldsymbol P\left\{\left|\frac 1 n\sum_{i=1}^nX_i-\mu\right|\lt\varepsilon\right\}=1\\\\\sim\quad&\frac 1 n\sum_{i=1}^nX_i-\mu\stackrel{P}\longrightarrow0\\\sim\quad&\frac 1 n\sum_{i=1}^nX_i\stackrel{P}\longrightarrow\mu\end{aligned}

这表明独立同分布的随机变量序列的平均值依概率趋于其数学期望。

列维-林德伯格定理

独立同分布的中心极限定理X1,X2,,Xn,X_1,X_2,\cdots,X_n,\cdots 相互独立同分布,且EXi=μ,DXi=σ2EX_i=\mu,DX_i=\sigma^2,则有:

xlimnFn(x)=limnP{i=1nXinμnσx}=12πxet22dt=Φ(x)\begin{aligned}\forall x\quad\lim_{n\to\infty}F_n(x)&=\lim_{n\to\infty}\boldsymbol P\left\{\frac{\sum\limits_{i=1}^nX_i-n\mu}{\sqrt n\sigma}\leq x\right\}\\\\&=\frac 1 {\sqrt{2\pi}}\int_{-\infty}^xe^{-\frac{t^2}{2}}\mathrm dt\\\\&=\varPhi(x)\end{aligned}

其中,Φ(x)\varPhi(x) 是标准正态分布的分布函数.

该定理表明当nn 很大时,独立同分布随机变量的XiN(nμ,nσ2)\sum X_i\sim N(n\mu,n\sigma^2).

棣莫弗-拉普拉斯定理

也就是 中心极限定理。它描述二项分布以正态分布为其极限分布
设随机变量XB(n,p)X\sim B(n,p) ,则有:

xlimnP{Xnpnp(1p)x}=Φ(x)\begin{aligned}\forall x\quad&\lim_{n\to\infty}\boldsymbol P\left\{\frac{X-np}{\sqrt{np(1-p)}}\leq x\right\}=\varPhi(x)\end{aligned}

该定理表明当xx 很大时,服从于参数为n,pn,p 的二项分布的随机变量XN(np,npq)X\sim N(np,npq).

利用 中心极限定理 进行估计一般比用 切比雪夫不等式 要好得多

数理统计基础

总体、样本和统计量

在数理统计中:

总体 表示所研究对象的某项数量指标XX 取值的全体,总体 中的每一个元素称为 个体

样本 全称为 简单随机样本,是与总体同分布的nn 个相互独立的随机变量X1,X2,..,XnX_1,X_2,..,X_n ,其中nn 称为样本容量。

x1,x2,..,xnx_1,x_2,..,x_n 分别是X1,X2,..,XnX_1,X_2,..,X_n观测值,则称x1,x2,..,xnx_1,x_2,..,x_n样本值,又称为总体XXnn 个独立观测值。

样本X1,X2,..,XnX_1,X_2,..,X_n 的联合分布函数、联合概率密度、概率分布:

F(x1,x2,..,xn)=i=1nF(xi),xiRf(x1,x2,..,xn)=i=1nf(xi),xiRP{X1=x1,X2=x2,..,Xn=xn}=i=1nP{Xi=xi}\begin{aligned}F(x_1,x_2,..,x_n)&=\prod_{i=1}^nF(x_i),\quad x_i\in R\\f(x_1,x_2,..,x_n)&=\prod_{i=1}^nf(x_i),\quad x_i\in R\\P\{X_1=x_1,X_2=x_2,..,X_n=x_n\}&=\prod_{i=1}^nP\{X_i=x_i\}\end{aligned}

统计量

统计量是样本X1,X2,..,XnX_1,X_2,..,X_n 的函数:T=f(X1,X2,..,Xn)T=f(X_1,X_2,..,X_n).
统计量不依赖任何未知参数,作为随机样本的函数,统计量也是随机变量.
样本的 数字特征 是最常用的统计量.

  • 样本均值

X=1ni=1nXi\overline{X}=\frac 1 n\sum_{i=1}^nX_i

  • 样本方差

S2=1n1i=1n(XiX)2=1n1(i=1nXi2nX2)S^2=\frac{1}{n-1}\sum_{i=1}^n\left(X_i-\overline{X}\right)^2=\frac{1}{n-1}\left(\sum_{i=1}^nX_i^2-n\overline{X}^2\right)

  • 样本标准差

S=1n1i=1n(XiX)22S=\sqrt[2]{\frac{1}{n-1}\sum_{i=1}^n\left(X_i-\overline{X}\right)^2}

  • 样本kk 阶原点矩

Ak=1ni=1nXikA_k=\frac 1 n\sum_{i=1}^nX_i^k

  • 样本kk 阶中心矩

Bk=1ni=1n(XiX)kB_k=\frac 1 n\sum_{i=1}^n\left(X_i-\overline{X}\right)^k

注:B2=n1nS2<S2\begin{aligned}B_2=\frac{n-1}{n}S^2\lt S^2\end{aligned}

如果总体XX 具有数学期望E(X)=μ\boldsymbol E(X)=\mu 和方差D(X)=σ2\boldsymbol D(X)=\sigma^2 (无论总体服从于什么分布),则总有:

E(X)=E(X)=μD(X)=D(X)/n=σ2nE(S2)=D(X)=σ2\begin{aligned}\boldsymbol E\left(\overline{X}\right)=\boldsymbol E(X)=\mu\\\boldsymbol D\left(\overline{X}\right)=\boldsymbol D(X)/n=\frac{\sigma^2}{n}\\\boldsymbol E\left(S^2\right)=\boldsymbol D(X)=\sigma^2\\\end{aligned}

常见的抽样分布

统计量的分布也称为抽样分布. 而抽样分布可能含有未知参数,即当样本函数含位置参数但是其分布又不依赖于该参数时,可以将这种样本函数的分布也称为抽样分布。

卡方分布

卡方分布(χ2\chi^2 分布)是一种非负连续型随机变量的分布,其密度函数f(x)f(x) 的图像位于第一象限,峰值向左偏,峰值随着自由度nn 的增大向右移动,如下图所示。

卡方分布示意图

典型的卡方分布对应的统计量为:

χ2=X12+X22++Xn2=i=1nXi2\chi^2=X_1^2+X_2^2+\cdots+X_n^2=\sum_{i=1}^nX_i^2

其中,样本均相互独立,且都服从于标准正态分布N(0,1)N(0,1).

我们称统计量χ2\chi^2 服从于自由度为nn 的卡方分布,记作χ2χ2(n)\chi^2\sim\chi^2(n).

  • 分位点

对于给定的α  (0<α<1)\alpha\;(0\lt\alpha\lt1),如果满足:

P{χ2>χα2(n)}=χα2(n)+f(x)dx=αP\{\chi^2\gt\chi^2_{\alpha}(n)\}=\int_{\chi^2_{\alpha}(n)}^{+\infty}f(x)\mathrm dx=\alpha

则称χα2(n)\chi^2_{\alpha}(n)χ2(n)\chi^2(n) 分布的α\alpha 分位点。

  • ♾️性质
    • 卡方分布的数学期望E(χ2(n))=n\boldsymbol E(\chi^2(n))=n,方差D(χ2(n))=2n\boldsymbol D(\chi^2(n))=2n
    • 卡方分布具有可加性χ12+χ22χ2(n1+n2)\chi^2_1+\chi^2_2\sim\chi^2(n_1+n_2). 其中,χ12,χ22\chi^2_1,\chi^2_2 分别服从自由度为n1,n2n_1,n_2 的卡方分布

根据卡方分布的概率密度函数,我们可以得出,若随机变量Xχ2(2)X\sim\chi^2(2) 则它也有XE(1/2)X\sim E(1/2) 。即服从于λ=1/2\lambda=1/2指数分布

t 分布

t 分布 是一种连续型随机变量的分布,其密度函数f(x)f(x) 的图像关于yy 轴对称,形状与标准正态分布类似.

典型的 t 分布对应的统计量为:

t=XY/nt=\frac{X}{\sqrt{Y/n}}

其中,X,YX,Y 相互独立,且XN(0,1),  Yχ2(n)X\sim N(0,1),\;Y\sim \chi^2(n).

我们称统计量tt 服从于自由度为nn 的 t 分布,记作tt(n)t\sim t(n).

  • 分位点

对于给定的α  (0<α<1)\alpha\;(0\lt\alpha\lt1),如果满足:

P{t>tα(n)}=tα(n)+f(x)dx=αP\{t\gt t_{\alpha}(n)\}=\int_{t_{\alpha}(n)}^{+\infty}f(x)\mathrm dx=\alpha

则称tα(n)t_{\alpha}(n)t(n)t(n) 分布的α\alpha 分位点。

由图像对称性可知,对于分位点有:t1α(n)=tα(n)t_{1-\alpha}(n)=-t_{\alpha}(n).

  • ♾️性质
    • t 分布的概率密度函数为偶函数,且:limnf(x)=12πet2/2\begin{aligned}\lim_{n\to\infty}f(x)=\frac{1}{\sqrt{2\pi}}e^{-t^2/2}\end{aligned}. 即 当nn 足够大时,t 分布近似于标准正态分布,但对于较小的nn 差距较大
    • tt(n)t\sim t(n)t2F(1,n)t^2\sim F(1,n),且tα/22(n)=Fα(1,n)t^2_{\alpha/2}(n)=F_{\alpha}(1,n)
    • t(1)t(1) 分布就是柯西分布,若X,YN(0,σ2)X,Y\sim N(0,\sigma^2),二者相互独立,则Y/Xt(1)Y/X\sim t(1) 也服从于 柯西分布

F 分布

F 分布 是一种非负连续型随机变量的分布,其密度函数f(x)f(x) 的图像与χ2\chi^2 分布类似.

典型的 F 分布对应的统计量为:

F=X/n1Y/n2F=\frac{X/n_1}{Y/n_2}

其中,X,YX,Y 相互独立,且Xχ2(n1),  Yχ2(n2)X\sim \chi^2(n_1),\;Y\sim \chi^2(n_2).

我们称统计量FF 服从于自由度为n1,n2n_1,n_2 的 F 分布,记作FF(n1,n2)F\sim F(n_1,n_2).

  • 分位点

对于给定的α  (0<α<1)\alpha\;(0\lt\alpha\lt1),如果满足:

P{F>Fα(n1,n2)}=Fα(n1,n2)+f(x)dx=αP\{F\gt F_{\alpha}(n_1,n_2)\}=\int_{F_{\alpha}(n_1,n_2)}^{+\infty}f(x)\mathrm dx=\alpha

则称Fα(n1,n2)F_{\alpha}(n_1,n_2)F(n1,n2)F(n_1,n_2) 分布的α\alpha 分位点。

  • ♾️性质
    • 如果FF(n1,n2)F\sim F(n_1,n_2),则:1FF(n1,n2)\begin{aligned}\frac{1}{F}\sim F(n_1,n_2)\end{aligned}.
    • F1α(n1,n2)=1Fα(n2,n1)\begin{aligned}F_{1-\alpha}(n_1,n_2)=\frac{1}{F_{\alpha}(n_2,n_1)}\end{aligned}.
样本均值与方差的分布

样本均值的分布

1.XN(μ,σ2/n)2.Xμσ/n=n(Xμ)σN(0,1)3.XYN(μ1μ2,σ12n1+σ22n2)4.(Xμ1)(Yμ2)σ12/n1+σ22/n2N(0,1)\begin{aligned}1.&\quad \overline{X}\sim N(\mu,\sigma^2/n)\\2.&\quad \frac{\overline{X}-\mu}{\sigma/\sqrt{n}}=\frac{\sqrt{n}(\overline{X}-\mu)}{\sigma}\sim N(0,1)\\\\\\3.&\quad \overline{X}-\overline{Y}\sim N\left(\mu_1-\mu_2,\frac{\sigma_1^2}{n_1}+\frac{\sigma_2^2}{n_2}\right)\\4.&\quad \frac{(\overline{X}-\mu_1)-(\overline{Y}-\mu_2)}{\sqrt{\sigma_1^2/n_1+\sigma_2^2/n_2}}\sim N(0,1)\end{aligned}

样本方差的分布

1.1σ2i=1n(Xiμ)2χ2(n)2.(n1)S2σ2=i=1n(XiXσ)2χ2(n1)3.XμS/n=Xμσ/n/(n1)S2σ2/(n1)N(0,1)χ2(n1)/(n1)t(n1)4.SX2/σ12SY2/σ22=[(n11)SX2/σ12]/(n11)[(n21)SY2/σ22]/(n21)χ2(n11)/(n11)χ2(n21)/(n21)F(n11,n21)5.(n1+n22)SXY2σ2χ2(n11)+χ2(n21)=χ2(n1+n22)6.(Xμ1)(Yμ2)SXY1/n1+1/n2=(Xμ1)(Yμ2)σ1/n1+1/n2/SXY2σ2t(n1+n22)where SXY2=(n11)SX2+(n21)SY2n1+n22,  σ1=σ2=σ\begin{aligned}1.&\quad \frac 1 {\sigma^2}\sum_{i=1}^n(X_i-\mu)^2\sim \chi^2(n)\\2.&\quad \frac{(n-1)S^2}{\sigma^2}=\sum_{i=1}^n\left(\frac{X_i-\overline{X}}{\sigma}\right)^2\sim\chi^2(n-1)\\3.&\quad \frac{\overline{X}-\mu}{S/\sqrt{n}}=\frac{\overline{X}-\mu}{\sigma/\sqrt{n}}\bigg/\sqrt{\frac{(n-1)S^2}{\sigma^2}\bigg/(n-1)}\sim \frac{N(0,1)}{\sqrt{\chi^2(n-1)/(n-1)}}\sim t(n-1)\\\\\\4.&\quad \frac{S_X^2/\sigma_1^2}{S_Y^2/\sigma_2^2}=\frac{[(n_1-1)S_X^2/\sigma_1^2]/(n_1-1)}{[(n_2-1)S_Y^2/\sigma_2^2]/(n_2-1)}\sim\frac{\chi^2(n_1-1)/(n_1-1)}{\chi^2(n_2-1)/(n_2-1)}\sim F(n_1-1,n_2-1)\\\\\\5.&\quad\frac{(n_1+n_2-2)S_{XY}^2}{\sigma^2}\sim\chi^2(n_1-1)+\chi^2(n_2-1)=\chi^2(n_1+n_2-2)\\6.&\quad \frac{(\overline{X}-\mu_1)-(\overline{Y}-\mu_2)}{S_{XY}\sqrt{1/n_1+1/n_2}}=\frac{(\overline{X}-\mu_1)-(\overline{Y}-\mu_2)}{\sigma\sqrt{1/n_1+1/n_2}}\bigg/\sqrt{\frac{S^2_{XY}}{\sigma^2}}\sim t(n_1+n_2-2)\\\\&\quad\text{where }S_{XY}^2= \frac{(n_1-1)S_X^2+(n_2-1)S_Y^2}{n_1+n_2-2},\;\sigma_1=\sigma_2=\sigma\end{aligned}

两个重要结论

求证

(n1)S2σ2χ2(n1)\frac{(n-1)S^2}{\sigma^2}\sim \chi^{2}(n-1)

样本方差与卡方分布的自由度

(n1)S2σ2=i=1n(xix)2σ2=i=1n(xiσxσ)2=i=1n(xiμσxμσ)2=i=1n(ZiZ)2=i=1nZi2nZ2\begin{aligned}\frac{(n-1)S^2}{σ^2}&=\frac{\sum\limits_{i=1}^n(x_i-\overline{x})^2}{\sigma^2}\\&=\sum_{i=1}^n(\frac{x_i}{\sigma}-\frac{\overline{x}}{\sigma})^2\\&=\sum_{i=1}^n(\frac{x_i-\mu}{\sigma}-\frac{\overline{x}-\mu}{\sigma})^2\\&=\sum_{i=1}^n(Z_i-\overline{Z})^2\\&=\sum_{i=1}^nZ_i^2-n\overline{Z}^2\\\end{aligned}

其中ZiZ_i表示一个标准正态分布变量。

下面利用 “一组相互独立的标准正态随机变量经过正交变换后仍然是相互独立的标准正态随机变量”的性质作定量考察。

构造一个正交矩阵AA,对ZZ 做正交变换,即Y=AZY=AZ.

其中Z=(Z1,Z2,,Zn)TZ=(Z_1,Z_2,\dots,Z_n)^T,Y=(Y1,Y2,,Yn)TY=(Y_1,Y_2,\dots,Y_n)^T,于是有:

i=1nYi2=YTY=ZTATAZ=ZTZ=i=1nZi2\sum_{i=1}^nY_i^2=Y^TY=Z^TA^TAZ=Z^TZ=\sum_{i=1}^nZ_i^2

令正交阵AA 的第一行A1=(1/n,1/n,,1/n)A_1=(1/\sqrt n,1/\sqrt n,\cdots,1/\sqrt n),其他行不用处理,只需可以配合使得AA 正交即可。于是可以得到:

nZ=(i=1nZi)2n=(A1Z)n=Y12n\overline{Z}=\frac{(\sum_{i=1}^nZ_i)^2}{n}=(A_1Z)^n=Y_1^2

从而:

i=1nZi2nZ2=i=1nYi2Y12=i=2nYi2\sum_{i=1}^nZ_i^2-n\overline{Z}^2\\=\sum_{i=1}^nY_i^2-Y_1^2=\sum_{i=2}^nY_i^2

实际上只有n1n-1 个独立的自由变量。


定性的分析,对于i=1n(ZiZ)2\begin{aligned}\sum_{i=1}^n(Z_i-\overline{Z})^2\end{aligned} 来说,这是一个nn 阶二次型f=zTAzf=z^TAz ,如果我们简单地对其作变量替换,使Yi=ZiZY_i=Z_i-\overline{Z},就不难发现,存在约束条件Yi=0\sum Y_i=0,即该变量替换是不可逆的,这也说明替换后的二次型矩阵的秩不为nn
而如果我们采用配方的方式对ZiZ_i 重新进行替换,就只能将原式化为n1n-1 个平方之和。

这就是为什么样本方差相关的自由度是n1n-1(对应二次型矩阵的秩为n1n-1

参考:1. 《概率论与数理统计》.浙江大学.第四版
2. 样本标准差与自由度 n-1 卡方分布关系的证明|CSDN
3. 概率中自由度真理理解?n-1怎么来的?|心一学长

求证 服从于正态分布的样本方差与样本均值相互独立

参考:为什么样本方差和样本均值是相互独立的,如何理解? - 知乎 https://www.zhihu.com/question/60963502/answer/2304102337

参数估计与假设检验

参数估计

参数点估计

在数理统计一章中,我们已经学习到了统计量这一概念,统计量是不依赖于任何未知参数,是样本X1,X2,..,XnX_1,X_2,..,X_n 的函数。

而假设总体XX 的分布形式已知,但仍然含有未知参数θ\theta ;抑或总体的某数字特征(如数学期望或方差)存在但未知时,借助 样本X1,X2,..,XnX_1,X_2,..,X_n 对未知参数给出一个估计值的这种参数估计问题就称为 点估计问题

显然,参数θ\theta 的估计是样本X1,X2,..,XnX_1,X_2,..,X_n 的函数,即θ^(X1,X2,..,Xn)\hat{\theta}(X_1,X_2,..,X_n),称其为 估计量.
它的观测值θ^(x1,x2,..,xn)\hat{\theta}(x_1,x_2,..,x_n) 称为 估计值,作为θ\theta 的近似值.

矩估计法

  1. 计算总体XX 的前kk 阶原点矩:

μl=E(Xl)=+xlf(x;θ1,θ2,,θk)  dx(对连续型)μl=E(Xl)=i=1xilp(xi;θ1,θ2,,θk)(对离散型)其中 l=1,2,,k  ,μl=μl(θ1,θ2,,θk)\begin{aligned}\mu_l=E(X^l)=\int_{-\infty}^{+\infty}x^lf(x;\theta_1,\theta_2,\cdots,\theta_k)\;\mathrm dx\quad(\text{对连续型})\\\mu_l=E(X^l)=\sum_{i=1}^\infty x_i^lp(x_i;\theta_1,\theta_2,\cdots,\theta_k)\quad(\text{对离散型})\\\text{其中 }l=1,2,\cdots,k\;,\mu_l=\mu_l(\theta_1,\theta_2,\cdots,\theta_k)\end{aligned}

  1. 令 样本矩 = 总体矩,即:

1ni=1nXil=E(Xl)(l=1,2,,k)\frac 1 n\sum_{i=1}^nX_i^l=E(X^l)\quad(l=1,2,\cdots,k)

  1. 由上述式子可得到kk 个包含kk 个未知参数的方程,整个方程组称为矩法方程。求解该方程即可得到:

θl^=θl^(X1,X2,..,Xn)(l=1,2,,k)\hat{\theta_l}=\hat{\theta_l}(X_1,X_2,..,X_n)\quad(l=1,2,\cdots,k)

最大似然估计法

如果已知连续型随机变量总体XX 的概率密度函数f(x;θ),θΘf(x;\theta),\theta\in\varTheta ,其中Θ\varTheta 为待估参数θ\theta 可能的取值范围,那么 来自该总体的样本X1,X2,..,XnX_1,X_2,..,X_n 的联合密度函数为θ\theta 的函数L(θ)L(\theta),即:

L(θ)=L(x1,x2,,xn;θ)=i=1nf(xi;θ),  θΘL(\theta)=L(x_1,x_2,\cdots,x_n;\theta)=\prod_{i=1}^nf(x_i;\theta),\;\theta\in\varTheta

称其为 样本的似然函数,若存在θ^\hat{\theta} 使得似然函数取得最大值,则称其为θ\theta最大似然估计值,对应的θ^(X1,X2,..,Xn)\hat{\theta}(X_1,X_2,..,X_n)最大似然估计量。有:

θ^=argmaxθΘL(x1,x2,,xn;θ)\hat{\theta}=\arg\max_{\theta\in\varTheta}L(x_1,x_2,\cdots,x_n;\theta)

L(θ)L(\theta) 可微时,可利用如下公式解得估计值:

lnL(θi)θi=0,  i=1,2,...,k\frac{\partial\ln L(\theta_i)}{\partial \theta_i}=0,\;i=1,2,...,k

注:一般我们对其取对数进行化简,即采用lnL(θ)\ln L(\theta) 作为目标优化函数)

注:求导的目的是为了找到θ\theta 取何值时,似然函数能达到最大值,所以,如果做题时发现求导结果不为零时,一定要记得最初的目的,通过考察函数的单调性来找出最大值点,从而得到估计值。

最大似然估计不变性 若存在一一映射的函数使得u=u(θ)u=u(\theta),则uu 的最大似然估计u^=u(θ^)\hat{u}=u(\hat{\theta}).

估计量的评选标准

  1. 无偏性 若参数θ\theta 的估计量θ^\hat{\theta} 的数学期望存在且E(θ^)=θE(\hat{\theta})=\theta ,则称θ^\hat{\theta}θ\theta 的无偏估计量
  2. 有效性θ1^,θ2^\hat{\theta_1},\hat{\theta_2} 都是未知参数θ\theta 的无偏估计量,那么当D(θ1^)D(θ2^)D(\hat{\theta_1})\leq D(\hat{\theta_2}) 时,称θ1^\hat{\theta_1}θ2^\hat{\theta_2} 有效
  3. 一致性(相合性)nn\to\infty 时,参数θ\theta 的估计量θ^(X1,X2,,Xn)\hat{\theta}(X_1,X_2,\cdots,X_n) 依概率收敛于θ\theta ,则称之为 一致估计量(或 相合估计量)

🔔即若limnP{θθ^ε}=0\lim\limits_{n\to\infty}P\{|\theta-\hat{\theta}|\geq\varepsilon\}=0 则称θ^\hat\thetaθ\theta 的一致估计量。

X1,X2,..,XnX_1,X_2,..,X_n 独立同分布,则:

  • ES2=σ2ES^2=\sigma^2,即S2S^2σ2\sigma^2 的无偏估计量。但是SS 未必是σ\sigma 的无偏估计量
  • 由辛钦大数定律,1/ni=1nXi21/n\sum\limits_{i=1}^nX_i^2 依概率收敛于E(X2)E(X^2)X2\overline{X}^2 依概率收敛于(EX)2(EX)^2. 进而可推导出S2Pσ2,  SPσS^2\stackrel{P}\longrightarrow\sigma^2,\;S\stackrel{P}\longrightarrow\sigma. 即SSσ\sigma一致估计量

参数区间估计

区间估计就是用统计量作为端点形成的某个随机区间来刻画总体未知参数所在的范围

置信区间

若总体XX 的分布函数F(x;θ)F(x;\theta) 含有一个未知参数θΘ\theta\in\varTheta,对于给定值α(0<α<1)\alpha\quad(0\lt\alpha\lt1),若由来自总体样本X1,X2,..,XnX_1,X_2,..,X_n 确定的两个统计量θ1^,θ2^(θ1^θ2^)\hat{\theta_1},\hat{\theta_2}\quad(\hat{\theta_1}\leq \hat{\theta_2}) ,有:

θΘ,  s.t.P{θ1^<θ<θ2^}1α\forall \theta\in\varTheta,\;s.t.P\{\hat{\theta_1}\lt\theta\lt\hat{\theta_2}\}\geq 1-\alpha

则称随机区间(θ1^,θ2^)(\hat{\theta_1},\hat{\theta_2})θ\theta 置信水平/置信度1α1-\alpha置信区间

单侧置信区间从略.

假设检验的基本概念

假设检验的基本概念

  1. 假设检验问题 在总体分布函数未知或只知形式不知参数时,提出某种关于其分布函数或参数的假设,并抽取样本构造合适的统计量,再根据样本对所提假设进行接受或拒绝,这样的问题称为 假设检验问题
  2. 检验法 借助于样本值来判断接受或拒绝假设的法则
  3. 原假设与备择假设 称需要着重考察的假设为 原假设,常记为H0H_0(原假设一般需要包括等于号),与之对立的假设称为备择假设或对立假设,记为H1H_1
  4. 检验统计量 如果基于某一个统计量的观测值来确定接受H0H_0 与否时,这一统计量被称为 检验统计量
  5. 拒绝域与临界点 检验统计量的观测值落在某个区域时就拒绝H0H_0,则该区域就称 拒绝域,拒绝域的边界点称为 临界点
  6. 假设检验的两类错误H0H_0 实际为真但拒绝之的错误称为“弃真”;反之,实际为假却接受之的错误称为“取伪”
  7. 显著性水平 在作检验时,要求犯第一类错误(弃真)的概率α\leq \alphaα\alpha 称为检验的显著性水平,通常取 0.1、0.05、0.01、0.005 等

简单假设 被定义为 “该假设成立,总体分布就完全确定” 。
例如,若XX 服从于泊松分布,作原假设H0:DX=3H_0:DX=3,则该假设是简单假设。因为泊松分布的数学期望与方差都是参数λ\lambda ,一旦λ\lambda 确定,整个分布就完全确定。
但是,若XX 服从于正态分布,作原假设H0:EX1H_0:EX\leq1 ,则该假设就不是简单假设

假设检验的一般步骤

  1. 提出所要检验的原假设H0H_0 与备择假设H1H_1
  2. 选择检验的统计量(通常根据参数的已知与未知情况选取),并在H0H_0 成立下求出其分布;
  3. 给定显著性水平α\alpha,在H0H_0 成立下确定临界值和拒绝域RR,即:

α=P{ 拒绝域 R    H0 成立 }\alpha=P\{\text{ 拒绝域 }R\;|\;H_0\text{ 成立 }\}

  1. 由样本值确定统计量的值,并判断该值是否落在拒绝域内,以此判断对原假设拒绝与接受.